Superpage
Bladder & urothelial tract

Authors: Turki Al-Hussain, M.D., Mohammed Alghamdi, M.B.B.S., Nicole K. Andeen, M.D., Daniel A. Anderson, M.D., M.B.A., Daniel Athanazio, M.D., Ph.D., Harsh Batra, M.B.B.S., D.C.P., D.N.B., Maria Carolina Beeter, M.D., Megan L. Brown, M.D., Alcides Chaux, M.D., Jesus Adrian Chavez, M.D., Bonnie Choy, M.D., Min Dai, M.D., Ph.D., Michelle R. Downes, M.D. , Lindsey Durowoju, M.D., Reima El Naili, M.D., Jonathan I. Epstein, M.D., Gong Feng, M.D., Ph.D., Jatin S. Gandhi, M.B.B.S., M.D., Mustafa Goksel, M.D., Lucy Jager, M.D., Jennifer Lee, M.D., Varsha Manucha, M.D., Timothy Isaac Miller, M.D., M.A., Hunter Monroe, B.S., Manasa Morisetti, M.D., Ngoc-Nhu Jennifer Nguyen, M.D., Erdener Özer, M.D., Ph.D., Vaishali Pansare, M.D., Rugvedita Parakh, M.D., Anil Parwani, M.D., Ph.D., M.B.A., Nat Pernick, M.D., Susan Prendeville, M.D., Nathan Rinehouse, M.D., Carol Rizkalla, M.B.B.Ch., Monika Roychowdhury, M.D., Iryna Samarska, M.D., Ph.D., Ruhani Sardana, M.B.B.S., Judy Sarungbam, M.D., Moe Thuzar, M.D., Mirna Toukatly, M.D., Maria Tretiakova, M.D., Ph.D., Theodorus van der Kwast, M.D., Ph.D., Sean R. Williamson, M.D., Huihui Ye, M.D., M.S., Y. Albert Yeh, M.D., Ph.D., Yan Hong Shirley Yu, M.D., Bohdan Zoshchuk, M.D., Chunlai Zuo, M.D., M.S., Debra L. Zynger, M.D.
Resident / Fellow Advisory Board: Alcino Pires Gama, M.D.
Editorial Board Members: Nicole K. Andeen, M.D., Bonnie Choy, M.D., Maria Tretiakova, M.D., Ph.D., Debra L. Zynger, M.D.
Deputy Editor-in-Chief: Maria Tretiakova, M.D., Ph.D.
Editor-in-Chief: Debra L. Zynger, M.D.

Copyright: 2003-2024, PathologyOutlines.com, Inc.

GU related: Jobs, Fellowships, Conferences, Cases, CME, Board Review

Related chapters: Penis and scrotum, Prostate, Testis

Editorial Board oversight: Maria Tretiakova, M.D., Ph.D. (last reviewed February 2021), Debra Zynger, M.D. (last reviewed July 2020)
Page views in 2024 to date: 14

Acute cystitis
Definition / general
  • A clinical diagnosis, usually with a triad of frequency, lower abdominal pain and dysuria (pain or burning during urination)
  • There is usually no surgical specimen for acute cystitis, although it may be a finding in a specimen obtained for other purposes, or at autopsy
Epidemiology
  • Common in young women of reproductive age; also older men and women
Sites
  • Bladder or lower urinary tract (urethra)
Etiology
  • Most common bacterial agents are E. coli, Proteus, Klebsiella, Enterobacter
  • Also due to Candida or Cryptococcus in immunocompromised, Schistosoma haematobium in Egypt; also adenovirus, chlamydia, mycoplasma
  • Noninfectious causes are chemotherapy, radiation therapy and trauma
  • For E. coli infections, the host's fecal flora (and in women, vaginal flora) is the most common immediate source for the infecting strain
  • The E. coli strain may represent the most prevalent fecal/vaginal E. coli clones of the individual (the prevalence hypothesis) or a distinctive, highly selected subset of the fecal/vaginal E. coli population with enhanced virulence potential (the special-pathogenicity hypothesis, J Clin Microbiol 2008;46:2529)
Clinical features
  • Patients may be asymptomatic
  • May also be caused by obstruction, cystocele or diverticula
  • May lead to pyelonephritis
  • Uncomplicated:occurs in otherwise healthy nonpregnant adult women
  • Complicated: associated with conditions that increase the risk of therapy failure, such as an upper tract infection or drug resistant pathogen; a broader spectrum antimicrobial is recommended in these cases
  • Excellent prognosis; symptoms usually resolve within 1-2 days after treatment
Treatment
  • Suggested clinical algorithm: treat empirically with antibiotics if 2 of 3 variables present: dysuria, urine WBC > trace, urine nitrites); otherwise obtain culture and wait for results (Arch Intern Med 2007;167:2201)
  • Complicated cases require a broader spectrum antibiotic for a longer period of time
Gross description
  • The bladder may show no gross abnormalities
  • Occasionally, the mucosa may be hyperemic with variable exudate
Microscopic (histologic) description
  • Usually neutrophils

Adenocarcinoma
Definition / general
  • Carcinoma derived from the urothelium and showing pure glandular differentiation
  • Not including urachal adenocarcinoma or urothelial carcinoma with glandular differentiation
Essential features
  • Rare, derived from the urothelium of the bladder with pure glandular differentiation
  • Histology: intestinal (enteric) type, mucinous type, signet ring cell type, NOS and mixed
  • Immunohistochemistry: often CK7+ and CK20+ or CK7- and CK20+, no nuclear staining for beta catenin
  • Diagnosis made after excluding secondary adenocarcinoma either by metastasis or by direct extension
Terminology
  • Primary adenocarcinoma of the bladder
  • Primary adenocarcinoma of the ureter
  • Primary adenocarcinoma of the renal pelvis
ICD coding
  • ICD-10:
    • C65 - malignant neoplasm of renal pelvis
    • C66 - malignant neoplasm of ureter
    • C67 - malignant neoplasm of bladder
Epidemiology
Sites
  • Usually lateral, posterior wall or trigone of bladder (if dome, should consider urachal adenocarcinoma by default)
  • Second most common site in urethra; rare in ureters and renal pelvis
Etiology
  • Some cases may be due to progression of extensive intestinal metaplasia (cystitis glandularis) or villous adenoma; these cases arise at trigone and are usually enteric
  • Exstrophy: diffuse intestinalization; 4 - 7% risk of developing adenocarcinoma, even after repair (J Urol 1970;104:699)
  • Diverticula: usually develop urothelial carcinoma, occasionally adenocarcinoma
  • Also Schistosoma haematobium (Urol Oncol 2006;24:13)
  • Chronic irritation of bladder, including nonfunctioning bladder or obstruction (Urol Oncol 2006;24:13)
  • Urethral adenocarcinomas may arise from metaplastic surface mucosa or from periurethral glands secondary to chronic inflammation, stricture, diverticula, fistula and infections
Clinical features
  • Usually presents with hematuria, rarely with mucusuria, dysuria (Cancer 1991;67:2165)
  • Patients are older and mucusuria is more common than in urachal adenocarcinoma
  • 5 year disease free survival rate is 40 - 50% (Urol Oncol 2006;24:13)
Diagnosis
  • Screen test: urine cytology
  • Imaging modalities include ultrasound, computerized tomography (CT) urogram, retrograde pyelogram or magnetic resonance imaging (MRI) urogram
  • Cystoscopic examination of the entire urethra and bladder, with diagnostic specimens collected by either biopsy or transurethral resection of bladder tumor (TURBT) (Eur Urol Focus 2021 [Epub ahead of print])
Radiology images

Contributed by Chunlai Zuo, M.D., M.S. and Huihui Ye, M.D., M.S.
Bladder wall mass by CT

Bladder wall mass by CT



Images hosted on other servers:
Bladder neck mass by ultrasound

Bladder neck mass by ultrasound

Prognostic factors
  • Stage is most important prognostic factor
  • Compared with urothelial carcinoma, patients with bladder adenocarcinoma have a worse prognosis, as they commonly present at a more advanced stage
  • It is controversial whether it is more aggressive if controlled for tumor stage
  • After adjusting pathologic stage, the presence of lymphovascular invasion and lymph node status, a study demonstrated that the primary adenocarcinoma has a worse prognosis than conventional urothelial carcinoma (J Urol 2006;175:2048)
Case reports
Treatment
  • Radical cystectomy and pelvic lymph node dissection
  • Adjuvant therapy (radiation or chemotherapy) may be given in some cases
Clinical images

Images hosted on other servers:
Bladder neck mass by ultrasound

Bladder neck tumor cystoscopy

Gross description
  • 67% are solitary lesions (while urothelial carcinoma tends to be multifocal)
  • Tumor surface may be covered by gelatinous material
  • Papillary, nodular, flat or ulcerated
Gross images

Images hosted on other servers:
Missing Image

Primary signet ring
cell carcinoma of
upper urinary tract

Missing Image

Adenocarcinoma of urinary bladder

Microscopic (histologic) description
  • Intestinal type (or enteric type):
    • Resembles colonic adenocarcinoma (Cancer 1991;67:2165)
    • Neoplastic glands are lined by pleomorphic mucin producing pseudostratified columnar epithelium
    • Central dirty necrosis is commonly seen
  • Mucinous (colloid) type:
    • Nests of neoplastic cells floating in abundant extravasated mucin
    • Singly dispersed or groups of signet ring cells can be seen in the mucin pools
    • Mucin usually deeply invades muscularis propria
  • Signet ring cell type (diffuse poorly differentiated adenocarcinoma):
    • Discohesive round cells with large intracellular mucin vacuoles displacing nuclei to the periphery without extracellular mucin
    • Diffusely infiltrating stromal tissue
  • Association with in situ component is an important clue to differentiate primary adenocarcinoma of the bladder from secondary adenocarcinoma involving the bladder
  • Intestinal metaplasia or villous adenoma may be seen
Microscopic (histologic) images

Contributed by Chunlai Zuo, M.D., M.S. and Huihui Ye, M.D., M.S.
Villoglandular growth

Villoglandular growth

Cytologic atypia

Cytologic atypia

Desmoplastic reaction

Desmoplastic reaction

Intraluminal mucin

Intraluminal mucin

Muscularis propria invasion

Muscularis propria invasion


Signet ring cells

Signet ring cells

Mucin pool

Mucin pool

Malignant glandular epithelium

Malignant glandular epithelium

Intestinal / enteric type Intestinal / enteric type

Intestinal / enteric type

Virtual slides

Images hosted on other servers:
Missing Image

Mucinous adenocarcinoma of the bladder

Cytology description
  • Clusters of tumor cells, intracytoplasmic vacuoles, eccentrically located nuclei, hyperchromatic irregular nuclear contours, conspicuous to prominent nucleoli (Cancer 1998;84:335)
  • Tumor cells of signet ring cell carcinoma show large intracytoplasmic mucin vacuole, hyperchromatic crescent shaped nuclei
  • Background necrosis or mucin may be present, depending on the subtype
Cytology images

Images hosted on other servers:
Missing Image

Vacuolated cells

Missing Image

Penile metastasis

Positive stains
Negative stains
Molecular / cytogenetics description
Molecular / cytogenetics images

Images hosted on other servers:
Missing Image Missing Image

Abnormal FISH

Sample pathology report
  • Bladder and prostate, radical cystoprostatectomy:
    • Bladder, invasive adenocarcinoma, mixed mucinous type with signet ring cell component (see synoptic report)
    • Prostate, negative for carcinoma
Differential diagnosis
Board review style question #1

A bladder TURBT is shown. Which of the following immunohistochemistry stains is most helpful in distinguishing the metastatic colon cancer to the bladder from the primary adenocarcinoma of the bladder, intestinal type?

  1. Beta catenin
  2. CDX2
  3. CK7
  4. CK20
  5. STAB2
Board review style answer #1
A. Beta catenin. Up to 80% of metastatic colon cancers are positive for nuclear beta catenin (in addition to cytoplasmic stains), an indicator of WNT pathway activation. Primary adenocarcinoma of the bladder, intestinal type can express CK20, CDX2 and STAB2, however, lacks nuclear beta catenin staining.


Comment Here

Reference: Adenocarcinoma

Adenocarcinoma in situ
Definition / general
  • Primary noninvasive adenocarcinoma of bladder
  • Findings below are for patients without concurrent invasive adenocarcinoma or villous adenoma
Terminology
Clinical features
  • Mean age 70 years, 75% male
  • Associated with urothelial CIS or papillary urothelial carcinoma without invasion
  • 50-74%% developed invasive carcinoma, including high incidence of poorly differentiated, micropapillary and small cell urothelial carcinoma (Am J Surg Pathol 2001;25:892)
Treatment
  • Transurethral excision; intravesical chemotherapy; intravesical bCG
Gross description
  • Exophytic, papillary to flat
Microscopic (histologic) description
  • Papillary, flat, and cribriform architecture
  • Neoplastic cells are columnar with luminal cytoplasm with occasional intracytoplasmic mucin
  • True glandular differentiation (by definition)
  • Moderate to severe nuclear pleomorphism, apoptosis, 5+ mitotic figures/10 HPF
  • Noninvasive, by definition
  • Often associated with other urothelial carcinoma patterns including small cell, micropapillary and poorly differentiated
  • Rare necrosis
  • Not commonly associated with invasive adenocarcinoma
Differential diagnosis
  • Urothelial carcinoma in situ: no invasion, no glandular formation, no/rare intracytoplasmic mucin
  • Urothelial carcinoma with gland like spaces: invasive, no true glandular formation, no/rare intracytoplasmic mucin
  • Florid cystitis glandularis with or without intestinal metaplasia: insufficient atypia to diagnosis as carcinoma in situ
  • Villous adenoma: insufficient atypia to diagnosis as carcinoma in situ
  • Noninvasive micropapillary carcinoma: has characteristic retraction-like spaces; no glandular formation, no/rare intracytoplasmic mucin
  • Clear cell adenocarcinoma: invasive, hobnail cells

Amyloidosis
Definition / general
  • Deposition of amyloid protein in urinary bladder
  • Almost always primary (amyloid tumor); rarely part of systemic disease
Sites
  • Preferentially affects posterior and posterolateral walls
Etiology
Systemic
  • Can be primary (AL type amyloid), secondary (AA type amyloid) or familial (ATTR type amyloid)
  • Systemic secondary bladder amyloidosis is associated with autoimmune disease and chronic infection
  • Familial cases are associated mostly with mutations in the transthyretin gene

Localized
Clinical features
Treatment
  • Transurethral resection and fulguration of amyloid tumor is usually curative, since not associated with myeloma (Am J Surg Pathol 1978;2:141); also controls bleeding
  • Partial cystectomy for large, mass forming lesions
Microscopic (histologic) description
  • Large masses of eosinophilic proteinaceous material with hemorrhage in lamina propria
  • Variable foreign body giant cell reaction to amyloid
  • May have associated atypical epithelium due to attenuation of urothelium
  • Rarely perivascular amyloid deposits, especially in systemic amyloidosis
  • Rare / no inflammatory cells
Positive stains
  • Congo red shows apple green birefringence when exposed to polarized light
  • Amyloid panel (kappa and lambda light chains, prealbumin, beta-2-microglobulin, SAA1)
  • Immunofluorescence with Thioflavin T
Electron microscopy description
  • Nonbranching, randomly distributed, rigid fibrils (8 - 10 nm) and associated ground substance
Differential diagnosis
  • Fibrosis: positive for trichrome stain, negative for Congo Red, no Thioflavin T immunofluorescence

Anatomy
Definition / general
  • Hollow organ that serves as a reservoir for urine
  • Adult bladder can hold up to 400 - 500 ml of urine without a change in intraluminal pressure
  • In addition, can initiate and sustain a contraction until empty
  • Distended adult bladder may reach level of umbilicus
  • ICD-O-3 coding
Embryology
  • Bladder develops during first 12 weeks of gestation
  • Bladder and trigone development are regulated by complex epithelial-mesenchymal signaling events (Curr Opin Urol 2009;19:427)
  • Urorectal septum divides cloaca into dorsal rectum and ventral urogenital sinus
  • Urogenital sinus is further divided into 3 parts: cranial vesical part (forms most of bladder, except trigone), middle pelvic part (forms urethra in bladder neck, prostatic urethra in males and entire urethra in female) and caudal phallic part (primordium of penis or clitoris)
  • Trigone develops from dilation, fusion and incorporation of caudal mesonephric ducts into urogenital sinus, forming a triangular area that is site of future ureters
  • Mesonephric ducts are gradually absorbed and replaced by endodermal epithelium of urogenital sinus
  • Posterior walls, dome and part of lateral walls arise from mesenchyme surrounding urogenital sinus
  • Anterior wall and part of lateral walls develop with closure of infraumbilical portion of abdominal wall
  • Note: neither urachus or allantois are involved in formation of bladder
Parts of bladder
  • Apex / dome: most anterosuperior point; is covered by peritoneum; site of insertion of median umbilical ligament (obliterated urachus) which anchors the bladder to the abdominal wall
  • Base: posterior surface, is also covered by peritoneum
  • Neck: most distal portion of bladder
  • Trigone: area between ureteral and urethral orifices, continuous with bladder neck
Relationship to other structures
  • Located in part within the abdomen in children, enters pelvis major at age 6, found entirely within pelvis minor ("true" or obstetric pelvis) after puberty
  • Adult bladder rests on rectum and seminal vesicles (males) or cervix and vagina (females); thus, cystectomy for tumor may be combined with removal of prostate and seminal vesicles (males) or hysterectomy and partial vaginectomy (females)
  • Ureters enter the bladder posteroinferiorly and obliquely, travel 1.5 - 2 cm through the bladder wall (intravesical portion) and open at the ureteral orifices
  • Bladder neck is formed by interlacing and converging fiber of detrusor muscle forming 3 well defined layers; occasionally contains prostatic ducts (males)
  • Anteroinferiorly and laterally, there is abundant fat and loose connective tissue (space of Retzius)
  • Bladder is relatively free within pelvis (which permits its expansion as it distends) but neck is secured by puboprostatic ligaments (male) and pubovesical ligaments (female)
Blood supply
  • Superior, middle and inferior vesical arteries, derived from the anterior trunk of the internal iliac artery
  • In addition, minor branches are derived from obturator and inferior gluteal arteries and, in women, uterine and vaginal arteries
  • Venous drainage by a rich vesical venous plexus, which empties into internal iliac veins
Lymphatic drainage
  • Vesical, internal and external iliac nodes; bladder neck drains to sacral or common iliac nodes
Nerve supply
  • Sympathetic from T11-L2 nerves, plays no role in micturition
  • Parasympathetic from S2-4, travel to bladder via pelvic nerve and inferior hypogastric plexus, cause contraction of muscularis propria fibers, which puts traction on bladder neck, which opens internal sphincter; important in micturition
Ureters
  • 30 cm long, 5 mm in diameter
  • Areas of ureteral narrowing are at ureteropelvic junction, crossing of external / common iliac artery and where ureters enter bladder; these are common sites of obstruction and stone impaction
  • Ureters enter bladder in oblique manner
  • Bladder muscle contraction mechanically closes off ureteral orifice
  • Defects cause reflux
Diagrams / tables

Images hosted on other servers:

Development of bladder

Position of bladder in female pelvis

Interior

Position of bladder in male pelvis

Gross description
  • Hollow viscus resembling inverted pyramid when empty, sphere when distended
  • Internal (mucosal) surface is flat when distended but show abundant folds in an empty bladder
Microscopic (histologic) description
  • Allantois: rudimentary structure lined by endoderm that is connected to urachus
  • Urachus: formed during descent of abdominal wall, connects umbilicus to apex (dome) of bladder, torn apart as embryo elongates but remnants persist in anterior abdominal wall and may persist in bladder wall (see patent urachus)

Angiosarcoma
Definition / general
  • Rare vascular tumor with anastomosing vascular channels
  • Endothelial cells usually exhibit marked cytologic atypia
Epidemiology
  • Mean age 64 years, 89% occur in males
Etiology
  • Due to radiation therapy for pelvic malignancy in 1/3; also exposure to arsenic, thorium dioxide, polyvinyl chloride or chemotherapeutic agents
  • It has been reported to arise in bladder with preexisting hemangioma
Clinical features
Case reports
Treatment
  • Wide radical resection, possibly radiotherapy/chemotherapy
Gross description
  • Hemorrhagic, raised mass (mean 7 cm) of trigone or dome
Microscopic (histologic) description
  • Anastomosing vascular channels lined by plump hyperchromatic cells
  • Also solid growth and epithelioid cytology
  • Typically invades the muscularis propria (Am J Surg Pathol 2008;32:1213)
Positive stains
Differential diagnosis
  • Hemangioma: typically 1 cm or less, 80% are cavernous, no atypia, no anastomosing or solid areas
  • Kaposi’s sarcoma: associated with HIV and HHV8; extravasated red blood cells present; usually less atypia than angiosarcoma
  • Sarcomatoid urothelial carcinoma: undifferentiated spindle cells; associated urothelial carcinoma; vascular markers are negative

Arteriovenous malformation
Definition / general
  • By definition, direct communication is present between arterioles and venules
Sites
  • More common in CNS, intestine, lung, extremities
  • Very rare in urinary bladder
Etiology
  • Can be congenital or acquired (post-traumatic)
Clinical features
  • The most common symptom is hematuria (gross or micro, persistent or intermittent, may be massive)
  • Other symptoms include dysuria, difficulty in voiding and urinary retention
  • Some cases are asymptomatic
Case reports
Treatment
  • Excision is adequate therapy
Gross description
Microscopic (histologic) description
  • Admixture of malformed vessels such as capillaries, arteries and venules
  • Abrupt changes in thickness of medial and elastic layers of vessels, abnormal vascular dilation
  • Often advanced small vessel disease, hemorrhage, ulceration (Hum Pathol 1986;17:94)
  • Involves submucosa but not muscularis propria
  • May be associated with pseudocarcinomatous epithelial hyperplasia of bladder (Am J Surg Pathol 2008;32:92)
Microscopic (histologic) images

Contributed by @Andrew_Fltv on Twitter
Contributed by @Andrew_Fltv on Twitter (see original post here)"> Arteriovenous malformation Contributed by @Andrew_Fltv on Twitter (see original post here)"> Arteriovenous malformation

Arteriovenous malformation


Bullous cystitis
Definition / general
  • Reversible inflammatory condition of bladder mucosa with extensive submucosal edema
Terminology
  • An endoscopic term referring to bullous appearing edematous bladder mucosa, that mimics neoplastic bladder lesions
  • A variant of polypoid cystitis, but with broader based and larger lesions
Epidemiology
  • Wide age variation from young children to adults
  • Associated with chronic cystitis in girls
Etiology
  • Chronic irritation (chronic catheter use, neuromuscular ureteral dysplasia, vesicoureteral reflux or urolithiasis)
  • Association with bullous pemphigoid (BP) has been suggested in at least on case report; expression of BP180 has been demonstrated in the transitional epithelium of the bladder by indirect immunofluorescence
    • In addition, BP230 has been demonstrated by immunofluorescence in a rat bladder tumor cell line
  • May be associated with an acute infection (hip arthritis, gonorrheal pelvic inflammatory disease)
Clinical features
  • May be asymptomatic or present with dysuria, extreme frequency and urgent micturition responsible for pseudo-urinary incontinence
  • If located in the trigone and periureteral zone, may produce transient acute ureteral obstruction
Radiology description
  • Pelvic CT and MRI may demonstrate thickening of bladder wall
  • Intravenous urography and sonography may show hypoechoic mucosa and nodular filling defects caused by the edema, and if prominent, ureteral obstruction
  • This may be radiographically indistinguishable from bladder carcinoma
Prognostic factors
  • Intermediate severity; usually reversible
Case reports
Treatment
  • Usually treated by removing the offending agent (catheter) or the infectious or inflammatory condition (such as in BP)
  • Surgery may be needed to correct a vesicoureteral reflux
Gross description
  • On cystoscopy, bladder shows numerous bullae with a pseudoneoplastic appearance
  • Changes seen are a thickened mucosa that is smooth in early stages and becomes redundant and polypoid in later stages
Microscopic (histologic) description
  • Broad bulbous projections of mucosa with prominent submucosal edema, acute and chronic inflammation, lymphangiectasis
  • Urothelium may show reactive atypia, focal or diffuse thickening
  • Congenital mucosal variants include overdevelopment of lymphoid tissue as massive lymphoid follicles and a lymphangiectatic form (Urologiia 2000;1:3)
Microscopic (histologic) images

Images hosted on other servers:
Missing Image

Many eosinophils

Differential diagnosis

Carcinoma in situ
Definition / general
  • Flat lesion composed of cells in mid to upper epithelium with high cytologic grade
  • By definition, no invasion into lamina propria
  • Note: high grade non-invasive papillary lesions are NOT designated carcinoma in situ to avoid confusion
Terminology
  • Also known as high grade intraurothelial neoplasia (HG IUN), severe dysplasia (sometimes)
Epidemiology
  • De novo CIS constitutes less than 3% of all urothelial neoplasms, but occurs in 45% with concurrent invasive bladder carcinoma
Clinical features
  • Symptoms are similar to cystitis; hematuria is common
  • 20 - 80% of CIS patients develop invasive disease if left untreated
  • Confers poorer prognosis in patients with coexisting noninvasive papillary urothelial carcinoma
  • Often involves urothelium in other areas of GU tract
  • Associated with multifocal high grade invasive carcinoma
  • Presence should be included in pathology reports
Prognostic factors
Case reports
Treatment
Gross description
  • Flat, grossly erythematous, granular or cobblestone mucosa
  • No mass
  • May involve large areas of mucosal surface, ureters, urethra
Microscopic (histologic) description
  • Flat lesion composed of cells with large, irregular, hyperchromatic nuclei, prominent nuclear pleomorphism, high N/C ratio, mitotic figures in mid to upper epithelium
  • Atypia may not be full thickness
  • Epithelium is often denuded
  • Nuclear size is 5x that of lymphocytes versus 2x lymphocytes for normal urothelium (Hum Pathol 2001;32:997)
  • Also (but less important) loss of polarity, nuclear crowding, irregular thickness of urothelium
  • Cells are not cohesive, leading to shedding into urine
  • Occasionally present in prostatic ducts, spreads by intramucosal extension
  • Note: high grade non-invasive papillary lesions are NOT designated as carcinoma in situ to avoid confusion
  • Large cells with pleomorphism, large cells without pleomorphism, small cell, clinging (single layer of atypical cells on denuded urothelium), cancerization of urothelium (pagetoid - Hum Pathol 1993;24:1199, undermining or overriding)
  • Pattern need not be included in surgical pathology report
  • Microinvasion (2 mm or less) demonstrates invasive cells with retraction artifact mimicking vascular invasion (77% of cases of microinvasion)
  • Also nests or irregular cords, rarely invades as isolated single cells with or without desmoplasia (Am J Surg Pathol 2001;25:356)
Microscopic (histologic) images

Contributed by Sean R. Williamson, M.D. and Bhavesh Papadi, M.D. (Case #331)
Caption Caption Caption

Ureter CIS

Caption

Denuded urothelium


Various images


Additional images



Contributed by @AnaPath10 on Twitter
Bladder carcinoma in situ Bladder carcinoma in situ Bladder carcinoma in situ

Bladder carcinoma in situ



Images hosted on other servers:

H&E

Loss of polarization

Broadened urothelium

Narrowed urothelium

Pagetoid pattern

Cytology description
  • Cytology is 95% sensitive for carcinoma in situ
  • Nuclear changes of carcinoma with minimal pleomorphism
  • Numerous high-grade neoplastic cells
  • Relatively clean background
  • Image analysis of bladder wash cytology may be comparable to "expert" cytologic review (Mod Pathol 1997;10:976)
Videos


Histopathology Bladder Transitional Carcinoma in situ

Positive stains
Molecular / cytogenetics description
  • Monoclonal with aneuploid DNA and more abnormal microsatellites than corresponding invasive carcinoma, if present (Hum Pathol 2009;40:988)
  • Has somatic mismatch repair protein down-regulation and accumulation of tumor suppressor gene microsatellite abnormalities
  • Molecular pattern of CIS is divergent from coexistent muscle invasive urothelial carcinoma
  • Deletion of 9p21 or polysomy of #9 (Hum Pathol 2008;39:527)
Differential diagnosis
  • Denuding cystitis: cells may look malignant
  • Denuded urothelium:
    • Extensively denuded epithelium is often seen in CIS ("clinging CIS" or "denuding cystitis")
    • However residual malignant cells required for diagnosis
    • Deeper sectioning of tissue block or examination of von Brunn nests may be helpful if epithelium is denuded
  • Dysplasia: less severe atypia although distinction may be difficult
  • Post-topical therapy for high grade urothelial carcinoma: still has capillaries
  • Radiation effect: cells still cohesive, may have distinctive nuclear borders, may resemble pagetoid variant of CIS
  • Reactive atypia: less pleomorphic nuclei than CIS; patchy CK20 in umbrella cells only, p53 weak/negative, CD44 diffusely or focally positive vs. CIS which is intensely CK20+ (81%), p53+ (57%), CD44- (100%, Am J Surg Pathol 2001;25:1074)

Clear cell (adeno)carcinoma
Definition / general
  • Resembles clear cell carcinoma (Müllerian type) of female genital tract
Terminology
  • Also called mesonephric or mesonephroid carcinoma / adenocarcinoma
Etiology
  • Either arises from Müllerian elements in the bladder (endometriosis), represents a peculiar variant of vesical adenocarcinoma of non Müllerian derivation, or represents a peculiar morphologic expression of urothelial carcinoma, with glandular differentiation often uncertain (Am J Surg Pathol 2002;190)
Clinical features
  • Usually women, in bladder or urethra
  • Often high stage at diagnosis; stage is important prognostic factor
Case reports
  • 35 year old woman with endometriosis (Mod Pathol 1993;6:225)
  • 70 year old woman with multiple recurrences of nephrogenic metaplasia of the urinary bladder and subsequent clear cell adenocarcinoma (Hum Pathol 2006;37:117)
Gross description
  • Usually papillary, also sessile
  • If presents as polypoidal growth, frequent ulceration is seen
Microscopic (histologic) description
  • Often papillary or tubulocystic (Am J Surg Pathol 1985;9:816)
  • Flat or cuboidal neoplastic cells have abundant clear or eosinophilic cytoplasm with glycogen and frequent hobnailing
  • More pleomorphic cells and more mitotic figures than adenomatoid tumor; variable necrosis
  • Resembles urothelial carcinoma more than adenocarcinoma
  • The papillae sometimes have hyalinized cores
  • Myxoid stroma may be seen
Microscopic (histologic) description

Contributed by @SueEPig on Twitter and Case #194
Clear cell (adeno)carcinoma Clear cell (adeno)carcinoma

Clear cell (adeno)carcinoma

Clear cell (adeno)carcinoma Clear cell (adeno)carcinoma

Clear cell (adeno)carcinoma


Various images


CK7

CD15

Ki67

p53

Positive stains
Negative stains
Differential diagnosis

Condyloma acuminatum
Definition / general
  • Rare lesion of the urinary bladder, composed of hyperplastic papillary fronds of squamous epithelium with koilocytosis and viral atypia
  • Caused by human papillomavirus (HPV) (both low risk HPV and high risk HPV)
Essential features
  • Polypoid lesion composed of hyperplastic papillary fronds of squamous epithelium with koilocytosis and viral atypia
  • Caused by HPV (both low risk HPV and high risk HPV) but negative RNA in situ hybridization does not exclude condyloma acuminatum
  • Often immunocompromised patients and history of anogenital condylomata
  • Differential diagnosis includes squamous papilloma, squamous metaplasia, verrucous squamous hyperplasia and papillary urothelial carcinoma
ICD coding
  • ICD-10: N32.9 - Bladder disorder, unspecified
Epidemiology
Sites
  • Urethra (more common) and urinary bladder (rare)
Pathophysiology
Etiology
  • Caused by HPV (both low risk HPV and high risk HPV)
Clinical features
Diagnosis
  • Cystoscopy
  • Diagnosis is by biopsy or surgical resection
Prognostic factors
  • Often recurs
  • At risk for in situ and invasive squamous cell carcinoma
Case reports
Treatment
  • Transurethral resection
  • Some studies suggest that BCG, which is often used for recurrent or multiple noninvasive papillary urothelial carcinomas, is also an effective therapy for recurrent penile or uterine cervical condylomata and thus can be used for treatment of condyloma acuminatum of the urinary bladder
Clinical images

Images hosted on other servers:

Cystoscopy

Gross description
  • Depends on the size of the lesion and often not remarkable
  • Smooth, pink-tan, papillary
Microscopic (histologic) description
  • Exophytic lesion
  • Nonkeratinizing, hyperplastic papillary fronds of squamous epithelium
  • Lack free floating papillary fronds
  • Cells with koilocytosis and viral atypia (Am J Surg Pathol 2006;30:883)
Microscopic (histologic) images

Contributed by Jonathan Epstein, M.D.

Prostatic urethra papillary lesion

Urinary bladder hyperplastic lesion

Urinary bladder koilocytosis

Molecular / cytogenetics description
  • RNA in situ hybridization testing for low risk and high risk HPV is needed
  • Can be positive for either low risk or high risk HPV or both (Am J Surg Pathol 2019;43:1547)
  • Negative RNA in situ hybridization for HPV does not exclude condyloma acuminatum if the histology is characteristic (Am J Surg Pathol 2019;43:1547)
Sample pathology report
  • Urinary bladder, papillary lesion, transurethral resection:
    • Condyloma acuminatum (see comment)
    • Comment: The lesion is positive for low risk HPV. Condyloma acuminatum of bladder can be associated with in situ or invasive squamous cell carcinoma. Therefore, close followup is warranted. Additional sampling is recommended to exclude concomitant squamous cell carcinoma if clinically indicated.
Differential diagnosis
  • Squamous papilloma:
    • More common in women
    • Shows free floating papillary fronds
    • Well established fibrovascular cores
    • Lined by a benign squamous nonkeratinizing epithelium
    • Lacks koilocytosis or viral atypia
    • Unrelated to HPV infection
  • Squamous metaplasia:
    • Can be divided into nonkeratinizing and keratinizing types
    • Nonpolypoid flat lesion consisting of a thickened squamous epithelium with an overlying thick keratin layer
    • Squamous dysplasia may be seen
    • Glycogenated squamous metaplasia have cleared out cytoplasm but lacks papillary growth and viral atypia
  • Verrucous squamous hyperplasia:
    • Spiking squamous hyperplasia with marked hyperkeratosis, parakeratosis and elongation of the rete pegs
    • No viral atypia
    • This lesion might be related to invasive squamous cell carcinoma
  • Papillary urothelial carcinoma:
    • Can have clear cytoplasm
    • Lacks viral atypia
    • Typically has more exophytic free floating papillary fronds
  • High-grade noninvasive urothelial carcinoma with FGFR3 mutation:
    • Has a urothelial lining with polygonal cells, distinct cell borders, clear cell to eosinophilic cytoplasm and prominent koilocytic nuclei
Board review style question #1
    A 30 year old woman presents with micturition pain and urinary obstruction symptoms. Transurethral bladder resection revealed a polypoid mass, composed of hyperplastic papillary fronds of squamous epithelium with koilocytosis and viral atypia. What would be the most appropriate ancillary test?

  1. Immunohistochemistry for p40 and p63
  2. Immunohistochemistry for CK20 and p53
  3. RNA ISH for low risk and high risk HPV
  4. Immunohistochemistry for p16
Board review style answer #1
C. RNA ISH for low risk and high risk HPV. The lesion may be a urinary bladder condyloma acuminatum.

Comment Here

Reference: Condyloma acuminatum
Board review style question #2


    Transurethral resection of a bladder mass in a 45 year old man with a history of renal transplantation revealed a condyloma acuminatum of the prostatic urethra. Which lesion can be related to the condyloma acuminatum?

  1. No clinically significant relation known
  2. Squamous cell carcinoma (either invasive or in situ)
  3. Invasive urothelial carcinoma
  4. Squamous metaplasia
Board review style answer #2
B. Squamous cell carcinoma (either invasive or in situ). Patients are at risk for concomitant or subsequent squamous cell carcinoma (either invasive or in situ).

Comment Here

Reference: Condyloma acuminatum

Cystitis cystica and cystitis glandularis
Definition / general
  • Proliferative or reactive changes occurring in von Brunn nests which acquire luminal spaces, become cystically dilated (cystitis cystica), undergo glandular metaplasia (cystitis glandularis) or intestinal type metaplasia (cystitis glandularis, intestinal type)
Essential features
  • Extremely common incidental finding
  • Develops in the setting of chronic mucosal irritation or inflammation
  • Cystitis cystica and cystitis glandularis are frequently coexisting interrelated lesions
  • Benign mimickers of invasive carcinoma
Terminology
  • Cystitis cystica et glandularis (when combined features)
  • Cystitis glandularis, conventional type
  • Cystitis glandularis, intestinal type (preferred term: intestinal metaplasia)
  • Proliferative cystitis (discouraged terminology)
ICD coding
  • ICD-10: N30.80 - Other cystitis without hematuria
  • ICD-10: N30.81 - Other cystitis with hematuria
Epidemiology
Sites
  • Bladder; neck and trigone are most common sites
  • Also ureters (ureteritis) and renal pelvis (pyelitis)
Pathophysiology
  • Reactive process in response to chronic irritation, infection, calculi, outlet obstruction, catheterization (Pathol Annu 1995;30:95)
  • Urothelium proliferates and invaginates into underlying lamina propria
Etiology
  • Chronic irritation / local inflammatory insult
Clinical features
  • Majority are asymptomatic incidental findings
  • May infrequently cause recurrent urinary tract infections
  • Occasionally appears as polypoid or papillary mass on cystoscopy
  • Cystitis glandularis can be associated with pelvic lipomatosis and bladder exstrophy
  • Reference: Pathol Annu 1995;30:95
Diagnosis
  • Diagnosis is by microscopic examination of resected tissue
Radiology description
Radiology images

Images hosted on other servers:

T1 weighted MRI

Transabdominal ultrasound

Prognostic factors
  • Reactive process without malignant potential
  • May regress if the cause of irritation is removed
Case reports
Treatment
  • Elimination of underlying source of irritation
  • Antibiotic therapy if associated with chronic urinary tract infections
  • Occasionally surgical resection (transurethral) may be necessary
  • References: BJU Int 2013;112:27, Indian J Urol 2016;32:329
Clinical images

Images hosted on other servers:

Cystoscopy

Gross description
  • Mucosa may appear grossly unremarkable
  • Cystitis cystica appears as translucent submucosal cysts, mostly < 5 mm diameter
  • Cystitis glandularis appears as irregular or nodular lesions with a cobblestone pattern or as a polypoid mass
  • References: Pathol Annu 1995;30:95, Histopathology 2011;58:811
Microscopic (histologic) description
  • Abundant urothelial von Brunn nests
    • Often exhibit a vaguely lobular distribution of invaginations
    • Noninfiltrative growth and variable connection to surface
  • Gland-like lumens with columnar or cuboidal cells (cystitis glandularis)
  • Cystically dilated lumens or cystic cavities filled with eosinophilic fluid (cystitis cystica)
  • Majority of cases show coexistence of both patterns
  • Cells lack significant atypia, mitotic activity, stromal reaction and muscular invasion (Ann Diagn Pathol 2019;38:11)
    • Degenerative atypia occasionally present
Microscopic (histologic) images

Contributed by Maria Tretiakova, M.D., Ph.D.

Cystitis cystica

Cystitis glandularis

Immunofluorescence description
  • Cystitis glandularis demonstrates uniform membranous expression of beta catenin without cytoplasmic or nuclear localization (J Urol 2003;170:1892)
Positive stains
Sample pathology report
  • Bladder, trigone, biopsy:
    • Urothelium with florid cystitis cystica et glandularis
    • No neoplasm identified
    • Muscularis propria absent
Differential diagnosis
Additional references
Board review style question #1

A 75 year old man presents with a polypoid bladder lesion on cystoscopy. Which feature could be present in nested variant of invasive urothelial carcinoma but should not be found in this entity?

  1. Background inflammation
  2. Expression of CK7 and CK20
  3. Invasion into muscularis propria
  4. Irregular nests variably distributed within lamina propria
  5. Mild cytologic atypia
Board review style answer #1
C. Invasion into muscularis propria. This is cystitis cystica.

Comment Here

Reference: Cystitis cystica and cystitis glandularis
Board review style question #2
Which is true about cystitis cystica?

  1. Considered a premalignant lesion
  2. Immunohistochemically negative for urothelial markers
  3. Part of morphologic spectrum with cystitis glandularis
  4. Rare finding
  5. Variably sized cysts are filled with extravasated mucin
Board review style answer #2
C. Part of morphologic spectrum with cystitis glandularis

Comment Here

Reference: Cystitis cystica and cystitis glandularis

Cytology-general, normal findings & biomarker testing
Definition / general
  • Cytology is useful to detect carcinoma in situ or marked chronic inflammation (i.e. when there is no specific lesion to biopsy), carcinoma hidden in diverticula, or for detecting residual tumor from urine specimens
  • Cystoscopic biopsy of visible lesions is more sensitive than cytology in most cases
  • Bladder irrigation is superior to collecting voided urine
  • Most sensitive and highly specific for high grade tumors (diagnosis or follow-up) whether flat (carcinoma in situ), papillary or mixed
  • Low sensitivity (difficult to diagnose) for papilloma and low malignant potential lesions because they have normal histology (Mod Pathol 1995;8:394)
  • Follow up examination of urine with FISH may improve sensitivity and specificity of cytology (Am J Clin Pathol 2001;116:79)
Types of specimens
Voided urine:
  • Non invasive, easiest to obtain
  • Obtaining three second “morning voided” midstream urine samples collected over three consecutive days appears to optimize the detection of urothelial malignancies

Instrumented urine:
  • Catheterization of the bladder or irrigation of bladder

Ileal conduit urine:
  • Ileal conduit and neobladder are the most common urine diversion techniques used in patients who have undergone cystectomy
  • A portion of the ileum is anastomosed with the ureters to the skin or to the urethra
Processing / preservation of specimen
  • Immediate processing is recommended or refrigerate if immediate processing cannot be done
  • If fixation if needed, use equal volumes of 50% ethanol or a methanol based fixative (Cytolyt® or similar)
Cytology description
Voided urine
  • Paucicellular
  • Urothelial cells, squamous cells, inflammatory cells and possibly red blood cells
  • The umbrella cells have one nucleus, abundant delicate cytoplasm, low N/C ratio, smooth nuclear contours with fine chromatin pattern and a small chromocenter
  • The basal and intermediate layers are smaller and have a columnar or cuboidal appearance; the nuclei are round with smooth nuclear contours with a fine chromatin pattern
  • Nucleoli are either absent or single and minute
  • Reactive conditions (e.g. urolithiasis, post instrumentation) demonstrate clusters of basal and intermediate urothelial cells


Instrumentation related urine
  • More cellular
  • Presence of clusters of urothelial cells forming cell balls and pseudopapillary clusters
  • The urothelial cells have a low N/C ratio, round nuclei, fine chromatin pattern and smooth nuclear contours
  • Instrumentation effect: crowded cluster formations, hyperchromasia and distinct nucleoli; may lead to a false positive diagnosis


Ileal conduit urine
  • Cellular
  • Many degenerated single cells (oval with pyknotic nuclei) and few glandular cells (vacuolated cytoplasm, hyperchromatic nuclei but preserved N/C ratio of normal cells)
  • Dirty background
  • Non-specific eosinophilic cytoplasmic inclusions
  • A potential pitfall due to hyperchromasia
Cytology images

Images hosted on other servers:
Missing Image

Urine cytology

Biomarker testing
Bladder Tumor Antigen (BTA) test
  • BTA Stat: done at physician's office or home
  • BTA Trak: done by reference laboratory; a quantitative sandwich assay
  • Based on the detection of the human complement factor H-related protein
  • Only indicated in patients with prior history of urothelial carcinoma
  • Sensitivity up to 50% for low grade urothelial carcinomas, which is higher than cytology (Can Urol Assoc J 2008;2:212)
  • Sensitivity of 69% for BTA Trak test
  • The main problem with BTA is relatively high false positive rate


Nuclear Matrix Protein 22
  • Part of the structural framework of the nucleus
  • Involved in DNA replication, ribonucleic acid transcription and regulation of gene expression
  • Nuclear mitotic apparatus proteins (NMP22) have a concentration in urothelial carcinomas of 25x normal urothelial cells
  • Either an office based test or a reference laboratory test (BladderChek®, Matritech)
  • Used to detect new cancers and to follow-up patients with a prior history of urothelial carcinoma
  • Reported sensitivity of 34.6%–100% for NMP22 Bladder Cancer Test and 49.5%–65.0% for BladderChek assays


Bladder Cancer Immunofluorescence Assay (former Immunocyt®)
  • An immunofluorescence assay
  • A cocktail of 3 monoclonal antibodies; M344, LDQ10 and 19A211
  • M344 and LDQ10 detect a mucin-like antigen
  • 19A211 recognizes a high molecular weight glycosylated form of carcinoembryonic antigen in exfoliated tumor cells
  • The test requires the correlation of the immumunofluorescence results with cytology
  • The overall sensitivity of the combined Bladder Cancer Immunofluorescence Assay and cytology assay is approximately 84%


UroVysion®
  • Fluorescent in situ hybridization (FISH) probe set
  • A mixture of 4 fluorescent labeled DNA probes; a locus specific probe to the 9p21 band on chromosome 9 and to the centromere of chromosomes 3, 7 and 17
  • The individual sensitivity of the centromeric probes is 73.7% for chromosome 3, 76.2% for chromosome 7, 61.9% for chromosome 17
  • The sensitivity of homozygous 9p21 deletion for urothelial carcinoma is 28.6%
  • The UroVysion® test: 72% sensitivity and 83% specificity
  • According to stage of the disease, for pTa tumors sensitivity ranges from 65 to 73% and for pT1-T4 urothelial carcinomas 95-100%
  • UroVysion® and cytology have similar specificity but superior specificity to BTA
  • Patients with a positive UroVysion® test result, equivocal (atypical or suspicious) cytology diagnosis and negative cystoscopy have a higher incidence of urothelial carcinoma on subsequent follow-up
  • The UroVysion® test seems to have limited value due to a higher incidence of false positives and false negatives
  • The positive predictive value of UroVysion® for bladder carcinoma is 65% in patients with a 40+ pack year history of smoking, but only 20% in nonsmokers

Cytology-neoplastic
Definition / general
  • Urine cytology is more sensitive in detecting high grade urothelial carcinoma than low grade (Diagn Cytopathol 2013;41:852, Urol Oncol 2014;32:27.e1)
    • Sensitivity: 10 - 43.6% for low grade, 50 - 85% for high grade
    • Specificity: 26.3 - 88% depending on type of urine sample collection
  • Rate of malignancy in urine cytology: 1.7 - 5.8% (Am J Clin Pathol 2007;127:946, Cytopathology 2011;22:329, Cancer Cytopathol 2013;121:15)
  • Bladder washings and upper urinary tract specimens have higher percentage of malignant cases when compared with voided urine specimens
  • Positive predictive value and specificity for urine cytology designated as positive for malignancy is very high
    • Specificity: 78 - 100%, majority > 90%
Essential features
  • The Paris System for Reporting Urinary Cytology is the recommended system to report results (Rosenthal: The Paris System for Reporting Urinary Cytology, 1st Edition, 2016)
  • Urine cytology cannot distinguish invasive high grade urothelial carcinoma from noninvasive high grade urothelial carcinoma or carcinoma in situ
  • Low grade urothelial neoplasia is a combined cytologic term to include: low grade papillary urothelial neoplasms (i.e. urothelial papilloma, papillary urothelial neoplasm of low malignant potential and low grade papillary urothelial carcinoma) and flat, low grade intraurothelial neoplasia
  • Cell block can also be prepared from the residual specimen for ancillary studies
CPT coding
  • 88108: cytopathology, concentrated preparation (e.g. cytospin or Saccomanno)
  • 88112: cytopathology, enriched / concentrated preparation, nongynecologic (e.g. liquid based slide preparation: ThinPrep, SurePath)
  • 88305: cell block
  • 88342: immunohistochemical stain (qualitative), first stain
  • 88341: immunohistochemical stain (qualitative), second and subsequent stains
Sites
  • Urinary bladder, upper urinary tracts (renal pelvis, ureters), urethra
  • Urinary bladder diversion (ileal conduit, Indiana pouch, neobladder)
Case reports
Cytology description
High grade urothelial carcinoma
  • Combined cytologic term to include invasive high grade papillary urothelial carcinoma, noninvasive high grade papillary urothelial carcinoma and carcinoma in situ
  • Diagnostic criteria based on The Paris System consensus:
    • At least 5 - 10 abnormal cells
    • High N/C ratio (0.7 or greater; nucleus occupies more than 70% of the cytoplasm)
    • Moderate to severe hyperchromatic nuclei (however, nuclear hypochromasia has been reported) (APMIS 2018;126:705)
    • Marked irregular nuclear membrane
    • Coarse / clumped chromatin
  • Other cytomorphologic features:
    • Individual and cohesive clusters of tumor cells
    • Pleomorphic nuclei
    • Marked variation in tumor cell size and shapes (i.e. oval, round, elongated, plasmacytoid - comet cells)
    • Scant, pale or dense cytoplasm
    • Jet black and smooth or glassy chromatin (Cancer Cytopathol 2018;126:64)
  • Background of necrosis and inflammation
  • Prominent nucleoli can be seen in high grade urothelial carcinoma and reactive urothelial cells

Low grade urothelial neoplasia
  • Combined cytologic term to include low grade papillary urothelial neoplasms (i.e. urothelial papilloma, papillary urothelial neoplasm of low malignant potential and low grade papillary urothelial carcinoma) and flat, low grade intraurothelial neoplasia
  • Definitive cytologic diagnosis can only be made in the presence of this feature (Acta Cytol 1996;40:676):
    • 3 dimensional papillary clusters with nuclear overlapping of tumor cells around fibrovascular cores
  • Cytologic diagnosis may be considered in the presence of the following features (Diagn Cytopathol 2014;42:555):
    • 3 dimensional clusters without fibrovascular cores
    • Increased number of monotonous singe cells (e.g. cercaria shaped cells with elongated tails and eccentrically placed nuclei) that are not umbrella cells
    • Note: these cases should still be categorized as negative for high grade urothelial carcinoma with optional comment suggestive of low grade urothelial neoplasia
  • Cytologic diagnosis may be considered in the presence of the following features, which can also be associated with high grade urothelial carcinoma, in the absence of other high grade urothelial carcinoma characteristics (Cancer 1994;74:1621, Mod Pathol 1996;9:225):
    • Cytoplasmic homogeneity
    • Slightly irregular nuclear contours
    • Increased N/C ratio
    • Note: these cases should still be categorized as negative for high grade urothelial carcinoma with optional comment suggestive of low grade urothelial neoplasia

Other malignancies: primary, metastatic and miscellaneous lesions

Primary nonurothelial tumors
  • Epithelial malignancies
    • Squamous cell carcinoma
      • Essentially similar to squamous cancers arising in other sites
      • Cellular specimen in clusters and singly
      • Tumor cells with:
        • Cytoplasm: abundant, polygonal, dense, keratinized (characteristically orangeophilic), intercellular bridges, low N/C ratio
        • Nuclei: hyperchromatic, nuclear enlargement, irregular nuclear contours, frequently pyknotic
      • Tadpole cells, fiber cells, squamous pearls and cell-in-cell arrangements may be seen
      • Background: hyperkeratosis, parakeratosis, squamous metaplasia, anucleated squamous cells (ghost cells), keratin debris, necrosis, inflammatory cells
      • Nonkeratinizing squamous cell carcinoma: dense cytoplasm, relatively uniform tumor cells, high N/C ratio, prominent nucleoli
      • Well differentiated squamous cell carcinoma: may present as minimally atypical cells (corresponding to ASCUS or LSIL in Pap) (Diagn Cytopathol 2005;33:394, Diagn Cytopathol 2012;40:798)
      • Note: definitive diagnosis of squamous cell carcinoma should be deferred to resection specimens as urothelial carcinoma with squamous differentiation cannot be excluded on urine cytology
    • Adenocarcinoma
      • Variable cellularity
      • Adenocarcinoma, not otherwise specified
        • Clusters of cells
        • Tumor cells with:
          • Cytoplasm: finely vacuolated
          • Nuclei: eccentrically placed, irregular nuclear contours, prominent nucleoli
      • Enteric (colonic type) adenocarcinoma (Cancer 1998;84:335)
        • Clusters of cuboidal to columnar cells and single degenerated cells
        • Tumor cells with:
          • Cytoplasm: scant, may be vacuolated, contains mucin
          • Nuclei: elongated, vesicular or hyperchromatic, irregular nuclear contours, noticeable or prominent nucleoli
        • Background: necrosis and mucin
      • Mucinous (colloid) adenocarcinoma
        • 3 dimensional clusters
        • Tumor cells with:
          • Cytoplasm: scant to moderate amount, lacy, occasional vacuoles
          • Nuclei: bland appearing, crowded, conspicuous nucleoli
        • Background: mucin
      • Signet ring cell carcinoma (Pathol Res Pract 1987;182:130, Acta Cytol 2009;53:309, Acta Cytol 2012;56:177)
        • Tumor cells with:
          • Cytoplasm: clear or finely vacuolated, large cytoplasmic mucin vacuole
          • Nuclei: crescent shaped (due to nucleus being pushed to the periphery by cytoplasmic vacuole), hyperchromatic
      • Clear cell adenocarcinoma (Diagn Cytopathol 1996;14:150, Korean J Pathol 2012;46:210, Am J Pathol 2014;184:584)
        • Tumor cells with:
          • Cytoplasm: abundant, vacuolated, clear or granular
          • Nuclei: centrally located, vesicular chromatin, irregular nuclear contours, prominent nucleoli
        • Singly or in clusters with hobnail appearance
    • Neuroendocrine tumors
      • Small cell carcinoma (Acta Cytol 2000;44:403, Diagn Cytopathol 2000;23:92, Diagn Cytopathol 1996;14:292, Cancer 1997;79:356, Cell J 2014;16:95)
        • Similar to small cell carcinoma arising in other sites
        • Hypercellularity
        • Tumor cells in various arrangements: singly, linear pattern, rosettes, loose or tight clusters
        • Tumor cells with:
          • Cytoplasm: scant, high N/C ratio
          • Nuclei: slightly enlarged (small to medium sized), round to ovoid, hyperchromatic, smudged chromatin, molding (may be minimal in liquid based preparations), crush artifact (may be less in liquid based preparations), absent or inconspicuous nucleoli (Diagn Cytopathol 2001;24:46)
        • Frequent mitosis and apoptosis
        • Background: hemorrhage, necrotic debris (may be clumped or clings to tumor cells in liquid based preparations), inflammation
        • Note: small cell carcinoma is commonly seen in combination with urothelial carcinoma; both cell populations may be seen
    • Large cell neuroendocrine carcinoma (Acta Cytol 2010;54:303)
      • Similar to large cell neuroendocrine carcinoma arising in other sites
      • Arranged in rosette pattern
      • Tumor cells with nuclear molding, finely or coarsely granular, hyperchromatic nuclei

  • Nonepithelial malignancies
    • Sarcoma
      • Leiomyosarcoma (Diagn Cytopathol 2004;31:281, J Cytol 2017;34:171)
        • Individual or sheets of atypical spindle cells
        • Tumor cells with:
          • Cytoplasm: moderate amount, ill defined cytoplasmic borders, low N/C ratio
          • Nuclei: large, spindled, hyperchromatic, irregular nuclear contours, occasional nucleoli
      • Angiosarcoma (Cytopathology 1999;10:137)
        • Individual cells and in small clusters
        • Rather monomorphic spindled or epithelioid tumor cells with:
          • Cytoplasm: moderately abundant, granular to dense, intracellular lumina containing erythrocytes may be seen
          • Nuclei: large, round to oval, eccentrically placed, irregular nuclear membrane, hyperchromatic, prominent cherry-red nucleoli
      • Note: confirmatory workup by histologic and immunohistochemical evaluation is required
    • Hematologic malignancy
    • Melanoma (Acta Cytol 2001;45:631, Diagn Cytopathol 2014;42:1091)
      • Individual to sheets and clusters
      • Tumor cells with:
        • Cytoplasm: abundant, dark brown melanin pigment may be present
        • Nuclei: large, round to ovoid, eccentrically located, irregular nuclear contours, prominent nucleoli, occasional nuclear pseudoinclusions

Direct extension and metastatic tumors to urinary bladder
  • Direct extension
    • Prostatic adenocarcinoma (Cancer 1998;84:335, Am J Clin Pathol 2000;113:29, Acta Cytol 2013;57:184)
      • Uniform, cuboidal cells arranged in clusters, some may show acinar formation
      • Tumor cells with:
        • Cytoplasm: abundant, dense, granular
        • Nuclei: round, smooth nuclear membrane, fine evenly distributed chromatin, may be eccentrically placed, prominent nucleoli OR hyperchromatic
      • Background usually clean
        • Note: prostatic adenocarcinoma detected in urine cytology has high Gleason scores and presents at advanced clinical stage
    • Colorectal adenocarcinoma (Cancer 1998;84:335)
      • Elongated or columnar cells in glandular arrangement
      • Tumor cells with:
        • Cytoplasm: frequent vacuolation
        • Nuclei: degenerated, hyperchromatic, angulated, irregular nuclear contours, coarse chromatin, inconspicuous nucleoli
      • Background tumor necrosis
      • Note: correlation with clinical history is necessary
    • Squamous cell carcinoma of uterine cervix
      • Similar to squamous cell carcinoma arising in other sites
      • Note: clinical history with ancillary studies are important to establish tumor origin
  • Metastatic tumors
    • Renal cell carcinoma (Acta Cytol 1983;27:383, Cancer 1998;84:335)
      • For clear cell renal cell carcinoma: tumor cells in clusters with large nuclei, evenly distributed chromatin, prominent nucleoli and granular, vacuolated cytoplasm
      • Multinucleated cells with distinct nucleoli or large vacuolated cells with eccentric nuclei may be seen
      • Granular eosinophilic cells with pyknotic nuclei may be degenerative changes caused by urine
      • Note: primary renal cell carcinoma involving the renal pelvis may be seen in upper urinary tract specimens
      • Note: metastasis to bladder is very rare; most frequently reported type for bladder metastasis is clear cell renal cell carcinoma
    • Breast carcinoma
      • For metastatic lobular carcinoma: small tumor cells in linear arrangement or in small clusters, eccentrically placed nucleus, small nucleolus and vacuolated cytoplasm with occasional targetoid mucin
      • For metastatic ductal carcinoma: tumor cells are arranged in cellular sphere or morula
      • Note: metastatic breast carcinoma is more frequently lobular than ductal
    • Gastric carcinoma
      • Tumor cell shows signet ring morphology

Nonepithelial benign tumors and tumor-like conditions
  • Nonepithelial benign tumors
    • Pheochromocytoma / paraganglioma (Diagn Cytopathol 2017;45:350)
      • Individual and nests of epithelioid cells
      • Tumor cells with:
        • Cytoplasm: moderate to abundant, finely granular
        • Nuclei: relatively regular and smooth nuclear contours, fine chromatin, inconspicuous nucleoli
      • Occasional spindle sustentacular cells present in association with the tumor cells
  • Tumor-like lesions
Cytology images

Contributed by Bonnie Choy, M.D.
High grade urothelial carcinoma High grade urothelial carcinoma

High grade urothelial carcinoma

High grade urothelial carcinoma High grade urothelial carcinoma High grade urothelial carcinoma

High grade urothelial carcinoma

Low grade urothelial neoplasia

Low grade urothelial neoplasia


Squamous cell carcinoma

Squamous cell carcinoma

Small cell carcinoma

Small cell carcinoma

Prostatic adenocarcinoma

Prostatic adenocarcinoma

NKX3.1

NKX3.1

Colorectal adenocarcinoma

Colorectal adenocarcinoma

Renal cell carcinoma

Renal cell carcinoma

Molecular / cytogenetics description
Sample pathology report
  • Bladder, voided urine:
    • Specimen adequacy:
      • Satisfactory for evaluation
    • Interpretation:
      • Positive for malignancy
    • Diagnosis:
      • High grade urothelial carcinoma
Differential diagnosis
Board review style question #1

A voided urine from a 55 year old man with hematuria shows cells with high nuclear to cytoplasmic ratios, nuclear hyperchromasia and pleomorphism. What is the correct diagnosis?

  1. Benign urothelial cells
  2. High grade urothelial carcinoma
  3. Low grade urothelial carcinoma
  4. Polyoma virus infected urothelial cells
Board review style answer #1
B. High grade urothelial carcinoma. Based on The Paris System for Reporting Urinary Cytology, the diagnostic criteria for high grade urothelial carcinoma include high N/C ratio, moderate to severe hyperchromatic nuclei, markedly irregular nuclear membrane and coarse chromatin. The voided urine contains cells meeting the diagnostic criteria for high grade urothelial carcinoma. While urothelial cell with polyoma (BK) viral cytopathic effect show high N/C ratio, other features of high grade urothelial carcinoma are not seen.

Comment Here

Reference: Cytology-neoplastic
Board review style question #2

A bladder washing from a 70 year old man with hematuria. Tumor cells are present and immunohistochemistry was performed on the cell block. The tumor cells are positive for NKX3.1 and negative for GATA3, PAX8 and CDX2. What is the correct interpretation?

  1. Colonic adenocarcinoma
  2. Nephrogenic adenoma
  3. Prostatic adenocarcinoma
  4. Urothelial carcinoma
Board review style answer #2
C. Prostatic adenocarcinoma. Nonurothelial carcinomas can involve the urinary bladder either by direct extension from adjacent organs or metastasis from distant site. The tumor cell shows NKX3.1 positivity. GATA3, PAX8 and CDX2 are negative. This immunoprofile supports the diagnosis of prostatic adenocarcinoma.

Comment Here

Reference: Cytology-neoplastic

Cytology-nonneoplastic
Definition / general
  • Benign and reactive cytologic changes based on the Paris System for Reporting Urinary Cytology include nonneoplastic entities such as changes associated with lithiasis, bacterial, fungal and parasitic infections, viral cytopathic effect and posttreatment effect
  • Nonneoplastic entities may mimic malignant cells; clinical correlation, as well as the use of ancillary testing when needed, is advised
Essential features
  • The Paris System for Reporting Urinary Cytology is the recommended system to report results (Rosenthal: The Paris System for Reporting Urinary Cytology, 1st Edition, 2016)
  • Negative for high grade urothelial carcinoma category encompasses nonneoplastic entities that pose no significant risk for the development of high grade urothelial carcinoma, including
    • Changes associated with urinary lithiasis
    • Bacterial, fungal and parasitic infections
    • Viral cytopathic effect
    • Posttreatment effect
  • Atypical urothelial cells category should be used only if there are cellular alterations (mild to moderate cytologic atypia) that warrant concern but fall short of suspicious for high grade urothelial carcinoma or high grade urothelial carcinoma categories
CPT coding
  • 88108: cytopathology, concentrated preparation (e.g. cytospin or Saccomanno)
  • 88112: cytopathology, enriched / concentrated preparation, nongynecologic (e.g. liquid based slide preparation: ThinPrep, SurePath)
Sites
  • Urinary bladder, upper tracts (renal pelvis, ureters), urethra
  • Urinary bladder diversion (ileal conduit, Indiana pouch, neobladder)
Case reports
Cytology description
Urothelium with lithiasis (J Am Soc Cytopathol 2015;4:30)
  • 3 dimensional pseudopapillary clusters of urothelial cells
    • Most clusters with smooth borders
    • Rim of cytoplasm (collarette) around clusters
    • Urothelial cells with uniform round nuclei, finely granular chromatin and inconspicuous nucleoli; however, reactive atypia (pleomorphism, coarsely granular chromatin, hyperchromasia, occasional mitotic figures) can be seen
  • Calcific concretions
  • Background of blood or inflammatory cells

Infections
  • Bacterial infections
    • Acute bacterial infection
      • Numerous neutrophils
      • Urothelial cells with reactive changes: slight nuclear enlargement, conspicuous nucleoli but chromatin is fine and uniformly distributed and nuclei remain round
      • Bacteria seen in the background
      • Note: presence of bacteria without neutrophils is a nonspecific finding
    • Malakoplakia
      • Histolytic inflammatory condition, often resulting from bacterial infection
      • Histiocytes with intracytoplasmic:
        • Bacteria
        • Basophilic, concentric, laminated structures (Michealis-Gutmann bodies)
  • Fungal infections
    • Candida species
      • Most common cause of fungal urinary tract infections
      • Seen in yeast forms and pseudohyphae
      • Reactive urothelial cells and mixed inflammatory background
      • Note: presence of Candida may be due to vaginal contamination, often seen with numerous squamous cells but few neutrophils
    • Other less common fungal organisms reported in urine cytology
  • Viral cytopathic effects
  • Parasitic infections
    • Schistosoma haematobium
      • Oval shaped eggs with terminal spine
        • Hatched eggs with empty shells
        • Unhatched eggs contain miracidia
      • Eosinophils in the background
    • Trichomonas vaginalis (Acta Cytol 2000;44:484)
      • Pear shaped parasites with small and oval nucleus and red cytoplasmic granules
      • Note: presence of Trichomonas may be due to contamination from vaginal infection, often seen with numerous squamous cells and vaginal flora

Treatment effect (J Clin Pathol 2002;55:641)
  • Radiation
    • Urothelial cells show significant cytomegaly and nucleomegaly but maintain N/C ratio (not increased)
    • Multinucleation and nuclear and cytoplasmic vacuoles may be seen
    • Nucleus and cytoplasm often have degenerative changes
    • Finely granular chromatin and smooth nuclear membrane
    • Cytologic changes can be seen for weeks to years
  • Immunotherapy
    • Intravesical Bacillus Calmette-Guérin (BCG) immunotherapy can cause granulomatous inflammation
    • Granulomas composed of epithelioid histiocytes with carrot shaped nuclei and lymphocytes
    • Langhans type multinucleated giant cells from fused macrophages have small, hyperchromatic nuclei clustered at one cytoplasmic pole
  • Chemotherapy
    • Mitomycin and thiotepa
      • Intravesical treatment usually affects superficial cells
      • Nuclear enlargement, multinucleation and hyperchromasia
      • Background eosinophils for mitomycin
    • Cyclophosphamide
      • Systemic treatment may cause similar cytologic changes as intravesical mitomycin and thiotepa
      • Hyperchromasia, degeneration, large nuclei and increased N/C ratio
Cytology images

Contributed by Bonnie Choy, M.D.
Acute bacterial infection

Acute bacterial infection

Candida Candida

Candida

Polyomavirus Polyomavirus

Polyomavirus


HPV

Human papilloma virus

Schistosoma

Schistosoma

Trichomonas

Trichomonas

Molecular / cytogenetics description
Sample pathology report
  • Bladder, voided urine:
    • Specimen adequacy:
      • Satisfactory for evaluation
    • Interpretation:
      • Negative for high grade urothelial carcinoma
    • Diagnosis:
      • Urothelial cells with viral cytopathic effects consistent with polyomavirus
Board review style question #1

A bladder washing from a 65 year old man shows urothelial cells with large basophilic, glassy nuclear inclusions. Which of the following features differentiated polyomavirus infected urothelial cells from high grade urothelial carcinoma cells?

  1. Comet shaped cells
  2. Homogenous chromatin with ground glass appearance
  3. Increased N/C ratio
  4. Irregular nuclear membrane
  5. Prominent nuclear membrane
Board review style answer #1
D. Irregular nuclear membrane. Both polyomavirus infected cells and high grade urothelial carcinoma cells have increased N/C ratio. When high grade urothelial carcinoma cells are infected by polyomavirus, they also can show ground glass chromatin, prominent nuclear membrane and comet shaped cells. However, only high grade urothelial carcinoma cells have irregular nuclear membranes.

Comment Here

Reference: Cytology-nonneoplastic
Board review style question #2
A voided urine sample from a 50 year old woman shows urothelial cells admixed with histiocytes with round, laminated and basophilic cytoplasmic inclusions. What is the most likely etiology?

  1. Candida infection
  2. Cytomegalovirus infection
  3. Malakoplakia
  4. Melamed-Wolinska bodies
  5. Polyomavirus infection
Board review style answer #2
C. Malakoplakia. The described structures are Michealis-Gutmann bodies, which are seen in malakoplakia. Fungal yeast forms and pseudohyphae with a mixed inflammatory background are seen in Candida infection. Cytomegalovirus infected cells show nuclear inclusions with an owl's eye appearance as well as cytoplasmic inclusions. Melamed-Wolinska bodies are intracytoplasmic eosinophilic inclusions seen in degenerated urothelial cells. The classic appearance of cells infected by polyomavirus shows a single, large, basophilic, glassy nuclear inclusion.

Comment Here

Reference: Cytology-nonneoplastic

Diverticula
Definition / general
  • Outpouchings of urothelial mucosa through bladder muscular wall
Epidemiology
Sites
  • Most diverticula are solitary lesions located in lateral walls of bladder
Etiology
  • Congenital cases are due to bladder outlet obstruction or failure of muscle development (eMedicine: Surgery for Pediatric Bladder Anomalies [Accessed 11 July 2018])
  • Acquired cases are more common and usually due to prostatic enlargement causing muscular hypertrophy and focal mucosal herniation without muscularis propria in areas of weakness, often near ureteral orifices, bladder dome or urethral orifice
Clinical features
  • Most bladder diverticula are small and asymptomatic
  • When symptomatic, usually associated with infections and stones (due to urine stasis), urinary retention or perforation
  • Some cases are associated with additional GU anomalies (urethral strictures, neurogenic bladder, duplicated collecting system) or hereditable syndromes (Ehlers-Danlos, others)
  • Malignancy occurs in 1 - 10% of bladder diverticula
  • Tumors are often large because location is hidden
  • All variants of urothelial carcinoma have been reported in bladder diverticula, with a relatively higher frequency of unusual subtypes when compared to the general population
Case reports
Treatment
  • Varies by patient - nonoperative treatment, surgical bladder outlet reduction or removal of the diverticulum
Gross description
  • Narrow necks, round / ovoid pouch from 1 - 18 cm (average 5.0 cm)
Gross images

Images hosted on other servers:

Image with case history

Microscopic (histologic) description
Nonneoplastic findings
  • Wall consists of fibrous tissue with no / scant muscularis propria
  • Common findings include inflammation, granulation tissue formation, erosion, cystitis cystica et glandularis, nonkeratinizing squamous metaplasia, reactive urothelial atypia

Neoplastic findings
  • Most neoplastic lesions that arise in bladder diverticula are urothelial carcinomas, mainly noninvasive papillary (high / low grade) or invasive

Duplication
Definition / general
  • Bladder is separated into complete or incomplete compartments, either in the sagittal or coronal plane
  • Variants include double bladder, septal bladder and hourglass bladder
Epidemiology
  • Rare, < 100 cases reported
  • Equal male:female ratio
Etiology
  • No single explanation for all cases of bladder duplication
  • Complete duplication of bladder and hindgut probably occurs because of partial twinning of the tail portion of the embryo
  • Excessive constriction of ventral cloaca or a supernumerary cloacal septum may lead to bladder duplication in the coronal plane
  • Complete bladder duplication with diphalia may be explained by duplication of the cloaca
Clinical features
  • Varies from intravesical septum to complete duplication of lower urinary tract
  • Complete duplication means two bladders, each with a full thickness muscularis propria and its own ipsilateral ureter, half the trigone, and a urethra; may occur in coronal or sagittal planes
  • Sagittal duplication (the most common): each bladder receives the ureter of ipsilateral kidney, separated from the other by a peritoneal fold and loose areolar tissue, is more often associated with non-urological anomalies than coronal ones
  • Coronal duplication: two bladders lying in front of each other, separated by an oblique (posterosuperior to anteroinferior) fibromuscular septum
  • Incomplete duplication consists of two bladder halves separated by a thick muscular wall, both draining into a common urethra
  • Associated with other GI (36-42%) and GU (38-90%) anomalies, including caudal duplication syndrome (Am J Dis Child 1993;147:1048)
  • Anomalies (and treatment) may vary by patient (Urology 2000;55:578)
  • Incomplete emptying may cause urinary tract infections, although most patients are asymptomatic
Case reports
Treatment
  • In some cases, needed for infections or to achieve continence
Radiology images

Images hosted on other servers:

Cystourethrogram shows complete bladder duplication


Dysplasia
Definition / general
  • Preneoplastic urothelial lesion that consists of cytological and architectural atypia that does not meet the criteria of a urothelial carcinoma in situ
Essential features
  • Flat urothelial lesions with normal thickness but may be increased or decreased (Hum Pathol 2010;41:155)
  • No maturation in basal and intermediate cell layers; umbrella cells are usually present
  • Mild degree of loss of polarity (Hum Pathol 2010;41:155)
  • Slight nuclear enlargement with variation in nuclear size and shape, hyperchromasia but not to the severity of urothelial carcinoma in situ
  • Some authors report a lack of reproducible criteria to distinguish from atypia of unknown significance (Histopathology 2019;74:68)
Terminology
  • Urothelial dysplasia
  • Urothelial atypia, cannot exclude dysplasia
  • Dysplasia / atypia of unknown significance (AUS)
  • Low grade urothelial neoplasia (regarded as obsolete)
ICD coding
  • ICD-10: N32.9 - bladder disorder, unspecified
  • ICD-11: GC01.Z - disorder of bladder, unspecified
Epidemiology
Sites
Pathophysiology
  • Deletion of chromosome 9 occurs in early step of tumorigenesis
  • TP53 mutation is involved in the development of urothelial dysplasia
  • Dysplasia may progress to carcinoma in situ after loss of RB1 (Cancers (Basel) 2018;10:100)
Etiology
Diagrams / tables

Images hosted on other servers:

Pathogenesis of bladder cancer

Clinical features
Diagnosis
Prognostic factors
  • Significant risk for progressing to urothelial carcinoma in situ and invasive urothelial carcinoma (19% of cases with follow up intervals ranging from 6 months to 8 years) (Am J Surg Pathol 1999;23:443)
Treatment
Gross description
  • Usually unremarkable; mucosal inflammation or erythema may be present
Microscopic (histologic) description
  • Flat urothelial lesions with normal thickness but may be increased or decreased (Hum Pathol 2010;41:155)
  • No maturation in basal and intermediate cell layers; umbrella cells are usually present
  • Mild degree of loss of polarity (Hum Pathol 2010;41:155)
  • Slight nuclear enlargement with variation in nuclear size and shape, hyperchromasia but not to the severity of urothelial carcinoma in situ
  • Early low grade papillary urothelial carcinoma for flat dysplasia with foci of early papillary formation (Adv Anat Pathol 2021;28:179)
  • Some authors report a lack of reproducible criteria to distinguish from atypia of unknown significance (Histopathology 2019;74:68)
Microscopic (histologic) images

Contributed by Moe Thuzar, M.D. and Y. Albert Yeh, M.D., Ph.D.
Loss of polarity

Loss of polarity

Nuclear enlargement and hyperchromasia

Nuclear enlargement and hyperchromasia

Decreased urothelial thickness

Decreased urothelial thickness

Hyperplastic atypical urothelium

Hyperplastic atypical urothelium

Urothelial atypia Urothelial atypia

Urothelial atypia

Cytology description
Positive stains
Negative stains
Molecular / cytogenetics description
  • UroVysion fluorescence in situ hybridization (J Clin Pathol 2008;61:272)
  • Polysomy (at least 1 of the chromosomes 3, 7, 17) in half of dysplasia cases and in > 90% of urothelial carcinomas in situ and invasive bladder carcinomas versus 17% of urothelial hyperplasia, reactive atypia and normal urothelium
Sample pathology report
  • Urinary bladder, posterior wall lesion, biopsy:
    • Urothelial mucosa with urothelial dysplasia (see comment)
    • Comment: The bladder lesion biopsy shows urothelial mucosa with slightly increased cell layers. There is loss of polarity. Nuclear crowding with nucleomegaly and hyperchromasis is noted. However, the degree of nuclear atypia is less severe than that of carcinoma in situ. These changes are consistent with urothelial dysplasia.
Differential diagnosis
Board review style question #1

A 65 year old man presented with hematuria and urinary urgency. Cystoscopic examination showed a 1 cm erythematous lesion in the posterior wall. Biopsy of the lesion was performed. The photomicrograph is shown above. What is the diagnosis?

  1. High grade urothelial carcinoma
  2. Low grade urothelial carcinoma
  3. Normal urothelium
  4. Urothelial carcinoma in situ
  5. Urothelial dysplasia
Board review style answer #1
E. Urothelial dysplasia. Urothelial dysplasia is characterized by variable thickness of urothelium with slightly enlarged nuclei and variable hyperchromasia but these do not reach the threshold of urothelial carcinoma in situ. Answer C is incorrect because there is hyperplasia and atypia. Answers A, B and D are incorrect because the enlarged nuclei and variable hyperchromasia fall short of urothelial carcinoma.

Comment Here

Reference: Dysplasia
Board review style question #2

A 72 year old man presented with hematuria and dysuria. Cystoscopic examination showed a 1.5 cm erythematous lesion in the right lateral wall. Biopsy of the lesion was performed. The photomicrograph is shown above. What is the diagnosis?

  1. High grade urothelial carcinoma
  2. Low grade urothelial carcinoma
  3. Normal urothelium
  4. Urothelial carcinoma in situ
  5. Urothelial dysplasia
Board review style answer #2
E. Urothelial dysplasia. The urothelium shows hyperplasia with some enlarged urothelial cells and hyperchromasia but the degree is less severe than that of a carcinoma in situ. Answers A, B and D are incorrect because their degree of dysplasia is greater than that of urothelial dysplasia. Answer C is incorrect because the lesion shows urothelial hyperplasia and cellular atypia.

Comment Here

Reference: Dysplasia

Ectopic prostate
Definition / general
  • Prostatic-type glands within bladder mucosa
Epidemiology
  • Rare; < 50 cases reported
  • Usually in young males
Sites
  • Mainly at bladder neck but also trigone and interureteral ridge
  • Very rarely at retrotrigone and ureteral orifices
Etiology
  • Due to vestigial remains of embryonic prostate remnants, metaplasia in response to chronic inflammation or abnormal migration of undifferentiated embryonic prostatic tissue with hormonal stimulation during adult life (Int J Urol 2005;12:1066)
Clinical features
  • May cause hematuria (most common symptom), dysuria, bladder outlet obstruction or persistent urinary tract infection
  • May simulate a neoplastic process
  • Benign behavior; no reported cases of malignant transformation
Case reports
Treatment
  • Excision; may persist / recur if incomplete excision
Gross description
  • Firm, smooth, dome-shaped, broad-based polypoid mass
  • Flat lesions also reported but very rare
Microscopic (histologic) description
  • Prostatic-like glands with luminal secretions beneath partially/totally denudated urothelium
  • May be associated with cystitis cystica and cystitis glandularis
  • Prostate-specific immunohistochemical markers are confirmatory but not mandatory
Microscopic (histologic) images

Images hosted on other servers:

Prostatic glands with luminal secretion underlying urothelium (right: PSA+)

Positive stains
Differential diagnosis

Emphysematous cystitis
Definition / general
  • Rare entity, associated with diabetes, with gas filled cysts in bladder wall
Epidemiology
Etiology
  • Due to gas forming bacteria (Clostridium perfringes, E. coli, Enterobacter aerogenes), which produce gas filled cysts in bladder wall
  • In one study, E. coli was most commonly isolated organism (J Microbiol Immunol Infect 2009;42:393)
  • CO2 may be produced by microorganisms through fermentation of glucose
  • There may also be impaired transportation of gas, due to local inflammation or other obstructive process (Braz J Infect Dis 2008;12:552)
Clinical features
  • Associated with diabetes (50% of cases), chronic cystitis, neurogenic bladder, leukemia / lymphoma
  • Commonly presents with pneumaturia
  • Diagnosed by radiologic evidence of gas in bladder mucosa
  • Rare complication is subcutaneous emphysema (Int J Emerg Med 2011;4:26)
Radiology images

Images hosted on other servers:

CT scan shows gas in bladder and bladder wall

CT scan of 77 year old woman without diabetes

Case reports
Treatment
  • Antibiotics, relieve bladder obstruction
Microscopic (histologic) description
  • Cysts often lined by multinucleated giant cells

Encrusted cystitis
Definition / general
Essential features
Terminology
ICD coding
  • ICD-10: N30.21 - other chronic cystitis with hematuria
  • ICD-11: GC00.1 - infectious cystitis
  • ICD-11: GC00.Y - other specified cystitis
Epidemiology
  • Incidence: 0.016 - 0.32% of clinical samples for bacterial cultures (Infect Drug Resist 2015:21;8:129)
  • 0.6% occurred in patients who received renal transplant (Front Med (Lausanne) 2021;7:609024)
  • Age and gender:
  • Natural colonizer of human skin and urinary tract
  • Transmitted by air or nosocomially (Infect Drug Resist 2015:21;8:129)
  • Risk factors (Front Med (Lausanne) 2021;7:609024):
    • Urological instrumentation (bladder catheterization)
    • Surgical or endoscopic procedure (cystoscopy, cystography)
    • Urological disease (urinary tract infections, intravesical mitomycin or bacillus Calmette-Guérin [BCG] instillations, radiation cystitis, drug induced cystitis, urolithiasis, malakoplakia, neurogenic bladder)
    • Chronic debilitating disease (diabetes, chronic renal insufficiency, liver cirrhosis, neurological disease, prolonged hospitalization)
    • Broad spectrum antibiotic therapy
    • Immunocompromised status (kidney transplantation, cancer, cytotoxic drug treatment, immunosuppressive therapy)
Sites
Pathophysiology
  • Urinary colonization and adherence to host tissue:
    • Colonization: asymptomatic, positive urine cultures
    • Adherence mediated by adhesive pili that covalently anchored to Corynebacterium cell wall (Front Med (Lausanne) 2021;7:609024)
  • Formation of struvite and carbonated apatite crystals
    • Corynebacterium urealyticum has strong urease activity that hydrolyses urea (pH ≥ 7.1) leading to formation of NH3 and CO2 (see diagram 1)
    • NH3 combines with H+ to form NH4+
    • In alkaline urine, NH4+ reacts to Mg2+ and PO43-, leading to formation of struvite (magnesium ammonium phosphate, NH4MgPO4.6H2O)
    • In alkaline urine, CO2 converts to bicarbonate HCO3- which reacts to Ca2+ and PO43-, leading to formation of calcium phosphate crystals (carbapatite, Ca10(PO4)6CO3) (Front Med (Lausanne) 2021;7:609024)
    • Infected urine saturates with struvite and calcium phosphate and precipitates, resulting in bladder wall incrustation in a suitable urothelial environment injured by inflammation, malignancy, ischemia or urological instrumentation
  • Adherence to medical device:
Etiology
  • Corynebacterium urealyticum (formerly coryneform CDC group D2)
    • Gram positive, aerobic, urease positive, pleomorphic, lipophilic, acid fast, non-spore forming, asaccharolytic and usually multidrug resistant bacillus (Clin Microbiol Infect 2008;14:632)
    • Common skin colonizer and opportunistic pathogen mainly detected in the groin area in hospitalized elderly patients receiving broad spectrum antibiotics
    • Pathological even if colony forming units (CFU) < 100,000/mL on urine cultures (Front Med (Lausanne) 2021;7:609024)
    • Highly susceptible to norfloxacin and vancomycin (J Clin Microbiol 1985;21:788)
  • Other bacteria: Ureaplasma urealyticum, Proteus vulgaris, Streptococcus haemolyticus, Streptococcus viridans, Staphylococcus species, Pseudomonas aeruginosa, Escherichia coli, Corynebacterium gluconolyticum, Arcanobacterium pyogene (Front Med (Lausanne) 2021;7:609024)
Diagrams / tables

Images hosted on other servers:

Struvite and carbonated apatite

Flowchart of diagnosis

Clinical features
  • Fever (25 - 50%), urinary frequency, urgency, stranguria, nycturia, suprapubic / pelvic pain, vesical tenesmus, costovertebral angle tenderness, dysuria (Br J Urol 1993;72:571)
  • Less often nausea, anorexia and weight loss
  • Commonly macroscopic hematuria detected in 75 - 100%
  • Urination of mucus, purulent debris, calcified necrotic debris, gravel or renal stones
Diagnosis
  • Clinical manifestations: urinary frequency, urgency, dysuria
  • Past clinical history and predisposing conditions
  • Urinalysis: alkaline urine (pH > 7.0), leukocytes
  • Urine microbiological cultures: positive for urease producing bacteria, commonly Corynebacterium urealyticum (J Med Microbiol 1998;47:79)
  • Radiological findings: thickening bladder wall with calcifications (Xray, ultrasound, CT scan)
  • Other conditions in the differential diagnosis of urinary tract encrustations or calcifications need to be excluded by means of cystoscopy and histopathological examination (Front Med (Lausanne) 2021;7:609024)
  • Cystoscopic examination: ulcerations, erythema, increased vascularity, yellow or white plaques firmly attached to the bladder mucosa (World J Clin Cases 2020;8:4234)
  • See diagram 2
Laboratory
  • Urinalysis:
    • Urinary ammonia-like odor
    • Alkaline urine (pH > 7.0)
    • Microscopic hematuria
    • Pyuria
    • Struvite crystalluria
    • Gritty calcified debris or stones
    • Mucopurulent debris (Front Med (Lausanne) 2021;7:609024)
  • Microbiological culture:
    • Conventional urine cultures often negative for Corynebacterium urealyticum
    • No specific transport media
    • Prolonged incubation in 5% CO2, 35 - 37 °C, for 48 - 72 hours
    • Nonselective media including blood agar or cysteine lactose electrolyte deficient agar
    • Selective media including lipid enriched peptone based CBU agar or Loeffler serum medium (3 parts of sheep or horse serum plus 1 part of glucose broth) (J Med Microbiol 1998;47:79, Infect Drug Resist 2015:21;8:129)
Radiology description
Radiology images

Images hosted on other servers:

Linear calcifications

Irregular calcifications

Case reports
Treatment
  • Endoscopic mucosal resection of encrusted calcifications
  • Urinary acidification: acetohydroxamic acid
  • Systemic glycopeptides antibiotic (especially teicoplanin and vancomycin) therapy for a few weeks to a few months (Front Med (Lausanne) 2021;7:609024)
Clinical images

Images hosted on other servers:

Multiple thin calcifications

Microscopic (histologic) description
  • May form 3 distinct layers
  • First superficial layer:
    • Ulceration and necrotic urothelial tissue
    • Calcified necrotic debris (demonstrated by von Kossa staining)
    • Degenerative changes with squamous metaplasia in nonaffected urothelium
  • Second layer:
    • Lamina propria with edematous and granulation tissue
    • Inflammatory infiltrate with lymphocytes, plasma cells, neutrophils, sometimes granulomas
    • Less often eosinophils, mast cells, fibroblasts and histiocytes
    • Small vascular thrombosis (may present)
    • Crystal deposition
    • Microabcesses with bacterial microcolonies (may present)
  • Third layer:
Microscopic (histologic) images

Contributed by Bohdan Zoshchuk, M.D. and Y. Albert Yeh, M.D., Ph.D.
Dystrophic calcifications

Dystrophic calcifications

Encrusted calcifications

Encrusted calcifications

Encrusted necrotic debris

Encrusted necrotic debris

Encrusted calcified fragments

Encrusted calcified fragments

Calcified plaques

Calcified plaques

Calcifications and chronic inflammation

Calcifications and chronic inflammation


Encrustations and fibrin

Encrustations and fibrin

Calcifying encrustations

Calcifying encrustations

Calcifications and giant cells

Calcifications and giant cells

Calcifications in fibrotic stroma

Calcifications in fibrotic stroma

Calcium salts

Calcium salts

Positive stains
Negative stains
Electron microscopy description
Electron microscopy images

Images hosted on other servers:

Calcifying nanoparticles

Molecular / cytogenetics description
Sample pathology report
  • Urinary bladder, biopsy:
    • Bladder tissue with acute and chronic inflammation and abundant calcification (see comment)
    • Comment: The urothelial mucosa shows ulcerations and extensive calcified necrotic debris admixed with many multinucleated giant cells. Granulation tissue with acute and chronic inflammatory infiltrate, eosinophils and fibrin deposits are present. Clinical correlation with urinalysis and microbiological culture is highly recommended. These findings are consistent with encrusted cystitis in the right clinical context.
Differential diagnosis
Board review style question #1

A bladder biopsy is shown above from a male patient with a history of bladder instrumentation. The entity depicted above is characterized by deposition of inorganic salts within injured urothelial mucosa due to the action of the urea splitting bacteria. What is the diagnosis?

  1. Emphysematous cystitis
  2. Encrusted cystitis
  3. Eosinophilic cystitis
  4. Interstitial cystitis
  5. Malakoplakia
Board review style answer #1
B. Encrusted cystitis

Comment Here

Reference: Encrusted cystitis

Eosinophilic cystitis
Definition / general
  • Inflammatory condition of the urinary bladder, with recurrent episodes of urinary frequency, dysuria, gross hematuria and suprapubic pain during micturition
  • Not related to Langerhans cell granulomatosis
Epidemiology
  • Rare (about 200 reported cases)
  • Women or children with allergic disorders and peripheral eosinophilia, older men with prostate / bladder disorders or parasitic infestation
Clinical features
  • Reported at all ages with striking predominance in females
  • 20% occur in children; symptoms tend to disappear spontaneously (Arch Dis Child 2001;84:344)
  • Clinical and imaging findings are nonspecific; cystoscopic findings include ulcers, exudates, edematous bullae or polyps (which may simuate malignancy)
Case reports
Treatment
  • Treatments are typically not curative
  • Withdrawal of any identifiable precipitating factor
  • Nonsteroidal antiinflammatory agents and antihistamines are favored first line agents followed by corticosteroids or cyclosporine
  • Transurethral resection is used in refractory cases (J Urol 2001;165:805)
  • Longterm followup is recommended for all patients (Int J Clin Pract 2005;59:356)
Gross description
  • Edematous and erythematous mucosa with polypoid growths resembling allergic polyps of nasal septum
Microscopic (histologic) description
  • Hispathological findgings can be divided in "acute" and "chronic" phase:
  • Acute phase: prominent eosinophilic infiltrate (Yamada and Taguchi criteria are 20 or more eosinophils per five 20x fields) with edema and occasional muscle necrosis; Charcot-Leyden crystals may be present (Arch Pathol Lab Med 2009;133:289)
  • Chronic phase: fewer eosinophils but more prominent mast cells, plasma cells and muscle fibrosis
Microscopic (histologic) images

Contributed by Jian-Hua Qiao, M.D.
Inflammatory infiltrate with marked eosinophils

Inflammatory infiltrate with marked eosinophils


Exstrophy
Definition / general
  • Bladder mucosa is exposed to the surface exterior through a defect in the anterior abdominal wall
Terminology
  • Also called classic bladder exstrophy
  • Part of exstrophy-epispadias complex (bladder exstrophy, epispadias, failure of labia fusion, lack of symphysis pubis fusion)
Epidemiology
  • Prevalence at birth is 1 per 50,000-100,000; male predominance up to 5:1
  • May be associated with maternal tobacco use (J Urol 2008;179:1539)
  • High concordance in identical and non-identical twins
  • Documented cases of family clusters
Etiology
  • Developmental failure in lower abdominal wall or anterior wall of bladder due to failure of cloacal membrane to properly differentiate
  • Persistence or overgrowth of cloacal membrane on lower anterior abdominal area may prevent normal mesenchymal ingrowth, which causes divergence of lower abdominal muscular structures, and forces the genital ridges to fuse caudal to the cloacal membrane (J R Soc Med 1996;89:39P)
  • The stage of ingrowth of the urorectal septum at the time of rupture determines whether one will produce an exstrophic urinary tract alone (classic bladder exstrophy or epispadias) or cloacal exstrophy with the hindgut interposed between the hemibladders
Diagrams / tables

Images hosted on other servers:

Diagrams of exstrophy

Clinical features
  • Usually diagnosed with prenatal scans
  • Urine drips from ureteric orifices onto bladder surface (Orphanet J Rare Dis 2009 Oct 30;4:23)
  • Bladder mucosa is red at birth, and mucosal polyps may be seen
  • If closure is delayed, may have inflammatory changes of whitish coating, ulcerations and hyperplastic formations
  • Thin skin stripes mark transition between normal skin and squamous metaplastic area
  • Most patients have bilateral inguinal hernias
  • Male newborns have open (epispadic) urethral plate that covers whole dorsum of penis from open bladder to glandular grove
  • Female newborns have completely split clitoris next to the open urethral plate; also narrow vaginal opening anterior to a shortened perineum
Treatment
  • Surgical correction with several procedures over time, usually beginning in first week of life
  • Rarely, adults present to physicians with exstrophy (Indian J Urol 2008;24:164)
  • Even in surgically treated patients, bladder mucosa reverts to normal in only 25% of cases
Clinical images

Images hosted on other servers:

Exstrophy in male infants

Microscopic (histologic) description
  • Acute and chronic inflammation; lamina propria fibrosis in practically all cases at any age
  • Metaplastic changes are very common and include squamous and glandular metaplasia
  • Evidence of mucosal proliferation (von Brunn’s nests, cystitis cystica, and cystitis glandularis) in most cases
  • Associated with adenocarcinoma (<10% of exstrophied bladders), squamous cell carcinoma (~7% of patients) or very rarely urothelial (transitional) carcinoma
Additional references

Features to report, grossing & frozen section
Features to report: Definition / general
Features to report: cystectomy, cystoprostatectomy, pelvic exenteration
Mandatory:
  • Specimen type / procedure
  • Tumor size
  • Tumor histologic type
  • Tumor histologic grade
  • Adequacy of specimen to determine muscularis propria invasion (muscularis propria found / not found / indeterminate)
  • Lymphovascular invasion (present / absent / indeterminate)
  • Pathologic stage (includes depth of invasion) (pTNM)
  • Regional lymph nodes: either no nodes submitted or found, number examined / cannot be determined (explain), number involved (any size) / cannot be determined (explain); optional - extranodal extension of tumor
  • Surgical margins: specify margins involved by invasive carcinoma or in situ carcinoma; if uninvolved by invasive carcinoma, specify distance from invasive carcinoma to margin
  • Presence of tumor at margins of urethra, ureter, paravesicular soft tissue or pelvic soft tissue
  • Involvement of adjacent structures: perivesical fat, ureter (specify laterality), urethra, vagina, uterus and adnexae, pelvic sidewall (specify laterality), prostate, seminal vesicle (specify laterality), rectum, other
  • Additional epithelial lesions: papilloma, inverted papilloma, papillary neoplasm of low malignant potential

Optional but recommended:
  • Site of tumor
  • Tumor configuration (papillary, solid / nodule, flat, ulcerated, indeterminate)
  • Additional findings: multifocality, carcinoma in situ, dysplasia, hyperplasia, inflammation, therapy related changes, keratinizing squamous metaplasia, intestinal metaplasia, inflammation / regenerative changes, cystitis cystica glandularis, urothelial carcinoma involving urethra / prostatic ducts and acini (use protocol for carcinoma of urethra)
  • References: Mod Pathol 2009;22:S70, Arch Pathol Lab Med 2003;127:1263
Features to report: bladder biopsy / TURBT
    Mandatory:
    • None

    Optional / recommended:
    • Site of tumor / procedure
    • Tumor size
    • Tumor histologic type
    • Tumor histologic grade
    • Macroscopic pattern of growth (papillary, flat, invasive)
    • Type of invasion: broad spread or tentacular (like tentacles)
    • Depth of invasion
    • Layers of wall represented (muscularis propria identified or not)
    • Denuded / ulcerated
    • Involvement of prostate
    • Additional findings: carcinoma in situ, dysplasia, hyperplasia, inflammation, normal, other

    Tumor:
    • Histologic type - urothelial carcinoma with / without squamous differentiation, squamous cell carcinoma (classical or variant), adenocarcinoma (classical or variant), small cell carcinoma, undifferentiated, mixed cell type, undetermined
    • Histologic grade - based on tumor type
    • Tumor configuration - papillary, flat, nodule, invasive, ulcerated, undetermined
    • Invasion assessment - detrusor muscle (muscularis propria) present, absent or indeterminate
    • Lymphovascular invasion - present, not identified, indeterminate, should be assessed away from the main tumor (only if unequivocal, often is overdiagnosed, Mod Pathol 1990;3:83)
    • Microscopic extent of tumor - noninvasive flat carcinoma in situ, involves lamina propria, involves muscularis propria
    • Extension in prostatic chips sampled by TURBT - involvement of prostatic urethra, prostatic and acini ducts (by carcinoma in situ) or prostatic stroma (by invasive carcinoma)
    • Associated epithelial lesions - urothelial papilloma (classic or inverted type), papillary urothelial neoplasm of low malignant potential, other
    • Additional findings - carcinoma in situ, dysplasia, metaplasia, hyperplasia, inflammation, regenerative changes, treatment related changes, other
    • References: Mod Pathol 2009;22:S70
Grossing
  • TURBT (transurethral resection of bladder tumor): weigh fragments together and measure largest fragment in one dimension; submit all specimen fragments if possible
  • Cystectomy: ink entire external surface; either open with Y shaped incision through anterior wall, pin and fix overnight or fill with formalin, fix overnight, and divide into anterior and posterior halves

Gross inspection of cystectomy specimen:
  • Document and measure all organs received
  • Document number and location of lesions
  • Examine ureters carefully
  • Document gross tumor extension into surrounding soft tissues (for substaging of pT3 disease)
  • Evaluate surgical margins

Distal urethral margins:
  • Women: usually taken en face
  • Men: prostatic urethra at apex may be taken en face or as apical cone (perpendicular)
  • Apical margin in men may be involved by incidental prostate cancer

Soft tissue:
  • Careful gross examination will identify deepest point of invasion that should be sampled
  • Sections usually taken perpendicular to inked outer surface

Lymph node identification in both cystectomy and separately submitted specimen:
  • At least 10 to 14 nodes should be retrieved, although node numbers vary widely according to individual patient circumstances (J Urol 2004;171:1823)
  • Clearly report anatomic node groups sampled, if possible (e.g. paraaortic)
  • Submit 1 section from each grossly positive lymph node
  • Submit all possible nodal tissue if it can be done in 5 or fewer cassettes
  • All other lymph nodes should be entirely submitted, as presence of nodal disease may be used as an indication for adjuvant therapy
  • Lymph nodes may be grossly or microscopically detected in the perivesical fat (See CAP Cancer Protocols and Checklists, 2011)
  • Submit grossly normal fat for small (< 1 cm) lymph nodes

Ureters and urethra:
  • Examination is usually for carcinoma in situ
  • Invasion may rarely be seen in surrounding soft tissue

Sections to submit:
  • Tumor, representative, 1 per cm of tumor diameter (up to 10 cassettes); if initial sampling shows only noninvasive or non-muscle invasive tumor, submit additional sections as necessary to rule out invasion or muscle invasion
  • Tumor, deepest penetration into wall (multiple sections)
  • Tumor and adjacent normal bladder wall
  • Bladder neck
  • Bladder trigone (two sections)
  • Anterior and posterior wall (two sections each)
  • Left lateral and right lateral wall
  • Dome (two sections)
  • Ureteral orifices, including intramural portion
  • Ureteral margin, possibly additional sections if long segment is present
  • Urethral margin
  • Prostatic urethral margin (if present)
  • Margins of resection
  • Other grossly abnormal areas in bladder, ureters, urethra or other tissue
  • Prostate (peripheral zone, central zone, seminal vesicles, grossly suspicious areas)
  • Lymph nodes
  • Pelvic wall
Frozen section
Indications
  • Frozen sections usually performed for ureteral margin evaluation for carcinoma in situ or invasive carcinoma
  • Only 3% positive ureteral margins in one study, which suggests it should not be done routinely (Can Urol Assoc J 2010;4:28)
  • More useful if carcinoma in situ present in bladder (J Urol 2006;176:2409)
  • Frozen sectioning may be useful for evaluating lymph nodes (Urology 2007;69:83)

Procedure
  • Recommended to obtain cross section of distal ureter, not shaved margin
  • Frozen section is highly sensitive for malignant ureteral margins, but reresection often does not convert positive margins to negative margins (World J Urol 2011;29:451)

Microscopic (histologic) description
  • Technical artifact from freezing may induce atypical features in urothelium
  • Use stromal lymphocytes as a reference of nuclear size
  • Variant invasive patterns, such as plasmacytoid, may mimic inflammatory cells

Female urethral carcinoma
Definition / general
  • Rare primary neoplasm of epithelial origin
  • Secondary involvement by urothelial carcinoma of the bladder is much more common than a primary (Eur Urol 2013;64:823)
Essential features
  • Urethral carcinoma is usually due to secondary involvement
  • Primary urethral carcinoma is rare and the most frequent histologic types are urothelial carcinoma, squamous cell carcinoma and adenocarcinoma (not otherwise specified, clear cell)
Epidemiology
Sites
  • Type depends on sex and location:
    • Female urethra divided in proximal 2/3 and distal 1/3
      • Proximal 2/3 usually urothelial carcinoma
      • Distal 1/3 usually squamous cell carcinoma (BJU Int 2014;114:25)
      • Frequently initially misdiagnosed as caruncle
    • Adenocarcinoma present in both sexes; may originate anywhere along the urethra
      • May arise from urothelial metaplastic mucosa or from periurethral glands in both sexes
Pathophysiology
  • Predisposing factors include:
Clinical features
  • Most patients present with symptoms associated with locally advanced disease (Eur Urol 2013;64:823)
    • Gross hematuria or bloody urethral discharge, dysuria, extraurethral mass
    • Bladder outlet obstruction, pelvic pain, urethrocutaneous fistula
    • Abscess formation, dyspareunia
  • Approximately 1/3 of men and women present with involved regional lymph nodes
Diagnosis
  • Clinical examination with palpation of external genitalia for suspicious indurations and pelvic exam in women (Eur Urol 2013;64:823)
  • Urinary cytology
  • Diagnostic urethroscopy and biopsy
Radiology description
  • Aims to assess local extent and detect lymphatic and distant metastatic spread
  • Magnetic resonance imaging for evaluating extent of tumor and monitoring response to neoadjuvant chemotherapy (Eur Urol 2013;64:823)
Prognostic factors
Case reports
Treatment
Gross images

Contributed by Dr. Jesus Chavez and Dr. Debra Zynger
Urethra and periurethral tissue

Urethra and periurethral tissue

Microscopic (histologic) description
  • Urothelial carcinoma
  • Squamous cell carcinoma
    • Sheets of large, pleomorphic tumor cells with focal or abundant keratinization (depending of grade of differentiation), ample cytoplasm, intercellular bridges, high mitotic activity, prominent nuclear atypia
  • Adenocarcinoma
  • Clear cell adenocarcinoma
    • May have glandular, tubulocystic, solid / diffuse, papillary or micropapillary growth patterns
    • Cuboidal, variably sized cells with abundant clear or eosinophilic cytoplasm and cytoplasmic vacuoles
    • Nuclei that are hyperchromatic, pleomorphic and have prominent nucleoli
    • Hobnail changes and extracellular mucoid material may be present
    • Mitoses and necrosis are often seen
Microscopic (histologic) images

Contributed by Dr. Jesus Chavez and Dr. Debra Zynger
High power, low grade

High power, low grade

Noninvasive high grade

Noninvasive high grade

Invasive high grade

Invasive high grade

Necrosis and keratinization

Necrosis and keratinization

Elderly woman: metastatic

Elderly woman: metastatic



Clear cell adenocarcinoma
Low power, prominent necrosis

Low power, prominent necrosis

Can mimic nephrogenic metaplasia

Can mimic nephrogenic metaplasia

With hobnailing

With hobnailing

With prominent clear cells and diffuse, sheet-like growth

With prominent clear cells and diffuse, sheet-like growth



Case #194

Various images

CK7

p53

Positive stains
Negative stains
Differential diagnosis

Fibroepithelial polyp
Definition / general
  • Exophytic intraluminal mass of vascular connective tissue and variable inflammatory cells covered by normal urothelium
Epidemiology
  • Rare; usually reported in children
  • In adults, male predominance, median age 44 years, range 17-70 years
Sites
  • Usually near verumontanum or bladder neck
  • More common in proximal ureter than bladder
Etiology
  • Nonneoplastic
Clinical features
  • May be incidental / asymptomatic
Case reports
  • Boys ages 2 and 5 years (Pediatr Surg Int 2008;24:613)
  • 3 year old boy with 15 cm polyp (Pediatr Dev Pathol 2003;6:179)
  • 14 year old boy (Arch Ital Urol Androl 2005;77:118)
Treatment
  • Transurethral resection; don’t recur
Clinical images

Images hosted on other servers:

Cystoscopy

Ureteral polyp

Microscopic (histologic) description
  • Urothelial or rarely columnar epithelial lining
  • Either (a) polypoid mass with cloverleaf-like projections and florid cystitis cystica et glandularis of nonintestinal type in stalk, (b) papillary tumor composed of numerous small, rounded fibrovascular cores containing dense fibrous tissue, or (c) polypoid lesion with secondary tall, finger-like projections (Am J Surg Pathol 2005;29:460)
  • Broader stalks than papilloma; no prominent edema or inflammation
  • May have degenerative stromal atypia (Arch Pathol Lab Med 1986;110:241)
Microscopic (histologic) images

Images hosted on other servers:

Polyp of ureter

Differential diagnosis

Flat hyperplasia
Definition / general
  • Markedly thickened mucosa (up to 10 or more cell layers) without cytologic atypia
  • May be adjacent to low grade papillary urothelial neoplasm
  • By itself, has no malignant potential and requires no treatment
Epidemiology
  • True incidence is not known due to lack of large scale screening studies
Clinical features
  • Asymptomatic
Prognostic factors
  • Isolated lesion does not have a premalignant potential
  • Molecular studies suggest a preneoplastic potential in flat hyplerplasia concurrent with low-grade papillary tumors (Hum Pathol 2010;41:155)
Microscopic (histologic) description
  • May occur adjacent to low-grade papillary tumors or as an isolated lesion
  • Marked thickening of mucosa up to 10 or more cell layers
  • Maintains morphologic evidence of maturation from basal cells to superficial cells
  • Absence of cytologic atypia or mitoses
Positive stains
  • CK20 limited to umbrella cells (also stains deeper cells in dysplasia)
  • CD44 staining limited to basal cells (stains all layers in reactive atypia but no staining in dysplasia)
Negative stains
Molecular / cytogenetics description
  • Genetic alterations in chromosome 9 are frequent in flat hyperplasia with concurrent low-grade papillary tumors
Differential diagnosis

Flat lesions with atypia of unknown significance
Definition / general
  • Atypia between reactive and dysplasia
  • More pleomorphism and hyperchromasia than expected for the amount of inflammation present
  • Recommend close follow up and reevaluation after inflammation subsides
Differential diagnosis

Flat lesions with reactive changes
Definition / general
  • Inflammatory atypia characterized by mild nuclear abnormalities in acutely or chronically inflamed urothelium
  • Uniformly enlarged and vesicular nuclei, central prominent nucleoli
  • May have frequent mitotic figures involving the lower layers of urothelium
Clinical features
  • May have history of cystitis, infection, instrumentation, stones or therapy
  • Not neoplastic but may coexist with dysplasia or carcinoma in situ
  • Most patients present with hematuria or irritative symptoms (frequency, urgency, dysuria)
Prognostic factors
Gross description
  • Erythematous or inflamed urothelium cystoscopically
Microscopic (histologic) description
  • Urothelium is normal to slightly thickened and shows morphologic evidence of maturation
  • Cells have abundant cytoplasm, nuclei may be enlarged and may have prominent nucleoli
  • Lamina propria is usually inflamed with inflammation extending to the mucosa
  • No nuclear hyperchromasia, no pleomorphism, no coarse chromatin pattern
Positive stains
  • CK20 (limited to umbrella cells), CD44 (stains all cell layers)
Negative stains
Differential diagnosis

Follicular cystitis
Definition / general
  • Nonspecific inflammatory disease of the bladder, with lymphoid follicles in lamina propria, often with chronic cystitis
Terminology
  • Also called cystitis follicularis
Sites
Etiology
  • Associated with prolonged urinary tract infection secondary to bladder outlet obstruction, neurogenic or muscular dysfunction of the bladder (commonly Salmonella or other gram- infection), intravesical chemotherapy or bcg (Clin Cancer Res 2003;9:5550)
Case reports
Gross description
  • Mucosal nodularity or granularity
Microscopic (histologic) description
  • Large number of plasmacytic cells and lymphocytes with lymphoid follicles scattered within the bladder mucosa and submucosa
  • Overlying urothelium may have mild atypia
Microscopic (histologic) images

Images hosted on other servers:

Bladder biopsies

Follicular cystitis

Cytology description
  • Reactive epithelial cells (suggested by nuclear pleomorphism without alteration of nuclear/cytoplasmic ratio and glandular metaplasia including cystitis cystica and glandularis) mixed with tissue fragments and cellular aggregates reflecting the structure of lymphoid follicles with a pleomorphic lymphoid population, intermixed histiocytes and scattered tingible body macrophages (Diagn Cytopathol 2002;27:205)
Cytology images

Images hosted on other servers:

Numerous lymphocytes with varying maturity

Differential diagnosis
  • Follicular lymphoma: very rare in bladder; closely packed follicles containing small cleaved cells without nucleoli (centrocytes) and larger noncleaved cells with moderate cytoplasm, open chromatin and multiple nucleoli (centroblasts); minimal or no apoptotic cells or tingible body macrophages; BCL2+ within follicles, usually t(14;18)(q32;q21)
  • Other non-Hodgkin lymphoma: monomorphic atypical lymphoid population
  • Granulomatous process in cases with histiocyte predominance (low number of lymphocytes)
  • Cystitis with sporadic lymphocytes: no germinal centers, usually no overlying epithelial atypia
  • Tuberculosis: may resemble follicular cystitis at cystoscopy; histologically has granulomas, with or without central caseation

Grading-bladder
Definition / general
  • Nonmuscle invasive urothelial carcinomas are graded following the 2 tier WHO 2004 / 2016 (endorsed by the American Urologic Association and European Association of Urology) or the 3 tier WHO 1973 grading systems (endorsed by the European Association of Urology)
  • WHO 2004 / 2016 classification of urothelial neoplasms includes papillary urothelial neoplasm of unknown malignant potential (PUNLMP)
  • Grading of nonmuscle invasive papillary urothelial carcinomas determines the risk of stage progression in recurrent bladder cancer
  • Invasive urothelial carcinomas, independent of the degree of invasion, are generally graded as WHO 2004 / 2016 high grade (World J Urol 2019;37:41)
Essential features
  • Grading of (papillary) urothelial carcinomas is based on the level of orderedness of the urothelial lining at intermediate power and nuclear atypia
  • Orderedness represents a continuum, varying from very well ordered to chaotic with increasing nuclear atypia (see Diagrams / tables)
  • Substantial interobserver variation due to lack of landmarks separating the different grades
  • 2 grading systems (WHO 1973 and 2004 / 2016) cannot be translated directly into each other due to overlapping grades (Eur Urol 2010;57:1052)
  • WHO 1973 but not WHO 2004 / 2016 grading of pT1 bladder cancer is prognostic for stage progression (BJU Int 2011;107:404)
  • In urothelial carcinomas with grade heterogeneity, the highest grade is reported
Diagrams / tables

Contributed by Theodorus van der Kwast, M.D., Ph.D.
WHO 1973 versus WHO 2004

WHO 1973 versus WHO 2004

WHO 2004 / 2016 PUNLMP
WHO 2004 / 2016 low grade
WHO 2004 / 2016 high grade
WHO 1973 grade 1
  • Clinical description
    • Frequency: 35% in pTa, < 5% in pT1
    • Manifestation by micro or gross hematuria
    • Urine cytology almost always negative
    • Cystoscopy shows exophytic sessile or polypoid lesion
  • Microscopic description
    • Increased thickness of papillary structures with slender fibrovascular cores
    • Ordered layering with streaming of uniform nuclei
    • No or minimal nuclear enlargement
    • No or mild variation in nuclear size, contour or shape
    • Nuclear grooves
    • No nuclear hyperchromasia
    • Limited mitotic activity may extend to suprabasal cell layers
    • Presence of umbrella cell layer
  • References: WHO: Histological Typing of Urinary Bladder Tumours [Accessed 14 June 2021], Eur Urol Focus 2021 Mar 23 [Epub ahead of print]
WHO 1973 grade 2
WHO 1973 grade 3
Grade heterogeneity
  • Clinical description
    • Frequency: up to 30% (Cancer 2000;88:1663)
    • Manifestation by microscopic or gross hematuria
    • Occasionally positive urine cytology
    • Cystoscopy shows exophytic sessile, solid or polypoid lesion
  • Microscopic description
    • Distinct areas of low and high grade urothelial carcinoma
    • Clear demarcation of separate areas
  • Reporting
    • By convention, the highest grade is reported if comprising > 5% of the carcinoma
    • If < 5%, a comment on its presence is made
Microscopic (histologic) images

Contributed by Theodorus van der Kwast, M.D., Ph.D.
WHO 2004 / 2016, PUNLMP

WHO 2004 / 2016, PUNLMP

WHO 2004 / 2016, low grade

WHO 2004 / 2016, low grade

WHO 2004 / 2016, high grade

WHO 2004 / 2016, high grade

WHO 1973, grade 1

WHO 1973, grade 1


WHO 1973, grade 2

WHO 1973, grade 2

WHO 1973, grade 3

WHO 1973, grade 3

Grade heterogeneity

Board review style question #1

Low grade papillary urothelial carcinoma can be distinguished from high grade papillary urothelial carcinoma microscopically by

  1. Absence of suprabasal mitoses
  2. Number of cell layers of the urothelial lining
  3. Presence of umbrella cells
  4. Variation in nuclear size
Board review style answer #1
D. Variation in nuclear size

Comment Here

Reference: Grading-bladder

Granulomatous cystitis
Granulomatous cystitis
Definition / general
  • Granulomas in bladder, due to various infectious or treatment related causes

Etiology
  • Tuberculosis, bCG (bacillus Calmette-Guerin) treatment for papillary urothelial carcinoma, biopsy / resection, Schistosoma haematobium infection, actinomycosis

Clinical features
  • Tuberculosis: rare in most countries; bladder lesions near trigone, smaller lesions merge over time into large ulcers; may involve prostate or vagina; often secondary infection from kidney
  • bCG: used to treat high grade papillary carcinoma or carcinoma in situ of bladder
  • Post biopsy / resection: present in 14% with 2 surgical procedures

Case reports

Treatment

Gross description
  • Can present as mass / polypoid lesion

Microscopic (histologic) description
  • Tuberculosis: caseating granulomas with Langhans giant cells, mostly in lamina propria with mucosal ulceration
  • bCG: induces chronic inflammation, superficial ulceration and noncaseating granulomas with active and chronic inflammation; changes may extend into prostate (Am J Clin Pathol 1993;99:244)
  • Post biopsy / resection: either necrotizing and palisading, resembling rheumatoid nodules or foreign body type (without foreign material) or both (Am J Clin Pathol 1986;86:430)
  • Actinomycosis: scattered lymphoid follicles and nonspecific inflammation with or without intermixed colonies of Actinomyces
Postoperative granulomas
Definition / general

Etiology

Microscopic (histologic) description
  • Palisading histiocytes or giant cells surrounding central necrosis (resembling rheumatoid nodules) or foreign body type granuloma
  • Lesions heal by fibrous scarring

Differential diagnosis
  • Granulomatous cystitis:
    • Terms may overlap; often refers to patients with infectious etiology and no distinct mass
Xanthogranulomatous cystitis
Definition / general
  • Rare (< 50 cases reported in bladder) inflammatory disorder characterized by lipid laden macrophages

Clinical features
  • Benign but associated with malignancy in some cases (Histopathology 1998;33:212)
  • Associated with xanthogranulomatous pyelonephritis, urachal adenomas, inflammatory bowel disease

Case reports

Treatment
  • Broad spectrum antibiotics; possibly resection

Microscopic (histologic) description
  • Lipid laden macrophages, plasma cells and lymphocytes
  • May have some Touton type giant cells
  • Similar to malakoplakia but without Michaelis-Gutmann bodies

Positive stains

Negative stains

Differential diagnosis
Microscopic (histologic) images

Images hosted on other servers:
Granulomatous cystitis due to bCG treatment Granulomatous cystitis due to bCG treatment

Granulomatous cystitis due to bCG treatment

Granulomatous cystitis, postresection

Granulomatous cystitis, postresection

Postoperative granulomas, postresection

Postoperative granulomas, postresection


Hemorrhagic cystitis
Definition / general
  • Clinical syndrome of gross hematuria, often with irritative bladder symptoms
Etiology
Clinical features
  • Irritative voiding symptoms, gross hematuria
Case reports
Treatment
  • Clot evacuation and hydration, bladder irrigation with saline or alkalinized saline, intravesical instillation of aluminium hydroxide / magnesium hydroxide
  • More aggressive therapies include transurethral coagulation of bladder, fixation with silver nitrate and formalin, embolization of internal iliac arteries or their branches to the bladder; rarely requires cystectomy
Clinical images

Images hosted on other servers:

Inflammatory bleeding bladder mucosa

Gross images

Images hosted on other servers:

Hemorrhagic and ulcerative cystitis after cyclophosphamide

Microscopic (histologic) images

Images hosted on other servers:

Edema, bleeding and granulocyte infiltration

Hemorrhage and edema in the lamina propria

Cytology images

Case of the Week #353

47 year old man with herpes viral cytopathic effects in urine cytology

Differential diagnosis
  • Hemorrhage associated with malignancy

Histology
Definition / general
  • Hollow muscular elastic organ that sits on the pelvic floor and stores urine
Essential features
  • Layers of bladder wall (from inside out)
    • Urothelium: specialized stratified lining epithelium, impermeable barrier
    • Lamina propria: connective tissue containing vessels, lymphatics, nerve endings and a few elastic fibers
    • Muscularis propria: also called detrusor muscle; consists of 3 layers (inner longitudinal, middle circular and outer longitudinal)
    • Serosa / adventitia: serosa is loose connective tissue covering bladder dome; the remaining area is covered by adventitia
  • Important to histomorphologically differentiate the layers in order to properly stage the tumors, determine therapy and estimate survival
Physiology
  • Located in the extraperitoneal space of the pelvis, behind the pubic bones (StatPearls: Histology, Bladder [Accessed 14 April 2022])
  • Extends into the abdomen when distended
  • 2 main parts
    • Upper part is above the ureteric orifices and is composed of the apex and body
    • Lower part is beneath the ureteric orifices and is composed of the fundus, trigone and neck
  • Urothelium is 5 - 7 cell layers thick in a contracted bladder and 2 - 3 cells thick in a distended bladder (Am J Physiol Renal Physiol 2009;297:F1477)
Diagrams / tables

Images hosted on other servers:

Staging of bladder carcinoma

Clinical features
  • Urothelium is exclusive to the conducting passages of the urinary system (renal pelvis, ureter, bladder, upper urethra and prostatic ducts) (Am J Physiol Renal Physiol 2009;297:F1477)
  • > 90% of bladder carcinomas arise from the urothelium, mostly from the posterior wall of the bladder (Nat Cell Biol 2014;16:982)
  • In practice, sampling tangentially to the basement membrane can generate an artificially thick mucosa, hence urothelial thickness is of low utility in the assessment of urothelial neoplasms
  • Squamous epithelium is common in the trigone area of females; abundant intracytoplasmic glycogen and lack of keratinization makes it histologically similar to vagina or cervix (Am J Anat 1962;111:319)
  • von Brunn nests, cystitis cystica and cystitis glandularis represent a continuum of proliferative or reactive changes
  • von Brunn nests (proliferative cystitis)
    • Reactive proliferative change is present in 85 - 95% of bladders; frequency increases with age; more common at the trigone
    • Nests of cytologically benign urothelium in lamina propria; with regular spacing and same extent to horizontal level at base of proliferation
  • Bladder wall urachal remnants: cystically dilated epithelial lined structures are sometimes found in biopsies from the dome or anterior wall of the bladder
Gross description
  • Hollow muscular viscus resembling an inverted pyramid when empty and a sphere when distended (Microbiol Spectr 2015;3:10)
  • Flat internal (mucosal) surface when distended; throws into abundant folds when empty
  • Lies posterior to the pubic bone, anterior to the uterus in females and anterior to the rectum in males
  • Partially retroperitoneal, with its peritoneal covered dome projecting into the abdomen when fully distended
  • Triangular shaped area at the base of the bladder called the trigone
Microscopic (histologic) description
  • 4 layers (from inside out): urothelium, lamina propria, muscularis propria, serosa / adventitia
  • Urothelium: 3 layers of cells
    • Umbrella / apical cells (superficial layer)
      • Formerly called transitional epithelium, intermediate between nonkeratinizing squamous and pseudostratified columnar epithelium
      • Dome / pyramid shaped, large and ovoid, frequently binucleated cells with abundant eosinophilic cytoplasm
      • Scalloped surface outline often overlapping 2 or more of the underlying cells
      • Superficial cytoplasm is fuzzy, indistinct and more intensely stained than the rest of the cytoplasm
      • Luminal surface of the cells appears thickened and more densely stained
      • Form an impermeable barrier; tight junctions between the cells decrease paracellular flux while uroplakins form a superficial plaque
    • Intermediate cells
      • Multicell layering, depending on the stage of distension
      • Cuboidal to low columnar with well defined borders and amphophilic cytoplasm that is rich in glycogen
      • Uninucleated cells: ovoid nuclei with fine granular chromatin, no mitotic figures arranged regularly along the long axis at right angles to surface
    • Basal cells
      • Single layer in contact with basal lamina
      • Mononucleated, cuboidal cells with mitotic capability
      • Cylindrical, can be flat when bladder wall is stretched; some have longitudinal nuclear grooves
      • Gradual turnover but significant regenerative ability
  • Lamina propria
    • Dense connective tissue containing a rich vascular network, lymphatic channels, sensory nerve endings, elastic fibers and interspersed adipose tissue
    • Forms thick mucosal folds when the viscus is contracted
    • Thickness varies with the degree of distention and is generally thinner in the areas of the trigone and bladder neck
    • Discontinuous isolated bundles of muscularis mucosa (wisps of smooth muscle) may be present; 5% of bladders may have a well developed, continuous muscularis mucosa (Am J Surg Pathol 1987;11:668)
    • Hyperplastic muscularis mucosa (more common in women) may resemble muscularis propria (Am J Surg Pathol 2007;31:1420, Ann Diagn Pathol 2007;11:395)
    • Important to distinguish muscularis mucosa from muscularis propria for accurate staging; smoothelin antibody is negative, while vimentin is positive in the former (Int J Biol Markers 2017;32:e305)
    • Interstitial cells of Cajal act as nerve signal transducer between the smooth muscle cells and nerve endings
    • Adipose tissue may be present within deep lamina propria (in small localized aggregates), while it is always found within muscularis propria; carefully differentiate between the two, especially in transurethral resection of bladder tumor (TURBT) specimens as it can overstage the tumor (Am J Surg Pathol 2000;24:1286)
  • Muscularis propria
    • 3 layers: inner and outer longitudinal layers and a central circular layer
    • Layers are distinct near bladder neck, elsewhere there is no definite orientation
    • Bladder's body has a higher smooth muscle content compared with the trigone
    • Rarely nests or cords of paraganglia associated with nerves and vessels; clear or granular cytoplasm with round or vesicular nuclei
      • Carefully distinguish from invasive carcinoma
      • Cytokeratin negative and chromogranin positive
    • Significantly thickened in urinary flow obstruction
  • Serosa / adventitia
    • Thin connective tissue layer covering the bladder dome and continuous with the peritoneal layer of the abdominal wall
    • Contains blood vessels of various sizes
    • Adventitia, an outermost layer of loose connective tissue in areas where there is no serosa
Microscopic (histologic) images

Contributed by Anil Parwani, M.D., Ph.D., M.B.A.
Bladder wall

Bladder wall

Urothelium

Urothelium

von Brunn nests

von Brunn nests


Urothelium and lamina propria

Muscularis mucosa

Muscularis mucosa

Muscularis propria

Cytology description
  • In voided urines, transitional cells are shed singly; whereas in washings / brushings, the cells are aggregated in sheets and clusters
  • Superficial and intermediate urothelial cells are seen in voided urine
  • Superficial, intermediate and basal urothelial cells are seen in catheterized urine and bladder washings
  • Superficial / umbrella cells:
    • Low N:C ratio with pale finely granular chromatin, smooth nuclear contour and clear cytoplasm
  • Intermediate and basal cells:
    • High N:C ratio, smaller nuclei with even spacing and darker chromatin than superficial cells
  • Melamed-Wolinska bodies: round to oval, hyaline, red or green-blue cytoplasmic inclusions seen within degenerated, benign or malignant urothelial cells (Diagn Cytopathol 2011;39:117)
Positive stains
Negative stains
  • Basal / parabasal cells: CK20
  • Umbrella cells: CD44s
Electron microscopy description
  • Superficial urothelial cells form an impermeable barrier via numerous junctional complexes
  • Trilaminar (asymmetric) unit membrane composed of 2 dense layers of unequal thickness and a central lucent layer with interspersed normal areas which act like hinges
  • Surface plasma membrane consists of inflexible apical plaques containing uroplakins (Kidney Int 2016;89:612)
Videos

Shotgun histology bladder

Board review style question #1

Which of the following characteristics is true regarding the histology displayed in the picture?

  1. Muscular layer is consistently organized into 3 distinct layers
  2. Superficial layer consists of only umbrella cells, which have a high regenerative capacity
  3. Umbrella cells are CK20+ and CD44s-
  4. Well defined continuous muscularis mucosa can be always seen
Board review style answer #1
C. Umbrella cells are CK20+ and CD44s-. Superficial cells or the umbrella cells of urothelium are CK20+ and CD44s-, while basal cells are CK20- and CD44s+.

Comment Here

Reference: Bladder - Histology
Board review style question #2

What does the arrow indicate in the micrograph shown above?

  1. Desmoplastic response to invasive tumor
  2. Elastosis
  3. Muscularis mucosa
  4. Muscularis propria
Board review style answer #2
C. Muscularis mucosa. The wispy, incomplete fascicles of muscle represent muscularis mucosae. They are morphologically and histochemically distinct from muscularis propria.

Comment Here

Reference: Bladder - Histology

Inflammatory myofibroblastic tumor
Definition / general
  • Rare myofibroblastic spindle cell neoplasm of bladder with unknown neoplastic potential, more common at other sites (lung most common followed by soft tissue, bone)
  • Similar to postoperative spindle cell nodule, but without a history of surgery
  • Characterized by spindle cell proliferation with characteristic fibroinflammatory and pseudosarcomatous appearance
Terminology
  • Inflammatory myofibroblastic tumor (IMT) is terminology for neoplastic lesions
  • Inflammatory lesions are often called pseudotumors
  • Also known as pseudosarcoma, atypical myfibroblastic tumor, atypical fibromyxoid tumor, plasma cell granuloma
Epidemiology
  • More common in children and young adults
  • More common in females
Sites
  • Bladder is most common site in genitourinary tract
Etiology
  • Idiopathic, no known predisposing conditions
Clinical features
  • Pain, fever, weight loss, anemia, thrombocytosis, increased erythrocyte sedimentation rate and elevated gamma globulins are more common with IMTs arising at sites other than bladder
  • IMTs in bladder present with painless hematuria
  • Benign, but frequently misinterpreted as leiomyosarcoma or rhabdomyosarcoma
  • May recur locally, but doesn’t metastasize (Am J Surg Pathol 2006;30:1502)
Case reports
Treatment
Gross description
  • Polypoid mass with pale, firm cut surface
  • May be very large
  • Often gelatinous
Gross images

Images hosted on other servers:

White, gelatinous
mass with cystic
change brand
hemorrhage

Microscopic (histologic) description
  • Essential criteria: spindled myoepithelial cell proliferation and lymphocytic infiltrate
  • Patterns include:
    (1) loose stellate cells with myxoid background containing scattered inflammatory cells (nodular fasciitis-like)
    (2) spindle cells with a compact fascicular pattern (fibrohistiocytoma-like)
    (3) sparse cellular, collagenous areas (desmoid-like)
    (4) mixed

  • Cells are stellate myofibroblasts with abundant eosinophilic cytoplasm, elongated nuclei
  • May be cellular and infiltrative with mucosal ulceration, necrosis, cytologic atypia
Microscopic (histologic) images

AFIP images

Abundant thin walled vessels



Images hosted on other servers:

ALK1 expression

Focal inflammatory cells

Myofibroblastic cells
and inflammatory
cells in edematous
stroma

Spindle cells within fibromyxoid matrix and inflammatory cells

Electron microscopy description
  • Myofibroblasts: bipolar cells with eosinophilic, elongated, tapering cytoplasmic processes without striation, central oval nuclei with smooth contours, open chromatin, occasional nucleoli
  • No evidence of smooth muscle or skeletal muscle differentiation
Molecular / cytogenetics description
Differential diagnosis

Interstitial cystitis
Definition / general
  • Bladder syndrome of unknown etiology that clinically presents as chronic pelvic pain with lower urinary tract symptoms (Int J Urol 2020;27:578)
  • Clinical diagnosis of exclusion
Essential features
  • Idiopathic, debilitating syndrome that presents as chronic pelvic pain accompanied by lower urinary tract symptoms, diagnosed clinically after confusable diseases are ruled out
  • Conservative, multimodal approach recommended as initial treatment
  • Can be classified into non-Hunner lesion subtype (most common) and Hunner lesion subtype via cystoscopy with or without histopathology
  • Non-Hunner lesion subtype has a strong association with nonbladder pain syndromes and worse / unchanged therapeutic outcomes; bladder with no diagnostic pathology
  • Hunner lesion subtype has a strong association with more severe bladder symptoms and better outcomes from intravesical therapy; nonspecific histopathological features including but not limited to pancystitis with suburothelial lymphoplasmacytic inflammation and urothelial denudement
Terminology
  • Commonly referred to as bladder pain syndrome
  • Also called painful bladder syndrome and chronic interstitial cystitis
  • Subtypes: interstitial cystitis / bladder pain syndrome with Hunner lesions, interstitial cystitis / bladder pain syndrome without Hunner lesions
ICD coding
  • ICD-9: 595.1 - chronic interstitial cystitis
Epidemiology
Sites
  • Bladder
Pathophysiology
  • Poorly understood pathophysiology
  • Chondroitin sulfate, a glycosaminoglycan on the urothelial luminal surface that plays an important role in urothelial barrier function, is deficient, which leads to increased urothelial permeability (J Urol 2013;189:336)
  • Antiproliferative factor (APF), a sensitive and specific urinary biomarker of interstitial cystitis, plays a role in the inhibition of urothelial cell proliferation and in the alteration of urothelial cell adhesion (Urology 2001;57:104, J Urol 1996;156:2073, Mol Cell Proteomics 2011;10:M110.007492)
  • Upregulation of genes involved in pronociceptive inflammatory reactions and nociceptive pathways in the brain and spinal cord leads to pelvic pain (J Urol 2013;190:1925, Nat Rev Urol 2019;16:187)
  • Pelvic visceral organ cross sensitization theory: nonpelvic organs sharing sensory pathways with the bladder can undergo persistent physiologic changes induced by interstitial cystitis, which explains the high prevalence of extravesical pain in patients with interstitial cystitis
Etiology
  • No clear etiology for the onset of interstitial cystitis
  • Acidic food, caffeine and alcohol were found to be associated with acute exacerbation of interstitial cystitis (BJU Int 2012;109:1584)
Clinical features
  • Persistent or recurrent pelvic discomfort / pain with bladder filling, relieved by voiding (J Urol 2015;193:1545, Eur Urol 2010;57:35)
  • Pain is usually suprapubic but can radiate to the groin, vagina, rectum and sacrum
  • Accompanied by urinary frequency, urgency or nocturia
  • Often associated with extrapelvic pain, other systemic pain syndromes, somatic disorders and psychosocial difficulties / dysfunction (Nat Rev Urol 2019;16:187)
  • Interstitial cystitis with Hunner lesions are associated with more severe bladder symptoms (smaller bladder capacity and higher bladder pain), whereas the non-Hunner lesion subtype correlates with nonbladder syndromes (J Urol 2021;205:226, Urology 2011;78:301)
Diagnosis
  • Clinical diagnosis of exclusion without any invasive investigation required
  • Cystoscopy with or without cytology and biopsy / resection can be performed as clinically warranted to rule out other underlying pathology, especially in older patients and to subtype interstitial cystitis in the context of persistent, debilitating bladder pain syndrome (Neurourol Urodyn 2017;36:984, Can Urol Assoc J 2016;10:E136)
Laboratory
  • Basic laboratory investigations, including urine analysis and culture, are negative for confusable diseases
  • Many patients have hematuria (J Psychosom Res 2014;77:510)
Radiology description
  • Pelvic imaging is unnecessary for the diagnosis of interstitial cystitis (BJU Int 2018;122:729)
  • Diffuse and focal bladder thickening on CT is significantly more frequent in patients with Hunner lesion subtype than in patients with non-Hunner lesion subtype (Biomedicines 2021;9:1306)
Radiology images

Images hosted on other servers:
Bladder CT

Bladder CT

Prognostic factors
  • Patterns described: complete resolution, relapsing remitting, intermittent disease flares, chronic progression (Transl Androl Urol 2015;4:629)
  • Patients with no positive histopathological finding (non-Hunner lesion subtype) have worse or unchanged outcomes from intravesical therapy compared with those who have at least one histopathological finding consistent with Hunner lesion subtype (J Urol 2021;205:226)
  • No association between individual histopathological findings and treatment outcomes in the Hunner lesion subtype
  • No impact on survival
Case reports
  • 15 year old girl with ulcerative interstitial cystitis treated with complete transurethral ulcer resection (IJU Case Rep 2018;2:51)
  • 35 year old woman with interstitial cystitis and persistent arousal treated with combined site specific sacral neuromodulation and pudendal nerve release surgery (BMJ Case Rep 2016;2016:bcr2015213513)
  • 67 year old woman with bladder pain syndrome caused by iliohypogastric and ilioinguinal nerve injuries following a laparoscopic pyeloplasty (Urol Case Rep 2019;28:101056)
Treatment
  • Initial treatment for the non-Hunner lesion subtype should be conservative; however, cystoscopic techniques represent the first line treatment in the Hunner lesion subtype (J Urol 2022;208:34, Int J Urol 2016;23:542)
  • Conservative: multimodal therapy (pain management, behavioral, psychological, educational), stress management, dietary advice, physiotherapy
  • Oral therapies: amitriptyline, cimetidine, hydroxyzine, oral pentosan polysulphate
  • Intravesical therapies: dimethylsulfoxide, heparin, lignocaine
  • Cystoscopic techniques: fulguration of ulcers, injection of triamcinolone to ulcers, hydrodistension
  • Other treatments: botulinum toxin A, sacral neuromodulation, cyclosporin A
  • Major surgery (option of last resort): urinary diversion or substitution cystoplasty with or without cystectomy
Clinical images

Images hosted on other servers:
Cystoscopic findings

Cystoscopic findings

Gross description
  • Bladder mucosa is unremarkable or with Hunner lesions or glomerulations (Eur Urol 2008;53:60)
    • Hunner lesion (5 - 57% of cases): reddened mucosal area with small vessels that radiate towards a central scar, with fibrin deposition or an attached blood clot; can present as a denuded urothelium with oozing blood due to rupture and bladder distension (Int J Urol 2019;26:26)
    • Glomerulations (nonspecific): petechial mucosal hemorrhages (J Urol 2016;195:19)
  • Bladder wall is smooth or with focal to diffuse thickening
Gross images

Images hosted on other servers:
Inflamed mucosa

Inflamed mucosa

Microscopic (histologic) description
  • Histologic features are nonspecific
  • Features suggestive of the Hunner lesion subtype (at least one positive histological finding) (Int J Urol 2020;27:491, J Transl Med 2022;20:97, PLoS One 2015;10:e0143316, Scand J Urol 2020;54:91, Biomedicines 2021;9:1306)
    • Pancystitis with subepithelial lymphoplasmacytic inflammatory infiltrate with frequent lymphoid aggregates (B cell clonal expansion) with or without monocytes / macrophages or granulocytes
    • Urothelial detachment / denudation with or without fibrin deposition with or without granulation tissue
    • Lamina propria with hemorrhage, neovascularization, fibrosis or edema
    • Detrusor mastocytosis or fibrosis
  • Non-Hunner lesion subtype
    • No or scarce histopathological changes
    • Noninflammatory with preserved urothelium
  • Patients with diffuse bladder wall thickening on CT correlates with greatest inflammatory cell infiltration, urothelial denudation and granulation tissue, as well as obvious fibrosis in both superficial and deep lamina propria
    • Patients with focal bladder wall thickening have fewer inflammatory and reactive changes and typically demonstrate obvious fibrosis only in the deep lamina propria (Biomedicines 2021;9:1306)
Microscopic (histologic) images

Contributed by Michelle R. Downes, M.D.
Hemorrhage and edema

Hemorrhage and edema

Urothelial denudation and inflammation

Urothelial denudation and inflammation

Mixed inflammatory infiltrate

Mixed inflammatory infiltrate

Surface ulceration

Surface ulceration

Inflammation, microhemorrhages and congestion

Inflammation,
microhemorrhages
and congestion

Hemorrhage and fibrosis

Hemorrhage and fibrosis

Positive stains
Molecular / cytogenetics description
Sample pathology report
  • Bladder, biopsy:
    • Urothelial denudation with lamina propria edema, congestion and inflammation (lymphoid aggregates, plasma cells and granulocytes) (see comment)
    • Muscularis propria sampled
    • Comment: The histological features are not specific but would be in keeping with interstitial cystitis in the appropriate clinical context.

  • Bladder, cystectomy:
    • Inflammatory process with features in keeping with the clinical impression of interstitial cystitis (see comment and microscopic description)
    • Negative for malignancy
    • Comment: The histological features in this condition can be nonspecific but the findings in the current specimen would be in keeping with the clinical impression of interstitial cystitis.
    • Microscopic description: Bladder with marked urothelial denudation, vascular congestion and red cell extravasation. The lamina propria is markedly edematous and in areas polypoid, with focal surface fibrin deposition in keeping with ulceration. There is marked inflammation, predominantly chronic (lymphocytes and plasma cells) with neutrophils and some mast cells within the lamina propria. The muscularis propria is uninvolved by either inflammation or fibrosis.
Differential diagnosis
  • Includes other types of cystitis with nonspecific histopathological findings, such as
    • Drug or radiation mediated cystitis:
      • Urothelium with cellular enlargement, multinucleation, nuclear hyperchromasia and cytoplasmic vacuolation
      • Other classic findings include hemorrhage, fibrin deposition, fibrin thrombi, fibrosis, acute and chronic inflammation, edema, vascular congestion and thickened vessels
    • Infectious cystitis:
      • Neutrophilic inflammation
    • Follicular cystitis:
      • Lymphoid follicles within the urothelium and lamina propria
    • Granulomatous cystitis:
      • Granulomas in the lamina propria
      • Can have surface ulceration and chronic inflammation
    • Eosinophilic cystitis:
      • Prominent eosinophilic infiltrate in the acute phase
      • Fewer eosinophils but more prominent mast cells, plasma cells and muscle fibrosis in the chronic phase
    • Polypoid / papillary cystitis:
      • Exophytic bullous, polypoid or papillary structures covered by benign urothelium and composed of stromal cores
      • Varying degree of edema, vascular ectasia, congestion, inflammatory infiltrate and fibrosis
  • Clinical correlation is required
Board review style question #1

A 50 year old woman, known for fibromyalgia with otherwise unremarkable past medical history, has been having chronic pelvic pain with urinary frequency for 6 months. Recently, she has been having intermittent, mild hematuria. Physical examination and laboratory investigations, including urine analysis and culture, as well as abdominal / pelvic ultrasound, are negative. A cystoscopy was performed and showed reddened mucosal areas with small vessels radiating towards a central scar, which were biopsied. No mass or lesion suspicious for malignancy was seen on cystoscopy and urine cytology was negative. Bladder biopsies showed the histology above. What is the most likely diagnosis?

  1. Bladder stones
  2. Infectious cystitis
  3. Interstitial cystitis
  4. Lymphoma
  5. Papillary urothelial neoplasm
Board review style answer #1
C. Interstitial cystitis. Chronic pelvic pain with urinary frequency as well as intermittent mild hematuria can be seen in the Hunner lesion subtype of interstitial cystitis. Confusable diseases such as infectious cystitis and malignancy are ruled out, with negative urine analysis and culture as well as the absence of cytologic atypia and architectural disorder on pathology. Lesions characteristic of Hunner lesion (reddened mucosal areas with small vessels radiating towards a central scar) visualized on cystoscopy. Histopathology showing urothelial denudation and lamina propria inflammation are suggestive of the Hunner lesion subtype of interstitial cystitis.

Answer A is incorrect because no stones were detected on ultrasound and on cystoscopy, which cannot explain the persistent pelvic pain with urinary frequency. Answer B is incorrect because urine analysis and culture are negative. Answer D is incorrect because there is no diffuse infiltrate of atypical lymphoid cells seen on pathology. Answer E is incorrect because pathology shows no cytologic atypia and architectural disorder.

Comment Here

Reference: Interstitial cystitis
Board review style question #2
Pathological subtyping of interstitial cystitis is nonessential for diagnostic purposes but can guide treatment and aid in prognostication. What are the 2 subtypes of interstitial cystitis?

  1. With / without atypia
  2. With / without fibrosis
  3. With / without glomerulation
  4. With / without Hunner lesion
  5. With / without ulceration
Board review style answer #2
D. With / without Hunner lesion. Interstitial cystitis with Hunner lesions shows nonspecific histopathological features including but not limited to pancystitis with suburothelial lymphoplasmacytic inflammation and urothelial denudement. The non-Hunner lesion subtype shows no diagnostic pathology. The Hunner lesion subtype is associated with more severe bladder symptoms and better outcomes from intravesical therapy compared with the the non-Hunner lesion subtype.

Answer A is incorrect because no atypia is seen in interstitial cystitis. Answer B is incorrect because fibrosis can be seen in both the Hunner lesion subtype and non-Hunner lesion subtype of interstitial cystitis, although it is usually absent or scarce in the non-Hunner lesion subtype. Fibrosis is a nonspecific finding in interstitial cystitis and has no predictive or prognostic value. Answer C is incorrect because glomerulations are nonspecific findings in interstitial cystitis and have no predictive or prognostic value. Answer E is incorrect because ulceration can be found in some cases of the Hunner lesion subtype of interstitial subtype. It is a nonspecific finding.

Comment Here

Reference: Interstitial cystitis

Intestinal metaplasia
Definition / general
  • Replacement of urothelium by benign colonic or small intestinal epithelium, with presence of mucin producing goblet cells
    Essential features
    • Intestinal or colonic metaplasia: glandular epithelium with mucin secreting goblet cells (single or aggregates) replacing surface mucosa and von Brunn nests
    • May coexist with cystitis glandularis, characterized by formation of glandular structures in lamina propria within von Brunn nests that have an innermost lining of columnar or cuboidal cells bound by transitional cells at periphery
    Terminology
    • Also called glandular metaplasia, colonic metaplasia or goblet cell metaplasia
    ICD coding
    • ICD-10:
      • N30.80 - other cystitis without hematuria
      • N30.81 - other cystitis with hematuria
    Epidemiology
    • Identified in 71 - 90% of bladders at autopsy (Acta Pathol Jpn 1981;31:545)
    • Incidence increases with age; mean age of diagnosis 57 years (range 23 - 81 years)
    Sites
    • Lamina propria of trigone is most affected (Mod Pathol 2006;19:1395)
    • Also seen in ureter and renal pelvis
    • Mostly focal process but occasionally diffuse
    Pathophysiology
    • Metaplastic reactive alteration induced by chronic inflammation or irritation
    Etiology
    • Variation of normal histology
    • Chronic inflammation, ureteral reimplantation, neurogenic bladder, bladder exstrophy, paraplegia, stones, long term catheterization (J Urol 1981;126:822, J Urol 1970;104:699)
    Clinical features
    • Incidental finding
    • Hematuria, dysuria, urgency or obstructive symptoms
    • Rare cases may form mass lesion (< 1 cm) with mucin extravasation; may be confused cystoscopically with neoplastic process or malignancy (Am J Surg Pathol 1996;20:1462)
    Diagnosis
    • Cystoscopy finding of irregular or minimally elevated mucosa in a symptomatic patient
    • Microscopic findings of intestinal type epithelium with goblet cells replacing urothelium
    Prognostic factors
    • Long term follow up of intestinal metaplasia has confirmed a benign behavior (Urology 2008;71:915, Urology 1997;50:427)
    • If associated with dysplastic changes, considered as a harbinger of adenocarcinoma; close follow up is warranted (Clin Cancer Res 2007;13:6232)
    • If the lesion is extensive on initial transurethral resection, it is prudent to suggest area to be re-resected
    Case reports
    • 20 year old man with adenocarcinoma arising in residual native bladder in association with intestinal metaplasia and dysplasia of bladder mucosa, 17 years following gastrocystoplasty (J Pediatr Urol 2010;6:525)
    • 49 year old man with hematuria and radiologic abnormalities with no visible lesions (J Surg Case Rep 2018;2018:rjy193)
    • 60 year old man with spina bifida with bladder adenocarcinoma arising in intestinal metaplasia (Ann Diagn Pathol 2007;11:453)
    Treatment
    • May regress completely if pathogenic factor is removed
    • Frequent monitoring if symptoms persist or if associated with dysplasia
    Gross description
    Microscopic (histologic) description
    • Replacement of urothelium by tall columnar cells
    • Presence of goblet cells (single or aggregates) within von Brunn nests; variable presence of Paneth cells (small, intestine-like) or neuroendocrine cells (rare) (Mod Pathol 2006;19:1395)
    • May coexist with cystitis cystica or cystitis glandularis
    • Mucin may be occasionally extravasated into the stroma
    • No atypia, confined to the lamina propria, no involvement of muscularis propria
    Microscopic (histologic) images

    Contributed by Maria Tretiakova, M.D., Ph.D.
    Replacement of urothelium

    Replacement of urothelium

    Without dysplastic changes

    Without dysplastic changes

    Formation of cystic spaces

    Formation of cystic spaces

    GATA3

    GATA3


    CK20

    CK20

    CK20

    CK20

    CK20

    CK20

    HMWCK

    HMWCK



    Contributed by Rugvedita Parakh, M.D.
    Replacing urothelium

    Replacing urothelium

    Surface goblet cells

    Surface goblet cells

    Cytology description
    Positive stains
    Negative stains
    • CK7 (usually), HepPar1 (100%)
    • GATA3, HMWCK
    • Note: in the urinary bladder, intestinal metaplasia and typical cystitis glandularis have sharply contrasting immunoprofiles despite similar morphology (Mod Pathol 2006;19:1395)
    Electron microscopy description
    • Cystitis cystica:
    • Not needed for routine diagnostic purposes
    Molecular / cytogenetics description
    Sample pathology report
    • Bladder, biopsy:
      • Bladder mucosa with intestinal metaplasia (see comment)
      • Comment: No evidence of dysplasia or malignancy in submitted tissue.
    Differential diagnosis
    Board review style question #1

    The term intestinal metaplasia is recommended over cystitis glandularis to denote the presence of which of the following?

    1. Columnar cells
    2. Cuboidal cells
    3. Goblet cells
    4. Paneth cells
    Board review style answer #1
    C. Goblet cells. According to updated criteria, lesions containing mucin secreting goblet cells are classified as intestinal metaplasia and the term cystitis glandularis is discouraged.

    Comment Here

    Reference: Intestinal metaplasia
    Board review style question #2
    Which of the following features is considered to be a feature of intestinal metaplasia?

    1. Dysplasia (cytologic atypia)
    2. Invasion of muscularis propria
    3. Paneth cells
    4. Presence of dissecting mucin
    Board review style answer #2
    C. The presence of Paneth cells is a small intestinal type of metaplastic change. The other features, such as the presence of dissecting mucin, cytologic atypia and invasion of muscularis propria, are helpful to evaluate presence of dysplastic changes or malignancy (well differentiated adenocarcinoma).

    Comment Here

    Reference: Intestinal metaplasia

    Invasive urothelial carcinoma
    Definition / general
    • Urothelial carcinoma that has penetrated the basement membrane and invaded into the lamina propria or deeper
    Essential features
    • Histologic characterization and depth of invasion are the most important factors for determining prognosis (Adv Anat Pathol 2015;22:102)
    • Urothelial carcinoma is morphologically heterogenous with many variants and subtypes (Surg Pathol Clin 2018;11:713)
    • Invasive urothelial carcinoma involving the lamina propria (T1) is often treated with conservative intravesical therapy and mucosal resection (Ann Diagn Pathol 2007;11:395)
    • Invasive urothelial carcinoma involving the muscularis propria (T2) is often treated with radical cystectomy (Ann Diagn Pathol 2007;11:395)
    Terminology
    • Invasive transitional cell carcinoma (historical term)
    • WHO classification 2016:
      • Infiltrating urothelial carcinoma
      • Urothelial carcinoma with divergent differentiation
        • With squamous differentiation
        • With glandular differentiation
        • With trophoblastic differentiation
      • Nested urothelial carcinoma
        • Including large nested variant
      • Microcystic urothelial carcinoma
      • Micropapillary urothelial carcinoma
      • Lymphoepithelioma-like urothelial carcinoma
      • Plasmacytoid / signet ring / diffuse urothelial carcinoma
      • Sarcomatoid urothelial carcinoma
      • Giant cell urothelial carcinoma
      • Poorly differentiated urothelial carcinoma
      • Lipid rich urothelial carcinoma
      • Clear cell (glycogen rich) urothelial carcinoma
    ICD coding
    • ICD-O:
      • 8120/3 - infiltrating urothelial carcinoma
      • 8131/3 - micropapillary urothelial carcinoma
      • 8082/3 - lymphoepithelioma-like urothelial carcinoma
      • 8122/3 - sarcomatoid urothelial carcinoma
      • 8031/3 - giant cell urothelial carcinoma
      • 8020/3 - poorly differentiated urothelial carcinoma
    • ICD-10: C67.9 - malignant neoplasm of bladder, unspecified
    Epidemiology
    • Fourth most common malignancy in U.S. men
    • Eight most common malignancy associated with death in U.S. men
    • Median age: 69 years in men and 71 years in women (Adv Anat Pathol 2015;22:102)
    • Incidence:
      • 4x higher in men than women
      • 2x higher in white men than in black men
    Sites
    Pathophysiology
    • Smoking or other chemical exposures start the process of carcinogenesis by altering the DNA of the bladder mucosa; these genetic alterations lead to unregulated cell growth (Adv Anat Pathol 2019;26:251)
    Etiology
    Clinical features
    • Hematuria, irritation / pain during urination, increased urinary frequency or urgency (Urology 2006;67:3)
    Diagnosis
    Radiology description
    • CT image findings include urothelial thickening / hyperenhancement, asymmetric collecting duct system, urothelial calcifications or the presence of a nodule / mass (Urol Clin North Am 2018;45:389)
    • MRI T2 weighted images demonstrate hypointense thickening of bladder wall (Abdom Radiol (NY) 2019;44:3858)
    Radiology images

    Images hosted on other servers:
    CT with bladder thickening

    CT with bladder thickening

    CT and MRI urothelial carcinoma

    Bladder tumor and lung metastasis

    Retrograde pyelogram

    Retrograde pyelogram

    Pyeloureteral transitional cell

    Pyeloureteral transitional cell

    Prognostic factors
    Case reports
    Treatment
    Clinical images

    Images hosted on other servers:
    Cystoscopy pedunculated tumor

    Cystoscopy: pedunculated tumor

    Gross description
    Gross images

    Contributed by Debra L. Zynger, M.D., Nicole K. Andeen, M.D. and Maria Tretiakova, M.D.
    Muscularis propria invasion (pT2b) Muscularis propria invasion (pT2b)

    Muscularis propria invasion (pT2b)

    Prostatic invasion (pT4a)

    Prostatic invasion (pT4a)

    Renal pelvic and peripelvic fat invasion (pT3)

    Renal pelvic and peripelvic fat invasion (pT3)

    Friable, exophytic papillary mass, renal pelvis

    Friable, exophytic
    papillary mass,
    renal pelvis

    Frozen section description
    • Positive margin is defined by the presence of in situ or invasive urothelial carcinoma on frozen section (Ann Diagn Pathol 2015;19:107)
    • Ureteric intraoperative frozen sections have significant clinical limitations due to skip lesions, which often occur in multifocal urothelial carcinoma in situ (J Clin Pathol 2010;63:475)
    Frozen section images

    Images hosted on other servers:
    Urothelial carcinoma in situ in frozen section

    Urothelial carcinoma in situ in frozen section

    Microscopic (histologic) description
    Microscopic (histologic) images

    Contributed by Maria Tretiakova, M.D., Ph.D., Andrey Bychkov, M.D., Ph.D. and Nicole K. Andeen, M.D.
    Invasive high grade urothelial carcinoma

    Invasive high grade urothelial carcinoma

    Extensive invasion into lamina propria

    Extensive invasion into lamina propria

    Invasive urothelial carcinoma with comedo necrosis

    Invasive urothelial
    carcinoma with
    comedo necrosis

    Lymphovascular invasion

    Lymphovascular invasion


    Muscularis propria invasion Muscularis propria invasion Muscularis propria invasion

    Muscularis propria invasion

    IHC profile

    IHC profile


    Subepithelial invasion characterized by jagged, irregular nests and a desmoplastic response (pT1)

    Subepithelial invasion (pT1)

    Invasion into muscularis (pT2)

    Invasion into muscularis (pT2)

    Invasion of renal parenchyma (pT3) Invasion of renal parenchyma (pT3)

    Invasion of renal parenchyma (pT3)

    Cytology description
    Cytology images

    Contributed by Bonnie Choy, M.D.
    High grade urothelial carcinoma High grade urothelial carcinoma High grade urothelial carcinoma High grade urothelial carcinoma High grade urothelial carcinoma

    High grade urothelial carcinoma



    Contributed by Nicole K. Andeen, M.D. and Maria Tretiakova, M.D.
    Highly atypical cells

    Highly atypical cells

    Large nuclei with high nuclear to cytoplasmic ratios, coarse chromatin and irregular nuclear contours (Papanicolaou and DiffQuik) Large nuclei with high nuclear to cytoplasmic ratios, coarse chromatin and irregular nuclear contours (Papanicolaou and DiffQuik) Large nuclei with high nuclear to cytoplasmic ratios, coarse chromatin and irregular nuclear contours (Papanicolaou and DiffQuik)

    Large nuclei with high nuclear to cytoplasmic ratios, coarse chromatin and irregular nuclear contours (Papanicolaou and DiffQuik)

    Positive stains
    Molecular / cytogenetics description
    • Most commonly mutated genes are associated with cell cycle progression, including TP53, PIK3CA, RB1 and FGFR3 (Nature 2014;507:315)
    • UroVysion is an FDA approved fluorescent in situ hybridization (FISH) that detects aneuploidy in chromosomes 3, 7 and 17 and loss of 9p21 locus (72% sensitivity and 83% specificity) (BJU Int 2013;112:E372, Curr Opin Urol 2019;29:203)
    • Nuclear matrix proteins (NMP22): group of proteins produced by cancerous cells
      • NMP22 BladderChek test is an FDA approved qualitative assay for surveillance and detection of bladder cancer (50% sensitivity and 87% specificity) (Curr Opin Urol 2019;29:203)
    • Bladder tumor antigen (BTA): antigen produced by cancerous cells
      • BTA Trak (quantitative) and BTA Stat (qualitative) are FDA approved assays for surveillance (68 - 77.5% sensitivity and 50 - 75% specificity) (Curr Opin Urol 2019;29:203)
    Molecular / cytogenetics images

    Contributed by Nicole K. Andeen, M.D. and Maria Tretiakova, M.D.
    FISH, loss of 9p, normal disomy chromosomes 3, 7, 17

    FISH, loss of 9p, normal disomy chromosomes 3, 7, 17

    FISH, polyploidy chromosomes 3, 7, 17; 9p preserved

    FISH, polyploidy chromosomes 3, 7, 17; 9p preserved

    Sample pathology report
    • Bladder, TURB:
      • Urothelial carcinoma, high grade (see comment)
      • Type / grade comment: micropapillary (20%)
      • Associated lesions: carcinoma in situ
      • Extent: lamina propria invasion, extensive (pT1)
      • Angiolymphatic invasion: present
      • Muscularis propria: present, uninvolved
    Differential diagnosis
    Board review style question #1
    A 76 year old man with gross hematuria was found to have invasive urothelial carcinoma on transurethral biopsy. Which of the following is not a risk factor for urothelial carcinoma but is a risk factor for developing squamous cell carcinoma?

    1. Tobacco smoke
    2. Polycyclic aromatic hydrocarbon exposure
    3. Benzidine exposure
    4. Indwelling catheter
    Board review style answer #1
    D. Indwelling catheter

    Comment Here

    Reference: Invasive urothelial carcinoma
    Board review style question #2
    Which of the following histopathologic subtypes of urothelial carcinoma is most likely to be associated with a worse prognosis?

    1. Clear cell urothelial carcinoma
    2. Microcystic urothelial carcinoma
    3. Conventional high grade urothelial carcinoma
    4. Plasmacytoid urothelial carcinoma
    Board review style answer #2
    D. Plasmacytoid urothelial carcinoma

    Comment Here

    Reference: Invasive urothelial carcinoma
    Board review style question #3

    An 85 year old man has a mass involving the prostate and bladder shown above. Which of the following findings is most supportive of invasive urothelial carcinoma over prostatic adenocarcinoma?

    1. Sheets of monotonous cells with amphiphilic cytoplasm and prominent nucleoli
    2. Negative for PSA and CK7
    3. Positive for GATA3 and HMWCK
    4. Focus of squamous differentiation
    Board review style answer #3
    C. Positive for GATA3 and HMWCK immunohistochemical stains

    Comment Here

    Reference: Invasive urothelial carcinoma

    Inverted growth pattern
    Definition / general
    • Inverted (endophytic) growth pattern can be associated with papilloma, urothelial neoplasm of low malignant potential, low / high grade tumors that are noninvasive or invasive; this topic discusses invasive tumors
    • Anastomosing cords and columns of urothelium resembling inverted papilloma or broad pushing bulbous invaginations into lamina propria (broad front pattern), but with definitive evidence of invasion
    • Not a WHO diagnosis
    Epidemiology
    • Mean age 68 years
    • 75% male
    Clinical features
    • Pattern present in 11% of urothelial carcinomas
    • Associated with high grade and high stage tumors, and possibly poorer prognosis (Urol Oncol 2012;30:49)
    Microscopic (histologic) description
    • Anastomosing cords and columns of urothelium resembling inverted papilloma or broad pushing bulbous invaginations into lamina propria (broad front pattern)
    • The trabecuale are wider and more irregular than those of inverted papilloma
    • Invasive if there is irregularity of broad front of bulbous invaginations, retraction artifact, desmoplasia or other features of destructive invasion
    • Usually exophytic papillary urothelial carcinoma is also present
    • Substantial nuclear pleomorphism, readily apparent mitotic figures and architectural abnormalities
    • At least 25% of tumor should have an inverted component to be considered as inverted urothelial carcinoma (BJU Int 2011;107:532)
    • Findings suggestive of invasion – irregularly shaped nests with disruption or absence of the basement membrane, desmoplasia or fibrotic stromal response (note that microinvasion usually does not elicit a stromal response, which makes its identification more difficult)
    • Pattern is associated with invasion in 50% of papillary urothelial carcinomas (Am J Surg Pathol 1997;21:1057)
    • Urothelial carcinoma in situ, if present in the surface urothelium, provides support for a diagnosis of inverted urothelial carcinoma
    Differential diagnosis

    Inverted urothelial papilloma
    Definition / general
    • Inverted urothelial papilloma is a benign urothelial neoplasm with an endophytic growth of complex and interconnecting trabeculae
    Essential features
    • Endophytic proliferation of normal thickness urothelium forming anastomosing cords, islands and trabeculae (Mod Pathol 2015;28:612)
    • Consists of uniformly bland urothelial cells with preserved polarity and peripheral palisading lacking cytologic atypia
    Terminology
    • Inverted urothelial papilloma
    • Inverted papilloma of the urinary bladder
    • Brunnian adenoma (Hum Pathol 1978;9:229)
    ICD coding
    • ICD-O
      • 8121/0 - transitional cell papilloma, inverted, benign
      • 8121/1 - transitional (cell) papilloma, inverted, NOS
    • ICD-10: D30.3 - benign neoplasm of bladder
    • ICD-11
      • 2F35 & XH5A08 - benign neoplasm of urinary organs & urothelial papilloma, inverted
      • XH3HQ8 - transitional papilloma, inverted, NOS
    Epidemiology
    Sites
    • Urinary bladder: bladder neck (most common, 41%), trigone, lateral wall, posterior wall, some tumors multifocal (Urol Oncol 2013;31:1584)
    • Upper urinary tract and urethra
    Pathophysiology
    • RAS-ERK pathway activation
      • Mutations of HRAS or KRAS activating nuclear transcription of proteins and promoting cell growth, cell cycle progression and proliferation (J Pathol 2019;249:3)
    • No TERT promotor or FGFR3 mutations
    Etiology
    Clinical features
    Diagnosis
    Radiology description
    Radiology images

    Images hosted on other servers:

    Left ureter filling defect

    Prognostic factors
    Case reports
    Treatment
    Clinical images

    Images hosted on other servers:

    Tumor with smooth contour

    Gross description
    • Raised, sessile, pedunculated or polypoid mass with smooth contour
    • Single or multiple (1.3 - 4.4%) (J Urol 1996;155:1391)
    • Size ranges from 1 to 50 mm (mean of 12.8 mm), measured up to 8 cm (BJU Int 2011;107:532)
    Gross images

    Images hosted on other servers:

    Polypoid mass

    Microscopic (histologic) description
    • Sharply circumscribed, endophytic proliferation of thin and complex anastomosing cords, islands and trabeculae of cytologically bland urothelial cells with virtually no nuclear atypia (Mod Pathol 2015;28:612)
    • Invaginating trabeculae composed of 5 - 10 layers of urothelial cells with central streaming and peripheral palisading cells, embedded in lamina propria
    • Surface is lined by the normal urothelium with no or minimal exophytic papillary component
    • Urothelial cells with vacuolation and foamy xanthomatous cytoplasmic changes may be seen (Hum Pathol 2006;37:1577)
    • Mild degenerative cytologic atypia may be seen
    • Mitotic figures are absent or extremely rare
    Microscopic (histologic) images

    Contributed by Y. Albert Yeh, M.D., Ph.D., Daniel Athanazio, M.D., Ph.D. and Debra Zynger, M.D.

    Smooth surface contour

    Invagination of trabeculae

    Interconnecting trabeculae and cords

    Endophytic growth

    Bland urothelial cells


    Spindle urothelial cells

    Central streaming / peripheral palisading

    Focal mild reactive atypia

    Inverted papilloma

    Inverted papilloma

    Inverted papilloma – anastomosing cords

    Inverted papilloma - anastomosing cords


    Thickened epithelium

    Thickened epithelium

    Epithelium / no atypia

    Epithelium / no atypia

    Inverted PUNLMP

    Inverted papilloma

    Inverted PUNLMP - no atypia

    Inverted papilloma - no atypia


    Surface with thin, flat urothelium

    Anastomosing, basophilic cords

    Oval nuclei

    Immunofluorescence description
    • UroVysion FISH
      • Negative (0%) for gain of chromosomes 3, 7, 17 or loss of 9p21 in inverted urothelial papilloma versus positive (72%) in inverted pattern urothelial carcinoma (Am J Surg Pathol 2007;31:1861)
    Positive stains
    Negative stains
    Molecular / cytogenetics description
    Sample pathology report
    • Urinary bladder, neck, biopsy:
      • Inverted urothelial papilloma (see comment)
      • Comment: The bladder lesion shows an invaginating polypoid proliferation of anastomosing cords and trabeculae with peripheral palisading of cells. There is central streaming of urothelial cells. No exophytic components are noted. The tumor cells are characterized by small, uniform and cytologically bland nuclei. Mitotic figure is not seen. These features are consistent with an inverted urothelial papilloma.
    Differential diagnosis
    Board review style question #1

    A 74 year old man presented to the urology clinic with hematuria and dysuria. Cystoscopic examination showed a polypoid mass with smooth surface in the bladder neck. A transurethral bladder tumor resection was performed. An image of the histopathological examination is shown above. What is the diagnosis?

    1. Florid cystitis cystica proliferation
    2. Florid von Brunn nest proliferation
    3. Inverted high grade papillary urothelial carcinoma
    4. Inverted urothelial papilloma
    5. Paraganglioma
    Board review style answer #1
    D. Inverted urothelial papilloma. This image shows an inverted urothelial papilloma characterized by invaginating growth of anastomosing trabeculae with central streaming of bland urothelial cells. Answers B and A are incorrect because florid von Brunn nests and cystitis cystica are composed of large nests of urothelial cells arranged in a lobular pattern with regular spacing. There is no trabecular formation. Answer C is incorrect because inverted high grade papillary urothelial carcinoma is composed of irregular and complex anastomosing cords and trabeculae with disorganized, pleomorphic and hyperchromatic urothelial cells. Answer E is incorrect because no zellballen pattern of cells is seen.

    Comment Here

    Reference: Inverted urothelial papilloma
    Board review style question #2

    A 68 year old man presented to the urology clinic with lower urinary tract obstructive symptoms. Cystoscopic examination showed a polypoid mass with smooth contour in the posterior wall of the bladder. A transurethral bladder tumor resection was performed. An image of the histopathological examination showed an endophytic growth with tumor cells stained positive for immunomarkers GATA3 and p63. A histopathological image is shown above. What is the diagnosis?

    1. Carcinoid tumor
    2. Inverted high grade papillary urothelial carcinoma
    3. Inverted low grade papillary urothelial carcinoma
    4. Inverted urothelial papilloma
    5. Leiomyoma
    Board review style answer #2
    D. Inverted urothelial papilloma. The tumor is composed of trabeculae of urothelial cells (positive for GATA3 and p63) with central streaming and peripheral palisading of cells. Answer E is incorrect because is incorrect because leiomyoma is negative for GATA3 and has very focal and weak cytoplasmic staining for p63. Answers B and C are incorrect because the urothelial cells in both inverted low grade and high grade papillary urothelial carcinoma show marked dispolarity of cells with moderate to severe nuclear pleomorphism and hyperchromasia. Answer A is incorrect because carcinoid tumor cells do not have fine and coarse chromatin. Neuroendocrine cells are stained negative for GATA3, although p63 may be weakly or focally positive.

    Comment Here

    Reference: Inverted urothelial papilloma

    Large cell neuroendocrine carcinoma
    Definition / general
    • Extremely rare carcinoma composed of large cells with prominent nucleoli and neuroendocrine differentiation
    Essential features
    • High grade carcinoma with typical large cell neuroendocrine carcinoma (LCNEC) histology and positivity for ≥ 1 neuroendocrine immunohistochemical marker(s) (e.g., synaptophysin, chromogranin, CD56, INSM1, etc.)
    • Criteria used to define LCNEC are similar to classifying the pulmonary counterpart, including (Adv Anat Pathol 2008;15:218)
      • Neuroendocrine appearance by light microscopy
      • Large polygonal cells with low N:C ratio
      • Coarse chromatin and frequent prominent nucleoli
      • High mitotic rate of > 10 mitoses per 10 high power fields accompanied by areas of necrosis
      • Immunohistochemical or ultrastructural evidence of neuroendocrine differentiation
    Terminology
    • Large cell neuroendocrine carcinoma
    ICD coding
    • ICD-O: 8013/3 - large cell neuroendocrine carcinoma
    • ICD-10:
      • C64.1 - malignant neoplasm of right kidney, except renal pelvis
      • C64.2 - malignant neoplasm of left kidney, except renal pelvis
      • C66.1 - malignant neoplasm of right ureter
      • C66.2 - malignant neoplasm of left ureter
      • C67.0 - malignant neoplasm of trigone of bladder
      • C67.1 - malignant neoplasm of dome of bladder
      • C67.2 - malignant neoplasm of lateral wall of bladder
      • C67.3 - malignant neoplasm of anterior wall of bladder
      • C67.4 - malignant neoplasm of posterior wall of bladder
      • C67.5 - malignant neoplasm of bladder neck
      • C67.6 - malignant neoplasm of ureteric orifice
      • C67.7 - malignant neoplasm of urachus
      • C67.8 - malignant neoplasm of overlapping sites of bladder
      • C67.9 - malignant neoplasm of bladder, unspecified
    • ICD-11:
      • 2C94.Y & XH0NL5 - other specified malignant neoplasms of bladder & large cell neuroendocrine carcinoma
      • 2C90.Y & XH0NL5 - other specified malignant neoplasms of kidney, except renal pelvis & large cell neuroendocrine carcinoma
      • 2C9Y & XH0NL5 - other specified malignant neoplasms of urinary tract & large cell neuroendocrine carcinoma
    Epidemiology
    Sites
    Pathophysiology
    Etiology
    • Unknown
    Clinical features
    Diagnosis
    • Diagnosis is made through identification of typical morphology and demonstration of neuroendocrine differentiation through immunohistochemistry
    • Rare neoplasm in the urinary tract which is essentially a diagnosis of exclusion
    Prognostic factors
    Case reports
    Treatment
    Gross description
    • Tan to white tumors with gritty or necrotic cut surfaces
    • In the bladder, median size is 4 cm (range: 1 - 9 cm) may be polypoid or nodular or less often flat or ulcerative and often invasive into the muscularis propria (Front Oncol 2020;10:1291)
    • In the kidney, tumors are often large (frequently > 100 mm in diameter) and commonly extend into the renal sinus or perirenal tissue (Indian J Urol 2009;25:274, Urologia 2014;81:57)
    Microscopic (histologic) description
    • Medium to large cell size with round to polygonal nuclei, low N:C ratio, vesicular / fine chromatin, frequent nucleoli and nuclear pleomorphism (Am J Surg Pathol 2021;45:1399, Mod Pathol 2009;22:S96)
    • Neuroendocrine growth pattern such as organoid, nesting, palisading, acini, trabeculae, sheet-like or solid growth pattern; peripheral palisading and rosettes may be occasionally seen
    • Usually > 10 mitosis per high power fields (Am J Surg Pathol 2021;45:1399)
    • Frequent necrosis, apoptotic bodies and often with lymphovascular invasion and lymph node metastasis
    • May be pure or mixed with other histologic subtypes; subtypes include conventional urothelial carcinoma, carcinoma in situ, small cell carcinoma, glandular, squamous, etc. or prostatic adenocarcinoma if from the prostate gland (Am J Surg Pathol 2021;45:1399, Mod Pathol 2009;22:S96)
      • Little guidance on terminology but when mixed, may be labeled as mixed tumors with types and percentages of neuroendocrine and nonneuroendocrine components
    Microscopic (histologic) images

    Contributed by Daniel Anderson, M.D., M.B.A.
    Solid growth

    Solid growth

    Necrosis and nuclear detail

    Necrosis and nuclear detail

    Mitotic figures

    Mitotic figures

    CD56

    CD56

    Ki67

    Ki67

    Synaptophysin

    Synaptophysin

    Cytology description
    • Similar to LCNEC of the lung displaying the following features:
      • Pleomorphic medium to large cells
      • Cells are round or polygonal in shape with abundant cytoplasm
      • Nuclei round, oval or polygonal with thin and smooth nuclear membranes
      • Chromatin finely or coarsely granular
      • Nucleoli may be prominent or inconspicuous
      • Cells may appear in clusters, rosettes or singly
      • Necrosis in background and nuclear streaking may be present (Lung Cancer 2005;48:331)
      • Nuclear pleomorphism, molding and mitosis, peripheral nuclear palisading and naked nuclei may also be present (Cancer 2008;114:180)
    Cytology images

    Images hosted on other servers:
    Prominent nucleoli shown in pleural fluid

    Prominent nucleoli shown in pleural fluid

    Positive stains
    Negative stains
    Molecular / cytogenetics description
    • Molecular features are unclear due to low prevalence
    Sample pathology report
    • Bladder, tumor, transurethral resection:
      • Invasive carcinoma with large cell neuroendocrine carcinoma features; the tumor invades the muscularis propria (detrusor muscle) (see comment)
      • Comment: Immunohistochemistry was performed. The carcinoma cells are positive for synaptophysin, chromogranin and ISM1 and negative for GATA3, NKX3.1, CDX2, p63, LCA (CD45), S100, TTF1, CDX2 and CK20. AE1 / AE3 and CK7 show focal to variable staining in the carcinoma cells. Features of primary conventional urothelial carcinoma and in situ carcinoma are not identified within this specimen. Clinical and radiologic correlation is required to exclude a metastasis. If this tumor is determined to be primary to the bladder, a definitive diagnosis of large cell neuroendocrine carcinoma requires review of the resection specimen.
    Differential diagnosis
    Board review style question #1

    A 63 year old man presents to the urologist with hematuria. Cystoscopy shows a 3 cm polypoid mass in the right lateral wall of the urinary bladder. The tumor is subsequently biopsied which shows a high grade tumor with prominent nucleoli, necrosis, abundant mitosis, low N:C ratio, neuroendocrine differentiation (synaptophysin, CD56, chromogranin positive) and TTF1 positivity by immunohistochemistry. Which is the most correct answer?

    1. Advanced stage is not common
    2. CK20 shows dot-like cytoplasmic positivity
    3. May display conventional urothelial carcinoma or in situ carcinoma
    4. Must be metastatic from the lung due to TTF1 positivity
    Board review style answer #1
    C. Primary large cell neuroendocrine carcinomas may display conventional urothelial carcinoma or in situ carcinoma. The question describes a large cell neuroendocrine carcinoma, a very rare neoplasm of the bladder and urinary tract. In one study, ~64% of urinary LCNEC were mixed with other histologic types of carcinoma, most commonly urothelial carcinoma or urothelial carcinoma in situ (Am J Surg Pathol 2021;45:1399). These tumors can closely mimic other carcinomas which involve the urinary tract including through metastasis. TTF1 can be positive in primary urothelial large cell neuroendocrine carcinoma and is not a specific marker to determine the origin of the tumor. LCNEC of the bladder usually is diagnosed in an advanced stage and has an overall poor prognosis. CK20 is usually not expressed by LCNEC. Dot-like cytoplasmic CK20 positivity is more common in metastatic Merkel cell carcinoma, a rare neoplasm of the skin, many of which are associated with infection from Merkel cell polyomavirus.

    Comment Here

    Reference: Large cell neuroendocrine carcinoma
    Board review style question #2

    Compared with small cell carcinoma of the urinary bladder, large cell neuroendocrine carcinoma will demonstrate which of the following?

    1. Have a high N:C ratio and no prominent nucleoli
    2. Show a worse prognosis when metastatic
    3. Not be mixed with other nonneuroendocrine histologic types
    4. Not express TTF1
    Board review style answer #2
    B. Show a worse prognosis when metastatic. Small cell carcinoma and large cell neuroendocrine carcinoma are the 2 recognized histologic subtypes of neuroendocrine carcinomas that are primary to the bladder. These tumors are often mixed with a nonneuroendocrine carcinoma component(s). In small cell carcinomas, the estimate of mixed cases varies from 32 to 67% (Cancer 2004;101:957, Hum Pathol 2018;79:57). The percent of large cell neuroendocrine carcinomas cases with mixed histology vary between 43 and 64% (Front Oncol 2020;10:1291, Am J Surg Pathol 2021;45:1399). According to one study, when compared with a cohort of small cell carcinoma cases of the bladder, LCNEC had a worse median overall survival (8.5 versus 29.0 months, p = 0.26) as well as a worse cancer specific survival (8.9 versus 53 months, p = 0.039) (Am J Surg Pathol 2021;45:1399). The cancer specific survival was not significantly different between LCNEC and small cell carcinoma in stage I through III; however, LCNEC had significantly shorter median survival time compared with small cell carcinoma for patients with distant metastasis (stage IV) (1.6 versus 15.0 months, p = 0.02). Like LCNEC, primary urinary bladder small cell carcinoma may demonstrate TTF1 expression / positivity (Hum Pathol 2005;36:718). Therefore, TTF1 is an unreliable marker to distinguish between a metastatic lung neuroendocrine carcinoma and a primary urinary bladder neuroendocrine carcinoma. Though there is some overlap between small cell carcinoma and LCNEC, as opposed to small cell carcinoma, LCNEC will show lower N:C ratio, prominent nucleoli, more cytoplasm and may show more diffuse rather than dot-like cytokeratin staining.

    Comment Here

    Reference: Large cell neuroendocrine carcinoma

    Lipid-rich
    Definition / general
    • An uncommon WHO recognized variant of invasive urothelial carcinoma
    • First described by Mostofi et al in 1999
    Terminology
    • Lipid cell, lipoid cell
    Epidemiology
    Etiology
    Diagnosis
    • Presence of cytokeratin and CK7 positive lipoblast-like cells and conventional urothelial carcinoma
    • Consistently negative for mucin (Am J Surg Pathol 2010;34:371)
    • Has areas of conventional UC, distinguishing it from a metastasis
    • May have heterologous liposarcomatous elements but lacks the prominent high grade spindled features of sarcomatoid carcinoma
    • Does not usually coexist with the sarcomatoid variant
    Case reports
    Gross description
    • Distinct macroscopic features are not well documented
    Microscopic (histologic) description
    Microscopic (histologic) images

    Contributed by Ankur Sangoi, M.D.
    Treatment
    • Similar to conventional high grade invasive urothelial carcinoma
    Positive stains
    Molecular / cytogenetics description
    • Shows identical loss of heterozygosity (LOH) patterns with the coexistent conventional UC, supporting a shared clonal origin (Am J Surg Pathol 2010;34:371)
    Differential diagnosis
    Board review style question #1
    Which of the following is true about lipid-rich variant of urothelial carcinoma?

    1. It is S100 and vimentin positive
    2. The lipid cell component usually comprises greater than 50% of the urothelial carcinoma
    3. The lipid cells are negative for panCK and CK7, contributing to diagnostic challenges with lipoblasts and liposarcoma
    4. The lipid cells express panCK and CK7 and are negative for mucin, distinguishing this entity from others in the differential diagnosis
    Board review style answer #1
    D. The lipid cells express panCK and CK7 and are negative for mucin, distinguishing this entity from others in the differential diagnosis. This and the presence of conventional high grade urothelial carcinoma are helpful diagnostic features in the differential diagnoses of lipid-rich variant of urothelial carcinoma with adenocarcinomas with signet ring features, sarcomatoid carcinoma and liposarcoma.

    Comment Here

    Reference: Lipid-rich urothelial carcinoma

    Lymphoepithelioma-like
    Definition / general
    Essential features
    • Subtype of invasive urothelial carcinoma that resembles lymphoepithelioma
    • Composed of sheets of undifferentiated, pleomorphic carcinoma cells with syncytial appearance, admixed with polymorphic inflammatory infiltrate
    Terminology
    • Also called lymphoepithelial carcinoma but most authors prefer -like in the name (i.e., lymphoepithelioma-like urothelial carcinoma) since it is EBV negative, in contrast to the nasopharyngeal tumor
    ICD coding
    • ICD-O: 8082/3 - lymphoepithelioma-like carcinoma
    • ICD-10:
      • C67 - malignant neoplasm of bladder
        • C67.0 - malignant neoplasm of trigone of bladder
        • C67.1 - malignant neoplasm of dome of bladder
        • C67.2 - malignant neoplasm of lateral wall of bladder
        • C67.3 - malignant neoplasm of anterior wall of bladder
        • C67.4 - malignant neoplasm of posterior wall of bladder
        • C67.5 - malignant neoplasm of bladder neck
        • C67.6 - malignant neoplasm of ureteric orifice
        • C67.7 - malignant neoplasm of urachus
        • C67.8 - malignant neoplasm of overlapping sites of bladder
        • C67.9 - malignant neoplasm of bladder, unspecified
      • C66.1 - malignant neoplasm of right ureter
      • C66.2 - malignant neoplasm of left ureter
      • C65.1 - malignant neoplasm of right renal pelvis
      • C65.2 - malignant neoplasm of left renal pelvis
    • ICD-11:
      • 2C91.0 & XH8EH1 - urothelial carcinoma of renal pelvis & transitional cell carcinoma, NOS
      • 2C92.0 & XH8EH1 - urothelial carcinoma of ureter & transitional cell carcinoma, NOS
      • 2C94.2 & XH8EH1 - urothelial carcinoma of bladder & transitional cell carcinoma, NOS
      • 2C93.2 & XH8EH1 - urothelial carcinoma of urethra or paraurethral gland & transitional cell carcinoma, NOS
      • 2C95.2 & XH8EH1 - urothelial carcinoma involving overlapping sites of urinary organs & transitional cell carcinoma, NOS
      • Note: transitional is a term that is no longer recommended; ICD-11 will be updated
    Epidemiology
    • M:F = 2.8:1; occurs in late adulthood, mean age of 70 years (range: 54 - 84 years) (Am J Surg Pathol 2011;35:474)
    • Rare subtype, first reported in 1991 in the bladder
    • Estimated incidence is 0.4 to 1.3% of all bladder carcinomas but has also been described in upper ureter and renal pelvis (Can Urol Assoc J 2013;7:E590)
      • In renal pelvis, virtually all reported cases from patients 65 and older
    Sites
    • Urinary tract
    • Has been described in the bladder, ureter, renal pelvis and urethra
    Etiology
    Clinical features
    Diagnosis
    • Diagnosis is made in a similar fashion to conventional urothelial carcinoma with cystoscopy and biopsy or TURBT of the lesion
    • Essentially an H&E diagnosis with demonstration of characteristic findings of pleomorphic cells with syncytial growth and associated inflammatory infiltrate
    Radiology description
    • Tumors are seen filling gaps within the urinary bladder, commonly with a broader base compared to transitional urothelial carcinomas (Cureus 2022;14:e21281)
    • May show enhancing soft tissue mass in the renal pelvis or ureter, involvement of the renal parenchyma or hilar fat when in the upper urinary tract
    Radiology images

    Images hosted on other servers:
    Missing Image

    Enhancing lesion from right renal pelvis

    Prognostic factors
    Case reports
    Treatment
    • No clear treatment guidelines different than conventional urothelial carcinoma
    • One study suggests that a combination therapy including radical cystectomy could possibly yield the best outcome of disease free survival (BMC Urol 2017;17:34)
      • The impact of neoadjuvant chemotherapy, per the authors, is yet undetermined
      • Transurethral resection of bladder tumor alone was associated with both the lowest disease free survival and highest mortality rate
    • Conservative treatment should be considered only in cases of pure or predominant lymphoepithelioma-like carcinoma; 5 patients in one study who had pure or predominant lymphoepithelioma-like carcinoma (> 50%) and were treated with transurethral resection followed by chemotherapy were alive without evidence of disease at 2 - 5 years (Am J Surg Pathol 2011;35:474)
    • High degree of PDL1 expression could indicate potential for immunotherapy (Mod Pathol 2007;20:828, Am J Surg Pathol 2011;35:474, Virchows Arch 2017;470:703, Transl Androl Urol 2021;10:1521, Am J Pathol 2020;190:134)
    Gross description
    • In bladder, usually in dome, posterior wall or trigone
    Gross images

    Images hosted on other servers:
    Missing Image

    Renal pelvic tumor
    invading parenchyma
    and hilar fat

    Microscopic (histologic) description
    • Large pleomorphic nuclei with prominent nucleoli, ill defined borders and frequent mitosis
    • Arranged in syncytial sheets, nests or individual cells with abundant mixed inflammatory cells that often obscure the neoplastic cells (Am J Surg Pathol 1994;18:466)
    • Nonneoplastic cells are a mixture of polyclonal B and T lymphocytes, histiocytes, eosinophils, plasma cells and occasional neutrophils or eosinophils that invade the epithelial nests; T cells usually predominate (Histopathology 2019;74:77)
    • May be pure or mixed, either predominant or focal; if mixed, consider recording proportion of subtype (Int J Clin Exp Pathol 2009;2:194)
      • May be admixed with conventional urothelial carcinoma, urothelial carcinoma with squamous differentiation, glandular differentiation or sarcomatoid components (Am J Surg Pathol 2011;35:474)
    • Surface urothelium may demonstrate concurrent carcinoma in situ (CIS), up to 50% of cases in one report or papillary urothelial carcinoma (Am J Surg Pathol 2011;35:474)
    Microscopic (histologic) images

    Contributed by Maria Tretiakova, M.D., Ph.D.
    Syncytial growth, inflammation Syncytial growth, inflammation

    Syncytial growth, inflammation

    Prominent nucleoli

    Prominent nucleoli

    Syncytial growth, prominent nucleoli

    Syncytial growth, prominent nucleoli

    Syncytial growth, mitosis

    Syncytial growth, mitosis

    Cytology description
    • Large atypical, undifferentiated tumor cells, oftentimes in clusters, with high N:C ratio, vesicular chromatin and prominent nucleoli; may be seen singly or in mixture with inflammatory cells
    • May be impossible to differentiate from conventional urothelial carcinoma on cytology alone; a diagnosis of this subtype in urine should be rendered with caution and should always be confirmed by the histologic examination of tissue biopsy
    • Reference: Diagn Cytopathol 2008;36:600
    Positive stains
    Negative stains
    Molecular / cytogenetics description
    • Lymphoepithelioma-like carcinoma of the bladder showed basal-like molecular phenotype by RNA expression profiling with PDL1 IHC positive in 93% of cases; no evidence for loss of MMR
      • By UroVysion FISH gains of chromosome 3 were most common (76%), followed by gain of chromosome 17, 9p21 abnormality and gain of chromosome 7
      • In cases with other histologic subtypes, squamous, urothelial, sarcomatoid and glandular components showed patterns of gain / loss that were identical to those of the lymphoepithelioma-like carcinoma component (Histopathology 2019;74:77, Am J Pathol 2020;190:134, Am J Surg Pathol 2011;35:474)
    • Whole genome sequencing study of lymphoepithelioma-like carcinoma of the renal pelvis showed mutations in susceptibility gene (KDM6A) and driver gene (LEPR) (Mol Oncol 2022;16:3666)
    Sample pathology report
    • Bladder and prostate, cystoprostatectomy:
      • Invasive urothelial carcinoma (3.2 cm), lymphoepithelioma-like subtype (100%); the carcinoma invades muscularis propria (detrusor muscle), margins are negative (see comment and synoptic reports)
      • Prostatic adenocarcinoma, Gleason score 3 + 4 = 7 (grade group 2), 5 - 10% Gleason pattern 4; organ confined, margins are negative
      • 6 lymph nodes, negative for tumor
      • Therapy related changes including foreign body giant cell reaction and scar
      • Comment: Lymphoepithelioma-like urothelial carcinoma is a rare subtype of urothelial carcinoma composed of syncytial sheets and clusters of pleomorphic cells with associated inflammatory infiltrate. The tumor has morphologic similarities to lymphoepithelioma of the nasopharynx but differs due to lack of association with EBV infection. Some authors have suggested a better overall survival in patients with pure or predominant (> 50%) lymphoepithelioma-like urothelial carcinoma than conventional urothelial carcinoma. A recent study showed that these tumors are enriched for basal squamous molecular subtype markers and for PDL1 expression, consistent with the known sensitivity for chemotherapy and possibly indicating a potential use for immune checkpoint inhibitors as a therapeutic option.
    Differential diagnosis
    • Florid chronic cystitis:
      • No malignant epithelial component
      • Cytokeratin negative
      • Differential diagnosis can be challenging on crushed or limited biopsy; epithelial cells in lymphoepithelioma-like urothelial carcinoma can be difficult to identify
    • Metastases from other site:
      • Lack of urothelial immunohistochemical markers
      • Clinical, radiologic correlation
    • Lymphoma:
      • No syncytial cohesive clusters of malignant epithelial cells
      • Cytokeratin and urothelial marker negative
      • Lymphocytes are clonal and LCA positive
      • Usually lacks overlying carcinoma in situ or usual urothelial component
    • Small cell neuroendocrine carcinoma:
      • Molding or crush artifact, no prominent nucleoli and usually no prominent lymphocytic component
    • Squamous cell carcinoma, poorly differentiated:
      • Pure squamous lesion with no in situ or invasive urothelial component
    • Urothelial carcinoma with prominent lymphoid stroma:
      • Lacks syncytial growth pattern and typical cytology
    • Xanthogranulomatous pyelonephritis and other inflammatory lesions:
    Board review style question #1

    A 65 year old man presents with hematuria and is discovered to have a 3 cm urinary bladder mass located in the dome. Histological examination on transurethral resection of bladder tumor shows a brisk, predominately lymphocytic, inflammatory infiltrate with sheets / nests of large epithelial cells displaying syncytial growth (see photomicrograph above). The epithelial cells are positive for high molecular cytokeratins and GATA3. No other components are identified. Which of the following statements is true about this carcinoma subtype in its pure form?

    1. Always expresses both CK7 and CK20
    2. Associated with EBV infection and often positive for EBV ISH
    3. Low to no PDL1 expression
    4. Poorer prognosis and less responsive to chemotherapy than conventional urothelial carcinoma
    5. Shows a basal type molecular phenotype
    Board review style answer #1
    E. Shows a basal type molecular phenotype. Lymphoepithelioma-like carcinoma of the bladder shows a basal type molecular profile. Some authors contend this subtype has a more favorable prognosis and is more responsive to chemotherapy when pure or predominant than compared to conventional urothelial carcinoma. Recent studies show a high degree of PDL1 expression, which could indicate a potential target for immunotherapy. The carcinoma cells express high molecular weight keratins, CK7, 34BE12, p63, GATA3 and are mostly negative for CK20. This carcinoma is not associated with EBV infection.

    Comment Here

    Reference: Lymphoepithelioma-like carcinoma

    Müllerian lesions
    Definition / general
    Endocervicosis
    • Rare benign tumor-like lesions characterized by prominent endocervical type glands in muscularis propria

    Endometriosis
    • Presence of functional endometrial tissue within bladder
    • Ureter:
      • Rare in ureter but can result in renal failure due to silent obstruction
      • Associated with hydroureter / hydronephrosis
      • Tendency to involve distal third of left ureter (Hum Pathol 2008;39:954)
      • Extrinsic: involving serosal or peritoneal surface (adventitia or connective tissue)
      • Intrinsic: involving muscularis propria, lamina propria, lumen
      • Rarely malignant transformation or endometrial hyperplasia can be present

    Endosalpingiosis
    • Involvement of lamina propria and muscularis propria by tubules and cysts with tubal type epithelium (ciliated cells, intercalated cells, peg cells)
    Terminology
    Endosalpingiosis
    • Called Müllerianosis if 2 of 3 related entities (endocervicosis, endometriosis or endosalpingiosis) are present (Mod Pathol 1996;9:731)
    Epidemiology
    Endocervicosis
    • Women in reproductive years (mean age 39 years, range 34 - 65 years)
    • Also men receiving estrogen for prostate cancer

    Endometriosis
    • Women between the second and fifth decades
    • Uncommon, occurs in < 2% of all patients with endometriosis
    • Seen in post menopausal women receiving exogenous estrogen
    • Also can occur, very rarely, in men taking estrogens for prostate cancer

    Endosalpingiosis
    • Very uncommon
    • Usually seen in women of childbearing age
    Sites
    Endocervicosis
    • Posterior wall or posterior dome preferentially affected

    Endometriosis
    • Usually posterior wall of bladder above trigone or at dome

    Endosalpingiosis
    • Posterior wall or posterior dome
    Etiology
    Endocervicosis
    Endometriosis
    • Probably due to retrograde menstruation, which seeds surface of bladder serosa or postsurgical
    • Not due to metaplasia of Müllerian remnants or extension from anterior uterine adenomyosis (Am J Obstet Gynecol 2002;187:538)

    Endosalpingiosis
    • Metaplastic (Müllerian metaplasia) or implantative (similar to endometriosis)
    Clinical features
    Endocervicosis
    Endometriosis
    • Bladder is the most common site (70 - 80%) of endometriosis of the urinary tract
    • May develop into endocervicosis (mucinous metaplasia), endometrioid adenocarcinoma, clear cell carcinoma, adenosarcoma
    • Usually associated with prior surgery or female GU symptoms of urgency, frequency, suprapubic pain, rarely hematuria
    • Mass is frequently apparent either by palpation or cystoscopic examination
    • Bladder implants typically occur at vesicouterine pouch; may grow through muscularis into submucosa, producing a luminal bulge or rarely a polypoid mucosal mass (Radiographics 2006;26:1847)
    • Mucosa may appear blue at cystoscopy

    Endosalpingiosis
    • Suprapubic pain, urinary frequency, dysuria
    • May occur after surgery in some cases
    Case reports
    Endocervicosis
    Endometriosis
    Endosalpingiosis
    Treatment
    Endocervicosis
    • Excision

    Endometriosis
    Endosalpingiosis
    Clinical images

    Images hosted on other servers:

    Endometriosis

    Laparoscopic segmental cystectomy

    Gross description
    Endocervicosis
    • Mass between bladder and uterus in posterior bladder wall, dome or trigone
    • Up to 2.5 to 3.0 cm in size
    • Spongy cut surface with mucinous / milky fluid

    Endometriosis
    Endosalpingiosis
    • May form mass on posterior wall of bladder
    Microscopic (histologic) description
    Endocervicosis
    • Irregular proliferation of prominent endocervical type glands in muscularis propria, less frequently in lamina propria or subserosal connective tissue
    • Glands are irregular in size and shape and may be cystically dilated, containing mucinous secretions with neutrophils
    • Glands are lined by a single layer of tall mucinous columnar cells, less commonly flat or cuboidal cells, rarely ciliated or goblet-like cells
    • Focal glandular rupture leads to mucin accumulation within the stroma with a fibroblastic histiocytic response
    • Absent or mild nuclear atypia, no mitotic figures
    • No desmoplasia, no glandular crowding or back to back architecture

    Endometriosis
    • Resembles endometriosis elsewhere: endometrium-like glandular epithelium associated with endometrial stroma cells and recent or old hemorrhage
    • Rarely, only glands or stroma are found
    • Ureter:
      • Similar to endometriosis seen elsewhere, diagnosis based on the presence of endometrial glands (usually inactive or proliferative pattern) with endometrial stroma (usually normal appearing), sometimes restricted to a thin zone around the glandular component
      • Foamy or hemosiderin laden macrophages
      • Sometimes a fibrotic reaction

    Endosalpingiosis
    • Involvement of lamina propria and muscularis propria by tubules and cysts of Müllerian type epithelium
    • May replace urothelium and form polypoid projections into bladder lumen
    • Tubules and cysts are round / oval, may have prominent branching
    • Glands are lined by tubal type epithelium (ciliated cells, intercalated cells, peg cells)
    • No atypia, no mitotic figures, no necrosis
    Microscopic (histologic) images

    AFIP images

    Endosalpingiosis

    Ovary: glands lined by ciliated epithelium lie in fibrous stroma

    Ovary: ciliated,
    secretory and
    intercalated cells
    line the cystic space



    Images hosted on other servers:

    Endocervicosis

    Prominent
    endocervical
    glands in
    muscularis propria

    Complex cystic lesion

    Columnar cells with granular mucinous apical cytoplasm


    Endometriosis

    Endometrial glands and
    stroma involving the
    peri-ureteral soft tissue

    Positive stains
    Endocervicosis
    Endometriosis
    Differential diagnosis
    Endocervicosis
    • Adenocarcinoma:
      • Marked atypia, mitotic figures
    • Adenoma malignum from uterine cervix:
      • Infiltration of bladder serosa, deep cervical involvement
      • Glands are variable in shape or size with irregular or claw shaped outlines

    Endosalpingiosis
    • Adenocarcinoma:
      • Marked atypia, invasive borders, usually not ciliated and lacks 3 types of tubal cells

    Malakoplakia
    Definition / general
    • Rare histiocytic disease that occurs in all organs
    • Common in GU tract, particularly bladder
    • Also gastrointestinal tract (most commonly colon, followed by stomach and duodenum), central nervous system, female genital tract (Radiographics 2006;26:1847)
    • Single or multiple white-yellow soft raised plaques on the mucosal surface
    • Often misdiagnosed clinically as a malignant condition
    Terminology
    • First described by Michaelis and Gutmann in 1902 and later named by Von Hansseman
    • "Malakoplakia" comes from Greek words malakos (soft) and plakos (plaque)
    Epidemiology
    • More common in immunocompromised (HIV, renal transplant recipients) and women
    • Mean age at diagnosis is fifth decade
    • Rare in children
    Etiology
    • Caused by defects in phagocytic or degradative functions of histiocytes in response to gram negative coliforms (E. coli or Proteus) that results in chronic inflammatory state, followed by intracellular deposition of iron and calcium (known as Michaelis-Gutmann bodies)
    Clinical features
    Case reports
    Treatment
    • Antibiotics that concentrate in macrophages (quinolones or trimethoprim-sulfamethoxazole), antibiotics directed against E. coli plus surgery
    • Possibly bethanechol (may correct decreased cGMP levels that may interfere with complete bacterial killing, eMedicine)
    • Discontinuation of immunosuppressive drug therapy
    Clinical images

    Contributed by Zachary Gordon, M.D., G. Bailey, M.D. and the Genitourinary Pathology Society (Case #496)

    White light cystoscopy


    Narrow band cystoscopy

    Missing Image

    Cystoscopic view of bladder lesions

    Microscopic (histologic) description
    • Can be divided in 3 stages:
      • Initial inflammatory stage
      • Classic stage with abundant Michaelis-Gutmann bodies
      • Third stage with progressive fibrous tissue and scarring
    • Foamy epithelioid histiocytes with PAS+ granular eosinophilic cytoplasm in lamina propria, some lymphocytes and occasional giant cells
    • Histiocytes have increased number of phagosomes containing non-digested bacteria (usually E. coli or Proteus), contain Michaelis-Gutmann bodies (iron containing, cytoplasmic laminated mineralized concretions)
    Microscopic (histologic) images

    Contributed by Cheng Wang, M.D. and the Genitourinary Pathology Society (Case #496)
    Missing Image Missing Image

    Various images

    Missing Image

    PAS stain

    Missing Image

    Von Kossa stain




    Images hosted on other servers:
    Missing Image Missing Image Missing Image Missing Image

    Malakoplakia (arrows at Michaelis-Gutmann bodies)


    Missing Image Missing Image

    von Kossa calcium stain

    Missing Image

    CD68

    Virtual slides

    Images hosted on other servers:
    Missing Image

    Bladder malakoplakia

    Electron microscopy description
    • Macrophages have phagosomes that are packed with undigested bacterial products
    Board review style question #1
    What is the name of the targetoid bodies, which are pathognomonic for malakoplakia?

    1. Asbestos bodies
    2. Aschoff bodies
    3. Howell-Jolly bodies
    4. Michaelis-Gutman bodies
    5. Psammoma bodies
    Board review style answer #1
    D. Michaelis-Gutman bodies

    Comment Here

    Reference: Malakoplakia

    Malakoplakia
    Definition / general
    • Chronic inflammatory disorder that affects many organs, most commonly occurs in urinary bladder
    • Single or multiple white-yellow soft raised plaques on the mucosal surface, can mimic malignancy (Nephrourol Mon 2014;6:e18522)
    Essential features
    • Histiocytic infiltrate with characteristic cytoplasmic basophilic inclusions
    Terminology
    • First described by Michaelis and Gutmann in 1902 and later named by von Hansseman
    • Malakoplakia comes from Greek words malakos (soft) and plakos (plaque)
    ICD coding
    • ICD-10
      • N32.8 - other specified disorders of bladder
      • N28.8 - other specified disorders of kidney and ureter
      • N36.8 - other specified disorders of urethra
    • ICD-11
      • GC01.Y - other specified disorders of bladder
      • GB90.Y - other specified disorders of kidney or ureter
      • GC0Y - other diseases of urinary system
    Epidemiology
    • More common in immunocompromised patients, patients with diabetes mellitus, renal transplantation recipients (Arch Ital Urol Androl 2022;94:350)
    • Prevalence 4 times higher in women
    • Mostly affects people over 40 years old
    Sites
    Pathophysiology
    • Thought to be a defect in macrophage phagolysosomal response to bacterial infection, resulting in accumulation of macrophages with calcified inclusions composed of undigested bacteria and products (Am J Kidney Dis 2016;68:e27)
    Etiology
    • Exact etiology unknown
    • Recurrent urinary infections (E. coli and other gram negative bacilli) and immunosupression
    Diagrams / tables
    Not relevant to this topic
    Clinical features
    Diagnosis
    • Histological by biopsy
    Laboratory
    Not relevant to this topic
    Radiology description
    Not relevant to this topic
    Radiology images
    None available
    Prognostic factors
    Not relevant to this topic
    Case reports
    • 55 year old woman with history of kidney disease presented with general malaise and worsening renal failure and was found to have a bladder mass (Pathol Res Pract 2022;237:153852)
    • 61 year old immunocompromised man with symptoms of urinary obstruction was found to have exuberant malakoplakia of prostate presenting as prostatic abscess (J Endourol Case Rep 2020;6:231)
    • 82 year old woman with intravesical malakoplakia mimicking an aggressive transitional cell carcinoma (Bladder (San Franc) 2020;7:e44)
    Treatment
    • Antibiotics or surgical excision
    Clinical images
    HTML note: would you like to keep these images?
    Contributed by Zachary Gordon, M.D., G. Bailey, M.D. and the Genitourinary Pathology Society (Case #496)

    White light cystoscopy

    White light cystoscopy


    Narrow band cystoscopy

    Missing Image

    Cystoscopic view of bladder lesions

    Gross description
    Not relevant to this topic
    Gross images
    None available
    Frozen section description
    Not relevant to this topic
    Frozen section images
    Not relevant to this topic
    Microscopic (histologic) description
    • Sheets of histiocytes with granular eosinophilic cytoplasm (von Hansemann cells) and basophilic intracytoplasmic inclusion (Michaelis-Gutmann bodies)
    Microscopic (histologic) images

    Contributed by Mustafa Goksel, M.D.
    Sheets of eosinophilic histiocytes

    Sheets of eosinophilic histiocytes

    Urinary bladder with histiocytic infiltration

    Urinary bladder with histiocytic infiltration

    Numerous intracytoplasmic inclusions

    Numerous intracytoplasmic inclusions

    Von Kossa stain Von Kossa stain

    Von Kossa stain



    HTML note: would you like to keep these images?
    Contributed by Cheng Wang, M.D. and the Genitourinary Pathology Society (Case #496)
    Missing Image Missing Image

    Various images

    Missing Image

    PAS stain

    Missing Image

    Von Kossa stain

    Virtual slides
    HTML note: would you like to keep this slide?
    Images hosted on other servers:
    Missing Image

    Bladder malakoplakia

    Cytology description
    Not relevant to this topic
    Cytology images
    Not relevant to this topic
    Immunofluorescence description
    Not relevant to this topic
    Immunofluorescence images
    Not relevant to this topic
    Negative stains
    Electron microscopy description
    Not relevant to this topic
    Electron microscopy images
    Not relevant to this topic
    Molecular / cytogenetics description
    Not relevant to this topic
    Molecular / cytogenetics images
    Not relevant to this topic
    Videos
    None available
    Sample pathology report
    • Bladder lesions, biopsy:
      • Benign urothelial mucosa with acute and chronic inflammation and prominent histiocytic infiltrate, consistent with malakoplakia (see comment)
      • Comment: Special stains for von Kossa (calcium) were performed on block A1 and highlight concentrically layered basophilic inclusions (Michaelis-Gutmann bodies), supporting the diagnosis of malakoplakia.

    • Bladder lesions, biopsy:
      • Malakoplakia
    Differential diagnosis
    Additional references
    Not relevant to this topic
    Board review style question #1
    What is the name of the targetoid bodies that are pathognomonic for malakoplakia?

    1. Asbestos bodies
    2. Aschoff bodies
    3. Howell-Jolly bodies
    4. Michaelis-Gutmann bodies
    5. Psammoma bodies
    Board review style answer #1
    D. Michaelis-Gutmann bodies are characteristic inclusions seen in many cases of malakoplakia. They are rounded concentric intracytoplasmic basophilic inclusions, caused by accumulation of undigested bacteria and products that are calcified. They can be highlighted by von Kossa stain. Answer A is incorrect because asbestos bodies are golden brown colored and fusiform to beaded rod shaped structures. Answer B is incorrect because Aschoff bodies are microscopic nodules of cells composed of pleomorphic cells, lymphocytes, plasma cells and fibrinoid necrosis. Answer C is incorrect because Howell-Jolly bodies are spherical intracytoplasmic inclusions within red blood cells. Answer E is incorrect because psammoma bodies are a round collection of calcium, a form of calcification that is mostly extracellular and much bigger than an intracellular inclusion.

    Comment Here

    Reference: Malakoplakia
    Board review style question #2

    Which of the following stains used in the diagnosis of malakoplakia does not target structures shown with yellow arrows in the image above?

    1. Alizarin red
    2. CD68
    3. Prussian blue
    4. Von Kossa
    Board review style answer #2
    B. CD68. The CD68 stain highlights the cytoplasm of histiocytes similar to CD163, not the inclusions. These stains are used to show the extent of histiocytic involvement and are helpful to distinguish malakoplakia from infiltrating carcinoma. Answers A and D are incorrect because Alizarin red and von Kossa stains target the calcium in Michaelis-Gutmann bodies. Answer C is incorrect because Prussian blue stain targets the iron within the Michaelis-Gutmann bodies.

    Comment Here

    Reference: Malakoplakia

    Male urethral carcinoma
    Definition / general
    • Rare primary neoplasm of epithelial origin
    • Secondary involvement by urothelial carcinoma of the bladder is much more common than primary (Eur Urol 2013;64:823)
    Essential features
    • Urethral carcinoma is usually due to secondary involvement
    • Primary urethral carcinoma is rare and the most frequent histologic types are urothelial carcinoma, squamous cell carcinoma and adenocarcinoma (not otherwise specified, clear cell)
    Epidemiology
    Sites
    Pathophysiology
    • Predisposing factors include:
    Clinical features
    • Most patients present with symptoms associated with locally advanced disease (Eur Urol 2013;64:823)
      • Gross hematuria or bloody urethral discharge, dysuria, extraurethral mass
      • Bladder outlet obstruction, pelvic pain, urethrocutaneous fistula
      • Abscess formation, dyspareunia
    • Approximately 33% of men and women present with involved regional lymph nodes
    Diagnosis
    • Clinical examination with palpation of external genitalia for suspicious indurations and pelvic exam in women (Eur Urol 2013;64:823)
    • Urinary cytology
    • Diagnostic urethroscopy and biopsy
    Radiology description
    • Aims to assess local extent and detect lymphatic and distant metastatic spread
    • Magnetic resonance imaging for evaluating extent of tumor and monitoring response to neoadjuvant chemotherapy (Eur Urol 2013;64:823)
    Prognostic factors
    Case reports
    Treatment
    Gross images

    Contributed by Jesus Adrian Chavez, M.D. and Debra Zynger, M.D.

    Penile urethra with periurethral involvement

    Microscopic (histologic) description
    • Urothelial carcinoma
    • Squamous cell carcinoma
      • Sheets of large, pleomorphic tumor cells with focal or abundant keratinization (depending of grade of differentiation), ample cytoplasm, intercellular bridges, high mitotic activity, prominent nuclear atypia
    • Adenocarcinoma
    • Clear cell adenocarcinoma
      • May have glandular, tubulocystic, solid / diffuse, papillary or micropapillary growth patterns
      • Cuboidal, variably sized cells with abundant clear or eosinophilic cytoplasm and cytoplasmic vacuoles
      • Nuclei that are hyperchromatic, pleomorphic and have prominent nucleoli
      • Hobnail changes and extracellular mucoid material may be present
      • Mitoses and necrosis are often seen
    Microscopic (histologic) images

    Contributed by Jesus Adrian Chavez, M.D. and Debra Zynger, M.D.

    Low grade noninvasive papillary urothelial carcinoma

    Penile urethra, squamous cell carcinoma

    Penile urethra, HPV+ high risk ISH

    Necrosis and keratinization


    Clear cell adenocarcinoma:

    Can mimic nephrogenic metaplasia

    With hobnailing

    With prominent clear cells and diffuse, sheet-like growth

    Positive stains
    Negative stains
    Differential diagnosis

    Metastases
    Definition / general
    • Most common primaries are breast and melanoma; also lung, pancreas, ovary and stomach
    • Associated with widely disseminated disease
    • Urothelium is usually spared
    • Tumors may also arise from local extension from prostate, uterine cervix or rectum
    • Difficult to distinguish bladder adenocarcinoma morphologically from extension of colonic adenocarcinoma
    • Renal cell metastases: rare (< 40 reported cases); usually men age 35 - 69 years who present with gross hematuria or urinary obstruction if tumor is in bladder neck; metastases also present in other organs; poor prognosis (Mod Pathol 1999;12:351); may be due to hematogenous spread, retrograde spread from renal vein or renal hilar lymphatics or direct intraluminal transit
    Case reports
    Microscopic (histologic) description
    • Morphology varies based on the primary tumor
    • Metastatic renal cell carcinoma: delicate fibrovascular stroma with abundant sinusoidal vessels; nests of polygonal cells with abundant clear cytoplasm and nuclei ranging from small and hyperchromatic with inconspicuous nucleoli to large irregular nuclei with prominent nucleoli; may resemble urothelial carcinoma with clear cell features
    Microscopic (histologic) images

    Images hosted on other servers:
    Missing Image

    Metastatic appendiceal
    mucinous adenocarcinoma
    - various images

    Missing Image

    Metastatic breast cancer: H&E, PR, HER2

    Cytology images

    Contributed by Bonnie Choy, M.D.
    Prostatic adenocarcinoma

    Prostatic adenocarcinoma

    NKX3.1

    NKX3.1

    Colorectal adenocarcinoma

    Colorectal adenocarcinoma

    Renal cell carcinoma

    Renal cell carcinoma

    Positive stains
    Negative stains
    Differential diagnosis
    • Colorectal carcinoma (extension): positive for CK20, villin, beta catenin (nuclear)
    • Prostatic adenocarinoma (extension): positive for PSA, PSAP, AMACR, Leu7
    • Urothelial carcinoma with glandular differentiation: positive for CK7, CK20, 34betaE12, Uroplakin, Thrombomodulin
    Additional references

    Microcystic
    Definition / general
    • Invasive urothelial carcinoma with deceptively bland architecture and cytology, consisting of slit-like or tubular microcysts lined by cuboidal / flattened cells with minimal atypia and intraluminal granular eosinophilic material (Mod Pathol 2009;22:S96)
    Essential features
    • Very rare, predominantly case reports and small series (Histopathology 2014;64:872)
    • Main biologic significance is deceptively benign features, which may be mistaken for cystitis cystica or glandularis on small biopsies but is usually deeply invasive
    • Histologically may be distinguished by infiltrative growth pattern, as well as variability in cyst size and shape
    Terminology
    ICD coding
    • Location based ICD-10 coding:
      • Renal pelvis, including pelviureteric junction and renal calyces
        • C65.1 - malignant neoplasm of right renal pelvis
        • C65.2 - malignant neoplasm of left renal pelvis
        • C65.9 - malignant neoplasm of unspecified renal pelvis
      • Ureter, including ureteric orifice of bladder
        • C66.1 - malignant neoplasm of right ureter
        • C66.2 - malignant neoplasm of left ureter
        • C66.9 - malignant neoplasm of unspecified ureter
      • Bladder
        • C67.0 - malignant neoplasm of trigone of bladder
        • C67.1 - malignant neoplasm of dome of bladder
        • C67.2 - malignant neoplasm of lateral wall of bladder
        • C67.3 - malignant neoplasm of anterior wall of bladder
        • C67.4 - malignant neoplasm of posterior wall of bladder
        • C67.5 - malignant neoplasm of bladder neck
        • C67.6 - malignant neoplasm of ureteric orifice
        • C67.7 - malignant neoplasm of urachus
        • C67.8 - malignant neoplasm of overlapping sites of bladder
        • C67.9 - malignant neoplasm of bladder, unspecified
      • Other, unspecified urinary organs
        • C68.0 - malignant neoplasm of urethra
        • C68.1 - malignant neoplasm of paraurethral glands
        • C68.8 - malignant neoplasm of overlapping sites of urinary organs
        • C68.9 - malignant neoplasm of urinary organ, unspecified
    Epidemiology
    Sites
    • Bladder, renal pelvis
    Pathophysiology
    Etiology
    • No specific risk factors for development of microcystic variant reported
    • General risk factors for urothelial carcinoma: smoking, aromatic amine exposure, arsenic exposure
    Clinical features
    Diagnosis
    • Cystoscopy
    • CT scan
    Laboratory
    Radiology description
    Prognostic factors
    Case reports
    Treatment
    Gross description
    • Thickened bladder wall
    Frozen section description
    • Not usually diagnosed on frozen section
    • Deceptively bland appearance; depth of invasion is key to distinguish from benign mimics, such as cystitis cystica glandularis, on the rare occurrence of frozen diagnosis (Eur Urol Focus 2020;6:653)
    Microscopic (histologic) description
    • Invasive urothelial carcinoma with microcysts, macrocysts or tubular structures, lined by single to multilayered cuboidal or flattened cells with minimal cytologic atypia (Histopathology 2019;74:77)
    • Microcysts are irregular in shape, deeply infiltrative and range in size from microscopic to up to 2 mm (Histopathology 2019;74:77)
    • Has luminal granular eosinophilic secretions with necrotic cellular debris, sometimes mucinous material or a targetoid appearance (PAS+, Alcian blue+) (Mod Pathol 2009;22:S96, Arch Pathol Lab Med 2007;131:1244)
    • May be focally calcified
    • May be associated with nested variant of urothelial carcinoma
    • May be associated with focal high grade conventional urothelial carcinoma in approximately 40% of cases (Histopathology 2019;74:77)
    Microscopic (histologic) images

    Contributed by Megan L. Brown, M.D. and Maria Tretiakova, M.D., Ph.D.
    Microcysts, macrocysts and tubules

    Microcysts, macrocysts and tubules

    Invasion to muscularis propria

    Invasion to muscularis propria

    Cytokeratin 7

    Cytokeratin 7

    Positive stains
    Negative stains
    Molecular / cytogenetics description
    Sample pathology report
    • Bladder, transurethral resection:
      • Invasive high grade urothelial carcinoma with microcystic component (40%)
      • Invasive of muscularis propria (pT2)
      • Angiolymphatic invasion absent
    Differential diagnosis
    Board review style question #1

    A 40 year old man presents with gross hematuria, abdominal pain and bladder wall thickening on CT scan. He undergoes transurethral resection of bladder tumor (see above image). Which immunohistochemical profile would support the diagnosis of urothelial carcinoma with microcystic features?

    1. PAX8- / GATA3- / CK7+ / PSA+
    2. PAX8- / GATA3+ / CK7+ / p53 wild type / PSA-
    3. PAX8- / GATA3+ / CK7+ / p53+ / PSA-
    4. PAX8+ / GATA3- / CK7- / PSA-
    Board review style answer #1
    C. PAX8- / GATA3+ / CK7+ / p53+ / PSA-. This immunohistochemical profile is one that is classic to conventional urothelial carcinomas and shared by microcystic variant.

    Comment Here

    Reference: Microcystic urothelial carcinoma-invasive

    Micropapillary
    Definition / general
    • Aggressive histologic subtype of urothelial carcinoma (UC) comprised of small papillary clusters of neoplastic cells within lacunae and without fibrovascular cores (Am J Surg Pathol 1994;18:1224)
    • If noninvasive, appears as slender, delicate filiform processes on surface (Am J Surg Pathol 1994;18:1224)
    Essential features
    • Uncommon histologic subtype of UC with micropapillary architecture lacking fibrovascular cores
    • Multiple clusters of tumor cells with peripherally oriented nuclei sharing cleft-like spaces or lacunae, often resembling vascular invasion
    • More likely to be diagnosed at an advanced stage compared with conventional UC, conferring a poorer prognosis
    • If micropapillary urothelial carcinoma (MPUC) is seen on biopsy, it is highly associated with muscularis propria invasive disease
    Terminology
    • Micropapillary urothelial carcinoma (MPUC)
    • Conventional urothelial carcinoma (CUC)
    ICD coding
    • ICD-O: 8120/3 - transitional cell carcinoma, NOS
    Epidemiology
    Sites
    • Bladder
    • Renal pelvis
    • Ureter
    Pathophysiology
    Clinical features
    • Most common presenting symptom: painless gross hematuria
    • Other urinary symptoms: dysuria, urgency, frequency, painful micturition
    • More likely to present at advanced stage with extravesical disease and lymph node metastases (Urol Oncol 2019;37:48, Urol Oncol 2014;32:110, World J Urol 2012;30:801)
      • Meta analysis: at diagnosis, median rate of muscle invasive disease is 41%, lymph node involvement is 12.5% and metastasis is 5% (Eur Urol 2019;75:649)
    Diagnosis
    • Clinical symptoms prompt cystoscopy, which then results in a transurethral biopsy or resection of tumor
    • Imaging may show mass in bladder wall or diffuse bladder wall thickening, which results in cystoscopy with biopsy
    Laboratory
    Radiology description
    Radiology images

    Images hosted on other servers:

    CT with bladder tumor invading the left ureteral orifice

    CT with tumor in right posterolateral bladder wall

    Prognostic factors
    Case reports
    • 70 year old man presented with hematuria and found to have a bladder tumor involving the dome and extending to the perivesical fat (Can Urol Assoc J 2008;2:540)
    • 70 year old woman hospitalized for dyspnea with pulmonary microangiopathy from metastases and at autopsy, found to have MPUC in the bladder (Intern Med 2021;60:2843)
    • 73 year old woman presented with hematuria and found to have a mass of the left renal pelvis (Med Sci Monit 2007;13:CS47)
    • 74 year old man hospitalized for hematuria found to have multifocal papillary bladder tumors with histology showing both micropapillary and plasmacytoid urothelial carcinoma (Diagn Cytopathol 2016;44:124)
    • 82 year old man with end stage kidney disease on hemodialysis and found to have a sessile papillary tumor on the left bladder wall (Case Rep Oncol 2022;15:462)
    Treatment
    • Transurethral resection of bladder cancer (TURB) with intravesical bacillus Calmette-Guérin (BCG) may not be as effective (Eur Urol Focus 2020;6:653)
    • Early radial cystectomy (RC) may improve survival (J Urol 2015;193:1129)
    • Many patients are immediately upstaged at RC compared with TURB stage, suggesting one reason why RC is the preferred option (Pathology 2010;42:650)
    • Role of neoadjuvant chemotherapy before RC is not clear, although it is associated with pathological downstaging, it does not show increased overall survival (Eur Urol 2019;75:649)
    Gross description
    Microscopic (histologic) description
    • Small nests of tumor cells with surrounding lacunar (empty) space and without fibrovascular cores (Eur Urol Focus 2020;6:653)
      • Presence of multiple nests in one lacunae is a classic feature and may be the most helpful feature in making the diagnosis (Am J Surg Pathol 2010;34:1367)
      • Must be careful not to confuse lacunae with lymphovascular invasion; lacunae may be lined with flattened endothelial type cells that are not lymphovascular in origin
    • Epithelial ring forms and intracytoplasmic vacuolization may also be present (Eur Urol Focus 2020;6:653)
    • Stromal reaction to tumor cells may be absent (Mod Pathol 2009;22:S96)
    • Usually has high nuclear grade (Mod Pathol 2009;22:S96)
    Microscopic (histologic) images

    Contributed by Timothy Isaac Miller, M.D., M.A., Maria Tretiakova, M.D., Ph.D. and @katcollmd on Twitter

    Classic features

    Micropapillary
    and conventional
    urothelial
    carcinoma

    Multiple nests throughout stroma

    Epithelial ring morphology

    Ring forms and intracytoplasmic vacuolization

    Numerous nests in large lacunae


    Micropapillary urothelial carcinoma Micropapillary urothelial carcinoma

    Micropapillary urothelial carcinoma

    Micropapillary urothelial carcinoma Micropapillary urothelial carcinoma

    Micropapillary urothelial carcinoma

    Cytology description
    Cytology images

    Images hosted on other servers:
    Missing Image Missing Image

    Cohesive clusters in a 3 dimensional arrangement

    Molecular / cytogenetics description
    Videos

    Bladder urothelial carcinoma

    Sample pathology report
    • Bladder, transurethral resection:
      • Urothelial carcinoma, high grade
        • Histologic component: micropapillary (90%)
        • Adjacent flat carcinoma in situ: absent
        • Angiolymphatic invasion: absent
        • Muscularis propria: present, invaded by carcinoma
        • Depth of invasion: invasive of muscularis propria (pT2)
    • Bladder, cystectomy:
      • Invasive high grade urothelial carcinoma, micropapillary subtype (see synoptic report)
    Differential diagnosis
    Board review style question #1

    Which of the following is true regarding the histologic subtype of urothelial carcinoma shown above?

    1. Fibrovascular cores should be present in order to call it this histologic subtype
    2. Neoadjuvant chemotherapy is the standard of care for this subtype
    3. Patients are more likely to have better outcomes if this subtype represents > 10% of the tumor content
    4. Patients are more likely to present at an advanced stage compared with conventional urothelial carcinoma
    5. There is no concordant in situ histologic correlation to this subtype
    Board review style answer #1
    D. Patients are more likely to present at an advanced stage compared with conventional urothelial carcinoma

    The image above is the micropapillary histologic subtype of urothelial carcinoma. When micropapillary histology is found on biopsy, patients are more likely to already have muscularis propria invasion, lymph node metastasis and extravesicular extension, overall conferring a worse prognosis and why it is important to report this subtype. The role of neoadjuvant therapy before radical cystectomy in micropapillary cases is not clear and may not actually improve overall survival (choice B is incorrect). Fibrovascular cores should not be present within the clusters of cells (choice A is incorrect). If the micropapillary histology comprises > 10% of the tumor, then patients are more likely to have worse, not better outcomes (choice C is incorrect). There is an in situ micropapillary subtype that will look like slender filiform processes (choice E is incorrect) (Urol Oncol 2019;37:48, Urol Oncol 2014;32:110, World J Urol 2012;30:801, Eur Urol 2019;75:649, Histopathology 2004;45:55, Am J Surg Pathol 1994;18:1224).

    Comment Here

    Reference: Micropapillary urothelial carcinoma

    Nephrogenic metaplasia
    Definition / general
    • Nonneoplastic localized or diffuse, papillary, tubular or cystic metaplastic changes of the urothelium in response to chronic infection, calculi, injury or prolonged catheterization
    • Characterized by a single layer of cuboidal or hobnail epithelial cells with clear or eosinophilic cytoplasm and small, discrete nuclei without prominent nucleoli
    • First described by Davis in 1949; Friedman and Kuhlenbeck coined the term nephrogenic adenoma in 1950 (Northwest Med 1949;48:182, J Urol 1950;64:657)
    Essential features
    • Nonneoplastic reactive metaplastic lesion with tubular, glandular, papillary or cystic architecture and surrounded (at least focally) by thick basement membrane
    • Most cases confined to lamina propria
    • May mimic urothelial carcinoma, clear cell or prostatic adenocarcinoma
    Terminology
    • Also called nephrogenic adenoma, mesonephroid metaplasia, adenomatous metaplasia
    ICD coding
    • ICD-10: N32.9 - bladder disorder, unspecified
    Epidemiology
    Sites
    • Most commonly seen in the bladder neck and trigone (80%) but may occur in the urethra (12%) and ureter (8%) (Arch Pathol Lab Med 2010;134:1455)
    • More common at bladder neck, trigone and adjacent urethra
    Etiology
    • Commonly appears to be metaplastic and associated with secondary urothelial injury, such as past surgery (60%), calculi (14%) or trauma including chronic catheterization (9%) and urinary tract infection (Hum Pathol 1981;12:907)
    • ~8% associated with prior renal transplant recipients, supporting seeding / exfoliation of renal tubular cells in the urinary tract (N Engl J Med 2002;347:653, J Urol 1988;139:45)
    • Less commonly associated with intravesical therapy for urothelial carcinomas (bacillus Calmette-Guérin (BCG), thiotepa, etc.), NSAID use, diverticulum, exstrophy, interstitial cystitis, malakoplakia
    Clinical features
    Diagnosis
    • Made on histopathological evaluation
    Prognostic factors
    • Nonneoplastic lesion with a propensity for recurrences
    • Occasional cases with worrisome features may require resampling / frequent surveillances
    • Rare case reports progressing to clear cell adenocarcinoma (Hum Pathol 2006;37:117, Curr Urol 2012;6:106)
    Case reports
    Treatment
    Gross description
    • May be papillary (56%), polypoid (10%), fungating or sessile (10%) lesions (Arch Pathol Lab Med 2010;134:1455)
    • Most (62%) are small lesions (1 cm), although rarely may be as large as 7 cm
    • Multiple (~18 - 20%) and large, especially in the setting of end stage renal disease (Adv Anat Pathol 2006;13:247)
    Microscopic (histologic) description
    • Exhibit spectrum of mixed histologic patterns: tubular, tubulocystic, papillary, polypoid and less frequently, solid growth (Adv Anat Pathol 2006;13:247)
    • Multiple small caliber tubules lined by cuboidal, low columnar and hobnail cells, exhibiting irregular smudged nuclei protruding into the lumen
    • Intraluminal colloid-like eosinophilic secretions or blue tinged mucin
    • Tubules often surrounded by a thickened hyalinized basement membrane
    • Cytoplasm, in general eosinophilic but may focally have clear cytoplasm, especially in solid areas (Hum Pathol 1998;29:1451)
    • Mostly limited to superficial lamina propria, lack a desmoplastic stromal response, necrosis, cytological atypia or brisk mitosis
    • Background stroma may be variable depending upon the etiology and display edematous changes, mixed inflammatory infiltrates, reparative type fibrosis, granulomas or fibromyxoid changes
    • Atypical nephrogenic metaplasia is a term reserved for cases with unexpected nuclear enlargement, hyperchromasia and abnormally prominent nucleoli (Cancer 2000;88:853)
    • Uncommon patterns include:
      • Pseudoinfiltrative growth pattern composed of small tubules lacking a basal cell layer; may mimic neoplasia
      • Signet ring-like appearance with intraluminal blue tinged mucin and compressed tubules
      • Cystic pattern which may resemble atrophic renal tubules
    • Fibromyxoid variant:
      • Composed of compressed spindled cells and irregular epithelial cords within a fibromyxoid stroma
      • Scanty classic tubular pattern
      • Background with a light pinkish / lavender colored stroma, which is negative for amyloid (Congo red stain) and Tamm-Horsfall protein (Am J Surg Pathol 2007;31:1231)
    Microscopic (histologic) images

    Contributed by Jatin S. Gandhi, M.B.B.S., M.D., @MirunaPopescu13 on Twitter and @katcollmd on Twitter
    Nephrogenic metaplasia with BCG granuloma

    With BCG granuloma

    Peritubular basement membrane

    Peritubular basement membrane

    Nephrogenic metaplasia with papillary architecture

    With papillary architecture

    Nephrogenic metaplasia Nephrogenic metaplasia Nephrogenic metaplasia

    Nephrogenic metaplasia


    Nephrogenic metaplasia Nephrogenic metaplasia Nephrogenic metaplasia Nephrogenic metaplasia

    Nephrogenic metaplasia

    PAX8

    PAX8+

    HNF1 beta

    HNF1 beta+

    Cytology description
    • Variable sized clusters composed of cuboidal cells with high N/C ratio (Diagn Cytopathol 2008;36:47)
    • Nuclei are centrally or eccentrically located, have finely distributed chromatin with small nucleoli and regular nuclear membranes
    • Cytoplasm is granular or vacuolated resembling a signet ring cell appearance
    Positive stains
    Negative stains
    Electron microscopy description
    Molecular / cytogenetics description
    • Currently no known disease defining recurrent cytogenetic alterations
    Sample pathology report
    • Urinary bladder, transurethral resection:
      • Nephrogenic metaplasia and chronic and granulomatous inflammation
      • Muscularis propria identified
    Differential diagnosis
    Board review style question #1

    A 60 year old man presented with a bladder lesion. A transurethral resection of bladder tumour was performed. Which of the following statements is true regarding the lesion shown in the above image?

    1. Associated with endometriosis
    2. Associated with renal transplant
    3. Cytological atypia is common
    4. Generally infiltrates muscularis propria
    Board review style answer #1
    B. Associated with renal transplant

    Nephrogenic metaplasia is commonly seen in the superficial lamina propria, is cytologically bland and may be associated with post renal transplant. Endometriosis is commonly associated with clear cell adenocarcinoma.

    Comment Here

    Reference: Nephrogenic metaplasia
    Board review style question #2
    Which of the following combination of immunostains can be useful to differentiate nephrogenic metaplasia from prostatic adenocarcinoma?

    1. PAX8, HNF1 beta, AMACR
    2. PAX8, Napsin A, AE1/3
    3. PAX8, Napsin A, NKX3.1
    4. PAX8, Napsin A, GATA3
    Board review style answer #2
    C. PAX8, Napsin A, NKX3.1

    Nephrogenic metaplasia is commonly positive for PAX8 and Napsin A and less commonly for HNF1 beta and GATA3. NKX3.1 is expressed almost exclusively in prostatic adenocarcinoma. Both nephrogenic metaplasia and prostatic adenocarcinoma express AMACR and AE1/3.

    Comment Here

    Reference: Nephrogenic metaplasia

    Nested
    Definition / general
    • Morphologic variant of urothelial carcinoma characterized by unusually bland, nested pattern of invasion but with clinical outcomes similar to conventional invasive high grade urothelial carcinoma (Virchows Arch 2014;465:199)
    Essential features
    • Cytologically bland nested pattern that could be difficult to distinguish from florid von Brunn nests; particularly challenging in superficial biopsies
    • Often deeply invasive with increasing irregularity in the deeper portion
    • Clinical outcomes are similar to invasive high grade urothelial carcinoma of matched clinical stage
    Terminology
    ICD coding
    • Location based ICD-10 coding:
      • Renal pelvis, including pelviureteric junction and renal calyces
        • C65.1 - malignant neoplasm of right renal pelvis
        • C65.2 - malignant neoplasm of left renal pelvis
        • C65.9 - malignant neoplasm of unspecified renal pelvis
      • Ureter, including ureteric orifice of bladder
        • C66.1 - malignant neoplasm of right ureter
        • C66.2 - malignant neoplasm of left ureter
        • C66.9 - malignant neoplasm of unspecified ureter
      • Bladder
        • C67.0 - malignant neoplasm of trigone of bladder
        • C67.1 - malignant neoplasm of dome of bladder
        • C67.2 - malignant neoplasm of lateral wall of bladder
        • C67.3 - malignant neoplasm of anterior wall of bladder
        • C67.4 - malignant neoplasm of posterior wall of bladder
        • C67.5 - malignant neoplasm of bladder neck
        • C67.6 - malignant neoplasm of ureteric orifice
        • C67.7 - malignant neoplasm of urachus
        • C67.8 - malignant neoplasm of overlapping sites of bladder
        • C67.9 - malignant neoplasm of bladder, unspecified
      • Other, unspecified urinary organs
        • C68.0 - malignant neoplasm of urethra
        • C68.1 - malignant neoplasm of paraurethral glands
        • C68.8 - malignant neoplasm of overlapping sites of urinary organs
        • C68.9 - malignant neoplasm of urinary organ, unspecified
    Epidemiology
    Sites
    • Renal pelvis, ureter, bladder
    Pathophysiology
    Etiology
    • No specific risk factors to development of nested variant reported
    • General risks factors for urothelial carcinoma: smoking, aromatic amine exposure, arsenic exposure
    Clinical features
    Diagnosis
    • Cystoscopy
    • CT scan
    • MRI
    Laboratory
    • Hematuria
    Radiology description
    • Bladder with irregular wall thickening and little protrusion into the bladder lumen with gradual contrast enhancement on dynamic contrast enhanced MRI (Abdom Radiol (NY) 2020;45:2279)
    Prognostic factors
    Case reports
    Gross description
    • Thickened, irregular renal pelvis with or without associated papillary component
    Gross images

    Images hosted on other servers:
    Missing Image

    Gray-white tumor

    Frozen section description
    • Not typically diagnosed on frozen section
    Microscopic (histologic) description
    Microscopic (histologic) images

    Contributed by Megan L. Brown, M.D., Maria Tretiakova, M.D., Ph.D. and @SueEPig on Twitter
    von Brunn-like appearance

    von Brunn-like appearance

    Irregular projections and occasional lumens

    Irregular projections and occasional lumens

    Muscularis propria

    Invasion into muscularis propria

    Large nested carcinoma

    Large nested carcinoma


    Nested urothelial carcinoma Nested urothelial carcinoma Nested urothelial carcinoma Nested urothelial carcinoma

    Nested urothelial carcinoma

    Positive stains
    Negative stains
    Molecular / cytogenetics description
    Sample pathology report
    • Bladder, transurethral resection:
      • High grade urothelial carcinoma with nested growth (60%) (see comment)
      • Comment: Invasive of lamina propria (pT1)
      • Angiolymphatic invasion absent
      • Muscularis propria present, no tumor
    Differential diagnosis
    • Florid von Brunn nests:
      • Relatively round and evenly spaced with uniform linear border at base versus nested carcinoma, which has anastomosing, smaller, irregularly sized nests lacking orientation to overlying urothelium and more irregular projections into stroma and muscle invasion (Mod Pathol 2009;22:S96)
      • Wide variability in staining for Ki67, p53, p27 and CK20 seen in both florid von Brunn nests and nested variant of urothelial carcinoma (Am J Surg Pathol 2003;27:1243)
    • Inverted papilloma:
      • Has more cytoarchitectural organization within nests
      • Cells maintain peripheral palisading
    • Nephrogenic adenoma:
      • May be nested, tubular and have irregular stromal interface but nests are composed of only a single layer of cuboidal or flattened cells and is more likely to contain other architectural patterns (cystic, papillary) and have surrounding stromal edema and inflammatory infiltrate (Mod Pathol 2009;22:S96)
    • Paraganglioma and carcinoid:
      • Characteristic vascular pattern and chromatin features
      • Positive for neuroendocrine markers
    Board review style question #1
    Nested variant of urothelial carcinoma is most commonly identified in which patients?

    1. Women aged 30 - 50
    2. Women older than 80
    3. Men older than 60
    4. Men younger than 50
    5. Pediatric populations (< 18)
    Board review style answer #1
    C. Men older than 60. The nested variant of urothelial carcinoma is most commonly identified in men older than 60 with a similar occurrence to that of classic urothelial carcinoma.

    Comment Here

    Reference: Nested urothelial carcinoma
    Board review style question #2

    A 65 year old man with flank pain and hematuria presents with bladder wall thickening on MRI. Transurethral resection of bladder tumor is performed (see above image). What is the diagnosis?

    1. Florid von Brunn nests
    2. Inverted papilloma
    3. Nephrogenic adenoma
    4. Urothelial carcinoma, nested variant
    Board review style answer #2
    D. Urothelial carcinoma, nested variant

    Comment Here

    Reference: Nested urothelial carcinoma

    Noninvasive papillary urothelial carcinoma high grade
    Definition / general
    • Neoplastic proliferation of the urothelium with a papillary configuration and no invasion beyond the basement membrane
    • Moderate to marked architectural and cytologic atypia
    Essential features
    • Noninvasive papillary urothelial neoplasm with moderate to marked cytoarchitectural abnormality
    • Complex solid to fused papillary architecture, nuclear atypia, pleomorphism (may be focal), crowded and overlapping cells, brisk mitotic activity
    • Immunohistochemistry not required for diagnosis
    • Commonly presents with hematuria
    • High rate of progression to invasion
    Terminology
    • Current 2016 WHO Classification: high grade
    • Prior / older terminology - 1973 WHO classification: grade 2 and 3
    • Overlap between WHO 1973 (grade 1, 2 and 3) and 2004 (low / high grade) nomenclatures
    ICD coding
    • ICD-O: 8130/2 - papillary transitional cell carcinoma, noninvasive
    Epidemiology
    • M:F = 6 - 8:1
    • Median age: 70 years
    Sites
    • Most commonly found in posterior and lateral walls of bladder but may be found anywhere within urothelium
    • 85% of urothelial neoplasms of renal pelvis are papillary and 66% of them are high grade (Mod Pathol 2005;18:11)
    Pathophysiology
    • Normal urothelium develops hyperplasia, dysplasia or carcinoma in situ as it acquires further genetic alterations: loss of chromosome 9, activating mutations in FGFR3 or RAS (Int J Clin Oncol 2008;13:287)
    • Other mutations: PIK3CA, loss of 11p, CCND1, p53
    Etiology
    Clinical features
    • Painless, intermittent hematuria
    • Pure high grade carcinomas more aggressive than mixed high and low grade
    • High rate of progression to invasive disease
    Diagnosis
    • Cystoscopy: exophytic lesion, solitary or multiple, with varying size
    • Imaging: CT urography or ultrasound
    • Urine cytology
    Radiology description
    • CT urography shows hydronephrosis and filling defects (Abdom Radiol (NY) 2018;43:663)
    • Ultrasound can detect hydronephrosis and any intraluminal bladder masses
    Prognostic factors
    • WHO / ISUP histologic grade
    • Nuclear anaplasia: decreased time to recurrence and progression
    • Concomitant urothelial carcinoma in situ: higher recurrence rate
    • Multifocal disease: higher disease associated mortality and progression
    • High Ki67 poor prognosis (Int J Mol Sci 2018;19:2548)
    • PTEN deletions: increased recurrence rate (Virchows Arch 2018;472:969)
    • TP53 and RB alternations: adverse histopathological parameters (Anticancer Res 2018;38:3985)
    Case reports
    • 64 year old woman with high grade noninvasive papillary urothelial carcinoma develops Poncet disease (rare, nondestructive parainfective symmetric polyarthritis) after intravesical Bacillus Calmette-Guérin treatment (BMC Res Notes 2017;10:416)
    • 69 year old woman with high grade noninvasive urothelial carcinoma of bladder later metastatic to uterus (Int J Gynecol Pathol 2017;36:493)
    • 24 cases of noninvasive urothelial carcinoma of bladder with glandular differentiation (Am J Surg Pathol 2009;33:1241)
    Treatment
    • Surgical: transurethral resection of tumor
    • Adjuvant therapy:
      • Intravesical immunotherapy with Bacillus Calmette-Guérin
      • Intravesical chemotherapy with thiotepa or mitomycin C
    Gross description
    • Exophytic single or multiple lesions with variation in size
    • More likely nontranslucent and hyperemic than low grade lesions
    Frozen section description
    • Papillary architecture with nuclear pleomorphism, dyscohesion and mitotic activity
    Microscopic (histologic) description
    • Fibrovascular cores lined by neoplastic urothelium
    • Complex, solid to fused papillae common
    • Architectural disorder; nuclear pleomorphism readily visible at low and intermediate power
    • Crowded overlapping cells, dyscohesion common and partial denudation
    • Nucleomegaly present, irregular and clumped chromatin
    • Frequent prominent nucleoli and mitoses (brisk and maybe atypical)
    • Concomitant low grade carcinoma may be present
    • Inverted growth pattern may coexist (both exophytic and endophytic growth)
    • Reference: Mod Pathol 2009;22:S60
    Microscopic (histologic) images

    Contributed by Michelle R. Downes, M.D., Nicole K. Andeen, M.D. and Maria Tretiakova, M.D.
    Cystectomy section

    Cystectomy section

    Transurethral bladder resection

    Transurethral bladder resection

    Cross section of ureter

    Cross section of ureter

    Architecture and cytology

    Architecture and cytology

    Cytologic features

    Cytologic features


    Noninvasive papillary urothelial carcinoma, high grade (pTa) Noninvasive papillary urothelial carcinoma, high grade (pTa)

    Noninvasive papillary urothelial carcinoma, high grade (pTa)

    CK7 expression

    CK7 expression

    GATA3 expression

    GATA3 expression

    CK5/6 loss

    CK5/6 loss

    p53

    p53

    Cytology description
    • Moderate / severe nuclear hyperchromasia, irregular nuclear membrane, coarse chromatin, nuclear/cytoplasmic ratio ≥ 0.7; mitoses and necrotic debris may be present
    • Cytology cannot distinguish invasive from noninvasive high grade or carcinoma in situ
    • Shows high sensitivity (up to 84%) for high grade tumors (Urol Oncol 2015;33:66.e25)
    • Use of 2016 Paris Working Group Reporting system recommended (Acta Cytol 2016;60:185)
    Cytology images

    Contributed by Zeina Ghorab, M.D. and Bonnie Choy, M.D.
    High grade cytology

    High grade cytology

    High grade urothelial carcinoma High grade urothelial carcinoma High grade urothelial carcinoma High grade urothelial carcinoma High grade urothelial carcinoma

    High grade urothelial carcinoma

    Positive stains
    Negative stains
    Electron microscopy description
    • High grade papillary urothelial carcinoma shows focal loss of zonula occludens (tight junctions) and macula adherens (desmosomes) (Cancer 1971;27:71)
    Molecular / cytogenetics description
    • Genetic or epigenetic changes in TP53 gene or TP53 regulatory gene (CDKN2A or p16)
    • Somatic mutations in TERT in 70 - 80% noninvasive urothelial carcinomas
    • Mutations in PIK3CA (25% of cases), TSC1, HRAS, APC
    • Epigenetic silencing of tumor suppressor genes via promoter hypermethylation): RUNX3, CDKN2A, MLH1, MGMT, VHL, DAPK, TBX2, TBX3, GATA2, ZIC4, GSTP1, CDH1 (Eur Urol 2012;61:1245, Nat Rev Urol 2013;10:327)
    • MicroRNA alterations (J Pak Med Assoc 2018;68:759)
    • Loss of chromosome 9
    Videos

    Urothelial carcinoma, papillary and invasive

    Sample pathology report
    • Bladder lesion, transurethral resection:
      • Papillary urothelial carcinoma, high grade (grade 3/3 - WHO 1973)
      • Noninvasive, pTa
      • Muscularis propria sampled
      • No lymphovascular invasion
      • Negative for urothelial carcinoma in situ
    Differential diagnosis
    • Low grade urothelial carcinoma:
      • Cells more uniform and appear orderly on low power
      • Chromatin evenly distributed
      • Mitoses may be present but usually confined to lower half of urothelium
      • Prominent umbrella cells present
    • Papillary polypoid cystitis:
      • Broad based papillary fronds with edematous or fibrous stroma
      • No complex branching papillary structures
      • Reactive urothelial atypia may be present
    • Papillary nephrogenic adenoma:
      • Single layer of cuboidal cells lining papillae
      • Other patterns present: tubular, tubulocystic, diffuse, solid
      • Positive for PAX2, PAX8, AMACR expression
    • Prostatic type polyp:
      • Papillary architecture lined by both urothelial and prostatic cells
      • PSA and PAP expression in secretory cells
    Board review style question #1

    Which of the following is true about the noninvasive papillary bladder lesion in the above image?

    1. Immunohistochemistry is helpful for diagnosis
    2. Progression to invasive disease is rare
    3. Common presentation include dysuria, urgency, frequency
    4. Neoplastic cells dyscohesion and partial denudation of urothelium is rare
    5. Cytologic atypia, nuclear pleomorphism and loss of polarity are common
    Board review style answer #1
    E. Cytologic atypia, nuclear pleomorphism and loss of polarity are common

    Comment Here

    Reference: Noninvasive papillary urothelial carcinoma high grade
    Board review style question #2
    Which of the following molecular changes has been described in high grade noninvasive papillary urothelial carcinoma?

    1. TP53
    2. FGFR3
    3. STAG3
    4. Chromosome 11 point mutations
    5. Germline mutations in TERT promoter
    Board review style answer #2

    Noninvasive papillary urothelial carcinoma low grade
    Definition / general
    • Neoplastic proliferation of the urothelium in a papillary configuration, with no invasion through the basement membrane
    • Low grade architectural and cytologic abnormality, absence of high grade features, such as irregular nuclei with frequent, prominent nucleoli and mitoses, pleomorphism
    Essential features
    • Noninvasive papillary urothelial neoplasm with low grade cytoarchitectural abnormality
    • Loss of polarity, rare mitoses, subtle variation in nuclear size but no significant pleomorphism
    • Immunohistochemistry not helpful for diagnosis
    • Hematuria common presentation
    • Disease related death and progression is rare but recurrence common
    Terminology
    • Grade 1 and subset of grade 2 carcinomas from 1973 WHO classification
    ICD coding
    • ICD-O: 8130/2 - papillary transitional cell carcinoma, noninvasive
    Epidemiology
    Sites
    • Most commonly found in posterior and lateral walls of bladder
    • 85% of urothelial neoplasms of renal pelvis are papillary but only 34% of them are low grade (Mod Pathol 2005;18:11)
    • Anywhere with urothelium
    Pathophysiology
    • Normal urothelium, through loss of chromosome 9, becomes hyperplasia
    • Followed by further genetic alterations, such as mutations in FGFR3, which activates mitogen activated protein (MAP) kinase pathway, leading to development of low grade noninvasive papillary urothelial carcinoma (Int J Clin Oncol 2008;13:287)
    • Other genetic alterations: PIK3CA, loss of 11p or CCND1
    Etiology
    Clinical features
    • Most common presentation: painless hematuria (gross or microscopic)
    • Disease associated death and progression is rare (< 5%)
    • Recurrence common (48 - 71%)
    Diagnosis
    • Cystoscopy: exophytic lesion, solitary or multiple, with varying size
    • CT urography or ultrasound
    • Urine cytology
    Radiology description
    • CT urography shows hydronephrosis and filling defects (Abdom Radiol (NY) 2018;43:663)
    • Ultrasound can detect hydronephrosis and any intraluminal bladder masses
    Prognostic factors
    Case reports
    Treatment
    • Surgical: transurethral resection
    • Adjuvant: usually not indicated for low grade
    • Mitomycin C
    Gross description
    • Exophytic papillary lesions, single or multiple; can vary greatly in size
    Frozen section description
    Microscopic (histologic) description
    • Neoplastic urothelium lining fibrovascular cores
    • Long, slender papillae with minimal fusing or branching
    • Orderly architecture at low magnification, some loss of polarity and mild pleomorphism at medium magnification
    • Cells generally uniform in size, may have slight variation but no significant nuclear pleomorphism or nucleomegaly, occasional slight irregularities in nuclear contour
    • Mitoses may be present but not atypical and usually confined to lower half of urothelium
    • Inverted growth pattern (exophytic and endophytic components) may be present
    Microscopic (histologic) images

    Contributed by Michelle R. Downes, M.D., Nicole K. Andeen, M.D. and Maria Tretiakova, M.D.
    Low grade, noninvasive papillary urothelium carcinoma

    Fibrovascular cores lined by neoplastic urothelium

    Renal pelvis

    Noninvasive papillary, low grade urothelial carcinoma

    Papillary architecture

    Papillary architecture

    Architectural features

    Architectural features

    Endophytic growth pattern

    Endophytic growth pattern

    GATA3

    GATA3


    p53 staining

    p53

    Ki67

    Ki67

    Noninvasive papillary urothelial carcinoma, low grade (pTa) Noninvasive papillary urothelial carcinoma, low grade (pTa) Noninvasive papillary urothelial carcinoma, low grade (pTa)

    Noninvasive papillary urothelial carcinoma, low grade (pTa)

    Papillary urothelial<br>carcinoma, muscularis<br>is absent

    Papillary urothelial
    carcinoma, absent
    muscularis

    Cytology description
    Cytology images

    Contributed by Zeina Ghorab M.D. and Bonnie Choy, M.D.
    Cytology of non high grade lesion

    Cytology of
    non high grade lesion

    Low grade urothelial neoplasia

    Low grade urothelial neoplasia

    Negative stains
    Electron microscopy description
    • Subset with urothelial eddies show microvillous projections and increased intercellular space (Int J Clin Exp Pathol 2013;6:1458)
    • Low grade papillary urothelial carcinoma has zonula occludens (tight junctions) and well developed macula adherens (desmosomes), with fewer surface vesicles than normal epithelium
    Molecular / cytogenetics description
    • Activating mutations in FGFR3
    • Somatic mutations in TERT promoter in 50% of low grade papillary noninvasive urothelial carcinomas; more commonly associated with FGFR3 mutated tumors (Histopathology 2018;72:795)
    • Inactivating mutations in cohesion complex gene STAG2 in 32 - 36% noninvasive urothelial carcinomas; associated with low histologic grade and low tumor stage (Ann Surg Oncol 2017;24:4059)
    • PIK3CA (25% cases) TSC1, HRAS, APC
    • Epigenetic silencing of tumor suppressor genes through promoter hypermethylation: RUNX3, CDKN2A, MLH1, MGMT, VHL, DAPK, TBX2, TBX3, GATA2, ZIC4, GSTP1, CDH1 (Eur Urol 2012;61:1245, Nat Rev Urol 2013;10:327)
    • MicroRNA alterations (J Pak Med Assoc 2018;68:759)
    • Loss of chromosome 9 (9q and 9p in low grade)
    Videos

    Urothelial carcinoma, papillary and invasive

    Sample pathology report
    • Bladder, lesion, transurethral resection:
      • Noninvasive papillary urothelial carcinoma, low grade
      • Muscularis propria is present
    Differential diagnosis
    Board review style question #1

    A transurethral resection of bladder was performed on a 62 year old man. Histological examination showed a noninvasive papillary lesion. Which of the following is true about the lesion depicted above?

    1. Immunohistochemistry is helpful for diagnosis
    2. Mitoses are rare and usually confined to lower half of urothelium
    3. Nuclear pleomorphism is marked
    4. Progression to invasive disease is common
    Board review style answer #1
    B. Mitoses are rare and usually confined to lower half of urothelium. Diagnosis: low grade noninvasive papillary urothelial carcinoma.

    Comment Here

    Reference: Noninvasive papillary urothelial carcinoma low grade
    Board review style question #2

    Which of the following is the correct diagnosis?

    1. High grade noninvasive papillary urothelial carcinoma
    2. Invasive papillary urothelial carcinoma
    3. Inverted papilloma
    4. Low grade noninvasive papillary urothelial carcinoma
    5. Urothelial papilloma
    Board review style answer #2
    D. Low grade noninvasive papillary urothelial carcinoma

    Comment Here

    Reference: Noninvasive papillary urothelial carcinoma low grade

    Papillary urothelial neoplasm of low malignant potential
    Definition / general
    • Neoplastic proliferation of the urothelium in a papillary configuration, with no invasion through the basement membrane
    • Thickened urothelium or increased cellularity, without marked cytological atypia
    Essential features
    • Epithelial lining of fibrovascular cores is thicker than normal urothelium
    • No nuclear atypia or hyperchromatic nuclei in neoplastic urothelial cells
    • Maybe exophytic papillary or endophytic papillary (inverted)
    • Exclusion criteria is prior history of urothelial carcinoma
    Terminology
    ICD coding
    • ICD-O: 8130/1 - papillary transitional cell neoplasm of low malignant potential
    • ICD-11: 2F78 & XH5UU5 - neoplasms of uncertain behavior of urinary organs & papillary urothelial neoplasm of low malignant potential
    Epidemiology
    Sites
    • Most diagnosed in the bladder
    • Occurs anywhere along urinary mucosae
    Pathophysiology
    Etiology
    Clinical features
    • Gross or microscopic hematuria
    • Urine cytology is negative in most cases
    Diagnosis
    • Cystoscopy: exophytic lesion with varying sizes (no specific cut off) but usually a small and single lesion
    Radiology description
    • Bladder mass with polypoid configuration and papillary / exophytic surface
    • Inverted tumor shows a polypoid shape with a nonpapillary surface
    Radiology images

    Images hosted on other servers:

    Exophytic lesion with internal vascularity

    Prognostic factors
    • This diagnostic category is justified because the recurrence and progression rates of papillary urothelial neoplasm of low malignant potential (PUNLMP) are expected to be higher than urothelial papilloma and lower than low grade noninvasive urothelial carcinoma
    • Recurrence rate: 18 - 20% (Am J Clin Pathol 2010;133:788, Diagn Pathol 2015;10:3)
    • Progression rate: 1.9 - 11.1%, mainly to low grade noninvasive papillary urothelial carcinoma (Am J Clin Pathol 2010;133:788, Diagn Pathol 2015;10:3)
    • ~1% progress to invasive urothelial carcinoma (Diagn Pathol 2015;10:3)
    • Limited data on recurrence of inverted PUNLMP but it seems to be rare (Diagn Pathol 2015;10:3)
    • In a population based study from Sweden, 5 year recurrence rates of PUNLMP and noninvasive low grade papillary urothelial carcinoma were 21% and 42%, respectively; progression rates were 0.7% and 4%, respectively (Scand J Urol 2022;56:14)
    • Not all series show differences in recurrence and progression between PUNLMP and low grade noninvasive urothelial carcinoma:
      • 5 year recurrence rate is 51% for PUNLMP versus 48% for low grade papillary urothelial carcinoma
      • Progression to muscle invasive disease is 3.9% for PUNLMP versus 2.6% for low grade papillary urothelial carcinoma (Urol Oncol 2020;38:440)
    Case reports
    Treatment
    • Surgical: transurethral resection with no need of further topical treatments
    Clinical images

    Images hosted on other servers:

    Exophytic papillary - recurrent PUNLMP

    Cystoscopy: exophytic papillary

    Gross description
    • Variable size but most tumors are single and regular polypoid masses measuring < 2 cm
    Gross images

    Contributed by Daniel Athanazio, M.D., Ph.D.

    Fresh specimen

    Formalin fixed specimen

    Microscopic (histologic) description
    • Noninvasive papillary urothelial neoplasm with exophytic or endophytic (inverted) configuration; a cut off of > 80% is proposed by the Genitourinary Pathology Society to designate a urothelial neoplasm of inverted type (Adv Anat Pathol 2021;28:179)
    • Epithelial lining of fibrovascular cores is thicker than normal urothelium: urothelial cells show monotonous appearance and slight cytoplasmic and nuclear enlargement
    • No variation in nuclear size, shape or chromatin pattern
    • Preserved polarity of urothelial cells
    • Mitoses are rare and basally located
    • No hyperchromatic nuclei in urothelial cells in intermediate layers of neoplastic epithelium
    Microscopic (histologic) images

    Contributed by Daniel Athanazio, M.D., Ph.D. and Luciana Schultz, M.D., Ph.D. (source: Instituto de Anatomia Patológica)
    Exophytic intraluminal urothelial papillary neoplasm Exophytic intraluminal urothelial papillary neoplasm

    Exophytic intraluminal urothelial papillary neoplasm

    Preserved polarity and no atypia Preserved polarity and no atypia

    Preserved polarity and no atypia

    Thickened urothelium and hypercellularity

    Thickened urothelium and hypercellularity

    Thickened urothelium / no atypia

    Thickened urothelium / no atypia


    Papillary exophytic neoplasm

    Papillary exophytic neoplasm

    Preserved polarity and hypercellularity

    Preserved polarity and hypercellularity

    Basally located mitosis

    Basally located mitosis

    Transition zone between normal ureteral urothelium and a shoulder lesion Transition zone between normal ureteral urothelium and a shoulder lesion

    Transition zone between normal ureteral urothelium and a shoulder lesion


    Endophytic / inverted growth Endophytic / inverted growth

    Endophytic / inverted growth

    Exophytic and endophytic growth

    Exophytic and endophytic growth

    Preserved polarity

    Preserved polarity

    Thickened epithelium / no atypia

    Thickened epithelium / no atypia

    Thickened epithelium / inverted growth

    Thickened epithelium / inverted growth


    Thickened epithelium / inverted growth

    Thickened epithelium / inverted growth

    Preserved polarity

    Preserved polarity

    Papillary exophytic

    Papillary exophytic

    Thickened  epithelium

    Thickened epithelium

    No atypia No atypia

    No atypia


    Papillary exophytic

    Papillary exophytic

    Hypercellular monotonous urothelial lining Hypercellular monotonous urothelial lining Hypercellular monotonous urothelial lining

    Hypercellular monotonous urothelial lining

    No atypia

    No atypia

    Preserved polarity

    Preserved polarity


    GATA3

    GATA3

    Cytokeratin 5

    CK5

    p16

    p16

    Virtual slides

    Images hosted on other servers:

    H&E

    GATA3

    CK5

    p16

    Cytology description
    • Usually not detected in urinary cytology, unless entire papillary formations are seen (then a diagnosis of low grade urothelial neoplasm, including both PUNLMP and low grade noninvasive papillary urothelial carcinoma, may be suggested as a comment within the diagnosis of “negative for high grade urothelial carcinoma”)
    • The Paris System states that, in most cases, urinary cytology is unable to detected low grade urothelial neoplasms, including PUNLMP (J Am Soc Cytopathol 2016;5:177, J Am Soc Cytopathol 2022;11:62)
    Positive stains
    • Markers of urothelial differentiation (not helpful in the differential diagnosis)
    Negative stains
    • Not helpful
    Molecular / cytogenetics description
    Videos

    Differential diagnosis of noninvasive endophytic urothelial neoplasms

    PUNLMP at cytology

    Sample pathology report
    • Bladder, lesion, transurethral resection:
      • Low grade urothelial neoplasm of low malignant potential (see comment)
      • Muscularis propria is present
      • Comment: The sample shows urothelial neoplasia with thickened epithelial lining and no atypia. There are no invasive foci. The findings are consistent with the diagnosis of urothelial neoplasm of low malignant potential (PUNLMP), an indolent tumor expected to have lower rates of recurrence and progression when compared to noninvasive low grade papillary urothelial carcinoma. For a sample of an incompletely resected multifocal or larger tumor, complete excision of all visible lesions for pathologic analysis is recommended. The diagnosis of PUNLMP requires exclusion of prior history of urothelial carcinoma. In cases involving a prior history of carcinoma, the present tumor should be considered a recurrence of the original neoplasia.
    Differential diagnosis
    Board review style question #1

    A transurethral resection of bladder was performed on a 60 year old man. Histological examination showed a noninvasive papillary lesion with fibrovascular cores lined by thickened urothelium with cells with monotonous appearance and no nuclear atypia. Which of the following additional information would exclude the diagnosis of urothelial neoplasm of low malignant potential?

    1. Multicentric disease
    2. Prior history of prostate adenocarcinoma
    3. Prior history of urothelial carcinoma
    4. Size larger than 3 cm
    Board review style answer #1
    C. Prior history of urothelial carcinoma

    Comment Here

    Reference: Papillary urothelial neoplasm of low malignant potential
    Board review style question #2
    The differential diagnosis between urothelial neoplasm of low malignant potential and noninvasive low grade papillary urothelial carcinoma usually shows poor reproducibility in different studies. Which of the following is the most important criterion favoring the diagnosis of urothelial carcinoma?

    1. EGFR3 mutation
    2. Scattered hyperchromatic nuclei among intermediate neoplastic urothelial cells
    3. TERT promoter mutation
    4. TP53 mutation
    Board review style answer #2
    B. Scattered hyperchromatic nuclei among intermediate neoplastic urothelial cells

    Comment Here

    Reference: Papillary urothelial neoplasm of low malignant potential

    Paraganglioma
    Definition / general
    • Nonepithelial neuroendocrine neoplasm producing catecholamine excess and derived from neural progenitors (Endocr Pathol 2022;33:90)
    Essential features
    • In patients presenting with catecholamine related symptoms, initial workup should include measurement of plasma or urine metanephrine and normetanephrine levels; patients with nonfunctional paraganglioma of the urinary bladder can be asymptomatic or present only with hematuria or irritative symptoms
    • Classic histologic morphology: monomorphic cells with abundant eosinophilic / amphophilic cytoplasm arranged in a nested / zellballen pattern and separated by fibrovascular septa with sustentacular cells
    • Tumor cells are positive for GATA3 and neuroendocrine markers while negative for keratins; sustentacular cells are positive for S100 and SOX10
    • All paragangliomas have a metastatic potential; many grading systems have been proposed
    • SDHB germline mutation is the strongest genetic prognosticator of poor outcomes; SDHB immunohistochemistry is routinely used for its identification
    Terminology
    • Also called extra-adrenal paraganglioma and sympathetic paraganglioma (Endocr Pathol 2022;33:90)
    • Discontinuation of the term extra-adrenal pheochromocytoma in the 2022 WHO classification
    • Categorization into benign and malignant is discouraged since all paragangliomas hold a metastatic potential (Endocr Pract 2022;28:1253)
    ICD coding
    • ICD-O: 8693/3 - extra-adrenal paraganglioma, malignant
    • ICD-11
      • 2C9Y & XH1UN6 - other specified malignant neoplasms of urinary tract & extra-adrenal paraganglioma
      • 2C9Y & XH0EW6 - other specified malignant neoplasms of urinary tract & paraganglioma, NOS
      • 2C9Y & XH4G21 - other specified malignant neoplasms of urinary tract & sympathetic paraganglioma
    Epidemiology
    Sites
    • Genitourinary paragangliomas are rare among all paragangliomas and genitourinary malignancies (BMC Urol 2013:13:22, Urol Oncol 2017;35:457.e9)
    • In the urinary tract, paraganglioma involves most commonly the bladder and rarely the urethra, renal pelvis and ureter
    • Bladder paraganglioma can occur anywhere throughout the bladder
    Pathophysiology
    • Various pathogenetic classification groups
      • Pseudohypoxia group (Cancer Cell 2017;31:181, F1000Res 2018:7:1500, Best Pract Res Clin Endocrinol Metab 2020;34:101416, Cancers (Basel) 2021;13:2389)
        • VHL / EGLN1 / EPAS1 related (15 - 20% of paragangliomas)
        • Tricarboxylic acid cycle (Krebs cycle) related (10 - 15% of paragangliomas; germline mutations in SDH subunits A - D, SDHAF2 assembly factor for SDH and FH gene)
      • Kinase signaling group (50 - 60% of paragangliomas; germline or somatic mutations in RET, HRAS, NF1, TMEM127 and MAX genes)
      • Wnt altered group (5 - 10% of paragangliomas; somatic mutations in CSDE1 and MAML3 genes)
      • Disease modifying gene group (somatic mutations in ATRX, KMT2D, SETD2, TERT, TP53, KMT2C and SMARCA4 genes)
      • Cortical admixture group (mutations in CYP11B1, CYP21A2 and STAR genes)
    Etiology
    Clinical features
    • Can be asymptomatic or symptomatic (World J Urol 2022;40:2807)
    • Classic presentation consists of catecholamine related symptoms / signs: paroxysmal hypertension, headache, syncope, diaphoresis, palpitations / tachycardia, dizziness (Cancer 2000;88:844, BMC Urol 2013:13:22, Pathol Res Pract 2015;211:183)
      • Symptoms can be provoked / aggravated by the manipulation of the mass during cystoscopy or intraoperatively
    • Bladder paragangliomas can be accompanied by hematuria and irritative symptoms including micturition attacks (World J Urol 2022;40:2807)
    • Paragangliomas can cause mass effects
    Diagnosis
    • Recommended initial testing in patients with classic catecholamine related symptoms / signs consists of measuring plasma free or 24 hour urine fractionated metanephrines and normetanephrines (J Clin Endocrinol Metab 2014;99:1915, J Clin Med 2018;7:280)
    • Computed tomography (CT) / magnetic resonance imaging (MRI) for the detection of primary / metastatic paraganglioma in the context of positive biochemical testing
    • Cystoscopy can be performed in the context of hematuria or imaging detected bladder mass
    • Histological diagnosis based on morphology and immunoprofile
      • WHO essential criteria: morphology typical of paragangliomas
      • WHO desirable criteria
        • Identification of specific genetic mutations
        • Positivity for synaptophysin and chromogranin A on immunohistochemistry
        • Biochemical confirmation of catecholamine excess for functional tumors
        • Compatible imaging studies
    Laboratory
    Radiology description
    Radiology images

    Images hosted on other servers:
    Axial nonenhanced CT, Doppler ultrasound, MRI, PET

    Axial nonenhanced CT, Doppler ultrasound, MRI, PET

    Axial contrast enhanced CT

    Axial contrast enhanced CT

    Coronal contrast enhanced CT, MIBG scan

    Coronal contrast enhanced CT, MIBG scan

    Prognostic factors
    • All paragangliomas carry a metastatic potential (Urol Oncol 2015;33:167.e13)
    • Multiple grading systems have been proposed to predict metastasis
      • Grading system for adrenal pheochromocytoma and paraganglioma (GAPP), with unfavorable factors being large and irregular cell nest, pseudorosette (even focal), comedo type necrosis, moderate to high cellularity, Ki67 labeling index > 1 - 3%, vascular or capsular invasion and noradrenergic type (Endocr Relat Cancer 2014;21:405)
      • Modified GAPP: based on the same prognostic factors as GAPP with the addition of SDHB loss on IHC (PLoS One 2017;12:e0187398)
      • Age, size, extra-adrenal location, secretory type score (ASES), with unfavorable factors being age ≤ 35 years, tumor size ≥ 6.0 cm and tumor producing only norepinephrine (Surgery 2018;164:511)
      • COPPS (composite pheochromocytoma / paraganglioma prognostic score), with unfavorable factors being focal or confluent necrosis, PS100 loss, vascular invasion, SDHB loss, tumor size > 7 cm, capsular invasion, increased mitotic figures and MCM6 (minichromosome maintenance) > 30% (Virchows Arch 2019;474:721)
    • Unfavorable factors for disease progression in metastatic bladder paragangliomas: male, advanced age, synchronous metastases, large tumor size, high dopamine level and nonresection of the primary tumor (Urol Oncol 2015;33:167.e13)
    • Molecular features associated with poorer outcome: markers associated with poor aggressive disease free survival included MAML3 fusion gene, SDHB germline mutation, somatic mutation in SETD2 or ATRX, high somatic mutation total, Wnt altered and pseudohypoxia expression subtypes and the hypermethylated subtype (Cancer Cell 2017;31:181)
    • Molecular features associated with better outcome: kinase signaling expression subtype and the low methylated subtype
    Case reports
    Treatment
    • Medical preparation for treatment: alpha blockade with or without the addition of other antihypertensive drugs as needed
    • Primary treatment
      • Resection with preference for minimally invasive procedures
      • For locally unresectable or metastatic cases, watchful waiting is recommended if the patient is asymptomatic or if the tumor is small and slow growing
      • For secreting tumors that are unresectable / with distant metastases, the recommendation is to continue alpha blockade with initiation of one of the following modalities as clinically warranted: clinical trial, radiation therapy, cytoreductive resection, MIBG therapy, sunitinib, systemic chemotherapy, peptide receptor radionuclide therapy and somatostatin analogues
    • Reference: NCCN: NCCN Guidelines - Neuroendocrine and Adrenal Tumors [Accessed 28 November 2023]
    Clinical images

    Images hosted on other servers:
    Cystoscopic findings

    Cystoscopic findings

    Laparoscopic findings

    Laparoscopic findings

    Robot assisted laparoscopic findings

    Robot assisted laparoscopic findings

    Gross description
    • Genitourinary paragangliomas are typically tan-pink to violaceous, encapsulated and firm (Abdom Radiol (NY) 2022;47:1414, Abdom Radiol (NY) 2022;47:4032, BMC Urol 2023;23:21)
    • Bladder paragangliomas are often submucosal or intramural and less frequently subserosal, with size at diagnosis up to 9.1 cm (mean: 3.9 cm)
    • Small bladder paragangliomas: typically well circumscribed, round ovoid and homogeneous
    • Larger bladder paragangliomas: usually more complex appearance with lobulation, peri and intratumoral vascularization and cystic / necrotic / hemorrhagic areas
    • Bladder paragangliomas may have calcification (up to 20% of cases) or local invasion of the pelvic side wall, uterus, vagina and ovaries
    Gross images

    Contributed by Debra L. Zynger, M.D.
    Missing Image

    Partial cystectomy



    Images hosted on other servers:
    Partial cystectomy

    Partial cystectomy

    Nephroureterectomy

    Nephroureterectomy

    Frozen section description
    • Highly vascularized tissues showing on histologic examination nests / sheets of polygonal neoplastic cells with eosinophilic or amphophilic cytoplasm, surrounded by stromal cells and blood vessels (Int J Surg Pathol 2018;26:213)
    • Focal signet ring features or dilated blood vessels may be seen
    Frozen section images

    Images hosted on other servers:
    Sheet arrangement

    Sheet arrangement

    Dilated blood vessels

    Dilated blood vessels

    Dilated blood vessels

    Lobular / nested pattern

    Microscopic (histologic) description
    • Typical morphology: epithelioid cells with abundant, eosinophilic / amphophilic, granular cytoplasm and round / ovoid, vesicular / salt and pepper nuclei, arranged in a nested / zellballen pattern and separated by fibrovascular septae with sustentacular cells (Am J Surg Pathol 2004;28:94, World J Clin Cases 2014;2:591, Endocr Pathol 2022;33:90)
    • May have focal pleomorphism, nuclear hyperchromasia, low mitotic activity or clear cytoplasm
    Microscopic (histologic) images

    Contributed by Theodorus H. van der Kwast, M.D., Ph.D., Michelle R. Downes, M.D., Debra L. Zynger, M.D. and David Cohen, M.B.B.Ch., M.D.
    Transurethral bladder resection

    Transurethral bladder resection

    Zellballen pattern

    Zellballen pattern

    Typical cytologic features

    Typical cytologic features

    Transurethral bladder resection

    Transurethral bladder resection

    Zellballen pattern

    Zellballen pattern

    Invasion of muscularis propria

    Muscularis propria invasion


    Transurethral bladder resection

    Transurethral bladder resection

    Nuclear atypia

    Nuclear atypia

    Zellballen pattern

    Zellballen pattern

    Amphophilic cytoplasm

    Amphophilic cytoplasm

    Pseudopapillary architecture

    Pseudopapillary architecture


    Cellular spindling

    Cellular spindling

    Mitotic activity

    Mitotic activity

    Muscularis propria invasion

    Muscularis propria invasion

    Deep lamina propria invasion

    Deep lamina propria invasion

    Pseudocapsule

    Pseudocapsule


    Typical cytologic features

    Typical cytologic features

    Typical cytologic features

    Typical cytologic features

    Synaptophysin

    Synaptophysin

    Chromogranin

    Chromogranin

    GATA3

    GATA3


    S100

    S100

    SDHB retained

    SDHB retained

    Pankeratin

    Pankeratin

    p63

    p63

    Chromogranin

    Chromogranin


    Synaptophysin

    Synaptophysin

    S100

    S100

    HMB45

    HMB45

    Synaptophysin

    Synaptophysin

    GATA3

    GATA3

    Virtual slides

    Images hosted on other servers:
    Transurethral bladder resection

    Transurethral bladder resection

    Encapsulated bladder paraganglioma

    Encapsulated bladder paraganglioma

    Bladder paraganglioma with infiltrative pattern

    Bladder paraganglioma with infiltrative pattern

    Cytology description
    • Tumor cells arranged in clusters or singly (Diagn Cytopathol 2017;45:350)
      • Epithelioid cells with abundant, finely granular, pale cytoplasm
      • Ovoid nuclei with smooth contour, fine chromatin and inconspicuous nucleoli
    • Occasional spindle sustentacular cells admixed within clusters of tumor cells
    • Naked nuclei present
    Positive stains
    Molecular / cytogenetics description
    Sample pathology report
    • Bladder, transurethral resection:
      • Paraganglioma, SDHB staining intact (see comment)
      • Involvement of muscularis propria
      • Unremarkable surface urothelium
      • Comment: The risk of malignancy in these lesions is estimated at 5 - 15% of cases. Malignant behavior cannot be predicted based on morphology alone and relies on the identification of metastases. Tumors associated with SDHB mutations (loss of SDHB staining) are reported to have an increased risk of metastases.
      • Microscopic description: This specimen shows an unencapsulated, monomorphous population of round tumor cells in a nested / zellballen pattern with delicate vascular septa and intervening extravasated red blood cells. It predominantly involves the lamina propria of the bladder but in 1 tissue piece clearly involves muscularis propria. The tumor cells have abundant amphophilic cytoplasm and vesicular chromatin with occasional small nucleoli. There is no necrosis, diffuse growth pattern or vascular space invasion identified. The mitotic count is (maximally) 1/10 high power fields. Immunohistochemistry shows the tumor cells to be positive for GATA3 with strong and diffuse expression of synaptophysin and chromogranin, as well as retention of SDHB staining. The tumor cells are negative for AE1 / AE3, HMWK and p63. The Ki67 proliferation index is < 1%. S100 shows faint nuclear and cytoplasmic staining with occasional sustentacular cells identified.

    • Bladder, transurethral resection:
      • Paraganglioma extending to the base of the tissue fragment (see comment)
      • No muscularis propria sampled
      • Comment: Paragangliomas in the bladder can exhibit local recurrence or metastasis in up to 15% of cases (BMC Urol 2013:13:22).
      • Microscopic description: Sections show unremarkable surface urothelium with a lesion in the deep lamina propria comprised of large eosinophilic cells with hyperchromatic, focally enlarged nuclei with occasional intranuclear inclusions. A fine vascular plexus is identified in some areas. Necrosis and hemorrhage are not identified. Mitotic rate = 1/10 high power fields (field of view [FOV] = 0.55 mm). The lesion extends to the cauterized base of the specimen. Immunostains confirm the lesion expresses synaptophysin, chromogranin and GATA3. SDHB is retained. AE1 / AE3, CD45, p63 and NKX3.1 are negative.
    Differential diagnosis
    Board review style question #1

    A 50 year old woman reported paroxysmal hypertension with episodic headache, palpitations and micturition attacks. Biochemical studies demonstrated elevated 24 hour urine metanephrines and normetanephrine. CT revealed a 2 cm bladder mass lesion which was resected transurethrally. Pathology showed paraganglioma. Regarding this entity, what immunostain can be performed to identify the strongest genetic prognosticator of poor outcomes?

    1. BRAF
    2. FH
    3. p53
    4. SDHB
    Board review style answer #1
    D. SDHB. SDHB germline mutation is the strongest genetic risk factor for metastatic disease. The diagnosis of SDHB related paraganglioma can be supported by the loss of SDHB immunoreactivity in the tumor cells. Answer A is incorrect because while the presence of BRAF mutation has been described in a small subset of cases, it has not been proven to be associated with an increased risk for metastatic disease. Answer C is incorrect because while TP53 has been suggested to be a disease modifying gene, its association with a more aggressive disease behavior requires further studies. Answer B is incorrect because FH is a novel genetic risk factor for tumor aggressiveness but it is not the strongest genetic prognosticator of poor outcomes in paragangliomas.

    Comment Here

    Reference: Paraganglioma
    Board review style question #2

    A 45 year old man presented with macroscopic hematuria and underwent cystoscopy, which revealed a 4 cm highly vascularized, pink violaceous, ovoid mass in the posterior bladder wall. Transurethral resection of the bladder tumor was performed. Which stain combination would be most consistent with bladder paraganglioma?

    1. Keratin negative, GATA3 negative, synaptophysin positive, S100 positive
    2. Keratin negative, GATA3 positive, synaptophysin positive, S100 positive
    3. Keratin positive, GATA3 negative, synaptophysin positive, S100 negative
    4. Keratin positive, GATA3 positive, synaptophysin negative, S100 negative
    Board review style answer #2
    B. Keratin negative, GATA3 positive, synaptophysin positive, S100 positive. In paragangliomas, tumor cells are positive for GATA3 and neuroendocrine markers while negative for keratins; sustentacular cells are positive for S100. Answer D is incorrect because that stain combination is most consistent with urothelial carcinoma. Answer C is incorrect because that stain combination is most consistent with epithelial neuroendocrine neoplasm. Answer A is incorrect because paragangliomas are typically positive for GATA3.

    Comment Here

    Reference: Paraganglioma

    Paris system for urothelial neoplasia
    Definition / general
    • Urine Cytology represents a significant portion on non-gynecologic cytology specimens in daily practice, primarily because of the simplicity and ease of specimen procurement and its significant impact on management
    • However there was no consensus on the various categories and their cytomorphologic features used for reporting
    • Paris System for reporting urinary cytology was an effort to standardize terminology with standardized cytomorphologic criteria for reporting urine cytology (2016)
    • Paris System is based on the principle that the ultimate goal of urine cytology is detection of high grade urothelial carcinoma
    Essential features
    • Identification of high grade urothelial carcinoma (HGUC) is the ultimate goal of urinary cytology and should always be kept in mind while reporting urine cytology; these patients will receive active investigation to find the lesion
    • Suspicious for High Grade Urothelial Carcinoma (SHGUC) has similar management due to its strong association with HGUC
    • Carcinoma in Situ (CIS) cannot be distinguished from HGUC in urine cytology
    • Low grade urothelial neoplasia (LGUN) can only be definitively diagnosed in the presence of 3 dimensional cellular papillary clusters with fibrovascular cores; if not present, report as negative for high grade urothelial carcinoma (NHGUC) with a possible comment for consideration of LGUN
    • Atypical urothelial cells (AUC) should be reported using strict morphologic criteria with 1 major and 1 minor criterion (see below)
    Diagrams / tables

    Contributed by Vaishali Pansare, M.D.

    Adequacy of urine specimens

    Clinical features
    • Cytology is useful to detect carcinoma in situ or marked chronic inflammation (i.e. when there is no specific lesion to biopsy), carcinoma hidden in diverticula or for detecting residual tumor from urine specimens
    • Cystoscopic biopsy of visible lesions is more sensitive than cytology in most cases
    • Bladder irrigation is superior to collecting voided urine
    • Most sensitive and highly specific for high grade tumors (diagnosis or follow-up) whether flat (carcinoma in situ), papillary or mixed
    • Low sensitivity (difficult to diagnose) for papilloma and low malignant potential lesions because they have normal histology (Mod Pathol 1995;8:394)
    • Follow up examination of urine with FISH may improve sensitivity and specificity of cytology (Am J Clin Pathol 2001;116:79)
    Types of specimen
    Voided urine:
    • Non invasive, easiest to obtain
    • Obtaining 3 second morning voided midstream urine samples collected over 3 consecutive days appears to optimize the detection of urothelial malignancies

    Instrumented urine:
    • Catheterization of the bladder or irrigation of bladder

    Ileal conduit urine:
    • Ileal conduit and neobladder are the most common urine diversion techniques used in patients who have undergone cystectomy
    • A portion of the ileum is anastomosed with the ureters to the skin or to the urethra
    Processing / preservation of specimen
    • Immediate processing is recommended or refrigerate if immediate processing cannot be done
    • If fixation if needed, use equal volumes of 50% ethanol or a methanol based fixative (Cytolyt® or similar)
    Format of diagnostic report
    Recommendations for Diagnostic Format and Categories for Urinary Cytology Specimens Papanicolaou Society of Cytopathology Practice Guidelines Task Force


    I. Adequacy Statement (Optional)
    • Satisfactory for evaluation
    • List any quality factors affecting specimen
    • Unsatisfactory for evaluation (give reason)

    II. General Categorization
    • Negative for epithelial cell abnormality (see Descriptive Diagnoses)
    • Epithelial cell abnormality present (see Descriptive diagnosis)

    III. Descriptive Diagnosis
    • Negative for epithelial cell abnormality
    • Infectious agents
    • Bacterial organisms
    • Fungal organisms
    • Viral changes (CMV, herpes, adenovirus, polyomavirus)
    • Nonspecific inflammatory changes
    • Acute inflammation
    • Chronic inflammation
    • Changes consistent with xanthogranulomatous pyelonephritis
    • Cellular changes associated with:
      Chemotherapeutic agents
      Radiation
    • Epithelial Cell Abnormalities
      Atypical urothelial cells (*see comment)
      Low-grade urothelial carcinoma
      High-grade urothelial carcinoma (invasive carcinoma vs. carcinoma in situ)
      Squamous cell carcinoma
      Adenocarcinoma
      Other malignant neoplasms (specify type)

    IV. Other
    • An optional comment section should be used to list additional findings or further clarification of findings
    • Diagn Cytopathol 2004;30:24
    Specimen adequacy
    • Adequacy refers to the usefulness of the specimen to diagnose or raise suspicion of urothelial carcinoma
    • Adequacy is determined by the interplay of 4 specimen characteristics: collection type, cellularity, volume and cytomorphologic findings
    • Of these, the cytomorphologic findings must be considered first because any atypical, suspicious or malignant cells make the specimen intrinsically adequate regardless of the volume, cellularity or collection type
    • Similar to Pap smear adequacy, any atypical, suspicious or malignant cells on cytology automatically becomes an adequate specimen
      • Adequate number of benign urothelial cells also supercedes any volume requirements for adequacy
      • Volume is only considered for voided specimens; low volume samples are less likely to have adequate urothelial cells, but an optimal volume has not been established
      • The Paris system indicates that for SurePath®, 30 ml is an optimal volume for voided urines but specimens should not be rejected based on low volume
      • Benign urothelial cellularity cut offs should be validated for instrumented and voided urines
    Negative for High Grade Urothelial Carcinoma (NHGUC)
    • To minimize lumping everything into the "atypical" category, the terminology Negative for High Grade Urothelial Carcinoma includes all entities that pose no significant risk to the patient for developing HGUC based upon available studies
    • This term also clarifies the goal of the Paris System - to highlight those cases at risk for HGUC
    • For example, radiation associated atypia is classified as Negative for High Grade Urothelial Carcinoma and not atypical
    • A urine sample (voided or instrumented) is considered Negative for High Grade Urothelial Carcinoma if any of the following components are present:
      • Benign urothelial, squamous and glandular cells
      • Benign urothelial tissue fragments
      • Changes associated with stones
      • Viral cytopathic effect due to polyoma virus
      • Post therapy effect, including epithelial cells from urinary diversions

    Contributed by James Liu, M.D.
    Missing Image

    Normal

    Missing Image Missing Image

    Polyoma (BK) virus

    Atypical Urothelial Cells (AUC)
    • Defined as cellular changes that fulfill the major (required) criterion and only 1 minor criterion
      • Note: the presence of 2 or more minor criterion including nuclear hyperchromasia is diagnostic of Suspicious for HGUC (see below)
    • Major criterion (required):
      • Non superficial and non degenerated urothelial cells with an increased N/C ratio (> 0.5)
    • Minor criteria (one required):
      • Nuclear hyperchromasia
      • Irregular nuclear membranes
      • Irregular, coarse and clumped chromatin
    • Diagnosis of AUC is appropriate when cells are more abnormal than NHGUC
    • AUC is appropriate when there is suspicion of HGUC but also extensive degeneration

    Note:
    • Normal intermediate and basal urothelial cells, typically seen in instrumented urine, have high N/C ratio and frequently occur in groups; should be regarded as NHGUC

    Contributed by James Liu, M.D.
    Missing Image Missing Image Missing Image

    Atypical urothelial cells

    Suspicious for High Grade Urothelial Carcinoma (SHGUC)
    • Reflects the presence of urothelial cells with severe atypia that falls short for a diagnosis of high grade urothelial carcinoma but beyond atypical urothelial cells
    • A diagnosis of SHGUC is defined as non superficial and non degenerated urothelial cells showing:
      • Increased N/C ratio, at least 0.5 - 0.7 (required criterion)
    • Moderate to severe nuclear hyperchromasia (required criterion) and at least one of the following:
      • Irregular clumpy chromatin
      • Marked irregular nuclear membranes
    High Grade Urothelial Carcinoma (HGUC)
    • Sensitivity of urine cytology for HGUC is 50 - 85%
    • Positive urine cytology is clinically meaningful, is significantly associated with tumor recurrence and is independent of other clinicopathologic variables
      • Hence, positive urine cytology in primary upper urinary tract urothelial carcinoma is valuable to predict prognosis and preoperative positive urine cytology may be associated with higher prevalence of tumor recurrence
    • It can be useful to predict tumor progression

      Notes:
    • Urine cytology cannot distinguish invasive HGUC and carcinoma in Situ (CIS)
    • Squamous or glandular differentiation of urothelial carcinoma may be seen in urine cytology but a diagnosis of squamous cell carcinoma or adenocarcinoma of the urinary tract can only be made after examination of biopsy or cystectomy specimens
    • HGUC is diagnosed on the basis of this criteria according to the Paris System consensus:
      • Cellularity; at least 5 - 10 abnormal cells
      • N/C ratio: 0.7 or greater
      • Nucleus: moderate to severe hyperchromasia
      • Nuclear membrane: markedly irregular
      • Chromatin: coarse / clumped
    • Other notable cytomorphologic features of HGUC are:
      • Cellular pleomorphism
      • Marked variation in cellular size and shapes. i.e. oval, rounded, elongated or plasmacytoid (comet cells)
      • Scant, pale or dense cytoplasm
      • Prominent nucleoli
      • Mitoses
      • Necrotic debris
      • Inflammation

    Contributed by James Liu, M.D.
    Missing Image Missing Image Missing Image Missing Image

    High grade urothelial carcinoma (HGUC)

    Low Grade Urothelial Neoplasia (LGUN)
    • LGUN is a combined cytologic term for low grade papillary urothelial neoplasms, which includes urothelial papilloma, papillary urothelial neoplasm of uncertain malignant potential (PUNLMP) and low grade papillary urothelial carcinoma (LGPUC)
    • Definitive diagnosis of LGUN is possible only in the presence of this cytologic criteria (regardless of voided urine or instrumented urine):
      • 3 dimensional cellular papillary clusters with fibrovascular cores including capillaries
      • Cellular papillary clusters are defined as clusters of cells with nuclear overlapping forming papillae
    • The following cytologic features should be categorized as NHGUC:
      • 3 dimensional cellular clusters without fibrovascular cores
      • Increased numbers of single monotonous (non umbrella) cells
    • Cytoplasmic homogeneity
    • Nuclear border irregularity
    • Increased N/C ratio
    • A comment may be added to suggest LGUN in these cases without definitive cytomorphologic features
    • Rate of progression is 0% for papillomas, 3.6% for PUNLMP and 5 - 25% for LGPUC (WHO / ISUP classification (2004))
    Squamous Cell Carcinoma (SCC)
    • Accounts for 2 - 5% of all bladder cancer in West
    • In North Africa and Middle East where Schistosoma hematobium infestation is endemic, it accounts for 25 - 30% of bladder malignancies
    • Cases not associated with Schistosoma (non bilharzial) are usually associated with conditions causing urinary stasis with epithelial injury, such as spinal cord injury or paraplegia
    • Cytologic features of bladder SCC are similar to SCC elsewhere
    • Diagnostic criteria for SCC:
      • Cellular specimen with numerous individual and nests of squamous cells
      • Tumor cells are large, polygonal with keratinized cytoplasm, sharp borders and mildly to markedly atypical hyperchromatic nuclei
      • Fiber and tadpole cells, squamous pearls and "cell in cell" arrangement may be present
      • Background may show fragments of anucleated squamous cells, small atypical parakeratotic cells, necrosis, RBCs and neutrophils
      • Nonkeratinizing malignant cell groups with metaplastic appearance may be present
      • Liquid based preparations show similar morphology but the background is cleaner so cell details are better preserved
    Adenocarcinoma
    • Accounts for 0.5 - 2.5% of all primary bladder malignancies and includes vesical and urachal subtypes
    • Urachal adenocarcinoma develops within urachal remnants located in bladder dome
    • Primary urinary tract adenocarcinoma is less common than secondary involvement from adjacent organs
    • Risk factors include cystitis glandularis of intestinal type and bladder exstrophy
    • Diagnostic criteria:
      • Variable cellularity
      • Enteric / colonic type columnar cell clusters and single degenerated cells in a background of necrosis and mucin
      • Nuclei are large vesicular or hyperchromatic, with irregular shapes and prominent nucleoli
      • Cytoplasm may be vacuolated
    • Mucinous / colloid type has rounded 3-D clusters of crowded, bland cells with small to moderate amounts of lacy cytoplasm with occasional mucin vacuoles and medium sized nuclei with visible nucleoli in mucinous background
    • Signet ring cell adenocarcinoma: cells with large cytoplasmic mucin filled vacuole that appear optically clear or finely vacuolated, pushing the nucleus to the periphery
    • Clear cell carcinoma: cells with abundant vacuolated cytoplasm and centrally located nuclei that may be present in clusters with hobnail configuration
    Small cell carcinoma
    • Accounts for less than 1% of all bladder malignancies
    • Diagnostic criteria:
      • Moderate to high cellularity
      • Hemorrhagic and necrotic background with apoptosis, isolated or small groups of small, undifferentiated malignant cells, mitoses and numerous neutrophils
      • Cells are arranged singly, in linear pattern with rosettes, loosely or tightly cohesive clusters
      • Tumor cells are round to oval or irregular and small to medium in size (2 - 3 x lymphocytes)
      • Nuclei are small to oval, hyperchromatic with finely granular evenly distributed or smudged chromatin, ill defined membranes, prominent molding and display crush artifact
      • Nucleoli are inconspicuous
      • Scanty cytoplasm
      • High N:C ratio
    Secondary neoplasms
    • Renal cell carcinoma:
      • May be seen in cases of renal pelvis invasion by renal cell carcinoma
      • Degenerated cells
      • Cells maintain the same morphology as the tumors in the kidney
    • Prostatic carcinoma:
      • Bladder neck involvement by prostatic carcinoma
      • Large cells in clusters with ill defined cell borders, vacuolated cytoplasm, round nuclei and prominent nucleoli
      • Immunohistochemistry may be helpful
    • Colonic carcinoma:
      • Direct extension to bladder with possible fistulae formation
      • Cells arranged in acinar configuration
      • Coarse chromatin pattern and prominent nucleoli
      • Necrosis and abundant red blood cells
      • Fecal material present

    Persistent cloaca
    Definition / general
    • Defined as confluence of rectum, vagina and urethra into a single common chamber that opens into a single orifice where the normal urethral opening is located
    • Represents a wide spectrum of defects with the common denominator of a single perineal orifice
    • The length of the common chamber varies from 1-10 cm (average 3 cm)
    Terminology
    • Also called cloacogenic bladder
    Epidemiology
    • Occurs in 1 per 20,000 births, only in girls
    Etiology
    • Cloaca is single canal from which the urinary, genital and intestinal tracts arise at gestational weeks 5-6
    • Persistent cloaca is due to failure of urogenital septum to divide rectum from urogenital sinus
    • May be related to B-class Eph/ephrin signaling (J Pediatr Urol 2007;3:354)
    Diagrams / tables

    Images hosted on other servers:

    Normal embryo at 25 - 27 days

    Normal embryo at weeks 5 - 7

    Diagram of persistent cloaca

    Clinical features
    • Dilated sac like structure in center of abdomen
    • 80% have an associated anogenital anomaly, including imperforate anus, absent genital or urinary orifices, renal agenesis, absence of prostate or seminal vesicles or poorly developed foregut derivatives
    • 30% of patients present with hydrocolpos (dilated vagina filled with fluid, urine or mucus) and 40% have a duplicated Müllerian system
    • May be identified prenatally by ultrasound or MRI in a female fetus presenting with hydronephrosis and a large cystic lesion arising from the pelvis (Congenit Anom (Kyoto) 2009;49:116)
    Case reports
    Treatment
    Additional references

    Plasmacytoid
    Definition / general
    • Aggressive variant of urothelial carcinoma (UC) characterized by single infiltrating cells with eccentrically placed nuclei and abundant eosinophilic cytoplasm similar to plasmacytes or resembling signet ring cells due to intracytoplasmic vacuoles
    Essential features
    • Uncommon histologic variant of UC (~1 - 3%) with plasmacytoid morphology
    • Discohesive infiltrating tumor cells can spread extensively along tissue planes and peritoneal surfaces
    • Plasmacytoid tumor cells are often positive for CD138, which can result in a misdiagnosis for a plasma cell neoplasm; must use other immunohistochemistry markers to differentiate
    • Often has loss of membranous E-cadherin due to mutations in CDH1, similar to lobular breast carcinoma
    • More likely to be diagnosed at an advanced stage compared with conventional UC resulting in a poorer prognosis
    Terminology
    • PUC (plasmacytoid urothelial carcinoma)
    • Plasmacytoid variant (PCV)
    • Signet ring cell variant
    ICD coding
    • ICD-O: 8120/3 - transitional cell carcinoma, NOS
    Epidemiology
    Sites
    Pathophysiology
    Etiology
    Clinical features
    Diagnosis
    • Clinical symptoms prompt cystoscopy, which then results in a transurethral biopsy or resection of tumor
    • Imaging may show focal mass in bladder wall or diffuse bladder wall thickening, which results in cystoscopy with biopsy
    Laboratory
    Radiology description
    • On CT / MRI, PUC case series demonstrated focal bladder mass in 68% of cases and diffuse mural thickening in 32% of cases (Can Urol Assoc J 2017;11:E50)
    • On CT / MRI, imaging specific to PUC compared with conventional UC may be discontinuous thick sheets of tissue extending along fascial pelvic planes (Can Urol Assoc J 2017;11:E50)
    Radiology images

    Images hosted on other servers:

    MRI of PUC with bladder muscle invasion

    MRI and CT of PUC with peritoneal spread

    CT of PUC with
    peritoneal spread
    and bladder wall
    thickening

    Prognostic factors
    Case reports
    • 33 year old woman presenting with macroscopic hematuria and a solid lesion (Urol J 2019;16:86)
    • 53 year old man with plasmacytoid urothelial carcinoma metastatic to the duodenum (Case Rep Pathol 2017;2017:5209059)
    • 57 year old man presenting with renal colic and a solid lesion (Urol J 2019;16:86)
    • 60 year old man with a small bowel obstruction and an obstructive ureteric mass on CT scan (Am J Case Rep 2018;19:158)
    • 69 year old woman with a history of bilateral hydronephrosis, presenting with weight loss and vaginal spotting (Case of the Month #510)
    • 71 year old man with incidental plasmacytoid bladder cancer causing bilateral ureteral obstruction, hydroureteronephrosis and renal failure (Urol Case Rep 2020;33:101415)
    Treatment
    • More likely to receive neoadjuvant chemotherapy compared with conventional UC because it is discovered at a higher stage (J Urol 2020;204:215)
      • Literature review found overall pathological complete response to neoadjuvant chemotherapy to be 21% (J Urol 2020;204:215)
    • Same treatment for urothelial carcinoma of the same stage (Eur Urol Focus 2020;6:653)
    • Role of neoadjuvant therapy before cystectomy has been questioned for PUC
    Gross description
    Gross images

    Contributed by Timothy Isaac Miller, M.D., M.A. and Maria Tretiakova, M.D., Ph.D.

    Bladder with plasmacytoid urothelial carcinoma

    Frozen section description
    Microscopic (histologic) description
    • Discohesive single cells with eccentrically placed nuclei and abundant eosinophilic cytoplasm
    • Often deeply infiltrative but with minimal stromal reaction (Eur Urol Focus 2020;6:653)
    • Further subclassified into classic, pleomorphic and desmoplastic subtypes:
      • Classic: signet ring-like morphology, singly scattered and discohesive in loose aggregates forming cords (Hum Pathol 2019;90:27)
      • Pleomorphic: similar to classic but with pleomorphic nuclei and more atypia; can be rhabdoid and bizarre appearing (Hum Pathol 2019;90:27)
      • Desmoplastic: plasmacytoid neoplastic cells with a surrounding desmoplastic stromal response (Hum Pathol 2019;90:27)
    • Associated with sarcomatoid variant in 31% of cases (Hum Pathol 2019;90:27)
    • Often mixed with other histologic subtypes; in 1 case series, 53% of PUC had mixed histologic subtypes (Am J Clin Pathol 2017;147:500)
    • Despite resemblance to signet ring carcinoma cells, notably lack extracellular mucin, contrasting to signet ring adenocarcinoma (Acta Cytol 1991;35:277, Am J Surg Pathol 1991;15:569)
    Microscopic (histologic) images

    Contributed by Timothy Isaac Miller, M.D., M.A., Nicole K. Andeen, M.D. and Maria Tretiakova, M.D., Ph.D.

    PUC with carcinoma in situ

    Muscularis propria involvement

    Diffuse growth

    Pleomorphic subvariant

    Desmoplastic subvariant


    Signet ring and classic morphologies

    Lymphovascular invasion

    Perineural invasion


    PUC in bladder wall

    PUC at urethral margin

    AE1 / AE3

    CD138




    Contributed by Lisa Han, M.D. and Ricardo Lastra, M.D. (Case #510)

    Urothelial carcinoma, plasmacytoid variant

    Cytology description
    Cytology images

    Images hosted on other servers:

    Bladder washing with PUC (Papanicolau stain, x400)

    Immunohistochemical panels
    Molecular / cytogenetics description
    Videos

    Plasmacytoid variant histology in bladder cancer

    Sample pathology report
    • Bladder, transurethral resection:
      • Urothelial carcinoma, high grade
        • Histologic component: plasmacytoid (90%)
        • Adjacent flat CIS: absent
        • Angiolymphatic invasion: absent
        • Muscularis propria: present, invaded by carcinoma
        • pT: invasive of muscularis propria (pT2)
    • Bladder, cystectomy:
      • Invasive high grade urothelial carcinoma, plasmacytoid variant (see synoptic report)
    Differential diagnosis
    Board review style question #1

    Which of the following is true regarding the histologic variant of urothelial carcinoma shown above?

    1. Neoplastic cells usually express CD138
    2. Mutations / loss of CDH1 are uncommon
    3. Patients are less likely to present at an advanced stage compared with conventional urothelial carcinoma
    4. Primary ureteral carcinoma of this variant is common
    5. Neoplastic cells are usually positive for mucin
    Board review style answer #1
    A. Neoplastic cells usually express CD138. The histologic variant of urothelial carcinoma shown above is the plasmacytoid variant, consisting of discohesive cells with eccentrically placed nuclei and abundant cytoplasm. The neoplastic cells are usually positive for CD138 (making choice A the correct answer). Choice B is false, as truncating CDH1 mutations are common, causing loss of E-cadherin. Choice C is false, as these patients are generally diagnosed at an advanced stage and thus, have a worse prognosis. Choice D is incorrect because the primary ureteral of the variant has rarely been reported. Choice E is false since the cells do not express mucin, making it possible to distinguish from metastatic signet ring cell adenocarcinoma. (Hum Pathol 2019;90:27, Int J Clin Exp Pathol 2012;5:601, Nat Genet 2016;48:356, Eur Urol Focus 2019;5:104, Urology 2017;102:143, Bladder Cancer 2020;6:71, Am J Case Rep 2018;19:158, Am J Surg Pathol 2008;32:752).

    Comment Here

    Reference: Plasmacytoid
    Board review style question #2
    Which of the following immunoprofiles would be most helpful to differentiate primary plasmacytoid urothelial carcinoma of the bladder from metastatic lobular breast carcinoma?

    1. CK7, E-cadherin
    2. E-cadherin, uroplakin II
    3. GATA3, CK7
    4. GATA3, GCDFP-15
    5. Uroplakin II, ER
    Board review style answer #2
    E. Uroplakin II, ER. Membranous loss of E-cadherin will be seen in both lobular breast carcinoma and plasmacytoid UC. GATA3 and CK7 are usually expressed by both. GCDFP-15 can be positive in some cases of plasmacytoid UC (~25% in 1 study) and also positive in lobular breast carcinoma. ER is positive in most cases of lobular breast carcinoma and negative in plasmacytoid UC. Uroplakin II is negative in lobular breast carcinoma and may be positive in plasmacytoid UC. Overall, given these patterns, the immunoprofile most likely to discriminate between the 2 are uroplakin II and ER (choice E). (Breast Cancer Res 2015;17:12, Hum Pathol 2019;90:27, BMC Urol 2020;20:72, Ann Diagn Pathol 2015;19:6, Am J Surg Pathol 2017;41:1570, Cancers (Basel) 2021;13:3695, BMC Cancer 2014;14:546, Am J Clin Pathol 2017;147:500).

    Comment Here

    Reference: Plasmacytoid
    Board review style question #3
    Which panel of immunohistochemical stains would be most helpful to distinguish plasmacytoid urothelial carcinoma from its mimickers at other primary sites?

    1. CD138, CK20, CK7, ER, PR
    2. CD138, CK20, CDX2, GCDFP-15
    3. p63, GATA3, E-cadherin
    4. GATA3, mammaglobin, ER, Uroplakin II
    Board review style answer #3
    D. GATA3, mammaglobin, ER, Uroplakin II. A panel including GATA3, mammaglobin, ER and uroplakin II would be helpful. GATA3, more so than p63, is positive in plasmacytoid urothelial carcinoma (PUC) and while it does not differentiate between breast and urothelial origin, is helpful in ruling out gastrointestinal metastases. Mammaglobin and ER are consistently negative in PUC, which helps differentiate PUC from breast metastasis, particularly lobular carcinoma, which is more often ER positive. UroplakinII is helpful as it is negative in nonurothelial mimickers. CD138 has been reported to be positive in PUC although it is nonspecific with expression reported in squamous cell carcinoma, normal urothelium and other epithelial and lymphoid malignancies (Int J Surg Pathol 2016;24:614). E-cadherin expression is typically abnormal in both PUC and breast lobular carcinoma, which includes complete absence of staining or aberrant cytoplasmic staining. Of note, E-cadherin staining was normal in 27% of PUC cases in a recent study (Hum Pathol 2020;102:54).

    Comment Here

    Reference: Plasmacytoid

    Polypoid / papillary cystitis
    Definition / general
    • Exophytic polypoid to papillary structures characterized histologically by normal or mildly hyperplastic urothelium overlying a congested, chronically inflamed and edematous stroma (Arch Pathol Lab Med 2012;136:721)
    Essential features
    • Exophytic bullous, polypoid or papillary structures covered by benign urothelium and composed of stromal cores with varying degree of edema, vascular ectasia, congestion, inflammatory infiltrate and fibrosis (Am J Surg Pathol 1988;12:542)
    • Dense fibrosis with chronic inflammation in older polypoid cystitis (Surg Pathol Clin 2008;1:129)
    Terminology
    ICD coding
    • ICD-10:
      • N30.20 - other chronic cystitis without hematuria
      • N30.21 - other chronic cystitis with hematuria
    Epidemiology
    Sites
    Pathophysiology
    • Injury and inflammatory irritation to the urothelium leading to chronic inflammation and reactive changes
    • Edema of the lamina propria
    • Inflammation and fibrosis of the lesion in chronic phase (papillary cystitis) (Surg Pathol Clin 2008;1:129)
    Etiology
    • Catheterization: occurs in more than 80% of patients with indwelling catheters (Acta Pathol Microbiol Scand A 1979;87A:179)
    • Vesical fistulae: fistulae between the urinary bladder and the gastrointestinal tract
    • Radiation therapy for bladder, prostate or gynecologic cancer
    • Stones
    Clinical features
    • Urinary frequency, urgency, dysuria, hematuria, bladder obstruction
    • Pneumaturia and fecaluria in patients with vesico-intestinal fistula (Br J Surg 1964;51:644)
    • Associated with vesical fistulae resulting from Crohn's disease, colonic diverticulitis, colon cancer or appendicitis (Urology 1979;13:115)
    Diagnosis
    Radiology description
    Radiology images

    Images hosted on other servers:

    Irregular wall thickening

    Prognostic factors
    • Catheter associated lesions regress within 6 months of removal of the indwelling catheter (J Urol 1983;130:456)
    • In non-catheter associated lesions, no recurrence in 6 months to 2 years after initial diagnosis (Int Urol Nephrol 2002;34:293)
    • No increased risk for progressing to carcinoma
    Case reports
    Treatment
    • Removal of the indwelling catheter for catheter associated polypoid cystitis (J Urol 1983;130:456)
    • Removal of the irritating agent
    Clinical images

    Images hosted on other servers:

    Tumor-like lesion

    Gross description
    Microscopic (histologic) description
    • Variable exophytic lesions: bullous (width > height), polypoid (broad based, width < height) or papillary (narrow based, width < height)
    • No simple or complex branching of papillae
    • Stromal cores with varying degree of edema, vascular ectasia with congestion, acute and chronic inflammatory infiltrate and fibrosis
    • Overlying urothelium may be normal, metaplastic (squamous or mucinous) or hyperplastic (Am J Surg Pathol 1988;12:542)
    • Reactive urothelial atypia may be present
    • Dense fibrosis with chronic inflammation in older lesions (Surg Pathol Clin 2008;1:129)
    Microscopic (histologic) images

    Contributed by Maria Carolina Beeter, M.D. and Y. Albert Yeh, M.D., Ph.D.
    Polypoid mass

    Polypoid mass

    Inflammatory polypoid lesion

    Inflammatory polypoid lesion

    Nonneoplastic polypoid lesion

    Nonneoplastic polypoid lesion

    Polypoid lesion

    Polypoid lesion

    Small cobblestone nodule

    Small cobblestone nodule


    Small polypoid nodule

    Small polypoid nodule

    Fibrotic lamina propria

    Fibrotic lamina propria

    Somewhat fibrotic stroma

    Somewhat fibrotic stroma

    Polypoid papillary projections

    Polypoid papillary projections

    Edematous polypoid lesion

    Edematous polypoid lesion


    Narrow to broad based papillae

    Narrow to broad based papillae

    Focal urothelial hyperplasia

    Focal urothelial hyperplasia

    Bullous lesion

    Bullous lesion

    Normal urothelial lining cells

    Polypoid and papillary configuration

    Polypoid and papillary configuration

    Electron microscopy description
    • Pleomorphic microvilli present on surface epithelial cells in patients with catheter associated polypoid cystitis (J Urol 1984;131:242)
    Sample pathology report
    • Urinary bladder, posterior wall, biopsy:
      • Polypoid / papillary cystitis (see comment)
      • Comment: The bladder lesion biopsy shows a polypoid lesion with focal finger-like papillae lined by benign urothelium. There is marked edema of the lamina propria. Dilated capillaries and rare lymphocytes are noted. Muscularis propria is not identified. These changes are consistent with polypoid / papillary cystitis.
    Differential diagnosis
    Board review style question #1

      A 65 year old man presents with intermittent hematuria. He has a history of indwelling catheter use. His cystoscopy shows a small exophytic lesion, which is biopsied. The histologic image is shown above. What is the diagnosis?

    1. Noninvasive papillary urothelial carcinoma, low grade
    2. Polypoid cystitis
    3. Urothelial carcinoma in situ
    4. Urothelial papilloma
    Board review style answer #1
    B. Polypoid cystitis

    Comment Here

    Reference: Polypoid / papillary cystitis
    Board review style question #2

      A 64 year old man presented with dysuria and hematuria. Cystoscopic examination showed a 5 mm mass arising in the posterior bladder wall. Transurethral bladder tumor resection was performed. A histopathological image is shown above. What is the diagnosis of this lesion?

    1. Noninvasive papillary urothelial carcinoma, low grade
    2. Polypoid / papillary cystitis
    3. Urothelial neoplasm of low malignant potential
    4. Urothelial papilloma
    Board review style answer #2
    B. Polypoid / papillary cystitis

    Comment Here

    Reference: Polypoid / papillary cystitis

    Poorly differentiated variant (including osteoclast rich giant cells)
    Definition / general
    • Extremely rare variant of high-grade urothelial carcinoma with an aggressive behavior and poor outcome
    • Composed of a mixture of undifferentiated mononuclear carcinoma cells and osteoclast-like reactive giant cells (Mod Pathol 2006;19:161)
    Epidemiology
    • Mostly men in their 7th to 9th decade of life
    Clinical features
    • Usually gross hematuria, may present with flank pain, renal colic and dysuria
    Case reports
    Gross images

    Images hosted on other servers:

    Tumor of distal ureter

    Microscopic (histologic) description
    • Composed of a mixture of mononuclear malignant epithelial cells and multinucleated osteoclast-like reactive giant cells
    • Mononuclear cells have abundant cytoplasm, round to oval vesicular nuclei, mild atypia and variable mitotic activity
    • The giant cells are morphologically and immunohistochemically identical to osteoclasts and are regarded as being of histiocytic origin; are cytologically bland, may exhibit phagocytic activity but no mitotic activity; are generally evenly distributed among mononuclear cells but may condense around hemorrhagic foci
    Microscopic (histologic) images

    Case #339:

    Various images


    EMA

    CK7

    p53

    GATA3


    p63

    SMA

    S100

    CD68



    Images hosted on other servers:

    Background mononuclear tumor cells

    Osteoclast-like giant cell carcinoma

    Osteoclast rich undifferentiated urothelial carcinoma

    Cytoplasmic immunoreactivity with vimentin

    Immunoreactivity with CD68


    Focal immunoreactivity with AE1 / AE3

    Nuclear immunoreactivity with Ki67

    Nuclear immunoreactivity with p53

    Osteoclast-like giant cells positive for CD68

    Cytology images

    Images hosted on other servers:

    Diff Quick stain

    Pap stain

    Diff Quick stain

    Differential diagnosis
    • Giant cell carcinoma: obvious malignant bizarre giant cells, mitotic activity, invasion, positive staining for epithelial markers
    • Foreign-body type giant cell reaction: inflammatory infiltrate, no atypia, no invasion

    Postoperative spindle cell nodule
    Definition / general
    Terminology
    Epidemiology
    • Usually women
    Clinical features
    • Sometimes incidental
    • May present with hematuria or obstructive voiding symptoms
    Prognostic factors
    • Excellent prognosis
    Treatment
    Gross description
    • Friable nodule, mean size 1 cm
    Microscopic (histologic) description
    • Cellular, fascicular growth pattern of plump or elongated spindle cells which infiltrate the bladder wall and may focally destroy muscle
    • Delicate network of small blood vessels in edematous or myxoid stroma with red blood cell extravasation
    • Ulcerated surface with acute and chronic inflammatory infiltrate
    • May show low to high mitotic activity but no atypical mitotic figures, resembles sarcoma but no atypia
    • No necrosis, no significant pleomorphism
    Microscopic (histologic) images

    AFIP images

    Interlacing fascicles of myofibroblasts

    Uniform cells are mixed with inflammatory cells



    Images hosted on other servers:

    Left: muscularis
    propria,
    Right: spindle
    cell nodule

    Cytology description
    Positive stains
    Negative stains
    Molecular / cytogenetics description
    Differential diagnosis
    • Kaposi sarcoma:
      • No history of recent procedure, primarily a vascular tumor
    • Myxoid leiomyosarcoma:
      • No history of recent procedure; has smooth muscle morphology, pleomorphism, atypical mitotic figures, necrosis

    Prostatic type polyp
    Definition / general
    • Uncommon, nonneoplastic urothelial tract polyp comprised of prostatic glands and stroma
    Essential features
    • Polypoid or papillary lesion consists of nonneoplastic prostatic glandular epithelium with 2 distinct cell layers (luminal / secretory layer and basal layer) and corpora amylacea
    • Immunoreactive for prostate specific antigen (PSA) and prostatic acid phosphatase (PAP)
    • Most important differential to consider and exclude is prostatic ductal adenocarcinoma
    Terminology
    • Ectopic prostate
    • Benign prostatic epithelial polyp
    • Adenomatous polyp with prostatic type epithelium (historically used)
    • Papillary adenoma with prostatic type epithelium (historically used)
    ICD coding
    • ICD-10: N32.9 - bladder disorder, unspecified
    Epidemiology
    Sites
    Etiology
    Clinical features
    Diagnosis
    • Cystoscopy with biopsy or transurethral resection
    • Histologic examination of tissue
    Radiology description
    • Transrectal ultrasonography (Clin Imaging 2013;37:778)
      • Homogeneous isoechogenicity, similar to the bladder wall
      • No significant high blood flow on color Doppler ultrasound
    Prognostic factors
    Case reports
    Treatment
    Clinical images

    Images hosted on other servers:
    Smooth sessile mass on cystoscopy

    Smooth sessile mass on cystoscopy

    Spherical mass on cystoscopy

    Spherical mass on cystoscopy

    Gross description
    Microscopic (histologic) description
    • Polypoid or papillary
    • Nonneoplastic prostatic type glandular epithelium within stroma (Histopathology 1987;11:789, Am J Surg Pathol 1984;8:833, Int Urol Nephrol 1997;29:313, Histopathology 2011;58:750)
      • Prostatic acini vary in number and may contain corpora amylacea
      • 2 distinct cell layers: luminal epithelial cell layer and basal cell layer
        • Luminal layer: cuboidal to columnar cells with round nuclei and frothy, faintly eosinophilic cytoplasm, some containing eosinophilic granules
        • Basal cell layer: between luminal layer and basement membrane, typically elongated and flattened cells with slender nuclei and minimal cytoplasm
    • Interspersed urothelium may be associated with cystitis cystica and cystitis glandularis
    Microscopic (histologic) images

    Contributed by Bonnie Choy, M.D.
    Polypoid

    Polypoid

    Admixture of urothelium and prostatic acini

    Admixture of urothelium and prostatic acini

    Corpora amylacea

    Corpora amylacea

    PSA positive

    PSA positive

    Cytology description
    Sample pathology report
    • Urinary bladder, biopsy:
      • Prostatic type polyp (see comment)
      • Comment: Immunohistochemistry for PSA and PSAP highlight the glandular epithelium within the polypoid lesion, confirming prostatic origin.
    Differential diagnosis
    • Prostatic ductal adenocarcinoma:
      • More complex architecture, including cribriform and anastomosing cords
      • Pseudostratified columnar epithelium
      • Low grade nuclear features but with prominent nucleoli
      • IHC: PSA and PAP also positive
    • Urothelial carcinoma with glandular differentiation:
      • More complex architecture
      • Cytologic atypia seen in both urothelial and glandular components
      • Pseudostratified columnar epithelium lining glands
      • IHC: CK7 and GATA3 positive in urothelial component but negative in glandular component
    • Papillary urothelial neoplasm:
      • Papillary architecture with more complexity - secondary or tertiary branching
      • Stratified epithelium
      • No glandular component
      • IHC: CK7 and GATA3 positive
    • Benign prostatic hyperplasia:
      • Variable amounts of benign prostatic glands and spindle cells
      • May form polypoid mass protruding into the bladder near the bladder neck
    • Nephrogenic metaplasia (adenoma):
      • Variable patterns, including mixed papillary and polypoid with tubules in the underlying stroma
      • Single cuboidal layer of epithelial cells
      • IHC: PAX8, CK7 and AMACR positive, may occasionally show weak positivity for PSA or PAP
    • Polypoid / papillary cystitis:
      • Architecture may mimic prostatic type polyp
      • No prostatic glandular epithelium, no corpora amylacea
      • IHC: PSA and PAP negative
    • Cystitis cystica / glandularis:
      • Cystic and gland-like structures of varying size and shape lined by urothelium
      • May have polypoid appearance
      • No prostatic glandular epithelium, no corpora amylacea
      • IHC: PSA and PAP negative
    Board review style question #1
    Polypoid PSA positive


    A 40 year old man with gross hematuria is found to have a polypoid mass in the bladder trigone on cystoscopy. A transurethral resection of the bladder mass shows the above histologic features. Immunohistochemistry for PSA was performed. What is the likely diagnosis?

    1. Nephrogenic adenoma
    2. Polypoid cystitis
    3. Prostatic ductal adenocarcinoma
    4. Prostatic type polyp
    Board review style answer #1
    D. Prostatic type polyp. Prostatic type polyp consists of benign prostatic acini admixed with urothelium. PSA positivity confirms prostatic origin. Nephrogenic adenoma can demonstrate mixed papillary, polypoid, tubular and tubulocystic patterns lined by a single cuboidal layer of epithelial cells, unlike the 2 cell layers present in prostatic glands. Polypoid cystitis may have similar architecture as prostatic type polyp. However, polypoid cystitis does not contain prostatic glands and is negative for PSA. While PSA is immunoreactive in both prostatic type polyp and prostatic ductal adenocarcinoma, morphologic features, such as complex architecture, pseudostratified columnar epithelium and prominent nucleoli (not seen in the above image), support the latter.

    Comment Here

    Reference: Prostatic type polyp
    Board review style question #2
    Which of the following is true regarding prostatic type polyp?

    1. Behaves in benign fashion, with low recurrence rate
    2. Marked cytologic atypia present
    3. Most commonly seen at anterior wall of the bladder
    4. Patients often present with elevated PSA
    Board review style answer #2
    A. Behaves in benign fashion, with low recurrence rate. Prostatic type polyp is a nonneoplastic process, with low potential for recurrence. Patients most frequently present with hematuria. Other presenting symptoms include hematospermia, dysuria and urinary outflow obstruction. In the urinary bladder, the most common site is the trigone. Morphologic features show benign prostatic type glandular epithelium admixed with urothelium.

    Comment Here

    Reference: Prostatic type polyp

    Prune belly syndrome
    Definition / general
    • Prune belly syndrome (PBS) is a congenital disorder of the urinary system with renal, ureteral and urethral abnormalities
    • The syndrome is named for the mass of wrinkled abdominal skin often present
    • First described in 1839 by Frölich, but Osler gave the condition its name (Pediatr Surg Int.2012;28:219)
    Essential features
    • The three phenotypic features of the classical triad are:
      • Urinary tract malformations such as unusually large ureters (megaureter), distended bladder (megacystitis), vesicoureteral reflux
      • Partial or complete absence of the abdominal muscles
      • Bilateral cryptorchidism
    Terminology
    • Abdominal Muscle Deficiency Syndrome, Congenital Absence of the Abdominal Muscles, Eagle-Barrett Syndrome, Obrinsky Syndrome, Fröhlich Syndrome, Triad Syndrome
    Epidemiology
    • Rare, genetic birth defect affecting about 1 in 40,000 births
    • 97% male
    • Associated with twinning - 4% of all cases are products of twin pregnancies
    • Associated with trisomy 18 and 21
    • Reference: Fetal Pediatr Pathol 2013;31:13
    Pathophysiology
    • The prevailing theory is mesodermal arrest, which explains the involvement of the genitourinary tract, testis and abdominal wall (J Urol 1994;152:2328); a noxious insult would have to occur between gestational weeks 6 and 10
    • The distended bladder could result in the abnormal development of the abdominal wall musculature and prevent the descent of the testis
    Etiology
    • Autosomal dominant or X linked recessive inheritance has been suggested in some cases
    • A homozygous mutation in the muscarinic cholinergic receptor 3 gene (CHRM3) on chromosome 1q43 was reported in one family
    Clinical features
    • A partial or complete lack of abdominal wall muscles
    • Urinary tract abnormality such as unusually large ureters, distended bladder or vesicoureteral reflux due to obstruction; the site of obstruction may be as high as the pelviureteral junction or as low as the prostatic membranous urethra
    • Cryptorchidism (undescended testicles)
    • Frequent urinary tract infections
    • Ventricular septal defect, tetralogy of Fallot
    • Musculoskeletal abnormalities include pectus excavatum, scoliosis and congenital joint dislocations including the hip
    • Others: renal dysplasia, constipation, increased pulmonary secretions, malrotation of the gut, club foot, post-ejaculatory discomfort
    • Reference: Urology 2015;85:211
    Diagnosis
    • PBS can be diagnosed via ultrasound in utero based on an abnormally large abdominal cavity due to accumulated urine
    • A rise in BUN and creatinine levels associated with decreased urine output indicates urinary obstruction in the neonate
    • In young children, frequent urinary tract infections, typically uncommon, often herald PBS; diagnosis is suggested by a voiding cystourethrogram
    • Diagnosis necessitates a thorough orthopaedic evaluation because of the high prevalence of associated musculoskeletal abnormalities
    Radiology description
    • Renal and bladder ultrasonography is recommended as early as possible to evaluate the urinary tract
    • Contrast voiding cystourethrography (VCUG) should be performed
    Prognostic factors
    • PBS occurs with variable degrees of severity, with a mortality of 20%; with proper treatment, a longer, healthier life is possible
    • In severe cases, renal dysplasia and oligohydramnios in utero result in pulmonary hypoplasia; these infants may be stillborn or die shortly after birth due to respiratory complications
    • Those with less severe renal disease may survive infancy, but may have recurrent urinary tract infection or progressive renal insufficiency; some mild cases have little or no loss of renal function and therefore a better prognosis
    Treatment
    • Treatment depends on the severity of the symptoms (J Pediatr Urol 2015;11:276.e1)
    • Vesicostomy may prevent urinary tract infections
    • Consistent self-catheterization, often several times per day, may also prevent infections
    • A more drastic procedure is surgical "remodeling" of the abdominal wall and urinary tract
    • Boys often require orchiopexy
    • If renal failure develops, the child may need dialysis or a kidney transplant
    Differential diagnosis
    • For antenatal hydronephrosis with hydroureter, consider:
      • Posterior urethral valves: may show a keyhole sign and no evidence of cryptorchidism
      • Megacystis microcolon intestinal hypoperistalsis syndrome: tends to have polyhydramnios and more females affected
      • Pseudo-prune belly syndrome, which is associated with PBS uropathy, is characterized by normal abdominal wall examination findings and incomplete or absent cryptorchidism
      • An abdominal wall defect commonly confused with PBS has been termed prune belly-like variant

    Radiation cystitis
    Definition / general
    • Radiation cystitis: inflammation of the urinary bladder caused by radiation therapy for pelvic malignancies
    • Radiation induced pseudocarcinomatous hyperplasia: florid nonneoplastic urothelial proliferation with pseudoinfiltrative features (Am J Surg Pathol 2004;28:909)
    Essential features
    • Classic findings of hemorrhage, fibrin deposition, fibrin thrombi, fibrosis, acute and chronic inflammation, edema, vascular congestion and thickened vessels (Am J Surg Pathol 2004;28:909)
    • May have variable sized urothelial nests and cords with rounded or irregular edges present in the lamina propria, mimicking carcinoma (Am J Surg Pathol 2008;32:92)
    Terminology
    ICD coding
    • ICD-10:
      • N30.40 - irradiation cystitis without hematuria
      • N30.41 - irradiation cystitis with hematuria
      • N30.90 - cystitis, unspecified without hematuria
    Epidemiology
    • Incidence of acute radiation cystitis and urethritis: 50% of irradiated patients
    • Incidence of severe chronic cystitis and urethritis: 5 - 10% of irradiated patients
    • Age range: 40 - 85 years, average 69 years
    • M > F, 2.8:1 (Am J Surg Pathol 2008;32:92)
    Sites
    • Urinary bladder and urethra
    Pathophysiology
    • Acute phase: lasting for several weeks postradiation therapy (Urology 2016;88:14)
      • Urothelial cell damage
      • Damage to glycosaminoglycan layer
      • Irritability from urine leakage
      • Inflammation of the bladder wall
      • Pseudocarcinomatous epithelial hyperplasia
      • Edema of lamina propria
      • Telengiectasia
      • Stromal cell atypia
    • Latent phase: symptom free phase lasting for months to years (Rev Urol 2016;18:57)
      • Obliterative endarteritis
      • Urothelial proliferation
    • Chronic phase: irreversible, late response (Rev Urol 2016;18:57)
      • Loss of smooth muscle cells
      • Fibroblast proliferation, collagen deposition (fibrosis), atrophy
      • Telengiectasia
      • Edema
      • Hemorrhage
    Etiology
    • High energy radiation separates water molecule to free oxygen radicals - radiolysis (Urology 2016;88:14)
    • Free oxygen radicals cause injury to cell membrane by lipid peroxidation, leading to immediate cell death and cause genetic damage by reacting with DNA
    • High energy radiation directly absorbed by DNA, resulting in genetic damage including mutation or replication failure and cell death (Rev Urol 2016;18:57)
    Diagrams / tables

    Images hosted on other servers:

    Pathophysiology

    Clinical features
    • Past medical history of radiation therapy for pelvic malignancies
    • Acute radiation cystitis (Nat Rev Urol 2010;7:206)
      • Complications occur during or soon after therapeutic irradiation
      • Irritative voiding symptoms include dysuria, frequency, urgency, cystalgia with bladder spasms, usually no hematuria (Cells 2020;10:21)
    • Chronic radiation cystitis (Urol Int 2022;106:63)
      • > 3 months after radiotherapy and a clinical event free interval
      • 6.5 - 9% postradiotherapy, more frequently hematuria
    • 3 different types (Urol Int 2022;106:63)
      • Inflammation predominant
      • Bleeding predominant
      • Combination of inflammation and bleeding radiation cystitis and urethritis
    • Cystoscopy: irritated mucosa with edema, erythema, papillary, polypoid or atrophic lesions with congestion, telangiectasia, ulceration, stricture, necrosis or hemorrhage (J Urol 2013;189:2083)
    Diagnosis
    • Previous clinical history of pelvic irradiation
    • Hematuria, dysuria, urgency
    • Ultrasound, CT, MRI (Urol Int 2022;106:63)
    • Cystoscopic evaluation and biopsies of lesions
    Laboratory
    Radiology description
    Radiology images

    Images hosted on other servers:

    Bladder wall thickening

    Prognostic factors
    • Acute radiation cystitis: self limiting, favorable outcome
    • Chronic radiation cystitis: life threatening hemorrhage may occur (Nat Rev Urol 2010;7:206)
    Case reports
    Treatment
    • Discontinue insulting agents or aggravating factors, proper hydration and symptomatic treatment
    • Radiation cystitis with predominant hemorrhage:
      • Hydration, hyperbaric oxygen therapy (Nat Rev Urol 2010;7:206)
      • Oral drugs (sodium pentosan polysulfate, aminocaproic acid, immunokine WF10, conjugated estrogen or pentoxifylline plus vitamin E) if hyperbaric oxygen therapy is not available
      • Laser or electrocoagulation of bleeding vessels (Urol Int 2022;106:63)
    Clinical images

    Images hosted on other servers:

    Cystoscopy

    Gross description
    • Thickened bladder mucosal wall in acute phase and atrophy in chronic phase
    • Bladder mucosa with marked congestion, hemorrhage, telangiectasia, ulceration, stricture and necrosis (Urol Int 2022;106:63)
    Microscopic (histologic) description
    • Vessels with fibrin thrombi, fibrinoid vascular necrosis, vascular congestion, thickened walls, endothelial proliferation and telangiectasia
    • Acute and chronic inflammation
    • Reactive multinucleated stromal fibroblasts with smudgy nuclei
    • Fibrosis and edema
    • Hemorrhage and hemosiderin
    • Urothelium can show
      • Nuclei with variation in nuclear size and shape, hyperchromasia, vesicular or smudgy chromatin
      • Eosinophilic or amphophilic cytoplasm, cytoplasmic vacuoles
      • Multinucleated giant cells (Am J Surg Pathol 2008;32:92)
      • Usually no mitotic figures
      • Surface erosion, ulceration
      • Squamous metaplasia
    • Pseudocarcinomatous urothelial hyperplasia: variable sized urothelial cords and nests with rounded or irregular edges present in the lamina propria and enclose dilated blood vessels with fibrin deposition (Am J Surg Pathol 2008;32:92)
    • Acute phase: edematous, congested and telangiectatic vessels, acute and chronic inflammatory infiltrate, atypical stromal cells with multinucleated forms
    • Chronic phase: atrophic urothelium, atrophic smooth muscle layer, collagen deposition (fibrosis) (Am J Surg Pathol 2004;28:909)
    Microscopic (histologic) images

    Contributed by Y. Albert Yeh, M.D., Ph.D. and Jennifer Lee, M.D.
    Telangiectatic vessels with fibrin

    Telangiectatic vessels with fibrin

    Dilated vessels with fibrin

    Dilated vessels with fibrin

    Epithelial cells encircling fibrin

    Epithelial cells encircling fibrin

    Urothelial cells enclosing fibrin

    Urothelial cells enclosing fibrin

    Atypical urothelial proliferation

    Atypical urothelial proliferation


    Atypical urothelial proliferation

    Atypical urothelial proliferation

    Fibrinoid vascular necrosis

    Fibrinoid vascular necrosis

    Fibrin and proliferating cells

    Fibrin and proliferating cells

    Fibrinoid necrosis of vessels

    Fibrinoid necrosis of vessels

    Mucosal hemorrhage and inflammation

    Mucosal hemorrhage and inflammation


    Radiation induced urothelial changes

    Radiation induced urothelial changes

    Atypical von Brunn nests

    Atypical von Brunn nests

    Stromal cell changes

    Stromal cell changes

    Cytology description
    • Cohesive groups and dispersed urothelial cells with cellular and nuclear enlargement and maintained N:C ratio (Arch Pathol Lab Med 2023;147:1148)
    • Variable combinations of the following features: dense to vacuolated cytoplasm, nucleomegaly with increased N:C ratio (> 0.5), nuclear vacuolization, vesicular to finely granular chromatin, prominent nucleoli, nuclear hyperchromasia, smudgy / glassy nuclei, smooth to irregular membrane, nuclear grooves
    Cytology images

    Images hosted on other servers:

    Radiation induced urothelial cell changes

    Radiation induced urothelial reactive atypia

    Positive stains
    Negative stains
    Electron microscopy description
    • Increased stromal cellularity with enlarged fibroblast
    • Dilatation of rough endoplasmic reticulum
    • Prominent nucleoli
    • Enlargement of endothelial cells
    • Multiplication of vascular basal lamina around capillaries
    • Reference: Korean J Urol 1993;34:969
    Molecular / cytogenetics description
    • UroVysion® multiprobe FISH testing: tetraploidy with a balanced duplication of the whole genome or polyploidy in benign bladder lesions including radiation induced cystitis (Ann Pathol 2012;32:e52)
    Sample pathology report
    • Urinary bladder, right wall polypoid lesion, transurethral resection:
      • Radiation cystitis with pseudocarcinomatous urothelial hyperplasia (see comment)
      • Muscularis propria not identified
      • Comment: There is a history of radiation therapy per the medical record. The transurethral resection of the bladder lesion shows fragments of urothelial mucosa with total and partial denudation of urothelium. The urothelial lining cells show reactive changes. There is marked edema, hemorrhage and mixed inflammatory infiltrate composed predominantly of neutrophils and lymphocytes in the lamina propria. Reactive multinucleated stromal fibroblasts are seen. Telangiectatic vessels with fibrinoid necrosis and intravascular fibrin deposition are seen. Anastomosing cords and nests of urothelial cells encircling the vessels with fibrin deposition is evident. Urothelial cells with hyperchromatic nuclei, increased nuclear to cytoplasmic ratio and prominent nucleoli are seen. These features are consistent with radiation cystitis with pseudocarcinomatous urothelial hyperplasia. Muscularis propria is not identified in this specimen.
    Differential diagnosis
    • Invasive urothelial carcinoma (Am J Surg Pathol 2004;28:909):
      • Absence of urothelial nests surrounding ectatic vessels with intravascular and stromal fibrin deposition
      • More prominent urothelial atypia
      • Usually without pronounced acute and chronic inflammation
      • Invasion of muscularis propria (if present) is most diagnostic
    • Florid proliferation of von Brunn nests (Ann Diagn Pathol 2019;38:11):
      • Absence of urothelial nests surrounding ectatic vessels with intravascular and stromal fibrin deposition
      • Nests of benign urothelial cells with uniformly round and lobular configuration
      • Well demarcated from underlying lamina propria
    • Inverted urothelial papilloma (Ann Diagn Pathol 2019;38:11):
      • No history of radiation therapy
      • Absence of urothelial nests surrounding ectatic vessels with intravascular and stromal fibrin deposition
      • Thin anastomosing cords or trabeculae with bland urothelial cells invaginate into lamina propria
      • Periphery of trabeculae show palisading of cellular nuclei and central areas show cellular spindling (streaming)
    Board review style question #1

    A 68 year old man presented with hematuria and dysuria. He has a medical history of prostate cancer that was treated with radiation therapy. Cystoscopy revealed hemorrhage in the right bladder wall. A transurethral resection of the lesion was performed. A photomicrograph is shown above. What is the diagnosis?

    1. Cystitis cystica
    2. Invasive urothelial carcinoma, nested variant
    3. Radiation cystitis with pseudocarcinomatous hyperplasia
    4. Urothelial carcinoma in situ
    Board review style answer #1
    C. Radiation cystitis with pseudocarcinomatous hyperplasia. Answers A, B and D are incorrect because anastomosing cords and nests of urothelial cells encircling ectatic vessels with intravascular and stromal fibrin deposition in a background of mixed inflammation are not present in these entities.

    Comment Here

    Reference: Radiation cystitis

    Rhabdomyosarcoma
    Definition / general
    • Sarcoma with features of skeletal muscle differentiation
    • Either embryonal (botryoid or spindle cell), alveolar or pleomorphic rhabdomyosarcoma
    Epidemiology
    • Less than 0.5% of all bladder tumors, but one of most common tumors of the lower urogenital tract in children
    • Often children 2-6 years old; 75% male
    • Adult tumors are usually alveolar or unclassified, commonly with anaplasia, and resemble small cell carcinoma (Am J Surg Pathol 2008;32:1022)

      Botryoid variant:
    • Occurs in mucosal lined, hollow cavities (vagina, nasal cavity, bladder); most common bladder tumor in children
    • May be associated with Wilms tumor and Dandy-Walker syndrome
    Sites
    • Usually in trigone
    • Infiltrates adjacent tissue but distant metastases are rare
    Etiology
    • Arise from primitive muscle cells
    Clinical features
    Prognostic factors
    • Favorable prognostic factors: children vs. adults, embryonal histology, low stage, polypoid (exophytic) growth pattern [10 year survival is 92% vs. 68% for diffuse intramural (endophytic) growth pattern]
    Case reports
    Treatment
    Gross description
    • Mucoid, polypoid
    • Botryoid tumors resemble a bunch of grapes
    Microscopic (histologic) description
    • Myxomatous stroma with scattered, small, blue malignant cells, usually compressed beneath intact urothelium (cambium layer)
    • Cross striations are often difficult to identify in untreated cases
    Positive stains
    Negative stains
    Molecular / cytogenetics description
    • Alveolar subtype has translocations between #13 FKHR gene and either #2 - PAX3 or #1 - PAX7 gene
    • Embryonal subtype has 11p-
    Differential diagnosis
    • Carcinomas (urothelial or small cell) with rhabdomyosarcomatous differentiation:
      • Have distinct epithelial component

    Sarcomatoid variant
    Definition / general
    • Variant of urothelial carcinoma; morphologically indistinguishable from sarcoma
    • Heterologous elements may be present (Histopathology 2019;74:77)
    Essential features
    • Biphasic malignant neoplasm with morphologic and immunohistochemical evidence of both epithelial and mesenchymal differentiation
    • Rare (0.3 - 0.6% of all urothelial carcinomas) and biologically aggressive variant
    • Associated with radiation therapy or cyclophosphamide
    • If present, heterologous elements should be mentioned in the report
    Terminology
    • WHO recommends the term sarcomatoid variant of urothelial carcinoma for these biphasic malignant neoplasms
    • Terms carcinosarcoma, metaplastic carcinoma, spindle cell carcinoma and malignant mixed tumor have been used interchangeably but can lead to confusion and are therefore discouraged
    ICD coding
    • Location based ICD-10 coding:
      • Renal pelvis, including pelviureteric junction and renal calyces
        • C65.1 - malignant neoplasm of right renal pelvis
        • C65.2 - malignant neoplasm of left renal pelvis
        • C65.9 - malignant neoplasm of unspecified renal pelvis
      • Ureter, including ureteric orifice of bladder
        • C66.1 - malignant neoplasm of right ureter
        • C66.2 - malignant neoplasm of left ureter
        • C66.9 - malignant neoplasm of unspecified ureter
      • Bladder
        • C67.0 - malignant neoplasm of trigone of bladder
        • C67.1 - malignant neoplasm of dome of bladder
        • C67.2 - malignant neoplasm of lateral wall of bladder
        • C67.3 - malignant neoplasm of anterior wall of bladder
        • C67.4 - malignant neoplasm of posterior wall of bladder
        • C67.5 - malignant neoplasm of bladder neck
        • C67.6 - malignant neoplasm of ureteric orifice
        • C67.7 - malignant neoplasm of urachus
        • C67.8 - malignant neoplasm of overlapping sites of bladder
        • C67.9 - malignant neoplasm of bladder, unspecified
      • Other, unspecified urinary organs
        • C68.0 - malignant neoplasm of urethra
        • C68.1 - malignant neoplasm of paraurethral glands
        • C68.8 - malignant neoplasm of overlapping sites of urinary organs
        • C68.9 - malignant neoplasm of urinary organ, unspecified
    Epidemiology
    Sites
    • Kidney pelvis, proximal ureter and bladder
    Pathophysiology
    Etiology
    • Probably has a common malignant clonal origin, with epithelial and mesenchymal differentiation (J Pathol 2007;211:420)
    • Common final pathway of all forms of bladder tumors, supported by molecular and morphologic evidence (Histopathology 2019;74:77)
    • Risk factors include previous exposure to radiotherapy and intravesicular cyclophosphamide (Eur Urol Focus 2020;6:653)
    Clinical features
    • Gross hematuria, flank pain, an abdominal mass and hydronephrosis
    • Similar to those of conventional urothelial tumors
    • Reference: ISRN Urol 2014;2014:794563
    Diagnosis
    Laboratory
    • Hematuria
    Radiology description
    • Thickened bladder wall with or without intraluminal papillary or nodular mass; heterologous elements such as calcification in osteosarcoma may be demonstrated (Radiographics 2006;26:553)
    Prognostic factors
    • Poor prognosis, as frequently presents at an advanced stage and is associated with a worse overall survival when compared with pure urothelial carcinoma (Eur Urol Focus 2020;6:653)
    • Similar to bladder, sarcomatoid urothelial carcinoma of renal pelvis has a worse prognosis on univariate analysis (ISRN Urol 2014;2014:794563, Oncol Lett 2014;8:1208, J Urol 2012;188:398)
    • Associated with higher tumor stage, multifocality, tumor necrosis, frequent metastases at presentation; however, insufficient case numbers in kidney to confirm independent negative prognostic impact on multivariate analysis (J Urol 2012;188:398)
    • May have heterologous elements, without definite prognostic significance (Mod Pathol 2009;22:S96)
    Case reports
    Treatment
    Gross description
    • Gray fleshy cut surface with infiltrative margins similar to sarcoma in appearance
    Gross images

    Contributed by Nicole K. Andeen, M.D. and Maria Tretiakova, M.D., Ph.D.
    Mass involving hilum

    Mass involving hilum

    Frozen section description
    • Not typically diagnosed on frozen section
    Microscopic (histologic) description
    • Most common component is undifferentiated high grade spindle cell sarcoma (Histopathology 2019;74:77)
    • Sarcomatoid areas admixed with conventional high grade urothelial carcinoma
    • Spindle cell component may account for 10% to > 60% of tumor
    • Epithelial component has urothelial differentiation or less commonly squamous or glandular differentiation
    • Highly variable and can mimic nonepithelial neoplasms (J Biol Chem 2019;294:1579)
    • Most common heterologous element is osteosarcoma, followed by chondrosarcoma, rhabdomyosarcoma, leiomyosarcoma, liposarcoma and angiosarcoma (Histopathology 2019;74:77)
    Microscopic (histologic) images

    Contributed by Megan L. Brown, M.D., Nicole K. Andeen, M.D., Maria Tretiakova, M.D., Ph.D. and Kenneth A. Iczkowski, M.D.
    High grade spindled to epithelioid cells

    High grade spindled to epithelioid cells

    Elongated spindled cells

    Elongated spindled cells

    Classic urothelial carcinoma

    Classic urothelial carcinoma

    Neoplasm with admixed carcinomatous and sarcomatous elements Neoplasm with admixed carcinomatous and sarcomatous elements

    Neoplasm with admixed carcinomatous and sarcomatous elements


    Elongated spindled cells Elongated spindled cells

    Osteosarcomatoid differentiation

    GATA3 and CK7 GATA3 and CK7

    GATA3 and CK7

    Virtual slides

    Images hosted on other servers:
    Urothelial carcinoma with sarcomatoid differentiation, high grade Urothelial carcinoma with sarcomatoid differentiation, high grade

    Urothelial carcinoma with sarcomatoid differentiation, high grade

    Urothelial carcinoma with extensive sarcomatoid features

    Sarcomatoid urothelial
    carcinoma involving
    muscularis propria

    Positive stains
    Molecular / cytogenetics description
    Sample pathology report
    • Bladder, transurethral resection:
      • High grade urothelial carcinoma with sarcomatoid differentiation (80%) (see comment)
      • Comment: Invasive of muscularis propria (T2)
      • Angiolymphatic invasion absent
      • Muscularis propria present, involved by tumor
    Differential diagnosis
    Board review style question #1

    A 65 year old presented with bladder wall thickening on ultrasound. Transurethral resection of bladder tumor is performed and shows a lesion with features on H&E as seen in the image shown above. Which of the following is likely in the patient’s medical history?

    1. Asbestos exposure
    2. Family history of prostate adenocarcinoma
    3. High nitrate consumption in their diet
    4. Intravesical cyclophosphamide
    5. Woodworking hobby
    Board review style answer #1
    D. Intravesical cyclophosphamide has been associated with the development of sarcomatoid variant of urothelial carcinoma (Eur Urol Focus 2020;6:653)

    Comment Here

    Reference: Sarcomatoid variant
    Board review style question #2

    What is the most commonly identified heterologous element identified in association with the pictured lesion?

    1. Chondrosarcoma
    2. Leiomyosarcoma
    3. Liposarcoma
    4. Osteosarcoma
    5. Rhabdomyosarcoma
    Board review style answer #2
    D. Osteosarcoma is the most commonly identified heterologous element in the sarcomatoid variant of urothelial carcinoma (Histopathology 2019;74:77)

    Comment Here

    Reference: Sarcomatoid variant

    Schistosomiasis (bilharziasis)
    Definition / general
    • Parasitic infection of the bladder with blood flukes (trematodes), Schistosoma species (mainly Schistosoma haematobium)
    Essential features
    • Bladder schistosomiasis is most commonly caused by Schistosoma haematobium
    • Eggs of Schistosoma haematobium: 110 - 170 μm, oval with terminal spine
    • Eggs of Schistosoma mansoni: 115 - 175 μm, oval with lateral spine
    • Few case reports of bladder schistosomiasis caused by Schistosoma mansoni and Schistosoma haematobium - Schistosoma mansoni hybrid (Emerg Infect Dis 2019;25:365)
    • Coinfection with Schistosoma haematobium and Schistosoma mansoni occurs (East Afr Med J 2013;90:36)
    Terminology
    • Bladder bilharziasis is named after Theodor Bilharz, a German physician who discovered the first blood fluke causing urinary schistosomiasis in 1851 (Wikipedia: Schistosomiasis [Accessed 9 February 2022])
    • Bladder schistosomiasis
    • Genitourinary schistosomiasis
    • Schistosomiasis related cystitis
    ICD coding
    • ICD-10: B65.0 - schistosomiasis due to Schistosoma haematobium [urinary schistosomiasis]
    Epidemiology
    Pathophysiology
    • After penetration of skin or mucous membranes, schistosomulae migrate through the lungs to portal venous system and mature into adults
    • About 6 weeks later, adult worms migrate to bladder venules (Nat Rev Dis Primers 2018;4:13)
    • Females lay eggs that reach the bladder wall, penetrate bladder mucosa and excrete into urine
    • Bladder wall shows eggs and mixed inflammatory reaction with lymphocytic, neutrophilic and eosinophilic infiltrate and scarring of the vesicular vessels
    • Parasitic eggs provoke chronic granulomatous inflammation with calcifications and fibrotic thickening of the bladder wall, deposits of calcified eggs in later stage (Nat Rev Dis Primers 2018;4:13)
    • Eggs may block drainage of urine into the bladder, causing hydroureter
    • May develop secondary bacterial urinary tract infections, renal failure and pulmonary hypertension (Hum Pathol 1986;17:333)
    • May progress to bladder cancer (squamous cell carcinoma [80 - 85%], adenocarcinoma [5 - 15%] and rarely urothelial carcinoma) (Front Med (Lausanne) 2018;5:223)
    Etiology
    • Humans are infected with schistosomes after exposure to cercariae released by snails (Bulinus species) in fresh water
    • Penetration of skin or mucous membranes by cercariae (survives 3 - 4 days in fresh water)
    • Schistomulae migrate to bladder plexus and nearby veins and venules (Nat Rev Dis Primers 2018;4:13)
    • Female schistomulae deposit their eggs
    • Some eggs reach the bladder wall, penetrate bladder mucosa and excrete in urine
    • Eggs hatch and release miracidia
    • Miracidia penetrate snail intermediate hosts (Bulinus species)
    • After 2 generations of sporocysts, the snails produce cercariae and release them in fresh water (CDC: Parasites - Schistosomiasis [Accessed 9 February 2022])
    Diagrams / tables

    Images hosted on other servers:

    Life cycle

    Clinical features
    • Swimmer's itch: cercarial dermatitis (days after infection)
      • Local erythema, pruritic maculopapular rash
    • Acute schistosomiasis (2 - 8 weeks after infection) (Lancet Infect Dis 2007;7:218)
      • Katayama syndrome: fever ("snail fever"), cough, muscle pain, malaise, abdominal pain
      • Hematuria, dysuria, suprapubic pain, urinary tract infections
    • Chronic schistosomiasis (within months after infection)
    Diagnosis
    • Detection of Schistosoma haematobium or Schistosoma mansoni eggs in urine samples or bladder tissues (J Egypt Soc Parasitol 2017;47:211)
    • Schistosoma haematobium eggs: 110 - 170 μm × 40 - 70 μm, oval with terminal spine (Hum Pathol 1986;17:333)
    • Schistosoma mansoni eggs: 115 - 175 μm × 45 - 47 μm, oval with lateral spine
    Laboratory
    Radiology description
    • For bladder schistosomiasis: bladder wall thickening (usually > 4 mm) on ultrasound
    • For carcinoma arising in bladder schistosomiasis: focal bladder wall thickening or mass on ultrasound
    Prognostic factors
    • Good prognosis for patients with early disease
    • Prognosis is worse for chronic infections with complications:
      • Immune complex glomerulonephritis
      • Obstructive nephropathy
      • Obstructive uropathy
      • Chronic pyelonephritis
      • Amyloidosis
      • Bladder cancer (risk 1.72, further increased in smokers)
    • References: J Adv Res 2013;4:453, Nat Rev Dis Primers 2018;4:13
    Case reports
    Treatment
    • Single course of praziquantel treatment can be curative
    • Corticosteroids (severe disease), metrifonate (alternative therapy) for bladder schistosomiasis (ChemMedChem 2018;13:2374)
    Gross description
    • Early active stage: erythematous, sessile or pedunculated polyps, "grainy sandy" patches (Urology 2017;107:e7)
    • Chronic stage: fibrotic thickening of bladder wall, stellate ulcers, tumor-like lesions (Hum Pathol 1986;17:333)
    Microscopic (histologic) description
    Microscopic (histologic) images

    Contributed by Y. Albert Yeh, M.D., Ph.D., Susan Prendeville, M.D. and Kiran Alam, M.D., Anshu Jain, M.D.,
    Veena Maheshwari, M.D., Farhan A. Siddiqui, M.B.B.S., Ershadul Haq, M.B.B.S., M.D. and @ThatGlassTho on Twitter

    Calcified eggs and keratin

    Schistosoma eggs and keratin

    Schistosoma eggs with terminal spine

    Bladder calcified eggs

    Calcified eggs with cracks

    Parasitic calcified eggs


    Dysplastic keratinizing squamous cells

    Malignant squamous cells

    Malignant squamous cells with intercellular bridges

    <i>Schistosoma haematobium</i> eggs

    Schistosoma haematobium eggs

    Schistosoma eggs in ureter

    Ureter with Schistosoma eggs


    Muscularis propria with Schistosoma eggs

    Schistosoma eggs

    Schistosomiasis (bilharziasis) Schistosomiasis (bilharziasis)

    Schistosomiasis (bilharziasis)

    Cytology description
    • Uncommon in urine but if present, may show an oval calcified egg with a lateral or terminal spine
    Cytology images

    Contributed by Y. Albert Yeh, M.D., Ph.D.

    Calcified eggs in urine

    Schistosoma egg with lateral spine

    Positive stains
    Negative stains
    • Schistosoma haematobium egg shells: acid fast
    Molecular / cytogenetics description
    • Molecular testing of schistosome eggs in Africa showed hybrids of Schistosoma haematobium and livestock schistosoma species, S. mattheei and S. bovis (Emerg Infect Dis 2019;25:1245)
    • Common chromosomal aberrations in schistosomiasis associated bladder cancer (Carcinogenesis 2000;21:1721)
      • Loss: 8p, 18q, 17p, 5q, 9p, 10q, 11q, 18p, 11p, 9q, 20p, 6q (in descending order)
      • Gain: 17q, 5p, 7q, 7p, 1q (in descending order)
    • Most common genetic alterations in schistosomiasis associated bladder cancer (Nat Clin Pract Urol 2005;2:502)
      • Low grade urothelial carcinoma: chromosome 9 loss of heterozygosity, FGFR3 activating mutation
      • Dysplasia and carcinoma in situ: TP53 mutations, chromosome 9 loss of heterozygosity
    • Loss of heterozygosity in schistosomiasis associated bladder cancer versus non schistosomiasis associated bladder cancer (J Natl Cancer Inst 1995;87:1383)
      • 9p loss of heterozygosity (twofold higher in schistosomiasis associated bladder cancer)
      • 9q loss of heterozygosity (threefold lower in schistosomiasis associated bladder cancer)
    • Genes involved in schistosomiasis associated bladder cancer (Clin Microbiol Rev 1999;12:97)
      • Mutation of FGFR3 (> 30%, more frequent in low grade papillary tumors)
      • Activation of HRAS (7 - 17%, more frequent in low grade papillary tumors)
      • Homozygous deletions of p16INK4A (more common in high grade Ta tumors) (Am J Pathol 1999;155:105)
      • Inactivation of tumor suppressor genes TP53 and RB (more common in carcinoma in situ and high grade invasive tumors)
      • Overexpression of ERBB2 in advanced urothelial carcinoma
      • BCL2 expression in later stages of squamous cell carcinoma
    Sample pathology report
    • Urinary bladder, dome, biopsy:
      • Urothelial mucosa with ulcerations, chronic granulomatous inflammation with eosinophils and calcified parasitic eggs, consistent with schistosomiasis related cystitis (see comment)
      • Comment: The urothelial mucosa shows urothelial mucosa with granulation tissue and mixed inflammatory infiltrate composed predominantly of lymphocytes, neutrophils and eosinophils. There are many calcified oval parasitic eggs measuring approximately 120 microns with a sharp terminal spine. These changes of cystitis are consistent with Schistosoma haematobium infection. A small fragment of the muscularis propria is also present in the biopsy specimen.
    Differential diagnosis
    Board review style question #1
    What is the most common pathogen of bladder schistosomiasis?



    1. Schistosoma haematobium
    2. Schistosoma haematobium - Schistosoma mansoni hybrid
    3. Schistosoma intercalatum
    4. Schistosoma japonicum
    5. Schistosoma mansoni
    Board review style answer #1
    A. Schistosoma haematobium

    Comment Here

    Reference: Schistosomiasis (bilharziasis)
    Board review style question #2
    Which of the following parasitic eggs contains a terminal spine?

    1. Schistosoma haematobium
    2. Schistosoma japonicum
    3. Schistosoma mansoni
    4. Schistosoma mekongi
    Board review style answer #2
    A. Schistosoma haematobium

    Comment Here

    Reference: Schistosomiasis (bilharziasis)

    Small cell neuroendocrine carcinoma
    Definition / general
    • High grade neuroendocrine neoplasm characterized by small to medium sized cells with high N:C ratio and indistinct nucleoli arising from urothelium of urinary bladder, renal pelvis and ureters
    Essential features
    • Histologically characterized by small to medium sized cells with scant cytoplasm, large nuclei with absent or inconspicuous nucleoli, finely granular salt and pepper chromatin, nuclear molding, Azzopardi phenomenon (crush artifact), abundant mitoses and often pervasive necrosis
    • In the genitourinary tract, small cell neuroendocrine carcinoma most commonly arises from the bladder and less commonly from the renal pelvis and ureters
    • Usually shows mixed histology; a concomitant urothelial carcinoma, adenocarcinoma or other carcinomatous / sarcomatous component(s) is common
    • Typically reaches advanced clinical stage characterized by muscularis propria invasion, lymphovascular invasion or distant metastases by time of diagnosis and is thus associated with a poor prognosis
    • Treatment regimens are variable but surgical resection with platinum agent based neoadjuvant or adjuvant chemotherapy is most commonly implemented
    Terminology
    • Small cell neuroendocrine carcinoma (preferred terminology)
    • Small cell carcinoma
    • Oat cell carcinoma (no longer in use)
    ICD coding
    • ICD-10
      • C65 - malignant neoplasm of renal pelvis
        • C65.1 - malignant neoplasm of right renal pelvis
        • C65.2 - malignant neoplasm of left renal pelvis
        • C65.9 - malignant neoplasm of unspecified renal pelvis
      • C66 - malignant neoplasm of ureter
        • C66.1 - malignant neoplasm of right ureter
        • C66.2 - malignant neoplasm of left ureter
        • C66.9 - malignant neoplasm of unspecified ureter
      • C67 - malignant neoplasm of bladder
        • C67.1 - malignant neoplasm of trigone of bladder
        • C67.2 - malignant neoplasm of lateral wall of bladder
        • C67.3 - malignant neoplasm of anterior wall of bladder
        • C67.4 - malignant neoplasm of posterior wall of bladder
        • C67.5 - malignant neoplasm of bladder neck
        • C67.6 - malignant neoplasm of ureteric orifice
        • C67.7 - malignant neoplasm of urachus
        • C67.8 - malignant neoplasm of overlapping sites of bladder
        • C67.9 - malignant neoplasm of bladder, unspecified
    Epidemiology
    Sites
    Pathophysiology
    • Transdifferentiation / dedifferentiation of urothelial carcinoma (iScience 2020;23:101201, Clin Cancer Res 2018;24:1965)
      • BRCA1 / BRCA2, TP53 and RB1 with or without APOBEC mutations create high mutation rate and predisposition to malignancy in basal subtype conventional urothelial carcinoma
      • Urothelial to neural plasticity and activation of epithelial - mesenchymal transition
        • Urothelial to neural plasticity: downregulation of genes responsible for urothelial differentiation and upregulation of genes responsible for neural / neuroendocrine differentiation
        • Activation of epithelial mesenchymal transition: downregulation of p53 and p63 pathways in addition to loss of epithelial adhesion molecules (E-cadherin, claudin1, tight junction protein 1)
      • Overexpression of ADORA2A (adenosine receptor 2A) and loss of expression of immunostimulatory genes leads to immune desert phenotype
    • Other theories regarding pathogenesis
      • Multipotent / pluripotent stem cell in urothelium differentiates into small cell neuroendocrine carcinoma and other common bladder carcinomas (i.e., shares same clonal cell of origin as other bladder carcinomas) (Hum Pathol 2018;79:57)
      • Malignant transformation of endemic Kulchitsky or enterochromaffin cells in urothelial mucosa or submucosa (Hum Pathol 2018;79:57)
    Etiology
    • Smoking
    • Other proposed etiologies in bladder (Mol Clin Oncol 2018;9:335)
      • Chronic cystitis
      • Bladder calculi
      • History of cystoplasty
    Clinical features
    Diagnosis
    • Urinalysis
    • Urine / bladder washing cytology (Oncol Lett 2014;7:369)
    • Cystoscopy
    • Imaging (Clujul Med 2017;90:13)
      • Ultrasound (US): usually first line and may identify hydronephrosis
      • Computerized tomography (CT): identifies masses and tumoral extension
      • Magnetic resonance imaging (MRI): used to identify extravesical or extranephroureteric extension and distant metastases
        • Gadolinium enhanced MRI: may identify brain metastases
      • 99mTc MDP bone scan may identify bone metastases
    • Transurethral resection of the bladder (TURBT) or resection for confirmation
    Laboratory
    Radiology description
    Radiology images

    Images hosted on other servers
    CT Bladder

    CT of bladder

    CT Ureter

    CT of ureter

    US Bladder

    US of bladder

    MRI Bladder

    MRI of bladder

    Gadolinium enhanced MRI Renal Pelvis

    Gadolinium enhanced MRI of renal pelvis

    Prognostic factors
    Case reports
    Treatment
    Gross description
    Gross images

    Images hosted on other servers:
    Right ureter mass

    Right ureter mass

    Renal pelvic lesion

    Renal pelvic lesion

    Microscopic (histologic) description
    • General histologic features (Pol J Pathol 2014;65:15)
      • Solid sheet-like, nested and trabecular architectures
      • Small to medium sized cells with scant cytoplasm and large round to oval nuclei (i.e., high N:C ratio)
      • Finely granular, stippled salt and pepper chromatin
      • Absent or inconspicuous nucleoli
      • Nuclear molding
      • Azzopardi phenomenon (crush artifact)
      • Brisk mitotic activity and atypical mitoses
      • Extensive, often geographic necrosis
    • Frequently coexists with other malignant cell populations
      • Prevalence of additional histologic components (Hum Pathol 2018;79:57)
        • Urothelial carcinoma (59%)
        • Urothelial carcinoma in situ (48%)
        • Adenocarcinoma (26%)
        • Sarcomatoid carcinoma (7.4%)
        • Micropapillary carcinoma (7.4%)
        • Squamous cell carcinoma (5.6%)
        • Plasmacytoid carcinoma (1.9%)
        • Sarcomas (chondrosarcoma, leiomyosarcoma and rhabdomyosarcoma) have been described in rare case reports (Onco Targets Ther 2017;10:4105)
        • Lymphoma described in 1 case report (Onco Targets Ther 2017;10:4105)
      • May be triphasic or even tetraphasic in rare cases (World J Surg Oncol 2017;15:33)
    Microscopic (histologic) images

    Contributed by Reima El Naili, M.D.
    Sheets of basophilic cells.

    Sheets of basophilic cells

    Pleomorphism and nuclear molding.

    Pleomorphism and nuclear molding

    High N:C ratio.

    High N:C ratio

    Brisk mitotic activity.

    Brisk mitotic activity

    Hemorrhage and atypical mitoses.

    Hemorrhage and atypical mitoses

    Necrosis and Azzopardi effect.

    Necrosis and Azzopardi effect



    Mixed urothelial neuroendocrine carcinoma.

    Mixed urothelial neuroendocrine carcinoma

    Carcinoma with normal urothelium.

    Carcinoma with normal urothelium

    CK AE1 / AE3.

    CK AE1 / AE3

    Synaptophysin.

    Synaptophysin

    CD56.

    CD56

    TTF1.

    TTF1



    Ki67.

    Ki67

    CD56 in mixed tumor.

    CD56 in mixed tumor

    GATA3 in mixed tumor.

    GATA3 in mixed tumor

    INSM1.

    INSM1

    Cytology description
    • Reciprocates histologic features, including small to medium cell size, scant cytoplasm, round to oval nuclei, salt and pepper chromatin and nuclear molding
    • Cells are most commonly arranged into small clusters
    • Variable abundance or paucity of tumor cells
    • Inflammatory background primarily comprised of neutrophils
    • Tumor diathesis may be present
    • Limitation: additional histologic component, especially high grade urothelial carcinoma, may obfuscate the diagnosis
    • Reference: Oncol Lett 2014;7:369
    Cytology images

    Contributed by Bonnie Choy, M.D.
    Hypercellular specimen

    Hypercellular specimen



    Images hosted on other servers
    Bladder, pap

    Bladder, Pap

    Positive stains
    Negative stains
    Electron microscopy description
    Molecular / cytogenetics description
    Videos

    Bladder small cell neuroendocrine carcinoma

    Sample pathology report
    • Bladder, lateral wall, transurethral resection:
      • Small cell neuroendocrine carcinoma with invasion into muscularis propria (see comment)
      • Comment: The tumor consists of cohesive, small blue cells with nuclear molding and extensive necrosis. The tumor cells are positive for CK AE1 / AE3 (moderate, dot-like), synaptophysin (strong, diffuse), chromogranin A (moderate, diffuse), INSM1 (strong, diffuse) and TTF1 (strong, diffuse). GATA3 is negative. Ki67 index is > 95%. Small cell neuroendocrine carcinoma of the urinary bladder is the primary consideration based on the anatomical site of the specimen and absence of evidence of pulmonary or other extrapulmonary site of origin. The findings above should be considered in the context of imaging and clinical history.

    • Bladder, posterior wall, transurethral resection:
      • Small cell neuroendocrine carcinoma mixed with high grade invasive urothelial carcinoma (see comment)
      • Comment: Histologic evaluation shows small cell neuroendocrine carcinoma and adjacent high grade urothelial carcinoma. The small cell neuroendocrine carcinoma component is positive for synaptophysin, INSM1, CD56 and chromogranin while negative for GATA3. In contrast, the urothelial carcinoma component is positive for GATA3 while negative for the aforementioned neuroendocrine markers.
    Differential diagnosis
    • Large cell neuroendocrine carcinoma (Clujul Med 2017;90:13)
      • Shares high grade features including necrosis, mitotic activity and high Ki67 index
      • Comprised of larger polygonal cells
      • Lower N:C ratio with vesicular chromatin and prominent nucleoli
      • Positive for neuroendocrine markers
      • Diffuse cytoplasmic staining pattern of CAM5.2 and CK7
    • Metastatic pulmonary small cell neuroendocrine carcinoma:
      • Both entities may be positive for TTF1
      • Lacks TERT promoter mutation
      • Mixed histomorphology is rare
    • Metastatic prostatic small cell neuroendocrine carcinoma (Hum Pathol 2018;79:57):
      • Mixed histomorphology similar to small cell neuroendocrine carcinoma of urothelial origin may be present
      • Often arises in a setting of concomitant prostatic adenocarcinoma
      • PSA may be elevated
      • TMPRSS2::ERG fusion may be present
      • Lacks TERT promoter mutation
    • Well differentiated neuroendocrine tumor (Hum Pathol 2018;79:57):
      • In upper genitourinary tract, originates most commonly from kidneys
      • Histologically characterized by well defined cords, trabeculae and ribbons containing columnar or cuboidal tumor cells
      • Granular eosinophilic (oncocytic) cytoplasm is common
      • Substantially lower mitotic count and Ki67 index, especially if grade < 3
      • Necrosis is absent or less prominent
      • Also positive for neuroendocrine markers
    • Lymphoepithelioma-like carcinoma (Hum Pathol 2018;79:57):
      • Derivation of urothelial carcinoma characterized by large pleomorphic nuclei and prominent nucleoli, often with abundant mitoses
      • Cells are arranged into syncytial sheets with infiltration of epithelium by mixed inflammatory cells
      • Frequently mixed with other malignant cell populations
      • Positive for GATA3 and p63
      • Negative for neuroendocrine markers
    • Neuroblastoma (Pol J Pathol 2014;65:15):
      • Most commonly located in adrenal gland
      • Most common in kids
      • Serum and urine catecholamine assays may be positive
      • Differentiated type is characterized by Homer-Wright pseudorosettes containing central eosinophilic neurofilaments
      • Undifferentiated type is characterized by small to medium sized cells with scant cytoplasm, salt and pepper chromatin and prominent nucleoli may be present
        • Prominent nucleoli may be present
      • ALK1 may be positive in up to 90% of cases
      • May harbor N-MYC amplification
    • Ewing sarcoma (Pol J Pathol 2014;65:15):
      • Comprised of small round cells with sheet-like growth
      • Dense fibrous tissue often surrounds islands of tumor cells
      • Positive for CD99, FLI1 and NKX2.2
      • Variably positive for neuroendocrine markers (NSE is most common [50%])
      • Associated with pathognomonic EWSR1::FLI fusion
      • Elevated erythrocyte sedimentation rate (ESR) may be present
    • Pheochromocytoma / paraganglioma (Anticancer Res 2017;37:4529):
      • Classic clinical triad of episodic headaches, sweating and tachycardia in addition to palpitations, anxiety, paroxysmal hypertension and syncope
      • Serum and urine assays for catecholamine metabolites (e.g., VMA) may be positive
      • Comprised of nests (zellballen) or solid sheets of large polygonal cells with granular, red-purple cytoplasm
      • Chief cells are positive for neuroendocrine markers
      • Sustentacular cells are positive for S100 and often GATA3
      • Negative for cytokeratin stains
      • VHL, RET, NF1 and SDHB mutations may be present
    • Nephroblastoma / Wilms tumor (Pol J Pathol 2014;65:15):
      • Most common in kids
      • Monophasic blastema predominant type is histomorphologically similar
      • Focal epithelial and stromal components may be present
      • Blastema component is positive for WT1, PAX8 and vimentin
      • May be variably positive for CD56 but is negative for other neuroendocrine markers
    • Lymphoma (Pol J Pathol 2014;65:15, Case Rep Hematol 2015;2015:934374):
      • MALT lymphoma is the most common primary lymphoma of bladder
      • Nuclei are usually cleaved or lobulated
      • All lymphomas are positive for CD45 and often either CD3 or CD20
    • Desmoplastic small round cell tumor (Pol J Pathol 2014;65:15):
      • Comprised of nests and trabeculae of small uniform round cells abounded by diffuse desmoplastic stroma
      • Peripheral palisading may be observed
      • Solid pattern lacks desmoplastic stroma and may appear nearly identical to small cell carcinoma
      • Most common in kids and young adults
      • Positive for WT1, CD99, desmin and vimentin
      • EWSR1::WT1 fusion present
    • Melanoma (Pol J Pathol 2014;65:15):
      • Comprised of epithelioid or spindle tumor cells
      • Exhibits extensive nuclear pleomorphism with prominent eosinophilic nucleoli
      • Positive for melanocytic markers (S100, SOX10, HMB45 and MelanA)
    • Embryonal rhabdomyosarcoma (Pol J Pathol 2014;65:15):
      • Most common in kids and infants
      • Nuclear molding may be prominent
      • Mixed hypercellular and hypocellular areas are usually present
      • Typically harbors eosinophilic (rhabdoid) or clear cytoplasm
      • May harbor striations consistent with rhabdoid differentiation
      • Positive for desmin, myogenin, MyoD1 and vimentin
    • Poorly differentiated synovial sarcoma (Pol J Pathol 2014;65:15, Pathol Oncol Res 2001;7:63):
      • Comprised of overlapping round cells with hyperchromatic nuclei, brisk mitotic activity and extensive necrosis
      • Positive for TLE1, CD99 and BCL2
      • May be positive for select neuroendocrine markers including CD56
      • Fusions of SS18 and SSX1 / SSX2 / SSX4 are present
    Additional references
    Board review style question #1

    A 68 year old man with a 48 pack year smoking history presents with gross hematuria of 1 month's duration. Urinalysis reveals 9 red blood cells (RBCs) while a complete blood count (CBC) displays hemoglobin of 9.8 g/dL. An abdominopelvic ultrasound reveals a thickened lateral vesical wall while computerized tomography (CT) elucidates a 4.6 cm heterogeneous mass in the greatest dimension with possible extravesical extension into locoregional lymph nodes. Cystoscopy reveals a nodular mass with overlying necrosis. Due to suspicion of a bladder malignancy, a radical cystectomy is performed. An H&E stained image from this specimen is shown above. Immunohistochemistry is pertinent for positive staining with synaptophysin, CD56, NSE, INSM1 and CK AE1 / AE3, the latter of which exhibits a perinuclear dot-like pattern. Lesional cells are negative for GATA3, p63, CD99, WT1, HMB45, CD45, vimentin, desmin and NKX2.2. The Ki67 index exceeds 95%. What is the most likely diagnosis?

    1. Desmoplastic small round cell tumor
    2. Ewing sarcoma
    3. Large cell neuroendocrine carcinoma
    4. Pheochromocytoma
    5. Small cell neuroendocrine carcinoma
    Board review style answer #1
    E. Small cell neuroendocrine carcinoma. The high N:C ratio, nuclear molding, abundant mitoses and karyorrhectic debris on histology in tandem with the neuroendocrine immunohistochemical profile and history of smoking is most consistent with small cell neuroendocrine carcinoma. Answer A is incorrect because although the solid form of desmoplastic small round cell tumor may mimic the histomorphology of small cell carcinoma, the tumor cells in question are negative for CD99, desmin, vimentin and WT1. Answer B is incorrect because the tumor cells are negative for CD99 and NKX2.2. Answer C is incorrect because the tumor in question displays a high N:C ratio, stippled chromatin and inconspicuous nucleoli rather than the lower N:C ratio, vesicular chromatin and prominent nucleoli associated with large cell neuroendocrine carcinoma. Answer D is incorrect because the tumor cells do not exhibit granular reddish purple cytoplasm or GATA3 staining and are positive for cytokeratins; moreover, the patient does not display the characteristic symptoms of excess catecholamine secretion.

    Comment Here

    Reference: Small cell neuroendocrine carcinoma
    Board review style question #2

    Which of the following is true of primary upper genitourinary tract small cell neuroendocrine carcinomas?

    1. History of smoking tobacco use is common
    2. Surgery with neoadjuvant or adjuvant cyclophosphamide-based chemotherapy is the mainstay of treatment
    3. TERT promoter mutations are specific for small cell neuroendocrine carcinomas of renal origin
    4. The nephroureter is the most common site of origin within this region
    5. TMPRSS2::ERG fusions are prevalent in small cell neuroendocrine carcinomas of bladder origin
    Board review style answer #2
    A. History of smoking tobacco use is common. As with other carcinomas from this region, smoking tobacco use is a common historical factor in patients with upper genitourinary tract small cell neuroendocrine carcinomas. Answer B is incorrect because platinum based agents, especially cisplatin, are used most often in tandem with surgical removal of tumors, not cyclophosphamide based regimens. Answer C is incorrect because TERT promoter mutations have only been reported in small cell neuroendocrine carcinomas of bladder origin. Answer D is incorrect as the majority of reported upper genitourinary tract small cell neuroendocrine carcinomas arise from the bladder rather than the kidney and ureters. Answer E is incorrect because TMPRSS2::ERG fusions have only been reported in small cell neuroendocrine carcinomas of prostatic origin.

    Comment Here

    Reference: Small cell neuroendocrine carcinoma

    Squamous cell carcinoma
    Definition / general
    • Malignant epithelial neoplasm arising in the urinary bladder, renal pelvis or ureter demonstrating a pure squamous cell phenotype
    • This topic describes classic squamous cell carcinoma; please see separate topic on verrucous squamous cell carcinoma
    Essential features
    • Uncommon subtype of carcinoma showing pure squamous morphology without any component of conventional urothelial carcinoma
    • Predisposing factors include Schistosoma infection, smoking and conditions resulting in chronic irritation for bladder tumors and nephrolithiasis, anatomic anomalies of the kidney and chronic recurrent infections for upper urinary tract tumors
    • May be associated with keratinizing squamous metaplasia / dysplasia of the mucosa
    • Must be distinguished from urothelial carcinoma with squamous differentiation, secondary spread of squamous cell carcinoma primary to another site, e.g. cervix, penis, anus and metastasis
    Terminology
    • Classic / typical squamous cell carcinoma
    ICD coding
    • ICD-10: C67 - malignant neoplasm of bladder
    • ICD-10: C65 - malignant neoplasm of renal pelvis
    • ICD-10: C66 - malignant neoplasm of ureter
    • ICD-O: 8070/3 - squamous cell carcinoma, NOS
    Epidemiology
    • Bladder
      • Incidence varies by geographic region
      • 2 - 5% of bladder tumors in Western countries (nonbilharzial) (Arab J Urol 2016;14:183)
        • Most common nonurothelial subtype of bladder cancer
      • 20 - 30% of bladder tumors in geographic regions with high prevalence of schistosomiasis (bilharzial) including Egypt and other parts of Africa (Arab J Urol 2016;14:183)
      • Slight male predominance
    • Renal pelvis / ureter
    Sites
    Pathophysiology / etiology
    • Bladder
      • Schistosomiasis
        • Schistosoma haematobium
        • Major risk factor for bladder squamous cell carcinoma in geographic regions with high prevalence of schistosomiasis including Egypt and other parts of Africa
      • Smoking: 5x increased risk compared with nonsmokers (JAMA 2011;306:737)
      • Chronic bladder irritation / inflammation associated with:
        • Long term catheterization
        • Bladder calculi or foreign bodies
        • Neurogenic bladder
        • Bladder exstrophy (Urology 2006;67:199)
      • Renal pelvis / ureter
    Clinical features
    • Typically presents with hematuria
    • May also present with dysuria, urgency and frequent urination, recurrent urinary tract infection, urinary obstruction and flank or suprapubic pain
    Diagnosis
    • Urine cytology
    • Cystoscopy and biopsy / transurethral resection of bladder tumor
    Radiology description
    Radiology images

    Images hosted on other servers:

    MRI bladder

    Solitary mass lower right kidney

    Mass in right renal pelvis

    Prognostic factors
    Case reports
    • 45 year old woman with poorly differentiated squamous cell carcinoma arising on a background of recurrent condyloma acuminatum (Urol Case Rep 2016;7:61)
    • 46 year old woman with basaloid squamous cell carcinoma of the bladder and concomitant HPV infection of the genital tract (Int J Urol 2015;22:222)
    • 57 year old man with recurrent stone formation presenting with muscle invasive squamous cell carcinoma of the bladder (Clin Case Rep 2017;5:1616)
    • 61 year old woman with enterovesical fistula secondary to squamous cell carcinoma of the bladder (Urol Case Rep 2015;3:201)
    • 76 year old woman with squamous cell carcinoma of the upper urinary tract presenting with paraneoplastic syndrome (Urol Ann 2020;12:388)
    • 79 year old woman with squamous cell carcinoma of the renal pelvis associated with staghorn calculi (Case Rep Oncol 2015;8:399)
    • 80 year old woman with squamous cell carcinoma in the renal pelvis of a horseshoe kidney (Int J Urol 2004;11:782)
    Treatment
    Clinical images

    Images hosted on other servers:

    Bladder exstrophy

    Gross description
    • Usually large exophytic bulky tumor; some cases are predominantly flat with ulcerating / infiltrating appearance (Am J Surg Pathol 2007;31:1777)
    • Typically has tan-white coloration
    • Often necrotic with flaky keratin material on the surface
    Gross images

    Bladder

    Contributed by Susan Prendeville, M.D.
    Cystoprostatectomy specimen

    Cystoprostatectomy
    specimen
    Cross section of tumor

    Cross section of tumor

    Ulcerating tumor

    Ulcerating tumor




    Renal pelvis

    Contributed by Nicole K. Andeen, M.D.
    Dilated renal calyces due to reflux

    Dilated renal calyces due to reflux

    Solid, tannish white with central necrosis

    Solid, tannish white with central necrosis

    Microscopic (histologic) description
    • May be well, moderately or poorly differentiated
      • Moderately or poor differentiated tumors are most common
    • Irregular infiltrating nests or sheets of malignant squamous cells with destructive stromal invasion
    • Squamous differentiation in the form of keratin pearls, individual cell keratinization or intercellular bridges; findings may be focal in poorly differentiated tumors
    • Often associated with surface keratinizing squamous metaplasia and dysplasia / squamous carcinoma in situ (Am J Surg Pathol 2007;31:1777)
    • Must not show any component of conventional urothelial carcinoma or urothelial carcinoma in situ
      • If present, even focally, tumor is designated as urothelial carcinoma with squamous differentiation
    Microscopic (histologic) images

    Bladder

    Contributed by Susan Prendeville, M.D.
    Destructive stromal invasion

    Destructive stromal invasion

    Irregular squamous nests

    Irregular squamous nests

    Keratin pearls

    Keratin pearls

    Prominent keratinization

    Prominent keratinization

    Intercellular bridges

    Intercellular bridges


    Poorly differentiated tumor

    Poorly differentiated tumor

    Squamous dysplasia / carcinoma in situ

    Squamous dysplasia / carcinoma in situ

    Invading muscularis propria

    Invading muscularis propria

    Keratinizing squamous dysplasia

    Keratinizing squamous dysplasia

    <i>Schistosoma haematobium</i> eggs

    Schistosoma haematobium eggs




    Renal pelvis

    Contributed by Nicole K. Andeen, M.D.
    Invasive tumor

    Invasive tumor

    Entrapped glomeruli

    Entrapped glomeruli

    No urothelial differentiation

    No urothelial differentiation

    Individual cell keratinization

    Individual cell keratinization

    Entrapped renal tubules

    Entrapped renal tubules

    Virtual slides

    Images hosted on other servers:

    Cystectomy with squamous cell carcinoma

    Cytology description
    • Keratinized cells demonstrate dense orangeophilic cytoplasm on Pap stain
    • Hyperchromatic nuclei with irregular nuclear membranes
    • Often necroinflammatory background
    Cytology images

    Contributed by Susan Prendeville, M.D. and Bonnie Choy, M.D.

    Malignant squamous cells

    Keratinized malignant cells

    Squamous cell carcinoma

    Squamous cell carcinoma

    Positive stains
    Negative stains
    Molecular / cytogenetics description
    Sample pathology report
    • Bladder, tumor left lateral wall, transurethral resection:
      • Invasive poorly differentiated carcinoma with extensive squamous differentiation
      • No evidence of in situ carcinoma (urothelial or squamous) in this biopsy material (see comment)
      • Comment: The differential includes pure squamous cell carcinoma and urothelial carcinoma with extensive squamous differentiation. This distinction requires evaluation of the entire resected tumor. In addition, clinical / radiological correlation is advised to ensure the tumor is compatible with a bladder primary.
    • Bladder and prostate, robotic radical cystoprostatectomy:
      • Bladder with invasive squamous cell carcinoma, moderately differentiated (see synoptic report)
      • Prostate, negative for carcinoma
    Differential diagnosis
    • Urothelial carcinoma with squamous differentiation:
      • Squamous differentiation is common in urothelial carcinoma and may be extensive
      • Careful search for conventional urothelial carcinoma component or urothelial carcinoma in situ (may be focal)
      • Immunohistochemistry is of limited utility in this differential due to overlapping profiles; a panel approach may aid diagnosis in some cases: (squamous markers CK14 / desmoglein 3 with urothelial markers GATA3 / uroplakin / S100P) (Hum Pathol 2013;44:164)
    • Metastasis or direct extension of squamous cell carcinoma primary to another site, e.g. cervix, penis, anus:
      • Requires clinical and radiological correlation
      • p16 expression not of use in this differential
      • In situ hybridization for HPV DNA supports cervical primary (expressed in majority of cervical but not bladder squamous cell carcinoma) (Am J Surg Pathol 2014;38:e20)
    • Verrucous squamous cell carcinoma:
      • Well differentiated tumor showing minimal cytologic atypia
      • Hyperkeratotic papillary or undulating projections with a pushing type of invasion at the base comprising rounded, broad based tongues of epithelium
      • Absence of irregular infiltrating nests
    Board review style question #1

    A 65 year old man underwent radical cystectomy for a bladder mass and the entire tumor looked like the histologic image shown above. Which of the following is true regarding this type of tumor arising in the urinary bladder?

    1. Gross examination usually shows a small tumor
    2. Hematuria is not a clinical feature
    3. It is associated with Schistosoma haematobium infection
    4. It is the most common type of bladder carcinoma
    5. Smoking is not a risk factor
    Board review style answer #1
    C. It is associated with Schistosoma haematobium infection

    Comment Here

    Reference: Bladder squamous cell carcinoma
    Board review style question #2
    Which of the following is true about squamous cell carcinoma of the urinary bladder?

    1. It is associated with keratinizing squamous metaplasia / dysplasia of the bladder mucosa
    2. It is negative for CK5/6
    3. It is negative for p63
    4. It is the same as urothelial carcinoma with squamous differentiation
    5. p16 immunohistochemistry is reliable to distinguish between a bladder and cervical primary
    Board review style answer #2
    A. It is associated with keratinizing squamous metaplasia / dysplasia of the bladder mucosa

    Comment Here

    Reference: Bladder squamous cell carcinoma

    Squamous cell papilloma
    Definition / general
    • Rare, benign tumor of the urinary bladder showing delicate fibrovascular cores lined by squamous epithelium
    Essential features
    • Rare, benign bladder lesion with female predilection
    • Presents with gross hematuria and irritative bladder symptoms
    • Recurrence is rare
    • Not related to condyloma acuminatum
    Terminology
    • Squamous papilloma
    ICD coding
    • ICD-O: 8052/0 - squamous cell papilloma, NOS
    • ICD-10: D30.3 - benign neoplasm of bladder
    Epidemiology
    • Rare, benign urothelial lesion with < 20 cases reported in known literature
    • F > M (6:1) (Cancer 2000;88:1679)
    • Fourth to seventh decade (Cancer 2000;88:1679)
    • Can arise de novo or along with previous urothelial lesions
    Sites
    • Most cases involve the dome and the lateral and posterior walls of the bladder (Cancer 2000;88:1679)
    Pathophysiology
    • Largely unknown
    Etiology
    • Smoking
    • Occupational exposure to amines
    Clinical features
    Diagnosis
    Laboratory
    • Urine cytology is negative for malignant cells
    Radiology description
    • Ultrasonography may show thickened bladder wall
    Prognostic factors
    • Recurrence is very rare and progression to carcinoma has not been reported (Cancer 2000;88:1679)
    Case reports
    • 32 - 82 year old patients with squamous cell papilloma of urinary bladder, including squamous papilloma related to HPV (Cancer 2000;88:1679)
    • 74 year old man with irritative voiding symptoms and exophytic mass found at the mucosa of the floor and the posterior wall of the bladder (Case Rep Pathol 2013;2013:486312)
    • Noninvasive squamous lesions in the urinary bladder; a case series with 5 reported cases of squamous cell papilloma of the urinary bladder (Am J Surg Pathol 2006;30:883)
    Treatment
    Clinical images

    Images hosted on other servers:
    Cystoscopy finding: whitish exophytic lesion

    Cystoscopy finding: whitish exophytic lesion

    Gross description
    • Small, whitish, polypoid lesion
    Microscopic (histologic) description
    Microscopic (histologic) images

    Contributed by Harsh Batra, M.B.B.S., D.C.P., D.N.B. and Anil Parwani, M.D., Ph.D., M.B.A.
    Transverse section of papillae

    Transverse section of papillae

    Transverse section through a single papilla

    Transverse section through a single papilla

    Fibrovascular cores

    Fibrovascular cores

    Positive stains
    Molecular / cytogenetics description
    Sample pathology report
    • Urinary bladder, lateral wall, transurethral resection of bladder tumor:
      • Squamous papilloma (see comment)
      • Comment: Microscopic examination shows multiple papillae with fibrovascular core lined by keratinized squamous epithelium. No evidence of stromal invasion, atypia or koilocytic changes seen in the biopsy submitted. Clinical correlation with follow up is advised.
    Differential diagnosis
    • Condyloma acuminatum:
      • Multiple, extensive lesions
      • External genitalia condyloma or history of immunosuppression
      • Papillary fronds lined by hyperplastic, metaplastic squamous epithelium that may be hyperkeratotic
      • Presence of koilocytes
      • p53 overexpression by IHC
      • HPV DNA present
      • Aneuploid
    • Verrucous carcinoma:
      • Rare variant of squamous cell carcinoma
      • Diffuse, extensive lesions
      • Endophytic growth pattern with broad nests of tumor cells intimately associated with lamina propria or detrusor muscle with minimal stromal reaction
      • Margins of the lesion are pushing
      • Nuclear atypia is minimal to moderate
      • p53 staining is positive
      • HPV DNA negative
    • Keratinizing squamous metaplasia:
      • Nonpolypoid, flat lesion showing thick squamous epithelium with pronounced hyperkeratosis and hypergranulosis
      • Extensive lesions are associated with neoplasia, so it is recommended to report the presence and the extent (focal versus extensive) of keratinizing squamous metaplasia
    • Squamous cell carcinoma:
      • Irregular infiltrating nests or sheets of malignant squamous cells with destructive stromal invasion
      • Presence of keratin pearls, individual cell keratinization or intercellular bridges
      • Often associated with surface keratinizing squamous metaplasia and dysplasia
    Board review style question #1
    Squamous papilloma of the urinary bladder is which of the following?

    1. Benign lesion
    2. Carcinoma in situ
    3. Malignant
    4. Papillary urothelial neoplasm of unknown malignant potential (PUNLMP)
    Board review style answer #1
    A. Benign lesion

    Comment Here

    Reference: Squamous cell papilloma
    Board review style question #2


    Which of the following histopathological features distinguishes squamous papilloma of the bladder from condyloma acuminatum?

    1. Absence of fibrovascular cores in the papillae
    2. Absence of koilocytes
    3. Presence of invasion into submucosa
    4. Presence of umbrella cells
    Board review style answer #2
    B. Absence of koilocytes. Squamous papilloma shows absence of koilocytic change, whereas koilocytes are present in condyloma acuminatum.

    Comment Here

    Reference: Squamous cell papilloma

    Squamous metaplasia
    Definition / general
    • Replacement of urothelium with stratified squamous epithelium
    • 2 subtypes: nonkeratinizing and keratinizing
    Essential features
    • Presence of squamous epithelium
    • May show glycogenation (clearing of cytoplasm)
    • Nonneoplastic cytology: normally maturing squamous epithelium from basal to superficial
    • Hyperkeratosis and parakeratosis with presence of granular layer might be seen
    • Absence of koilocytosis
    Terminology
    • Keratinizing subtype is also called leukoplakia (clinical term)
    ICD coding
    • ICD-10: N32.89 - other specified disorders of bladder
    Epidemiology
    • All ages
    • Normally seen in the trigone of the bladder in women of reproductive age
    Sites
    • Bladder (most commonly), ureter and renal pelvis
    • In trigone of women, it is normal variation and not metaplasia
    Pathophysiology
    • Chronic irritation, inflammation, catheters, stones, infection (Urol Int 2008;81:247)
    • Reversible if the irritating or inciting factor is removed
    • With continued insult, can progress to preneoplastic / premalignant (in situ) and invasive squamous cell carcinoma (Histopathology 2019;74:68)
    Etiology
    • Urinary tract infections: Escherichia coli, Proteus and Streptococcus faecalis
    • Schistosomiasis is commonly associated, due to chronicity (J Cell Biol 2005;171:835)
    • Urinary tract irritants: indwelling catheters, urinary calculi, urinary outflow obstruction, fistula, tumors (J Urol 1999;161:1106)
    • Bladder exstrophy
    • Neurogenic bladder, previous bladder surgery
    • Vitamin A deficiency
    Clinical features
    • Vaginal (nonkeratinizing) subtype:
      • Also called pseudomembranous trigonitis; common in trigone
      • Normal finding in females
      • Rarely occurs in children (Urol Int 2006;77:46)
    • Keratinizing subtype:
      • Also called leukoplakia
      • More common in males
      • Associated with chronic irritation (catheters, stones, parasite eggs)
      • Rule out atypia
      • Risk factor for squamous cell carcinoma (Urol Int 2008;81:247, Eur Urol 2002;42:469)
    Diagnosis
    Prognostic factors
    • Keratinizing squamous metaplasia may transform to squamous dysplasia / carcinoma (Urol Int 2008;81:247)
    • Important to report extent of involvement
    • Sampling of all lesional tissue is important to rule out dysplastic in situ or invasive squamous lesion in the vicinity
    • Requires close follow up
    Case reports
    Treatment
    • Spontaneous resolution in some
    • Careful follow up of keratinizing squamous metaplasia, particularly if atypical, to detect dysplasia or carcinoma (Urol Int 2008;81:247, Am J Surg Pathol 2006;30:883)
    • Transurethral resection and fulguration
    • Prophylactic cystectomy is reserved for extensive disease (Eur Urol 2002;42:469)
    • Possibly sodium pentosan polysulfate (used for painful bladder syndrome / interstitial cystitis) (Drugs 2006;66:821)
    Clinical images

    Images hosted on other servers:

    Flaky, white, plaque-like lesions

    Cystoscopy of trigone in 15 year old girl

    Gross description
    • Epithelial lining appears paler or thicker than normal with irregular borders and a surrounding zone of erythema
    • Sometimes presents with subtle changes
    Microscopic (histologic) description
    • Nonkeratinized squamous metaplasia:
      • Epithelium has abundant intracytoplasmic glycogen, like vaginal or cervical epithelium
      • Trigone in women may have glycogenated nonkeratinizing squamous mucosa and is considered as a normal histologic variant with no significantly increased risk for squamous carcinoma
      • May not be reported if lacking associated dysplastic changes
    • Keratinizing squamous metaplasia:
      • Hyperkeratotic squamous epithelium with or without parakeratosis (Urol Int 2008;81:247)
      • Presence and extent of keratinizing squamous metaplasia should be reported
    Microscopic (histologic) images

    Contributed by Maria Tretiakova, M.D., Ph.D.

    Nonkeratinizing squamous metaplasia

    Keratinizing squamous metaplasia with parakeratosis

    Squamous metaplasia with cystitis glandularis

    Keratinizing squamous metaplasia

    Cytology description
    • Benign squamous cells
    Cytology images

    Images hosted on other servers:

    Benign squamous cells

    Positive stains
    Negative stains
    Molecular / cytogenetics description
    • Molecular drivers of keratinizing squamous metaplasia are poorly studied but at least 1 human tissue regeneration model implicates PPARy and PTEN in its development (Am J Pathol 2013;182:449)
    Sample pathology report
    • Bladder, biopsy:
      • Keratinizing squamous metaplasia; ~20% in submitted sample (see comment)
      • Comment: Entire tissue received was submitted to rule out premalignant or invasive lesions. If clinical concern remains on close clinical follow up, sampling of multiple areas is recommended.
    Differential diagnosis
    • Radiation atypia:
      • Degenerative type epithelial changes, pseudocarcinomatous hyperplasia, multinucleation, fibrin thrombi
    • Squamous papilloma:
      • Exophytic papillary fibrovascular cores lined by benign squamous epithelium
      • Lack of inflammation or koilocytosis
    • Squamous dysplasia:
      • Atypia, lack of maturation, increased mitoses, high N/C ratio
      • If full thickness, qualifies for in situ squamous cell carcinoma
    • Urothelial carcinoma in situ (CIS):
      • Marked full thickness urothelial atypia
      • No keratinization or intercellular bridges
    • Squamous cell carcinoma (renal pelvis, ureter and bladder):
      • Marked atypia, hemorrhage, necrosis, invasion into lamina propria
      • Rule out more common urothelial carcinoma with variable amount of squamous differentiation
    • Verrucous hyperplasia:
      • Church spire-like multilayering, benign cytology
    • Verrucous carcinoma:
      • Large size
      • Well differentiated tumor with broad based invasive front, lymphocytic inflammation at the base
    • Condyloma acuminatum:
      • Exophytic proliferation of nonkeratinizing squamous changes with viral cytopathic effect exhibited by koilocytosis (perinuclear clearing with nuclear membrane irregularity), binucleation
      • No deep extension into lamina propria
      • Associated with urethral changes in immunosuppressed
    Board review style question #1


    What is the most important aspect of reporting in keratinizing squamous metaplasia?

    1. Extent of disease
    2. Glycogenation
    3. Hyperkeratosis
    4. Parakeratosis
    Board review style answer #1
    A. Extent of disease. More extensive squamous metaplasia is associated with increased risk of in situ and invasive squamous cell carcinoma. The cut off percentage is not established but the patient will be recommended for frequent follow ups.

    Comment Here

    Reference: Squamous metaplasia
    Board review style question #2


    A 45 year old woman with a history of lupus erythematosus has urgency, frequency and irritation during micturition for 6 months. She undergoes cystoscopy with irregularity in bladder mucosa. A bladder biopsy reveals vaginal type mucosa without any dysplasia. The biopsy is most likely taken from which of the following areas?

    1. Anterior bladder wall
    2. Bladder neck
    3. Near ureteral orifices
    4. Trigone of bladder
    Board review style answer #2
    D. Trigone of bladder. Trigone of bladder may have vaginal type or glycogenated nonkeratinizing mucosa without any associated abnormality in females. In this patient, the biopsy was done to rule out flat carcinoma in situ, which may clinically mimic interstitial cystitis. Interstitial cystitis is a condition of unknown etiology, with some cases associated with autoimmune disorders, such as lupus erythematosus.

    Comment Here

    Reference: Squamous metaplasia

    Staging-bladder carcinoma
    Definition / general
    • All carcinomas of the bladder are covered by this staging system
    • The following topics are not covered: urachal carcinoma, paraganglioma, sarcoma or lymphoma
    Essential features
    • AJCC 7th edition staging was sunset on December 31, 2017; as of January 1, 2018, use of the 8th edition is mandatory
    Primary tumor (pT)
    • pTX: cannot be assessed
    • pT0: no evidence of primary tumor
    • pTa: noninvasive papillary carcinoma
    • pTis: carcinoma in situ
    • pT1: invades lamina propria
    • pT2a: invades inner half of muscularis propria
    • pT2b: invades outer half of muscularis propria
    • pT3a: microscopically invades perivesical tissue
    • pT3b: macroscopically invades perivesical tissue
    • pT4a: directly invades prostatic stroma, seminal vesicles, uterus or vagina
    • pT4b: directly invades pelvic wall or abdominal wall

      Lamina propria (pT1)
    • Lamina propria contains connective tissues between urothelium and detrusor muscle (muscularis propria), made of loose stroma, variably sized blood vessels and thin muscle bands of muscularis mucosae (see Histology)
    • Has nests, clusters or single tumor cells, sometimes with prominent retraction artifact (mimics lymphovascular invasion)
    • Often has desmoplastic or inflammatory stromal response and absent or irregular basement membrane (not seen with noninvasive low grade papillary carcinoma with inverted pattern)
    • Tumor cells often have abundant eosinophilic cytoplasm at advancing edge (paradoxical differentiation) but have enlarged, malignant appearing nuclei
    • If tumor cells hug the mucosa, they should be more anaplastic than benign mucosal cells
    • Pitfalls include tangential sectioning, poor specimen orientation, inflammation, thermal injury, deceptively bland cytology and pseudoinvasive nests of nonneoplastic proliferative urothelial cells (Pathology 2003;35:484)
    • Subcategorization of pT1 based on muscularis mucosae is difficult and may not correlate with progression (Mod Pathol 1996;9:1035, Arch Pathol Lab Med 2022;146:1131)
    • Other strategies for the subcategorization of pT1 (such as based on depth of invasion) have been proposed but are not a part of AJCC staging (Arch Pathol Lab Med 2022;146:1131)
    • pTa cases may actually be invasive when studied by electron microscopy; significance is unclear (Am J Clin Pathol 2003;120:188)
    • Invasion of lamina propria (pT1) is not as clinically crucial as invasion of muscularis propria (pT2)

      Muscularis propria (pT2)
    • Muscularis propria is thick aggregated muscle bundles of detrusor muscle; must distinguish from hypertrophic muscularis mucosae (Am J Surg Pathol 2007;31:1420)
    • Muscularis propria presence should be specified in transurethral resections and cold cup biopsies
    • Muscularis propria invasion prompts definitive therapy, typically cystectomy with or without neoadjuvant therapy
    • Can use muscle markers such as desmin or SMA to highlight muscle and keratins such as AE1 / AE3 to highlight carcinoma in challenging cases
    • Difficult to subcategorize as pT2a or pT2b unless the pathologist has full thickness bladder wall and well oriented section such as in a cystectomy specimen and thus would not be documented in biopsy or transurethral resection of bladder tumor (TURBT) specimens

      Perivesical fat (pT3)
    • Perivesicular adipose tissue is deep to muscularis propria but is also present within deep lamina propria, usually as small localized aggregates and within muscularis propria (superficial and deep)
    • Beware of inappropriate classification as pT3 due to tumor infiltration of adipose within the lamina propria, particularly in TURBT specimens (Am J Surg Pathol 2000;24:1286)
    • Note that the gross prospector must document the impression of perivesicular adipose invasion to allow for subcategorization as pT3a versus pT3b as pT3b requires macroscopic invasion
    • Many cases lack the gross impression of perivesicular adipose invasion and thus cannot be accurately classified (Hum Pathol 2017;61:190)
    • Subcategorization into pT3a and pT3b has an uncertain prognostic impact with some studies showing no difference in outcome (Virchows Arch 2012;461:467)

      Prostate gland (pT4 versus pT2)
    • If carcinoma of the bladder transmurally invades through the bladder wall and into the prostatic stroma, the tumor is designated as pT4a
    • If carcinoma of the bladder colonizes the prostatic urethra via in situ spread and then invades the underlying prostatic stroma, it is not pT4a (but can be considered a separate tumor of the prostatic urethra with a pT2 designation; see Staging of the prostatic urethra)
    • Survival of patient with pT4 tumors is poor (median: < 1 year; 1.5 year survival: < 15%) (Virchows Arch 2012;461:467)
    Regional lymph nodes (pN)
    • pNX: cannot be assessed
    • pN0: no regional lymph node metastasis
    • pN1: metastasis in 1 true pelvic lymph node
    • pN2: metastasis in greater than 1 true pelvic lymph node
    • pN3: metastasis in common iliac lymph node

      Notes:
    • Regional lymph nodes are the true pelvic lymph nodes, which include the following
      • Perivesical
      • Hypogastric / deep obturator / fossa of Marcille / internal iliac
      • Obturator
      • External iliac
      • Presacral / sacral / lateral sacral / sacral promontory
    • Be aware that neoadjuvant therapy may result in organ confined tumor or no residual tumor (ypT0 - 2a) yet still harbor lymph node metastases (10% in one study); this phenomenon was not observed in untreated cases (Virchows Arch 2012;461:467)
    • Survival of patient with pN1 - 3 tumors is poor (median: < 1 year; 1.5 year survival: 30%) (Virchows Arch 2012;461:467)
    Distant metastasis (pM)
    • pM1a: metastasis in nonregional lymph node (ex: caval / aortic, inguinal)
    • pM1b: metastasis in other distant site
    Prefixes
    • y: preoperative radiotherapy or chemotherapy
    • r: recurrent tumor stage
    AJCC prognostic stage groups
    Stage group 0a: Ta N0 M0
    Stage group 0is: Tis N0 M0
    Stage group I: T1 N0 M0
    Stage group II: T2a - 2b N0 M0
    Stage group IIIA: T3a - 4a N0 M0
    T1 - 4a N1 M0
    Stage group IIIB: T1 - 4a N2 - 3 M0
    Stage group IVA: T4b any N M0
    any T any N M1a
    Stage group IVB: any T any N M1b
    Registry data collection variables
    • Extranodal extension
    • Number of lymph nodes examined and number positive
    • Grade
    • Lymphovascular invasion
    • Concurrent pTa with pTis
    Histologic grade (G)
    • Urothelial carcinoma
      • LG: low grade
      • HG: high grade
    • Squamous cell carcinoma and adenocarcinoma
      • GX: cannot be assessed
      • G1: well differentiated
      • G2: moderately differentiated
      • G3: poorly differentiated

    Notes:
    • Clinical management with respect to intravesicular bacillus Calmette-Guérin (BCG) / chemotherapy and frequency of surveillance differs for high grade versus low grade tumors
    Histopathologic type
    • Noninvasive low grade papillary urothelial carcinoma
    • Noninvasive high grade papillary urothelial carcinoma
    • Urothelial carcinoma in situ
    • Invasive urothelial carcinoma
    • Invasive urothelial carcinoma subtypes
      • Squamous, glandular, trophoblastic or Mullerian divergent differentiation
      • Nested
      • Large nested
      • Tubular and microcystic
      • Micropapillary
      • Lymphoepithelioma-like
      • Plasmacytoid
      • Giant cell
      • Lipid rich
      • Clear cell (glycogen rich)
      • Sarcomatoid
      • Poorly differentiated
    • Squamous cell carcinoma
    • Adenocarcinoma
    • Small cell neuroendocrine carcinoma
    Gross images

    Contributed by Debra L. Zynger, M.D.
    Muscularis propria invasion (pT2b) Muscularis propria invasion (pT2b)

    Muscularis propria invasion (pT2b)

    Prostatic invasion (pT4a)

    Prostatic invasion (pT4a)

    Residual noninvasive tumor (ypTis / a)

    Residual noninvasive tumor (ypTis / a)

    Microscopic (histologic) images

    Contributed by Debra L. Zynger, M.D.
    Noninvasive low grade (pTa)

    Noninvasive low grade (pTa)

    Microinvasion (pT1)

    Microinvasion (pT1)

    Lamina propria invasion (pT1)

    Lamina propria invasion (pT1)

    Within muscularis mucosae (pT1)

    Within muscularis mucosae (pT1)

    Within muscularis propria (pT2)

    Within muscularis propria (pT2)

    Within perivesicular adipose (pT3)

    Within perivesicular adipose (pT3)


    Within prostate (pT4a)

    Within prostate (pT4a)

    Within prostate and prostatic glands (pT4a)

    Within prostate and prostatic glands (pT4a)

    Lung metastasis (pM1b)

    Lung metastasis (pM1b)

    Uterus metastasis (pM1b)

    Uterus metastasis (pM1b)

    Lymph node metastasis (pN3)

    Common iliac lymph node metastasis (pN3)

    Board review style question #1

    Treatment with cystectomy is typical if there is invasion into which of the following structures?

    1. Lamina propria
    2. Muscularis mucosae
    3. Muscularis propria
    4. Urothelium
    Board review style answer #1
    C. Muscularis propria. Muscularis propria is correct because invasion of the muscularis propria of the bladder is managed via cystectomy if the patient is a surgical candidate. Neoadjuvant chemotherapy can be given prior to surgery for suitable candidates. Answers A, B and D are incorrect because cystectomy is not typical for noninvasive tumors, tumors invading the lamina propria or tumors invading the muscularis mucosae, which is wispy muscle within the lamina propria. However, cystectomy is a management option for BCG refractory tumors that are noninvasive or invade into the lamina propria.

    Comment Here

    Reference: Staging-bladder carcinoma

    Staging-female urethra
    Definition / general
    • All carcinomas of the female urethra are covered by this staging system
    Essential features
    • AJCC 7th edition staging was sunset on December 31, 2017; as of January 1, 2018, use of the 8th edition is mandatory
    ICD coding
    • ICD-10: C68.0 - malignant neoplasm of urethra
    Primary tumor (pT)
    • pTX: cannot be assessed
    • pT0: no evidence of primary tumor
    • pTa: noninvasive papillary carcinoma
    • pTis: carcinoma in situ
    • pT1: invasion of urethral subepithelial connective tissue
    • pT2: invasion of periurethral muscle
    • pT3: invasion of anterior vagina
    • pT4: invasion of adjacent organs (example: bladder wall)
    Regional lymph nodes (pN)
    • pNX: cannot be assessed
    • pN0: no regional lymph node metastasis
    • pN1: metastasis in 1 regional lymph node
    • pN2: metastasis in greater than 1 regional lymph node

    Note:
    • Regional lymph nodes include superficial / deep inguinal, perivesical, obturator, internal iliac / hypogastric, external iliac, presacral, sacral NOS, pelvic NOS
    Distant metastasis (pM)
    • pM1: distant metastasis
    Prefixes
    • y: preoperative radiotherapy or chemotherapy
    • r: recurrent tumor stage
    AJCC prognostic stage groups
    Stage 0a:  Ta  N0  M0
    Stage 0is:  Tis  N0  M0
    Stage I:  T1  N0  M0
    Stage II:  T2  N0  M0
    Stage III:  T3  N0  M0
     T1 - 3  N1  M0
    Stage IV:  T4  N0 - 2  M0 - 1
     TX - 4  N2  M0 - 1
     TX - 4  NX - 2  M1
    Registry data collection variables
    • Urothelial carcinoma grade (high / low)
    • Squamous cell / adenocarcinoma grade (1 - 3)
    Histologic grade (G)
    • Urothelial carcinoma
      • LG: low grade
      • HG: high grade
    • Squamous cell / adenocarcinoma
      • GX: cannot be assessed
      • G1: well differentiated
      • G2: moderately differentiated
      • G3: poorly differentiated
    Histopathologic type
    • Noninvasive urothelial carcinoma
    • Invasive urothelial carcinoma
      • Conventional urothelial carcinoma
      • Urothelial carcinoma variants
    • Squamous cell carcinoma
    • Adenocarcinoma (including those arising from periurethral glands)
    • Small cell carcinoma
    Board review style question #1
    A urothelial carcinoma of the female urethra at the deepest point of invasion involves the bladder wall. Which is the correct pT category?

    1. pT1
    2. pT2
    3. pT3
    4. pT4
    Board review style answer #1

    Staging-male urethra
    Definition / general
    • All carcinomas of the male urethra are covered by this staging system
    Essential features
    • AJCC 7th edition staging was sunset on December 31, 2017; as of January 1, 2018, use of the 8th edition is mandatory
    ICD coding
    • ICD-10: C68.0 - malignant neoplasm of urethra
    Primary tumor (pT)
    Prostatic urethra
    • pTX: cannot be assessed
    • pT0: no evidence of primary tumor
    • pTa: noninvasive papillary carcinoma
    • pTis: carcinoma in situ of the prostatic urethra, periurethra or ducts
    • pT1: invasion of prostatic urethral subepithelial connective tissue
    • pT2: invasion of prostatic stroma
    • pT3: invasion of periprostatic fat
    • pT4: invasion of adjacent organs (example: bladder wall, rectal wall)

    Penile urethra
    • pTX: cannot be assessed
    • pT0: no evidence of primary tumor
    • pTa: noninvasive papillary carcinoma
    • pTis: carcinoma in situ
    • pT1: invasion of urethral subepithelial connective tissue
    • pT2: invasion of corpus spongiosum
    • pT3: invasion of corpus cavernosum
    • pT4: invasion of adjacent organs (example: bladder wall)
    Regional lymph nodes (pN)
    • pNX: cannot be assessed
    • pN0: no regional lymph node metastasis
    • pN1: metastasis in 1 regional lymph node
    • pN2: metastasis in > 1 regional lymph node

    Note:
    • Regional lymph nodes include superficial / deep inguinal, perivesical, obturator, internal iliac / hypogastric, external iliac, presacral, sacral NOS, pelvic NOS
    Distant metastasis (pM)
    • pM1: distant metastasis
    Prefixes
    • y: preoperative radiotherapy or chemotherapy
    • r: recurrent tumor stage
    AJCC prognostic stage groups
    • Stage 0a:Ta N0 M0
    • Stage 0is:Tis N0 M0
    • Stage I:T1 N0 M0
    • Stage II:T2 N0 M0
    • Stage III:T3 N0 M0
    • T1 - 3 N1 M0
    • Stage IV:T4 N0 - 2 M0 - 1
    • TX - 4 N2 M0 - 1
    • TX - 4 NX - 2 M1

    Registry data collection variables
    • Urothelial carcinoma grade (high / low)
    • Squamous cell / adenocarcinoma grade (1 - 3)
    Histologic grade (G)
    • Urothelial carcinoma
      • LG: low grade
      • HG: high grade
    • Squamous cell / adenocarcinoma
      • GX: cannot be assessed
      • G1: well differentiated
      • G2: moderately differentiated
      • G3: poorly differentiated
    Histopathologic type
    Gross images

    Contributed by Debra L. Zynger, M.D.
    Penile urethral urothelial carcinoma (pT3) Penile urethral urothelial carcinoma (pT3)

    Penile urethral urothelial carcinoma (pT3)

    Microscopic (histologic) images

    Contributed by Debra L. Zynger, M.D.
    Penile urethral urothelial carcinoma (pT3) Penile urethral urothelial carcinoma (pT3)

    Penile urethral urothelial carcinoma (pT3)

    Board review style question #1
    A urothelial carcinoma of the prostatic urethra at the deepest point of invasion involves the bladder wall. Which is the correct pT category?

    1. pT1
    2. pT2
    3. pT3
    4. pT4
    Board review style answer #1

    Staging-renal pelvic carcinoma
    Definition / general
    • All carcinomas of the renal pelvis are covered by this staging system
    • These tumors are not covered: renal cell carcinoma, lymphoma and mesenchymal tumors
    Essential features
    • AJCC 7th edition staging was sunset on December 31, 2017; as of January 1, 2018, use of the 8th edition is mandatory
    ICD coding
    • ICD-10: C65.9 - renal pelvis
    Primary tumor (pT)
    • pTX: cannot be assessed
    • pT0: no evidence of primary tumor
    • pTa: noninvasive papillary carcinoma
    • pTis: carcinoma in situ
    • pT1: invades subepithelial connective tissue
    • pT2: invades muscle
    • pT3: invades peripelvic fat or renal parenchyma
    • pT4: invades adjacent organs or perinephric fat
    Regional lymph nodes (pN)
    • pNX: cannot be assessed
    • pN0: no regional lymph node metastasis
    • pN1: 1 lymph node with tumor deposit ≤ 2 cm
    • pN2: 1 lymph node with tumor deposit > 2 cm or metastases in multiple nodes

    Notes:
    • Regional lymph nodes include hilar, paracaval, aortic and retroperitoneal
    Distant metastasis (pM)
    • pM1: distant metastasis

    Notes:
    • pM is not included in the surgical pathology report unless there is pathologic evidence of distant metastasis (pM1)
    • pMX is no longer utilized
    Prefixes
    • y: preoperative radiotherapy or chemotherapy
    • r: recurrent tumor stage
    AJCC prognostic stage groups
    Stage group 0a:TaN0M0
    Stage group 0is:TisN0M0
    Stage group I:T1N0M0
    Stage group II:T2N0M0
    Stage group III:T3N0M0
    Stage group IV:T4NX - 2M0 - 1
    TX - 4N1 - 2M0 - 1
    TX - 4NX - 2M1
    Registry data collection variables
    • Extranodal extension
    • Size of largest tumor deposit within a lymph node
    • Total number of lymph nodes
    • Presence of carcinoma in situ
    • Presence of noninvasive papillary carcinoma
    • Lymphovascular invasion
    • Urothelial carcinoma grade (high / low)
    • Squamous cell / adenocarcinoma grade (1 - 3)
    • Intratubular renal in situ spread
    Histologic grade (G)
    • Urothelial carcinoma
      • LG: low grade
      • HG: high grade
    • Squamous cell / adenocarcinoma
      • GX: cannot be assessed
      • G1: well differentiated
      • G2: moderately differentiated
      • G3: poorly differentiated
    Histopathologic type
    • Noninvasive urothelial carcinoma
      • Low grade papillary urothelial carcinoma
      • High grade papillary urothelial carcinoma
      • Urothelial carcinoma in situ
    • Invasive urothelial carcinoma
      • Conventional urothelial carcinoma
      • Urothelial carcinoma variants
    • Squamous cell carcinoma
    • Adenocarcinoma
    • Small cell carcinoma
    Gross images

    Contributed by Debra L. Zynger, M.D.
    Renal pelvic and peripelvic fat invasion (pT3)

    Renal pelvic and peripelvic fat invasion (pT3)

    Microscopic (histologic) images

    Contributed by Debra L. Zynger, M.D.
    Renal parenchyma invasion (pT3) Renal parenchyma invasion (pT3)

    Renal parenchyma invasion (pT3)

    Board review style question #1


    A urothelial carcinoma of the renal pelvis at the deepest point of invasion involves the renal parenchyma. Which is the correct pT category?

    1. pT1
    2. pT2
    3. pT3
    4. pT4
    Board review style answer #1
    C. pT3. Renal parenchyma invasion is categorized as pT3.

    Comment here

    Reference: Staging-renal pelvic carcinoma

    Staging-ureter carcinoma
    Definition / general
    • All carcinomas of the ureter are covered by this staging system
    Essential features
    • AJCC 7th edition staging was sunset on December 31, 2017; as of January 1, 2018, use of the 8th edition is mandatory
    ICD coding
    • ICD-10: C66.9 - malignant neoplasm of unspecified ureter
    Primary tumor (pT)
    • pTX: cannot be assessed
    • pT0: no evidence of primary tumor
    • pTa: noninvasive papillary carcinoma
    • pTis: carcinoma in situ
    • pT1: invades lamina propria
    • pT2: invades muscularis propria
    • pT3: invades periureteric fat
    • pT4: invades adjacent organs or perinephric fat
    Regional lymph nodes (pN)
    • pNX: cannot be assessed
    • pN0: no regional lymph node metastasis
    • pN1: 1 lymph node with tumor deposit ≤ 2 cm
    • pN2: 1 lymph node with tumor deposit > 2 cm or metastases in multiple nodes

    Notes:
    • Regional lymph nodes include hilar, periureteral, paracaval, iliac (common, internal / hypogastric, external), pelvic
    Distant metastasis (pM)
    • pM1: distant metastasis
    Prefixes
    • y: preoperative radiotherapy or chemotherapy
    • r: recurrent tumor stage
    AJCC prognostic stage groups
    Stage group 0a:TaN0M0
    Stage group 0is:TisN0M0
    Stage group I:T1N0M0
    Stage group II:T2N0M0
    Stage group III:T3N0M0
    Stage group IV:T4NX - 2M0 - 1
    TX - 4N1 - 2M0 - 1
    TX - 4NX - 2M1
    Registry data collection variables
    • Extranodal extension
    • Size of largest tumor deposit within a lymph node
    • Total number of lymph nodes
    • Presence of carcinoma in situ
    • Presence of noninvasive papillary carcinoma
    • Lymphovascular invasion
    • Urothelial carcinoma grade (high / low)
    • Squamous cell / adenocarcinoma grade (1 - 3)
    • Intratubular renal in situ spread
    Histologic grade (G)
    • Urothelial carcinoma
      • LG: low grade
      • HG: high grade
    • Squamous cell / adenocarcinoma
      • GX: cannot be assessed
      • G1: well differentiated
      • G2: moderately differentiated
      • G3: poorly differentiated
    Histopathologic type
    • Noninvasive urothelial carcinoma
      • Low grade papillary urothelial carcinoma
      • High grade papillary urothelial carcinoma
      • Urothelial carcinoma in situ
    • Invasive urothelial carcinoma
      • Conventional urothelial carcinoma
      • Urothelial carcinoma variants
    • Squamous cell carcinoma
    • Adenocarcinoma
    • Small cell carcinoma
    Board review style question #1
    A urothelial carcinoma of the ureter at the deepest point of invasion involves the muscularis propria. Which is the correct pT category?

    1. pT1
    2. pT2
    3. pT3
    4. pT4
    Board review style answer #1

    Treatment effect
    Definition / general
    • Histologic changes associated with chemotherapy (systemic or topical), radiation therapy or surgery (J Clin Pathol 2002;55:641)
    Clinical features
    • Cyclophosphamide causes hemorrhagic cystitis, and is associated with high grade bladder carcinoma and sarcoma
    Case reports
    Gross description
    • Chemotherapy may destroy tips of papillae in papillary tumors
    Microscopic (histologic) description
    • General characteristics include pseudoinvasive urothelial nests wrapping around vessels associated with fibrin deposition; also hemorrhage, fibrin thrombi, fibrosis, acute and chronic inflammation; usually edema and vascular congestion; occasionally ulceration; no mitotic figures (Am J Surg Pathol 2004;28:909)
    • Radiation therapy: causes endothelial swelling and necrosis, mural thickening and hyalinization with late luminal narrowing; also pseudoinfiltrative epithelial cords and nests extending into lamina propria and wrapping around dilated blood vessels containing fibrin; radiation fibroblasts with cytoplasmic or nuclear vacuoles and prominent nucleoli, stromal edema, extravasated red blood cells, destruction of bladder tumor papillae (Hum Pathol 2000;31:678)
    • Surgery: associated with granulomatous reaction, postoperative spindle cell nodules, trapping of epithelial cells by inflammatory reaction resembling invasive disease, regenerative atypia resembling carcinoma in situ, reactive bone / osteoid
    • Systemic chemotherapy: nuclear atypia, hemorrhagic cystitis, polyoma virus related changes
    • Topical (intravesicular) Mitomycin C / ThioTEPA: may cause exfoliation of normal and abnormal urothelial cells, degeneration, multinucleation and bizarre reactive nuclear changes
    • Topical bCG (immunotherapy): causes focal epithelial denudation with granulomatous inflammation of lamina propria
    Microscopic (histologic) images

    Images hosted on other servers:

    Various images

    Postradiation changes mimic dysplasia

    Differential diagnosis

    Urachal adenocarcinoma
    Definition / general
    • Adenocarcinoma arising from the epithelium lined tubular structure known as urachus, an embryological derivative of the urogenital sinus and allantois that connects the bladder to the umbilicus
    • Diagnosis of urachal adenocarcinoma is usually established after excluding other mimics
    Essential features
    • Criteria for urachal origin of adenocarcinoma
      • Centered in the anterior wall or dome of bladder
      • Predominant invasion of muscularis or deeper tissues with sharp demarcation between tumor and surface bladder urothelium
      • Surface urothelium is free of carcinoma in situ or glandular metaplasia or cystitis glandularis
      • Presence of urachal remnants is helpful but not always identifiable
      • No primary adenocarcinoma elsewhere confirmed clinically
    Terminology
    • Some authors prefer terminology of median umbilical ligament for completely regressed involuted urachus (fibrous cord-like structure) and reserve urachus for the patent urachus (tube-like structure lined by urothelium)
    ICD coding
    • ICD-10: C67.7 - malignant neoplasm of urachus
    Epidemiology
    Sites
    • Usually dome of bladder, occasionally anterior wall of bladder (intramural mass)
    Pathophysiology
    • The urachus lies in the space of Retzius, a space anterior to the peritoneum between the layers of umbilicovesical fascia (see Urachus)
    • Involution usually happens before birth (by the fourth month of fetal life) and the urachus becomes a fibrous cord known as the median umbilical ligament
    • In adults, it is usually 5 cm long and 6 mm broad
    • It can be divided into supravesical, intramural and intramucosal segments
    • Persistent remnant is a relatively normal condition found in 25 - 35% of bladders at the time of autopsy (J Urol 1982;127:40)
    • Persistent remnants of the allantois develop into cysts or epithelial neoplasms; not all neoplasms are glandular (see Urachus) (Urology 1985;26:218)
    • Among urachal epithelial neoplasms, 80% are adenocarcinomas with enteric features and are mucinous (J Urol 1986;135:1240, Cancer 1991;67:2165)
    Etiology
    • Embryologic derivative undergoing neoplastic transformation (J Urol 1984;131:1)
    • Risk factors include bladder exstrophy and urachal anomalies
    Diagrams / tables

    Images hosted on other servers:
    Urachus

    Fetal tube connecting bladder with umbilicus

    Clinical features
    Diagnosis
    • Urachal adenocarcinoma is a clinicopathologic diagnosis
    • Clinical confirmation of location of mass and presence of urachal remnant
    • Radiologic and colonoscopic exclusion of extension (colorectal, prostatic and gynecologic) or metastasis (hematogenous) from another organ (breast, pancreas, gastric and pulmonary)
    • Reference: Urol Oncol 2016;34:388
    Radiology description
    • Cystoscopy: usually single, solid irregularity / nodularity in the dome / anterior wall
    • Ultrasound findings
      • Uneven internal echogenicity with involvement of anterior bladder wall / dome
      • Solid / cystic mass, with or without calcification
    • Computed tomography (CT) findings
      • Solid / cystic mass and punctate calcification
      • Thickened bladder dome and anterior wall and urachal area
      • For staging and assessment of muscle invasion
    • Magnetic resonance imaging (MRI): for staging
    • References: J Clin Pathol 2006;59:1091, Indian J Med Res 2013;137:398
    Radiology images

    Images hosted on other servers:
    Low attenuating cystic mass

    Low attenuating cystic mass

    Mass in the anterior abdominal wall

    Mass in the anterior abdominal wall

    Prognostic factors
    • Since the entire length of the median umbilical ligament may harbor urachal remnants and carcinoma may present in perivesical tissue, staging is different than bladder carcinoma
    • Staging of urachal carcinoma has reduction in substages (applied to invasion of the bladder wall) and management differs according to the pathologic stage at the time of surgery (Hum Pathol 1996;27:240)
    • First staging proposed by Sheldon (J Urol 1984;131:1, Urol Oncol 2016;34:388)
      • I: confined to urachal mucosa (in situ)
      • II: urachus invasion confined to urachus itself
      • III: local extension
        • Bladder (IIIA)
        • Abdominal wall (IIIB)
        • Peritoneum (IIIC)
        • Other viscera (IIID)
      • IV
        • Metastasis to regional lymph nodes (IVA)
        • Distant sites (IVB)
    • Local recurrence and metastatic disease are common (J Urol 2007;178:74, Cancer 2006;107:712)
    • 5 year cancer specific survival rate is ~50% (median ~45 months, significantly longer than bladder urothelial carcinoma) (Cancer 2006;107:712, Hum Pathol 2015;46:1808, J Urol 2006;175:2042, Dis Markers 2018;2018:7308168)
    • In Sheldon classification, > 80% patients presented in stage III (bladder invasion) with 5 year survival rate from 25 - 61% (J Urol 1984;131:1, Urol Oncol 2016;34:388)
    • Mayo and Ontario staging systems simplify the Sheldon staging using larger dataset (2006) (Cancer 2006;107:712, J Urol 2006;175:2042)
    • In the Mayo system
      • Bladder mucosa and muscularis propria invasion are stage II instead of III
      • Regional lymph node involvement is stage III instead of IVA
      • Distant metastasis is stage IV instead of IVB
    • In 2023, a new TNM staging system was proposed based on the 626 cases included in Surveillance, Epidemiology, and End Results (SEER) database as follows (Cancer Med 2023;12:2752)
      • Stage I
        • pT0, Ta, Tis (confined to urachal mucosa)
        • N0, M0
      • Stage II
        • pT2, pT3a (invasion of bladder muscularis propria or microscopic invasion of perivesical tissue)
        • N0, M0
      • Stage III
        • pT3b, 4a (macroscopic invasion of bladder perivesical tissue or invasion of uterus, vagina, prostate)
        • N0, M0
      • Stage IV
        • pT4b (invasion of pelvic wall, abdominal wall or peritoneum)
        • Any nodal involvement or distant metastasis regardless of pT category
    • Staging systems have prognostic utility but lack validation because of the overall smaller sample size due to disease rarity (Cancer Med 2023;12:2752, Hum Pathol 2015;46:1808)
    Case reports
    Treatment
    • Umbilectomy with partial cystectomy, possibly laparoscopic (Clinics (Sao Paulo) 2008;63:731)
    • Since the entire length of median umbilical ligament may harbor urachal remnants, which may develop carcinoma synchronously or metachronously, the surgery of choice includes en bloc resection of the entire length of the ligament with umbilectomy with negative margins
    • Adjuvant treatment given depending on the stage includes chemoradiation
    Clinical images

    Images hosted on other servers:
    Missing Image

    Intraoperative image of cystic lesion at dome of bladder

    Missing Image

    Intraoperative image of urachal cyst

    Missing Image

    Intraoperative image of multilobulated cyst

    Gross description
    Gross images

    Images hosted on other servers:
    Missing Image

    Excised specimen with umbilicus and neoplasm

    Microscopic (histologic) description
    • Arises from intramural portion of urachus, grows into bladder wall, lacks mucosal involvement, is sharply demarcated from normal epithelium and lacks intestinal metaplasia
    • Most tumors are well differentiated adenocarcinomas with histologic variations of mucinous / colloid (57%), enteric (15%), signet ring (6%), mixed morphology (8%) or nonspecific / NOS (14%) (Dis Markers 2018;2018:7308168)
    • Mucinous neoplasms are predominant
    • Mucinous cystic tumors form a spectrum from cystadenoma to mucinous cystic tumors of low malignant potential to microinvasive and frankly invasive cystadenocarcinomas similar to other organs (i.e., appendix, ovary) (Am J Surg Pathol 2014;38:1033)
    • Residual benign urachal remnants can be intermingled with carcinoma and harbor preinvasive lesions
    • Enteric subtype is morphologically identical to colorectal primary carcinomas
    • Signet ring subtype is characterized by infiltrative stromal growth of signet ring cells
    • < 5% of cases present with other malignant epithelial neoplastic morphology such as usual urothelial, squamous cell carcinoma and neuroendocrine (Adv Anat Pathol 2016;23:71)
    Microscopic (histologic) images

    Contributed by Maria Tretiakova, M.D., Ph.D.
    Mucinous / colloid subtype of urachal carcinoma

    Mucinous / colloid subtype

    Enteric subtype of urachal carcinoma

    Enteric subtype

    Signet ring subtype of urachal carcinoma

    Signet ring subtype

    Mixed subtype of urachal carcinoma

    Mixed subtype

    Urachal carcinoma, NOS

    Urachal carcinoma, NOS


    Mucinous adenocarcinoma, bladder dome

    Mucinous adenocarcinoma, bladder dome

    CDX2 in bladder dome adenocarcinoma

    CDX2 in bladder dome adenocarcinoma

    CK20 in bladder dome adenocarcinoma

    CK20 in bladder dome adenocarcinoma

    Beta catenin expression

    Beta catenin expression

    High molecular weight cytokeratin expression

    High molecular weight cytokeratin expression

    Virtual slides

    Images hosted on other servers:
    Mucinous cystadenocarcinoma of urachus

    Mucinous
    cystadenocarcinoma
    of urachus

    Adenocarcinoma of bladder dome

    Adenocarcinoma of bladder dome

    Mucinous subtype of urachal adenocarcinoma

    Mucinous subtype of urachal adenocarcinoma

    Cytology description
    • Cytomorphology differs according to type of adenocarcinoma (e.g., mucinous versus signet ring)
    • Cluster of cells with eccentric irregular nuclei; prominent nucleoli and vacuolated cytoplasm
    • Background may show mucin (Cancer 1998;84:335, J Clin Diagn Res 2016;10:ED10)
    Cytology images

    Images hosted on other servers:
    FNAC smear of urachal adenocarcinoma

    FNAC smear of urachal adenocarcinoma

    Positive stains
    Negative stains
    Electron microscopy description
    • Ultrastructurally, by transmission and scanning electron microscopy, the tissue resembles gut mucosa (J Urol 1986;135:1240)
    Molecular / cytogenetics description
    • Bladder and urachal adenocarcinoma genomic profiles resemble colorectal adenocarcinoma with frequent TP53, KRAS, BRAF, NRAS and PIK3CA alterations (JCO Precis Oncol 2022;6:e2100392, Oncotarget 2016;7:39293)
    • Although mutation profiles of bladder and urachal adenocarcinoma overlap, their differences may aid in separating these 2 entities (Int J Surg Pathol 2020;28:51)
    • Mutations of GNAS, SMAD4 and BRAF are found in urachal but not bladder adenocarcinomas
    • The most common recurrent somatic mutations of urachal carcinomas involve 6 genes: APC, COL5A1, KIF26B, LRP1B, SMAD4 and TP53 (J Med Genet 2017;54:572)
    • MicroRNA expression profiling does not show distinct pattern of the main morphologic subtypes (Hum Pathol 2013;44:1605)
    • Extent of chromosomal amplification is highly associated with the patient's cancer stage; 35% of focal DNA amplifications were in fibroblast growth factor receptor family genes (J Med Genet 2017;54:572)
    Videos

    Mini tutorial of urachal tumors

    Urachal adenocarcinoma diagnosis and treatment

    Sample pathology report
    • Dome of bladder, biopsy:
      • Adenocarcinoma, NOS (see comment)
      • Comment: Given the presence of surface ulceration, an in situ component cannot be evaluated. A possibility of adenocarcinoma of urachal origin is favored at this site although adenocarcinoma of bladder origin is also a consideration. Negative beta catenin nuclear positivity and clinical information confirms that there is no metastasis or direct extension from another organ.
    Differential diagnosis
    Additional references
    Board review style question #1

    The photomicrograph depicts urachal adenocarcinoma. Which of the following is the most important criterion for the diagnosis of urachal adenocarcinoma?

    1. Beta catenin nuclear positivity on immunohistochemistry
    2. Intraluminal mass in posterior wall
    3. Intramural mass in dome or anterior wall
    4. Presence of carcinoma in situ in mucosal lining
    Board review style answer #1
    C. Intramural mass in dome or anterior wall. Considering the embryologic aspect, the most important criterion for urachal origin is the presence of an intramural mass in the dome or anterior bladder wall. Answers B and D are incorrect because lack of intraluminal mass and absence of carcinoma in situ favor urachal origin over primary bladder origin. Answer A is incorrect because nuclear beta catenin is often positive in colonic adenocarcinoma, which may extend or metastasize in rare cases and excludes urachal origin.

    Comment Here

    Reference: Urachal adenocarcinoma
    Board review style question #2
    What is the importance of diagnosing primary urachal origin in a malignant glandular neoplasm of the bladder and perivesical area?

    1. Familial predisposition
    2. It arises from embryologic remnant
    3. Rarity of disease representing scientific curiosity
    4. Staging and management are different than bladder primary
    Board review style answer #2
    D. Staging and management are different than that of primary adenocarcinoma of the bladder. The urachus is a fibrous cord extending from the bladder wall to the umbilicus; along the free surface of the bladder urachus, carcinoma is frequently amenable to partial cystectomy if it presents as a solitary mass. The entire tract of median umbilical ligament must be excised. Staging of urachal carcinoma differs from those that arise from the bladder surface. Most urachal carcinomas (80%) present at advanced stages but are still associated with a better survival rate than bladder urothelial carcinomas (Hum Pathol 2015;46:1808). Answer A is incorrect because no familial predisposition has been described.

    Comment Here

    Reference: Urachal adenocarcinoma

    Urachus
    Definition / general
    • The urachus, originated from remnants of allantois, is a fibrous cord connecting the umbilicus to the anterosuperior aspect of the bladder dome; usually obliterates at birth and becomes the median umbilical ligament (Kumar: Robbins & Cotran Pathologic Basis of Disease, 10th Edition, 2020)
    • Urachal pathology results from failure of involution of the embryonic structures, resulting in the formation of a urachal cyst, umbilical urachal sinus, vesicourachal diverticulum or patent urachus (completely patent, tubular remnant communicating the developing bladder and the umbilical cord) (StatPearls: Patent Urachus [Accessed 29 December 2022])
    Essential features
    Terminology
    • Patent urachus: completely patent tubular lesion communicating the bladder and the umbilical cord
    • Umbilical urachal sinus: tubulocystic lesion with an opening in the umbilicus while the other end is closed (BMJ Case Rep 2016;2016:bcr2016215374)
    • Vesicourachal diverticulum: tubulocystic lesion with an opening in the bladder while the other end is closed
    • Urachal cyst: cystic structure with both the umbilical and bladder ends closed (BMJ Case Rep 2016;2016:bcr2016215374)
    • Urachal remnants within bladder wall: tubulocystic lesion located in the bladder wall
    ICD coding
    • ICD-10: Q64.4 - malformation of urachus
      • Urachus, congenital
      • Patent urachus
      • Patent (congenital) urachus
    • ICD-11: LB03.0 - allantoic duct remnants or cysts
    Epidemiology
    Sites
    Pathophysiology
    Etiology
    Diagrams / tables

    Images hosted on other servers:
    Classification of urachal anomalies

    Classification of urachal anomalies

    Urachal cysts and fistulas

    Urachal cysts and fistulas

    Clinical features
    Diagnosis
    • Prenatal:
      • Ultrasonography (US): increased thickness of the umbilicus observed as an extra-abdominal cystic mass (Medicina (Kaunas) 2022;58:1621)
      • Early prenatal detection for appropriate counseling for postpartum corrective surgery
    • Postnatal:
    • Ultrasound: diagnostic for 82% of cysts, 100% of sinuses, 100% of patent urachus
    • Voiding cystourethrogram: diagnostic for 100% of patent urachus
    • CT scan: diagnostic for 71% of cysts (J Pediatr Urol 2007;3:500)
    Laboratory
    • Check creatinine level if urine in the umbilical drainage is suspected (StatPearls: Patent Urachus [Accessed 10 January 2023])
    • Microorganisms cultured from the umbilical drainage often include Staphylococcus aureus, Escherichia coli, Enterococcus, Citrobacter and rarely, Proteus species
    Radiology description
    Radiology images

    Contributed by Bohdan Zoshchuk, M.D.
    Patent urachus, US Patent urachus, US

    Patent urachus, ultrasound



    Images hosted on other servers:
    Hypoechoic mass

    Hypoechoic mass

    Patent urachus, fistulography

    Patent urachus, fistulography

    Patent urachus, CT scan

    Patent urachus, CT scan

    Cyst protruding through umbilicus

    Cyst protruding through umbilicus

    Prognostic factors
    Case reports
    Treatment
    Clinical images

    Images hosted on other servers:
    Persistent urachus

    Persistent urachus

    Before and after surgery

    Before and after surgery

    Urachal cysts - sinus tract between bladder dome and umbilicus

    Urachal cysts - sinus tract between bladder dome and umbilicus

    Infected urachal cyst

    Infected urachal cyst

    Urachal cyst attached to Meckel diverticulum

    Urachal cyst attached to Meckel diverticulum

    Gross description
    Gross images

    Images hosted on other servers:
    Patent urachus, autopsy specimen

    Patent urachus, autopsy specimen

    Benign urachal lesion

    Benign urachal lesion

    Infected urachal cyst and fibrous tract

    Infected urachal cyst and fibrous tract

    Urachal cyst containing stones

    Urachal cyst containing stones

    Urachal sigmoid fistula

    Urachal sigmoid fistula

    Inflamed urachal cyst

    Inflamed urachal cyst

    Microscopic (histologic) description
    • Patent urachus, umbilical urachal sinus, vesicourachal diverticulum
      • Completely or incompletely opened structures lined by benign urothelium with or without squamous metaplasia
      • Glandular metaplasia may present
    • Urachal cyst
      • Cystic remnants lined by benign cuboidal cells, flattened epithelial cells or urothelial cells
      • Intestinal (columnar cells with or without goblet cells) metaplasia may present
    • Urachal remnants in bladder wall (Cheng: Urologic Surgical Pathology, 4th Edition, 2019)
      • Embryonic remnants in the bladder wall that are lined by benign urothelial cells
      • Squamous or glandular (columnar cells with or without goblet cells) metaplasia may present
    Microscopic (histologic) images

    Contributed by Bohdan Zoshchuk, M.D. and Manasa Morisetti, M.D.
    Patent urachus Patent urachus Patent urachus

    Patent urachus

    Urachal remnant

    Urachal remnant in bladder wall


    Urachal remnant Urachal remnant

    Urachal remnant in bladder wall

    Urachal sinus Urachal sinus

    Umbilical urachal sinus

    Molecular / cytogenetics description
    Sample pathology report
    • Urinary bladder wall lesion, excision:
      • Tubulocystic lesion lined by urothelial epithelium consistent with urachal remnants (see comment)
      • Comment: Sections of the bladder wall lesion show a tubular structure lined by urothelium and attached to the muscularis propria. These findings are consistent with urachal remnants.
    Differential diagnosis
    Board review style question #1
    During bladder catheterization in a newborn boy, the tip of the catheter came out through his enlarged umbilical stump. What is the diagnosis?

    1. Patent omphalomesenteric duct
    2. Patent urachus
    3. Umbilical urachal sinus
    4. Urachal cyst
    5. Vesicourachal diverticulum
    Board review style answer #1
    B. Patent urachus. Patent urachus creates a tubular connection between the umbilicus and the anterosuperior wall of the bladder. Answer A is incorrect because patent omphalomesenteric duct has connection between umbilicus and small intestine. Answers C, D and E are incorrect because umbilical urachal sinus, urachal cyst and vesicourachal diverticulum cannot be probed through.

    Comment Here

    Reference: Urachus and patent urachus
    Board review style question #2

    A 50 year old man complained of abdominal pain for several weeks. A noncontrast CT scan of the abdomen was performed. There was a cystic lesion in the anterosuperior bladder wall. No connection between the bladder and the umbilicus was noted. Surgical excision followed by pathological examination was performed. The photomicrograph is shown above. What is the diagnosis?

    1. Mesonephric remnant
    2. Omphalomesenteric duct remnant
    3. Urachal remnant in bladder wall
    4. Vitelline duct remnant
    Board review style answer #2
    C. Urachal remnant in bladder wall. Urachal remnants within the bladder wall are usually lined by urothelium but can show metaplastic changes, such as squamous metaplasia seen here. Mesonephric remnants are composed of small tubules lined by low columnar to cuboidal epithelial cells. The cells are immunoreactive to CK903, CD10 and vimentin. Omphalomesenteric duct remnant is the same as vitelline duct remnant, which is lined by gastrointestinal epithelium.

    Comment Here

    Reference: Urachus and patent urachus

    Urachus
    Definition / general
    • The urachus, originated from remnants of allantois, is a fibrous cord connecting the umbilicus to the anterosuperior aspect of the bladder dome; usually obliterates at birth and becomes the median umbilical ligament (Kumar: Robbins & Cotran Pathologic Basis of Disease, 10th Edition, 2020)
    • Urachal pathology results from failure of involution of the embryonic structures, resulting in the formation of a urachal cyst, umbilical urachal sinus, vesicourachal diverticulum or patent urachus (completely patent, tubular remnant communicating the developing bladder and the umbilical cord) (StatPearls: Patent Urachus [Accessed 29 December 2022])
    Essential features
    Terminology
    • Patent urachus: completely patent tubular lesion communicating the bladder and the umbilical cord
    • Umbilical urachal sinus: tubulocystic lesion with an opening in the umbilicus while the other end is closed (BMJ Case Rep 2016;2016:bcr2016215374)
    • Vesicourachal diverticulum: tubulocystic lesion with an opening in the bladder while the other end is closed
    • Urachal cyst: cystic structure with both the umbilical and bladder ends closed (BMJ Case Rep 2016;2016:bcr2016215374)
    • Urachal remnants within bladder wall: tubulocystic lesion located in the bladder wall
    ICD coding
    • ICD-10: Q64.4 - malformation of urachus
      • Urachus, congenital
      • Patent urachus
      • Patent (congenital) urachus
    • ICD-11: LB03.0 - allantoic duct remnants or cysts
    Epidemiology
    Sites
    Pathophysiology
    Etiology
    Diagrams / tables

    Images hosted on other servers:
    Classification of urachal anomalies

    Classification of urachal anomalies

    Urachal cysts and fistulas

    Urachal cysts and fistulas

    Clinical features
    Diagnosis
    • Prenatal:
      • Ultrasonography (US): increased thickness of the umbilicus observed as an extra-abdominal cystic mass (Medicina (Kaunas) 2022;58:1621)
      • Early prenatal detection for appropriate counseling for postpartum corrective surgery
    • Postnatal:
    • Ultrasound: diagnostic for 82% of cysts, 100% of sinuses, 100% of patent urachus
    • Voiding cystourethrogram: diagnostic for 100% of patent urachus
    • CT scan: diagnostic for 71% of cysts (J Pediatr Urol 2007;3:500)
    Laboratory
    • Check creatinine level if urine in the umbilical drainage is suspected (StatPearls: Patent Urachus [Accessed 10 January 2023])
    • Microorganisms cultured from the umbilical drainage often include Staphylococcus aureus, Escherichia coli, Enterococcus, Citrobacter and rarely, Proteus species
    Radiology description
    Radiology images

    Contributed by Bohdan Zoshchuk, M.D.
    Patent urachus, US Patent urachus, US

    Patent urachus, ultrasound



    Images hosted on other servers:
    Hypoechoic mass

    Hypoechoic mass

    Patent urachus, fistulography

    Patent urachus, fistulography

    Patent urachus, CT scan

    Patent urachus, CT scan

    Cyst protruding through umbilicus

    Cyst protruding through umbilicus

    Prognostic factors
    Case reports
    Treatment
    Clinical images

    Images hosted on other servers:
    Persistent urachus

    Persistent urachus

    Before and after surgery

    Before and after surgery

    Urachal cysts - sinus tract between bladder dome and umbilicus

    Urachal cysts - sinus tract between bladder dome and umbilicus

    Infected urachal cyst

    Infected urachal cyst

    Urachal cyst attached to Meckel diverticulum

    Urachal cyst attached to Meckel diverticulum

    Gross description
    Gross images

    Images hosted on other servers:
    Patent urachus, autopsy specimen

    Patent urachus, autopsy specimen

    Benign urachal lesion

    Benign urachal lesion

    Infected urachal cyst and fibrous tract

    Infected urachal cyst and fibrous tract

    Urachal cyst containing stones

    Urachal cyst containing stones

    Urachal sigmoid fistula

    Urachal sigmoid fistula

    Inflamed urachal cyst

    Inflamed urachal cyst

    Microscopic (histologic) description
    • Patent urachus, umbilical urachal sinus, vesicourachal diverticulum
      • Completely or incompletely opened structures lined by benign urothelium with or without squamous metaplasia
      • Glandular metaplasia may present
    • Urachal cyst
      • Cystic remnants lined by benign cuboidal cells, flattened epithelial cells or urothelial cells
      • Intestinal (columnar cells with or without goblet cells) metaplasia may present
    • Urachal remnants in bladder wall (Cheng: Urologic Surgical Pathology, 4th Edition, 2019)
      • Embryonic remnants in the bladder wall that are lined by benign urothelial cells
      • Squamous or glandular (columnar cells with or without goblet cells) metaplasia may present
    Microscopic (histologic) images

    Contributed by Bohdan Zoshchuk, M.D. and Manasa Morisetti, M.D.
    Patent urachus Patent urachus Patent urachus

    Patent urachus

    Urachal remnant

    Urachal remnant in bladder wall


    Urachal remnant Urachal remnant

    Urachal remnant in bladder wall

    Urachal sinus Urachal sinus

    Umbilical urachal sinus

    Molecular / cytogenetics description
    Sample pathology report
    • Urinary bladder wall lesion, excision:
      • Tubulocystic lesion lined by urothelial epithelium consistent with urachal remnants (see comment)
      • Comment: Sections of the bladder wall lesion show a tubular structure lined by urothelium and attached to the muscularis propria. These findings are consistent with urachal remnants.
    Differential diagnosis
    Board review style question #1
    During bladder catheterization in a newborn boy, the tip of the catheter came out through his enlarged umbilical stump. What is the diagnosis?

    1. Patent omphalomesenteric duct
    2. Patent urachus
    3. Umbilical urachal sinus
    4. Urachal cyst
    5. Vesicourachal diverticulum
    Board review style answer #1
    B. Patent urachus. Patent urachus creates a tubular connection between the umbilicus and the anterosuperior wall of the bladder. Answer A is incorrect because patent omphalomesenteric duct has connection between umbilicus and small intestine. Answers C, D and E are incorrect because umbilical urachal sinus, urachal cyst and vesicourachal diverticulum cannot be probed through.

    Comment Here

    Reference: Urachus and patent urachus
    Board review style question #2

    A 50 year old man complained of abdominal pain for several weeks. A noncontrast CT scan of the abdomen was performed. There was a cystic lesion in the anterosuperior bladder wall. No connection between the bladder and the umbilicus was noted. Surgical excision followed by pathological examination was performed. The photomicrograph is shown above. What is the diagnosis?

    1. Mesonephric remnant
    2. Omphalomesenteric duct remnant
    3. Urachal remnant in bladder wall
    4. Vitelline duct remnant
    Board review style answer #2
    C. Urachal remnant in bladder wall. Urachal remnants within the bladder wall are usually lined by urothelium but can show metaplastic changes, such as squamous metaplasia seen here. Mesonephric remnants are composed of small tubules lined by low columnar to cuboidal epithelial cells. The cells are immunoreactive to CK903, CD10 and vimentin. Omphalomesenteric duct remnant is the same as vitelline duct remnant, which is lined by gastrointestinal epithelium.

    Comment Here

    Reference: Urachus and patent urachus

    Ureters-congenital anomalies
    Table of Contents
    Definition / general
    Definition / general
    • Diverticula:
      • Congenital diverticulum results from aberrant development of ureteric bud (true diverticulum including all layers of normal ureter wall)
      • Pseudodiverticulum results from mucosal protrusion through a defect in the ureter wall (due to instrumentation, surgery, stone, obstruction, et cetera)
      • Can be asymptomatic or cause recurrent urinary tract infections
      • Lined by urothelium, sometimes squamous metaplasia or ureteritis glandularis
    • Double / bifid ureters:
      • Relatively common incidental finding, associated with duplication of renal pelves
    • Ectopic ureter:
      • More common in women
      • Ureter may terminate into structures such as proximal urethra, seminal vesicle/ejaculatory duct, vas deferens, fallopian tube, uterus or vagina
    • Hydroureter:
      • May have neurogenic cause
    • Megaloureter (megaureter):
      • Often divided into primary obstructed and primary refluxing megaureter
      • May have smooth muscle defect (such as apoptosis of smooth muscle in primary obstructive cases)
      • Defective interstitial cell of Cajal-like cells have been hypothesized to be involved in refluxing cases (Urology 2009;74:318)
    • Ureteral agenesis:
      • Mesonephric diverticulum does not develop
      • Associated with failure of ipsilateral kidney to develop (usually unilateral, bilateral Potter’s syndrome is fatal)
    • Ureteropelvic junction obstruction:
      • One of the most common causes of hydronephrosis in children
      • More common in males and on left side, 20% bilateral
      • May be associated with agenesis of contralateral kidney
      • May be due to abnormal organization of smooth muscle bundles (intrinsic obstruction thought to be associated with less dense muscle and increased collagen, compared to obstruction due to crossing of renal vessel, Urology 2010;76:181)

    Urinary diversion / neobladder
    Definition / general
    • Portions of ileum or colon used in adults and children to treat congenital anomalies, dysfunctional bladder or post-cystectomy for malignancy
    • Options are to enlarge capacity of bladder (augmentation), channel urine into temporary artificial reservoir while a new bladder is being created or create a neobladder (new bladder after cystectomy)
    Terminology
      Patients who must have their bladder removed usually have three options for urine elimination:
      1. Ileal Conduit (Urostomy) – Conduit of small intestine or colon carries the urine to an opening on the abdomen
      2. Orthotopic neobladder – Neobladder made from loops of intestine to store the urine and individual can void through normal channels
      3. Continent urinary diversion – Creation of an internal pouch from loops of intestine which is connected to an opening on the abdomen through a “one way” passage
    Clinical features


    Complications:
    • Intestinal adenocarcinoma in colonic conduits, reflux but only rare renal failure in ileal conduits, highest risk of adenocarcinoma is in augmentation cystoplasty (J Urol 1997;157:482)
    • Frequent complications but low reoperation rate in conduit urinary diversion (J Urol 2011;185:562)
    • Monitor for carcinoma with cytology (direct smears after centrifugation)
    • Note: must also monitor nonfunctionalized bladder, if present (J Urol 2006;176:620)
    Case reports
    Clinical images

    Images hosted on other servers:

    Continent urinary diversion using ileum

    Microscopic (histologic) description
    • Inflamed, atrophic and partially denuded epithelium
    • Candida in ileal conduits
    Microscopic (histologic) images

    Images hosted on other servers:

    Tubular adenoma with
    high grade dysplasia
    after augmentation
    ileocystoplasty

    Differential diagnosis
    • Normal intestinal cells: aggregates are normally present in urinary diversion specimens, may resemble malignancy

    Urothelial papilloma
    Definition / general
    • Papillary urothelial neoplasm characterized by delicate fibrovascular cores lined by urothelium of normal thickness and cytology
    Essential features
    • Rare papillary urothelial neoplasm with a benign clinical course: rare recurrence if completely excised
    • Exophytic tumor composed of thin papillary fronds lined by prominent umbrella cells
    • Should be distinguished from its morphologic mimics, including papillary urothelial neoplasm of low malignant potential (PUNLMP), urothelial carcinoma, polypoid cystitis and fibroepithelial polyp
    Terminology
    • Transitional cell papilloma
    ICD coding
    • ICD-O: 8120/0 - transitional cell papilloma, benign
    • ICD-10: D30.3 - benign neoplasm of bladder
    Epidemiology
    Sites
    Etiology
    • Unknown at this time but often associated with KRAS / HRAS mutations
    Clinical features
    • Painless hematuria, gross or microscopic
    Diagnosis
    • Imaging: ultrasound (USG)
    • Cystoscopy: papillary / elevated lesions, variable size, often unifocal (Mod Pathol 2003;16:623)
    • Biopsy / transurethral resection of bladder tumor needed for diagnosis
    Laboratory
    • Urine cytology is negative for malignant cells
    Radiology description
    • Ultrasound: endovesical papillary lesion
    Radiology images

    Images hosted on other servers:
    Missing Image

    USG finding - solitary polypoid lesion

    Lobulated soft-tissue mass

    Prognostic factors
    Case reports
    Treatment
    • Resection: transurethral resection of bladder tumor (TURBT)
    Gross description
    • Soft, pink, small isolated growth with delicate papillary structures, usually pedunculated
    Microscopic (histologic) description
    • Predominantly exophytic tumor
    • Discrete papillary structures with central fibrovascular cores with hierarchical branching pattern but without fusion
    • Papillae appear to float above urothelial surface due to transverse sectioning of branching papillae
    • Papillae usually slender with scant stroma around fibrovascular cores
    • May have large papillae with marked stromal edema or cystitis cystica-like invaginations
    • Papillary structures are lined by urothelium of normal thickness and cytology; often with prominent umbrella cells layer
    • Umbrella cells may show cytoplasmic vacuolization and degenerative type atypia
    • There should be no marked cytologic atypia, increased thickness of the urothelium or increased mitotic / apoptotic figures
    • Diffuse papillomatosis: rarely bladder mucosa covered / replaced by multiple papillomas (Virchows Arch 2002;441:109)
    Microscopic (histologic) images

    Contributed by Judy Sarungbam, M.D.
    Missing Image

    Discrete papillary structures

    Missing Image

    Slender fibrovascular core

    Missing Image

    Prominent umbrella cells

    Missing Image

    Edematous fibrovascular core

    Positive stains
    • CK20 is positive in the umbrella cell layer
    Negative stains
    • No p53 abnormality
    Molecular / cytogenetics description
    • Targeted next generation sequencing (NGS) and whole exome sequencing (WES) showed (J Pathol 2019;248:260)
      • Activating mutations involving the MAPK / ERK pathway, in particular KRAS and HRAS genes, are common alterations
      • KRAS mutations more frequent than HRAS
      • Mutations in chromatin modifiers (often associated with urothelial carcinoma) are rare
      • Low tumor mutational burden
      • No APOBEC (apolipoprotein B mRNA editing catalytic polypeptide-like) mutational signature (identified in about 70% of muscle invasive urothelial carcinomas)
    • Other earlier studies have shown
    Sample pathology report
    • Urinary bladder, right lateral trigone, transurethral resection of bladder tumor:
      • Urothelial papilloma
    Differential diagnosis
    • Papillary urothelial neoplasm of low malignant potential (PUNLMP):
      • Complex papillary structures with branching and fusion
      • Increased cellular proliferation, resulting in increased thickness of the urothelium
      • No architectural abnormality (maintained polarity)
      • No nuclear atypia
      • Lacks prominent umbrella cell layer
    • Noninvasive papillary urothelial carcinoma low grade:
      • Complex papillary structures with branching and fusion
      • Usually increased thickness of urothelium with architectural and cytologic atypia
      • Papillary carcinoma without increased thickness shows marked cytologic and architectural abnormality
      • Lacks prominent umbrella cell layer
    • Polypoid / papillary cystitis:
      • Usually has history of urothelial irritation, e.g. stones, radiation, stents etc. (Am J Surg Pathol 2008;32:758, J Urol 2013;189:1091)
      • Best evaluated at low power microscopy
      • Predominantly nonbranching broad base fronds with edematous / fibrotic stroma associated with inflammation
      • Lining urothelium shows reactive atypia
      • Lacks prominent umbrella cell layer
    • Fibroepithelial polyp:
      • More often seen in children, though they can be seen in adults
      • Can be a polypoid mass or a papillary lesion
      • Usually has broader stalks with dense fibrous tissue cores
      • Lacks prominent umbrella cell layer
    Board review style question #1
    Urothelial papilloma Urothelial papilloma


      What is the correct diagnosis seen in the images?

    1. Low grade papillary urothelial carcinoma
    2. Papillary urothelial neoplasia of low malignant potential (PUNLMP)
    3. Polypoid cystitis
    4. Urothelial papilloma
    Board review style answer #1
    D. Urothelial papilloma. Note the papillary architecture with thin fibrovascular cores, a normal appearing urothelial lining and prominent umbrella cell layer.

    Comment Here

    Reference: Urothelial papilloma
    Board review style question #2
      Which of the following is true regarding urothelial papilloma?

    1. Associated with high rate of recurrence and progression
    2. Lined by urothelium of normal thickness and cytology
    3. Seen in an older age group than urothelial carcinoma
    4. Umbrella cell atypia excludes the diagnosis
    Board review style answer #2
    B. Urothelial papilloma is an exophytic papillary neoplasm lined by urothelium of normal thickness and cytology. Urothelial papilloma is often seen in a younger age group than urothelial carcinoma and has no or a very low recurrence rate. A prominent umbrella cell layer is characteristic. The umbrella cells often show marked atypia and this should not be used to change the diagnosis from urothelial papilloma to urothelial carcinoma.

    Comment Here

    Reference: Urothelial papilloma

    Verrucous carcinoma
    Definition / general
    • Almost all cases are associated with Schistosoma hematobium infection
    • Indolent growth, spreads by direct extension
    • Does not metastasize, although may develop foci of invasive squamous cell carcinoma
    Gross description
    • Exophytic, papillary tumor which projects into bladder lumen
    Microscopic (histologic) description
    • Bulbous fronds of well differentiated, acanthotic epithelium with pushing margin, minimal atypia
    • No/rare mitotic figures
    • May resemble condyloma focally

    Villous adenoma
    Definition / general
    Essential features
    • Villous adenoma of the urinary tract is a rare but important lesion
    • Morphologic features of these tumors are similar to those of colonic adenomas
    • Villous adenoma of the urinary tract should be removed completely and sampled thoroughly to avoid missing a more aggressive component
    Terminology
    • Also called villous tumor
    ICD coding
    • ICD-O: 8261/0 - villous adenoma, NOS
    Epidemiology
    Sites
    Pathophysiology
    • Mechanism underlying the origin of villous adenoma of the urinary bladder is still speculative (Mod Pathol 2009;22:1280)
    • It has been suggested that a cloacal rest may remain in the bladder and subsequently proliferate into a glandular neoplasm (Am J Surg 1955;90:693)
    • Alternative theory suggests that villous adenoma is the product of a chronic irritation metaplasia → dysplasia → carcinoma sequence (J Clin Pathol 1993;46:450)
    Clinical features
    Diagnosis
    • Cystoscopic findings with tissue resected
    • Imaging or colonoscopy findings to rule out a colonic malignancy
    • Thorough microscopic examination of excised tissue
    Radiology description
    • No specific findings on ultrasonography, CT or MRI
    Prognostic factors
    Case reports
    Treatment
    • Complete excision
    • Excellent prognosis if isolated (Am J Surg Pathol 1999;23:764)
    • Coexisting infiltrating carcinoma or urothelial carcinoma suggests a more aggressive course
    Gross description
    Microscopic (histologic) description
    Microscopic (histologic) images

    Contributed by Varsha Manucha, M.D.

    Villous adenoma

    Villous pattern

    Nuclear stratification

    Cytology description
    • Urine cytology may show mucin producing cells or cell clusters with gland formation or arrangement along basement membrane
    • When glandular cells with columnar mucin filled goblet cells are seen in urine cytology, the presence of a primary glandular lesion of the urinary bladder, such as villous adenoma, should be considered possible (Diagn Cytopathol 2016;44:632)
    Positive stains
    Negative stains
    Sample pathology report
    • Bladder, transurethral resection:
      • Villous adenoma (negative for carcinoma in situ or invasive carcinoma)
      • Muscularis propria is present and is negative for neoplasia
    Differential diagnosis
    • Adenocarcinoma in situ arising in villous adenoma:
      • Psuedostratification, loss of polarity, increased pleomorphism with vesicular and round nuclei
      • Intraluminal cribriforming
      • No invasion
    • Colonic adenocarcinoma, well differentiated extending into bladder:
      • Clinical history of colonic tumor
      • Definite invasion identified
      • Usually beta catenin nuclear positive
    • Papillary urothelial carcinoma with villoglandular differentiation:
      • Typically has concurrent high grade papillary urothelial carcinoma component
      • GATA3 positive
    • Nephrogenic adenoma, papillary variant:
      • Has bland cytology
      • PAX8 positive
    Board review style question #1
    A urinary bladder villous adenoma is most likely to show which of the following immunohistochemical profiles?

    1. CK7+, CK20+, beta catenin (nuclear)+, thrombomodulin+
    2. CK7-, CK20+, beta catenin (nuclear)+, thrombomodulin-
    3. CK7+, CK20+, beta catenin (nuclear)-, thrombomodulin-
    4. CK7+, CK20-, beta catenin (nuclear)-, thrombomodulin+
    Board review style answer #1
    C. CK7+, CK20+, beta catenin (nuclear)-, thrombomodulin-. The immunohistochemical profile of villous adenoma has been reported to be positive for CK20 and CEA, and ~50% of cases of urinary tract villous adenomas exhibit positive results for CK7. It is very important to distinguish from metastatic adenocarcinoma derived from the colon. Most of the cases of adenocarcinoma derived from the colon are reported to be positive for beta catenin (nuclear staining) and negative for thrombomodulin and CK7, while urothelial carcinoma cases are positive for CK7, thrombomodulin and negative for beta catenin (nuclear staining).

    Comment Here

    Reference: Villous adenoma
    Board review style question #2

    A 50 year old man with a long term history of urethral stricture and chronic cystitis presents with mucosuria. Cystoscopy reveals a papillary growth which is biopsied and is diagnosed as villous adenoma of the urinary bladder. What is the best treatment option for the patient?

    1. Bacillus Calmette-Guérin (BCG) therapy
    2. Chemotherapy
    3. Close followup
    4. Complete excision
    Board review style answer #2
    D. Complete excision. Pure and isolated villous adenomas have an excellent prognosis, even without radical resection. However, urinary tract villous adenomas have also been found in association with in situ or invasive adenocarcinoma, squamous cell carcinoma and flat in situ urothelial carcinoma. In these cases, the lesions have sometimes been clinically more aggressive, with disease recurrence or even distant metastasis. The heterogeneity of the lesions associated with urinary tract villous adenomas has important implications for diagnosis and management. Any urinary tract lesion diagnosed by biopsy as villous adenoma must be completely excised and thoroughly sampled.

    Comment Here

    Reference: Villous adenoma

    WHO classification
    Definition / general
    • This classification (WHO 2016) is applicable to tumors that occur throughout the urothelial tract including the bladder, ureter, renal pelvis and urethra
    • Urothelial carcinoma (UC) is the most common malignant neoplasm of the urothelial tract
      • High propensity for divergent differentiation and variant morphologies
    WHO (2016)
      Urothelial carcinoma ICD-O
    • Infiltrating urothelial carcinoma 8120/3
    • Urothelial carcinoma with divergent differentiation
      • With squamous differentiation
      • With glandular differentiation
      • With trophoblastic differentiation
    • Nested urothelial carcinoma (including large nested variant)
    • Microcystic urothelial carcinoma
    • Micropapillary urothelial carcinoma 8131/3
    • Lymphoepithelioma-like urothelial carcinoma 8082/3
    • Plasmacytoid / signet ring / diffuse urothelial carcinoma
    • Sarcomatoid urothelial carcinoma 8122/3
    • Giant cell urothelial carcinoma 8031/3
    • Poorly differentiated urothelial carcinoma 8020/3
    • Lipid rich urothelial carcinoma
    • Clear cell (glycogen rich) urothelial carcinoma

    Noninvasive urothelial lesions
    • Urothelial carcinoma in situ 8120/2
    • Noninvasive papillary urothelial carcinoma, low grade 8130/2
    • Noninvasive papillary urothelial carcinoma, high grade 8130/2
    • Papillary urothelial neoplasm of low malignant potential 8130/1
    • Urothelial papilloma 8120/0
    • Inverted urothelial papilloma 8121/0
    • Urothelial proliferation of uncertain malignant potential
    • Urothelial dysplasia

    Squamous cell neoplasms
    • Pure squamous cell carcinoma 8070/3
    • Verrucous carcinoma 8051/3
    • Squamous cell papilloma 8052/0

    Glandular neoplasms
    • Adenocarcinoma, NOS 8140/3
      • Enteric 8144/3
      • Mucinous 8480/3
      • Mixed 8140/3
    • Villous adenoma 8261/0

      Urachal carcinoma 8010/3

    Tumors of Müllerian type
    • Clear cell carcinoma 8310/3
    • Endometrioid carcinoma 8380/3

    Neuroendocrine tumors
    • Small cell neuroendocrine carcinoma 8041/3
    • Large cell neuroendocrine carcinoma 8013/3
    • Well differentiated neuroendocrine tumor 8240/3
    • Paraganglioma 8693/1

    Melanocytic tumors
    • Malignant melanoma 8720/3
    • Nevus 8720/0
    • Melanosis

    Mesenchymal tumors
    • Rhabdomyosarcoma 8900/3
    • Leiomyosarcoma 8890/3
    • Angiosarcoma 9120/3
    • Inflammatory myofibroblastic tumor 8825/1
    • Perivascular epithelioid cell tumor
      • Benign 8714/0
      • Malignant 8714/3
    • Solitary fibrous tumor 8815/1
    • Leiomyoma 8890/0
    • Hemangioma 9120/0
    • Granular cell tumor 9580/0
    • Neurofibroma 9540/0

    Urothelial tract hematopoietic and lymphoid tumors

    Miscellaneous tumors
    • Carcinoma of Skene, Cowper and Littre glands 8140/3
    • Metastatic tumors and tumors extending from other organs
    • Epithelial tumors of the upper urinary tract
    • Tumors arising in a bladder diverticulum
    • Urothelial tumors of the urethra
    Diagrams / tables

    Images hosted on other servers:

    Infiltrating urothelial carcinoma

    Major updates
    • New / renamed entities: poorly differentiated urothelial carcinoma - replaced undifferentiated urothelial carcinoma encompassing broad spectrum of poorly differentiated tumors including those with osteoclast-like giant cells, with mixed morphologies and undifferentiated phenotype
    • Removed entities: lymphoma-like urothelial carcinoma (now part of plasmacytoid urothelial carcinoma)
    • Refined morphologic and immunohistochemical criteria for the diagnosis of urothelial carcinoma variants and divergent differentiation:
      • Divergent differentiation occurs in a background of conventional urothelial carcinoma
      • Nested urothelial carcinoma: described large nested variant
      • Plasmacytoid urothelial carcinoma: clarified morphologic criteria in plasmacytoid urothelial carcinoma with signet ring cells as not associated with extracellular mucin production, unlike a true signet ring cell adenocarcinoma
      • Micropapillary urothelial carcinoma:
        • Diagnosis restricted to invasive component only
        • No specific threshold (percentage of presence) to classify
        • Refined morphologic criteria based on highest interobserver reproducibility: multiple nests in the same lacunar space
    • Distinct molecular alterations in some of urothelial carcinoma variants:
      • Micropapillary urothelial carcinoma: common HER2 / neu amplifications or mutations
      • Plasmacytoid urothelial carcinoma: loss of E-cadherin and CDH1 gene loss of function mutations or methylation
      • No significant association with Epstein-Barr virus (EBV) or human papillomavirus (HPV) infection and urothelial carcinoma development
    • Prognostic implications of the urothelial carcinoma variants:
      • Worse prognosis / more aggressive behavior associated with micropapillary and plasmacytoid urothelial carcinoma variants
      • Uniformly poor prognosis for sarcomatoid, poorly differentiated and giant cell urothelial carcinoma
      • Nested variant, lipid rich and urothelial carcinoma with divergent differentiation (squamous, glandular or trophoblastic) are more likely to present with advanced disease but when adjusted by stage had no survival differences with respect to conventional urothelial carcinoma
    Histological variations and provisional entities of urothelial carcinoma NOT included in the current WHO classification
    • Urothelial carcinoma, inverted growth (inverted papilloma-like)
    • Pseudoangiosarcomatous (angiosarcoma-like) urothelial carcinoma
    • Urothelial carcinoma with myxoid stroma / chordoid features
    • Urothelial carcinoma with rhabdoid features
    • Urothelial carcinoma with unusual stromal reactions
      • Pseudosarcomatous stroma
      • Stromal osseous metaplasia
      • Stromal cartilaginous metaplasia
      • Osteoclast-like giant cells
      • Prominent lymphoid infiltrate
    • Urothelial carcinoma in specific clinical setting
      • Urothelial carcinoma in augmentation cystoplasty
      • Urothelial carcinoma in neurogenic bladder (spinal cord injury)
      • Urothelial carcinoma in children and young adults
    Board review style question #1
    Which of the following tumors demonstrate invasion (tumor beyond the basement membrane)?

    1. Infiltrating urothelial carcinoma
    2. Inverted urothelial papilloma
    3. Papillary urothelial neoplasm of low malignant potential
    4. Urothelial carcinoma in situ
    5. Urothelial papilloma
    Board review style answer #1
    A. Infiltrating urothelial carcinoma

    Comment Here

    Reference: Bladder, ureter & renal pelvis - WHO classification

    WHO classification
    Definition / general
    • The new WHO 2022 classification of urinary and male genital tumors (5th edition) replaces the previous WHO 2016 classification
    • Diagnostic criteria, molecular correlates and nomenclature have been updated
    Major updates
    • 5th edition of WHO classification for bladder cancers is organized based on tumor lineage: urothelial, squamous and glandular tumors (Pathologica 2022;115:32)
      • Exceptions for urachal, diverticular and urethral accessory gland tumors
      • Separate chapters for neuroendocrine neoplasms, mesenchymal tumors, hematolymphoid malignancies, melanocytic tumors and metastatic tumors
      • Genetic tumor syndromes of urinary and male genital tract are covered in dedicated chapter
    • Histologic subtypes are now preferred over variants (Pathologica 2022;115:32)
      • Standardized terminology: genetic alterations as variants, distinct morphologies as histologic patterns and significant morphologies as tumor subtypes
      • All subtypes of urothelial carcinoma and those with divergent differentiation are classified as high grade tumors, regardless of bland morphology (e.g., microcystic / tubular, small or large nested)
      • WHO 5th edition stresses reporting the presence and percentage(s) of different subtypes and divergent differentiation in urothelial carcinomas
    • Papillary tumors are deemed high grade if containing ≥ 5% high grade component; < 5% is noted as low grade with < 5% high grade component, prompted by poor interobserver reproducibility (J Pathol Transl Med 2024;58:45, Pathologica 2022;115:32)
    • The descriptor inverted is reserved for papillary tumors with almost exclusively inverted architecture
    • Urothelial proliferation with undetermined malignant potential is no longer considered a distinct entity but rather an early low grade noninvasive papillary urothelial carcinoma or extension at the tumor edge / shoulder lesion (J Pathol Transl Med 2024;58:45, Pathologica 2022;115:32)
    • Urothelial dysplasia no longer has a separate section; the term has been retained for preneoplastic lesions falling short of carcinoma in situ diagnosis (Pathologica 2022;115:32, J Pathol Transl Med 2024;58:45)
    • Clear cell urothelial carcinoma was renamed clear cell urothelial carcinoma (glycogen rich) for clearer distinction from clear cell adenocarcinoma with Müllerian differentiation (Pathologica 2022;115:32)
    • Signet ring / diffuse has been removed from plasmacytoid subtype terminology (Pathologica 2022;115:32)
    • New edition advocates for pT1 tumor substaging via histoanatomical (tumor relative to muscularis mucosa and vascular plexus) or micrometric approach (measuring of invasive tumor component), however, it does not favor any specific methodology of subcategorization or tier system (Pathologica 2022;115:32)
    • Predictors of immune checkpoint inhibitor response: PDL1 expression in tumor and host immune cells, tumor mutation burden and microsatellite instability / mismatch repair defect status (Mod Pathol 2018;31:623)
    WHO (2022)
    Urothelial tumors ICD-O ICD-11
    • Noninvasive urothelial tumors
    8120/0 2F35 & XH5M82
    8121/0 2F35 & XH5A08
    8130/1 2F78 & XH5UU5
    8130/2 2C91.0 & XH12F0
    8130/2 2C91.0 & XH12F0
    8120/2 2C91.0 & XH5GH8
    • Invasive urothelial neoplasms
    8120/3 2C91.0 & XH8EH1

    Squamous cell neoplasms of the urinary tract
    8052/0 2F35 & XH50T2
    • Squamous cell carcinomas of the urinary tract
    8051/3 2C9Y & XH5PM0
    8070/3 2C94.1

    Glandular neoplasms
    • Adenomas
    8261/0 2F35 & XH90D6
    • Adenocarcinomas
    8140/3 2C94.0

    Urachal and diverticular neoplasms
    8010/3 2C9Y & XH74S1
    • Diverticular carcinoma
    8120/3 GC01.2 & XH8EH1

    Urethral neoplasms
    • Urethral accessory gland carcinomas
    • Littre gland adenocarcinoma of the urethra
    8140/3 2C93.0 & XH22Z8
    • Skene gland adenocarcinoma of the urethra
    8140/3 2C93.0 & XH22Z8
    • Cowper gland adenocarcinoma of the urethra
    8140/3 2C93.0 & XH22Z8

    Tumors of the Müllerian type
    8310/3 2C9Y & XH6L02
    • Endometrioid carcinoma of the urinary tract
    8380/3 2C94.Y & XH0SD2
    Diagrams / tables

    Images hosted on other servers:
    Infiltrating urothelial carcinoma

    Infiltrating urothelial carcinoma

    Microscopic (histologic) images

    Contributed by Maria Tretiakova, M.D., Ph.D.
    Urothelial carcinoma, clear cell (glycogen rich)

    Urothelial carcinoma, clear cell (glycogen rich)

    Urothelial carcinoma, giant cell

    Urothelial carcinoma, giant cell

    Urothelial carcinoma, lipid rich

    Urothelial carcinoma, lipid rich

    Urothelial carcinoma, lymphoepithelioma-like

    Urothelial
    carcinoma,
    lymphoepithelioma-
    like


    Urothelial carcinoma, microcystic / tubular

    Urothelial carcinoma, microcystic / tubular

    Urothelial carcinoma, poorly differentiated with osteoclast-like giant cells

    Urothelial carcinoma,
    poorly differentiated
    with osteoclast-like
    giant cells

    Urothelial carcinoma, trophoblastic differentiation

    Urothelial carcinoma, trophoblastic differentiation

    Board review style question #1
    What is the cut off for a high grade component in diagnosing noninvasive high grade papillary urothelial carcinoma?

    1. 1%
    2. 5%
    3. 10%
    4. 25%
    5. 50%
    Board review style answer #1
    B. Papillary tumors are deemed high grade if they contain ≥ 5% high grade component; < 5% is noted as low grade with < 5% high grade component, prompted by poor interobserver reproducibility. Wide adoption of the 5% cut off would standardize reporting and improve interobserver agreement and tumor prognostication.

    Comment Here

    Reference: Bladder & urothelial tract - WHO classification
    Board review style question #2

    What is the best diagnosis for the noninvasive lesion shown above?

    1. Early low grade noninvasive papillary carcinoma
    2. Low grade dysplasia
    3. Papillary urothelial hyperplasia
    4. Urothelial proliferation with undetermined malignant potential
    5. Urothelium with reactive atypia
    Board review style answer #2
    A. Early low grade noninvasive papillary carcinoma. This lesion is composed of short, nonbranching papillae covered by mildly atypical urothelium which has cytological features similar to those of low grade noninvasive papillary carcinoma. This lesion is no longer considered a distinct entity and should be designated as precursor or early low grade noninvasive papillary carcinoma.

    Comment Here

    Reference: Bladder & urothelial tract - WHO classification

    Well differentiated neuroendocrine tumor
    Definition / general
    • Rare low grade neuroendocrine carcinoma, similar to counterparts at other sites
    • Formerly called carcinoid tumor
    Epidemiology
    • Usually ages 47-69 years
    Sites
    • Bladder neck and trigone
    Etiology
    Clinical features
    • Rare, < 50 cases described
    • Patients present with hematuria
    • No carcinoid syndrome
    • Behavior is difficult to predict (Arch Pathol Lab Med 2006;130:1693); occasionally produces metastases and death
    Case reports
    Treatment
    • Excision is usually curative, but behavior may not be predictable
    Gross description
    • Usually 1-2 cm
    • Smooth-surfaced sessile polypoid nodules covered by urothelium
    Microscopic (histologic) description
    Positive stains
    Electron microscopy description
    • Neuroendocrine differentiation
    Differential diagnosis
    • Inverted papilloma: lacks salt and pepper chromatin of carcinoid tumors; not invasive
    • Metastasis: clinical history
    • Nested variant of urothelial carcinoma: classic urothelial carcinoma usually present elsewhere; usually no well defined neuroendocrine features
    • Paraganglioma: nests of tumor cells surrounded by S100+ sustentacular cells
    • Direct invasion of carcinoid tumor from appendix: clinical history

    With glandular differentiation
    Definition / general
    • Glandular differentiation is defined by the presence of true glandular spaces, usually tubular or gland-like lumina, or with morphology similar to enteric adenocarcinomas and variable mucin production
    • Cytoplasmic mucin containing cells are seen in normal urothelium and are not considered to represent glandular differentiation (Arch Pathol Lab Med 2007;131:1244)
    Essential features
    • Presence of true glandular spaces
      • Cytoplasmic mucin is seen in normal urothelium and not diagnostic of glandular differentiation
    • No known prognostic significance
    Epidemiology
    • Foci of glandular differentiation are less common than squamous differentiation, and seen in up to 10% of urothelial carcinomas (Mod Pathol 2009;22:S96)
    Sites
    • Bladder, kidney
    Clinical features
    • Similar to other urothelial carcinomas
    Prognostic factors
    Case reports
    Treatment
    • Too few cases to establish specific treatment recommendations
    Microscopic (histologic) description
    • True glandular structures within a conventional urothelial carcinoma
    • Glands are of either tubular or enteric type, with a single layer of neoplastic columnar cells radially arranged around a lumen, with or without mucin production
    • May resemble enteric adenocarcinomas
    • May have signet ring morphology, with neoplastic signet ring cells floating in pools of mucin
    Microscopic (histologic) images

    Contributed by Nicole K. Andeen, M.D. and Maria Tretiakova, M.D., Ph.D.

    Various images

    Differential diagnosis
    Additional references

    With squamous differentiation
    Definition / general
    • Mixed variant with malignant urothelial and squamous components (either component may be in situ only)
    Epidemiology
    Clinical features
    Microscopic (histologic) description
    • Associated with high grade urothelial carcinoma
    • By definition, must also have an identifiable malignant urothelial component (may be only carcinoma in situ)
    • Squamous component has nests of malignant squamous epithelium, characterized by polygonal cells and evidence of keratinization (dyskeratosis, keratin pearls) or intercellular bridges
    • Squamous component may have basaloid or clear cell features
    • Associated with eosinophils (J Clin Pathol 1984;37:500)
    • Recommended to report percentage of squamous component
    Microscopic (histologic) images

    Contributed by @katcollmd on Twitter and AFIP
    Urothelial carcinoma with squamous differentiation Urothelial carcinoma with squamous differentiation Urothelial carcinoma with squamous differentiation Urothelial carcinoma with squamous differentiation

    Urothelial carcinoma with squamous differentiation

    High grade tumor



    Images hosted on other servers:
    Missing Image

    Keratin pearl formation

    Missing Image

    Fig 1: Trigone mucosa

    Missing Image

    Poorly differentiated
    tumor with squamous
    differentiation

    Positive stains
    Negative stains
    • Squamous component: uroplakins (positive in urothelial component), CK20
    Differential diagnosis

    With squamous differentiation
    Definition / general
    • Urothelial carcinoma with variable amounts of squamous differentiation (intracellular keratin, intercellular bridges or keratin pearls)
    Essential features
    • Squamous differentiation is the most frequent line of divergent histology seen in high grade urothelial carcinoma
    • Approximate percentage of the squamous component should be noted in the report
    • Squamous differentiation may be seen in the context of chronic irritation such as with stones, Schistosoma infection and neurogenic bladder with indwelling catheters (Front Med (Lausanne) 2018;5:223, World J Urol 2021;39:1531)
    Terminology
    • Urothelial carcinoma with squamous differentiation
    ICD coding
    • ICD-O: 8120/3 - urothelial carcinoma with squamous differentiation
    • ICD-10
      • C67.9 - malignant neoplasm of bladder, unspecified
      • C66.9 - malignant neoplasm of unspecified ureter
      • C68.0 - malignant neoplasm of urethra
      • C65.1 - malignant neoplasm of right renal pelvis
      • C65.2 - malignant neoplasm of left renal pelvis
    Epidemiology
    Sites
    • Genitourinary tract (renal pelvis, ureter, bladder and urethra)
    Pathophysiology
    Not provided
    Etiology
    • Smoking: smoking may increase the risk due to harmful chemicals accumulating in the urine (BJU Int 2008;101:11)
    • Aging: urothelial carcinoma can occur at any age, however, most patients diagnosed with urothelial carcinoma are older than 55
    • Male gender: men are more likely to develop bladder cancer than women
    • Chemical exposure: including arsenic and chemicals used in the manufacture of dyes, rubber, leather, textiles and paint products (Lab Invest 2008;88:686)
    • Previous cancer treatment: anticancer drug cyclophosphamide and pelvic radiation treatment (Eur Urol 2019;75:319)
    • Longstanding bladder irritation: chronic or repeated urinary infections or inflammations (cystitis), long term use of a urinary catheter (World J Urol 2021;39:1531)
    • Family history of cancer: Lynch syndrome with mismatch repair gene mutations (Eur Urol 2013;63:379)
    Diagrams / tables
    None
    Clinical features
    Diagnosis
    Laboratory
    • Hematuria (red blood cells are identified in urine) is most common
    Radiology description
    Radiology images

    Images hosted on other servers:
    T1 precontrast demonstrates isoechoic mass

    T1 precontrast demonstrates isoechoic mass

    Ultrasound shows mass-like hyperechoic area

    Ultrasound shows mass-like hyperechoic area

    Prognostic factors
    • Invasive urothelial carcinoma with divergent differentiation has poor prognosis because of common presentation at high stage
    • Uncommonly noninvasive urothelial carcinoma showing divergent patterns may not have worse prognosis by limited data
    • Stage by stage, no significant association with worse cancer specific survival in comparison with conventional urothelial carcinoma (Urology 2007;70:69, J Urol 2012;188:405)
    Case reports
    • 62 year old man with differentiation of urothelial carcinoma into 2 distinct subtypes, glandular and squamous, in 3 different organs (Cureus 2020;12:e7280)
    • 69 year old White woman with invasive urothelial carcinoma with squamous differentiation and associated high risk human papillomavirus infection (Ann Diagn Pathol 2023;63:152103)
    • 72 year old man with a primary sarcomatoid urothelial carcinoma of the ureter with heterologous chondrosarcoma and osteosarcoma elements and concurrent divergent squamous differentiation (Urol Case Rep 2020;34:101484)
    Treatment
    Clinical images
    N/A
    Gross description
    • Lesion may be polypoid, sessile, fungating ulcerated or infiltrative
    Gross images

    Contributed by Huihui Ye, M.D., M.S.
    Dome tumor

    Dome tumor

    Posterior wall tumor

    Posterior wall tumor

    Frozen section description
    N/A
    Frozen section images
    N/A
    Microscopic (histologic) description
    • Most commonly associated with high grade urothelial carcinoma
    • By definition, must have clear cut evidence of urothelial component (invasive urothelial carcinoma or urothelial carcinoma in situ)
    • Squamous components are similar to typical squamous cell carcinoma: nests of squamous epithelial cells, characterized by large nuclei with abundant eosinophilic cytoplasm and evidence of keratinization (intracellular keratin, intercellular bridges or keratin pearls)
    • Squamous component may show basaloid or clear cell features
    • Tumor associated eosinophilia in the bladder may present (J Clin Pathol 1984;37:500)
    • Presence of squamous component should be reported and providing estimation of the percentage of differentiation is strongly recommended
    Microscopic (histologic) images

    Contributed by Huihui Ye, M.D., M.S.
    Basaloid features

    Basaloid features

    Clear cell feature

    Clear cell feature

    Nests of clear cells

    Nests of clear cells

    Keratin Pearls

    Keratin pearls


    Basaloid with central necrosis

    Basaloid with central necrosis

    Surface keratinization

    Surface keratinization

    Intercellular bridges

    Intercellular bridges

    Virtual slides
    N/A
    Cytology description
    • High grade urothelial carcinoma component: high N:C ratio, nuclear pleomorphism, nuclear membrane irregularity and severe hyperchromasia
    • Squamous component: keratinizing cells with high N:C ratio, large and hyperchromatic nuclei, irregular nuclear contour, polygonal cell shape, abundant eosinophilic cytoplasm or dense orangeophilic cytoplasm
    • Reference: Paris system for urothelial neoplasia
    Cytology images

    Contributed by Chunlai Zuo, M.D., M.S.
    Squamous differentiation

    Squamous differentiation

    Conventional urothelial carcinoma component

    Conventional urothelial carcinoma component

    HGUC and necrosis

    HGUC and necrosis



    Images hosted on other servers:
    High grade urothelial carcinoma with squamous differentiation

    High grade with squamous differentiation

    Immunofluorescence description
    Not provided
    Immunofluorescence images
    None
    Positive stains
    Negative stains
    • Squamous component: uroplakins (positive in urothelial component), CK20
    Electron microscopy description
    Not provided
    Electron microscopy images
    None
    Molecular / cytogenetics description
    • Basal squamous molecular type by gene expression signature subtyping (Cell 2017;171:540)
      • Basal squamous subtype is enriched in TP53 mutations
      • Tends to have elevated cell cycle activity and a significant inflammatory signature
    Molecular / cytogenetics images
    None
    Videos
    N/A
    Sample pathology report
    • Bladder, tumor, transurethral resection:
      • Invasive high grade urothelial carcinoma with squamous (~30%) and glandular (~10%) differentiation
      • Muscularis propria present and involved
      • Background urothelial mucosa with squamous metaplasia
    Differential diagnosis
    • Squamous cell carcinoma:
      • No associated urothelial component (i.e., pure squamous cell component)
        • Schistosomal associated squamous cell carcinoma
        • Nonschistosomal associated squamous cell carcinoma
        • Human papillomavirus (HPV) is not commonly associated with bladder squamous cell carcinoma
          • HPV related squamous cell carcinoma arising from cervical, anal or penile origin should be ruled out first
          • Clinical presentation and imaging studies are crucial
        • Metastatic squamous cell carcinoma secondarily involving bladder
      • Keratinizing squamous metaplasia with dysplasia or squamous cell carcinoma in situ may be present in adjacent bladder mucosa in cases of bladder primary squamous cell carcinoma
    Additional references
    None
    Board review style question #1
    Invasive urothelial carcinoma with squamous differentiation is characterized by which of the following?

    1. High risk HPV in situ hybridization positive
    2. Intercellular bridges or keratinization
    3. No clear cut evidence of urothelial component
    4. p63 positive on immunostain
    Board review style answer #1
    B. Intercellular bridges or keratinization. The diagnosis is established based on squamous morphology characterized by intercellular bridges or keratinization on H&E stains. Answer D is incorrect because p63 is also commonly positive in conventional urothelial carcinoma. Answer A is incorrect because HPV is not commonly associated with urothelial carcinoma with squamous differentiation. In contrast, high risk HPV positivity would support a secondary involvement by cervical, anal or penile squamous cell carcinoma. Answer C is incorrect because by definition clear cut evidence of urothelial component must be identified.

    Comment Here

    Reference: Urothelial carcinoma with squamous differentiation
    Board review style question #2

    A 50 year old woman presented with hematuria and was found to have a mass involving the bladder posterior wall shown above. Which of the following findings is most supportive of invasive urothelial carcinoma with squamous differentiation over secondary involvement by carcinoma from the other organs?

    1. Component of uroplakin II positive conventional urothelial carcinoma
    2. High risk HPV ISH positive
    3. Mass lesion found at lower uterine segment
    4. p16 positivity with a diffuse block-like staining pattern
    Board review style answer #2
    A. Component of uroplakin II positive conventional urothelial carcinoma. Answer A is correct because Uroplakin II is a highly specific immunohistochemical marker for urothelial differentiation. Answers B, C and D are incorrect since these findings would favor squamous cell carcinoma of cervical or uterine primary over bladder primary.

    Comment Here

    Reference: Urothelial carcinoma with squamous differentiation

    With trophoblastic differentiation
    Definition / general
    • Urothelial carcinoma with focal or diffuse trophoblastic / syncytiotrophoblastic morphology or beta hCG immunoreactivity
    Essential features
    • Carries a worse prognosis compared to conventional urothelial carcinoma
    • Multinucleated giant cells admixed with urothelial carcinoma is the most commonly encountered pattern
    • Beta hCG immunohistochemical stain highlights cells with trophoblastic differentiation
    • Reporting the presence of this element is recommended
    Terminology
    • Urothelial carcinoma with choriocarcinomatous differentiation
    ICD coding
    • ICD-O:
      • 8120/3 - urothelial carcinoma, NOS
      • 8130/3 - papillary urothelial carcinoma
    • ICD-10: C67.9 - malignant neoplasm of bladder, NOS
    Epidemiology
    Sites
    • Can be seen along the entire urothelial tract, including urinary bladder, prostatic urethra and upper urothelial tract (Urol Oncol 2021;39:732.e17)
    Pathophysiology
    • Some reports consider it as metaplastic / retrodifferentiation from urothelial to trophoblastic differentiation (Acta Cytol 1992;36:176)
    Etiology
    Clinical features
    Diagnosis
    • Transurethral resection for bladder tumors
    • Cystoscopic biopsy
    Laboratory
    • Serum beta hCG can be elevated (Br J Urol 1986;58:143)
    • Elevation of serum beta hCG is not uncommon in high grade invasive urothelial carcinoma even without obvious trophoblastic differentiation (J Urol 1986;136:403)
    Prognostic factors
    Case reports
    Treatment
    • Depending on the stage of the disease, modalities include transurethral resection, intravesical therapies, chemotherapy, radiotherapy or cystectomy
    Microscopic (histologic) description
    • Wide spectrum of trophoblastic differentiation, ranging from scattered isolated cells with trophoblastic differentiation to pure choriocarcinoma
    • Often admixed with conventional urothelial carcinoma or other variants / subtypes
    • When present as scattered isolated cells, they can be in the form of cytotrophoblasts (usually indistinguishable from high grade urothelial carcinoma or syncytiotrophoblast (recognizable by their multinucleated giant cells)
    • Resembles choriocarcinoma in other organs
    Microscopic (histologic) images

    Contributed by Anuradha Gopalan, M.D., Victor Reuter, M.D. and AFIP images
    Multinucleated giant cells Multinucleated giant cells Multinucleated giant cells

    Multinucleated giant cells

    hCG

    hCG

    GATA3

    GATA3

    Cytology description
    • Mixture of mononuclear and multinucleated giant cells with marginal indistinct vacuole-like cytoplasmic inclusions, which stains positively for human placental lactogen (HPL) (Cytopathology 2013;24:405)
    Molecular / cytogenetics description
    • Losses of chromosomes 10, 11p12-p14 and 13q22-qter and gains of chromosomes 1q and 12p12-p13
    • High level amplification at 12q14-q21 (Hum Pathol 2002;33:1234)
    Sample pathology report
    • Bladder, tumor, transurethral resection:
      • Invasive urothelial carcinoma, high grade, with trophoblastic differentiation (5%), invasive to lamina propria (see comment)
      • Comment: Muscularis propria is identified and is non-involved. Immunohistochemical studies reveal that a subset of tumor cells are positive for beta hCG and GATA3 supporting the presence of trophoblastic differentiation.
    Differential diagnosis
    Board review style question #1
    Which immunohistochemical stain is useful to confirm the trophoblastic differentiation on a bladder biopsy with multinucleated giant cells?

    1. Beta hCG
    2. CD68
    3. CK7
    4. OCT4
    Board review style answer #1
    Board review style question #2

    Which histology should be considered in the urothelial carcinoma shown in the photomicrograph?

    1. Glandular
    2. Plasmacytoid
    3. Sarcomatoid
    4. Trophoblastic
    Board review style answer #2
    Back to top
    Recent Bladder & urothelial tract Pathology books

    Amin: 2022

    Cheng: 2019

    Eble: 2022

    Epstein: 2016

    Haber: 2010

    Hansel: 2018

    IARC: 2022

    Jim Zhai: 2015

    Lopez-Beltran: 2016

    Raspollini: 2019

    VandenBussche: 2019

    VandenBussche: 2022

    Wobker: 2021

    Wojcik: 2022

    Yang: 2020

    Zhou: 2022

    Zhou: 2022



    Find related Pathology books: cytopathology, GU/adrenal, gynecologic, renal, body fluid/urinalysis
    Image 01 Image 02